You are on page 1of 365

McGILL UNIVERSITY

FACULTY OF SCIENCE

DEPARTMENT OF
MATHEMATICS AND STATISTICS

MATH 329 2005 01


THEORY OF INTEREST

Information for Students


(Winter Term, 2004/2005)
Pages 1 - 10 of these notes may be considered the
Course Outline for this course.
W. G. Brown
April 12, 2005
Information for Students in MATH 329 2005 01

Contents 8 Class Tests 34


8.1 Class Test, Version 1 . . . . . 34
1 General Information 1 8.2 Class Test, Version 2 . . . . . 40
1.1 Instructor and Times . . . . . 1 8.3 Class Test, Version 3 . . . . . 46
1.2 Course Description . . . . . . 1 8.4 Class Test, Version 4 . . . . . 52
1.2.1 Calendar Description . 1
1.2.2 Syllabus (in terms of sec- 9 Solutions, Third Problem Assign-
tions of the text-book) 1 ment 58
1.2.3 Verbal arguments . 4
1.3 Evaluation of Your Progress . 4 10 Fourth Problem Assignment 67
1.3.1 Term Mark . . . . . . 4
1.3.2 Assignments. . . . . . 4 11 Solutions to Problems on the Class
1.3.3 Class Test . . . . . . . 4 Test 70
1.3.4 Final Examination . . 5 11.1 Problems on rates of interest and
1.3.5 Supplemental Assessments 5 discount . . . . . . . . . . . . 70
1.3.6 Machine Scoring . . . 5 11.2 Problems on the values of annu-
1.3.7 Plagiarism . . . . . . . 5 ities and perpetuities with con-
1.4 Published Materials . . . . . 6 stant payments . . . . . . . . 73
1.4.1 Required Text-Book . 6 11.3 Problems on increasing and de-
1.4.2 Website . . . . . . . . 6 creasing annuities and perpetu-
1.4.3 Reference Books . . . 6 ities . . . . . . . . . . . . . . 76
1.5 Other information . . . . . . 7 11.4 Problems on combinations of an-
1.5.1 Prerequisites . . . . . 7 nuities and perpetuities . . . 78
1.5.2 Calculators . . . . . . 7 11.5 Problems on drop and balloon
1.5.3 Self-Supervision . . . . 7 payments . . . . . . . . . . . 79
1.5.4 Escape Routes . . . . 7
12 Fifth Problem Assignment 82
1.5.5 Showing your work; good
mathematical form; sim- 13 Solutions, Fourth Problem Assign-
plifying answers . . . . 7 ment 84
2 Timetable 9 14 Solutions, Fifth Problem Assign-
ment 92
3 First Problem Assignment 11
15 References 901
4 Second Problem Assignment 13
A Supplementary Lecture Notes 2001
5 Solutions, First Problem Assign-
A.1 Supplementary Notes for the Lec-
ment 16
tures of January 4th and Janu-
6 Third Problem Assignment 22 ary 5th, 2005 . . . . . . . . . 2001
A.1.1 These notes . . . . . . 2001
7 Solutions, Second Problem As- A.1.2 1.1 INTRODUCTION 2001
signment 25
Information for Students in MATH 329 2005 01

A.1.3 1.2 THE ACCUMULA- A.7.3 1.11 SUMMARY OF RE-


TION AND AMOUNT SULTS . . . . . . . . 2032
FUNCTIONS . . . . . 2002 A.7.4 2.1 INTRODUCTION 2033
A.2 Supplementary Notes for the Lec- A.7.5 2.2 OBTAINING NUMER-
ture of January 7th, 2005 . . 2007 ICAL RESULTS . . . 2033
A.2.1 1.2 THE ACCUMULA- A.8 Supplementary Notes for the Lec-
TION AND AMOUNT ture of January 21st, 2005 . . 2036
FUNCTIONS (conclusion) 2007 A.8.1 2.2 OBTAINING NUMER-
A.2.2 1.3 THE EFFECTIVE ICAL RESULTS (contin-
RATE OF INTEREST 2007 ued) . . . . . . . . . . 2036
A.3 Supplementary Notes for the Lec- A.8.2 2.3 DETERMINING TIME
ture of January 10th, 2005 . . 2009 PERIODS . . . . . . . 2037
A.3.1 1.3 THE EFFECTIVE A.8.3 2.4 THE BASIC PROB-
RATE OF INTEREST (con- LEM . . . . . . . . . . 2039
clusion) . . . . . . . . 2009 A.8.4 2.5 EQUATIONS OF VALUE 2039
A.3.2 1.4 SIMPLE INTEREST 2010 A.9 Supplementary Notes for the Lec-
A.3.3 1.5 COMPOUND INTER- ture of January 24th, 2005 . . 2041
EST . . . . . . . . . . 2012 A.9.1 2.5 EQUATIONS OF VALUE
A.4 Supplementary Notes for the Lec- (continued) . . . . . . 2041
ture of January 12th, 2005 . . 2018 A.9.2 2.6 UNKNOWN TIME 2042
A.4.1 1.6 PRESENT VALUE 2018 A.10 Supplementary Notes for the Lec-
A.5 Supplementary Notes for the Lec- tures of January 26th, 2005 . 2045
ture of January 14th, 2005 . . 2021 A.10.1 2.7 UNKNOWN RATE
A.5.1 1.7 THE EFFECTIVE OF INTEREST . . . . 2045
RATE OF DISCOUNT 2021 A.10.2 2.8 PRACTICAL EX-
A.5.2 1.8 NOMINAL RATES AMPLES . . . . . . . 2046
OF INTEREST AND DIS- A.10.3 2.9 MISCELLANEOUS
COUNT (barely begun 2025 PROBLEMS . . . . . 2049
A.6 Supplementary Notes for the Lec- A.10.4 3.1 INTRODUCTION 2050
ture of January 17th, 2005 . . 2026 A.11 Supplementary Notes for the Lec-
A.6.1 1.8 NOMINAL RATES ture of January 28rd, 2005 . . 2052
OF INTEREST AND DIS- A.11.1 3.2 ANNUITY-IMMEDIATE 2052
COUNT (continued) . 2026 A.12 Supplementary Notes for the Lec-
A.6.2 1.9 FORCES OF INTER- ture of January 31st, 2005 . . 2056
EST AND DISCOUNT A.12.1 3.3 ANNUITY-DUE 2056
(barely begun) . . . . 2030 A.13 Supplementary Notes for the Lec-
A.7 Supplementary Notes for the Lec- ture of February 2nd, 2005 . . 2058
ture of January 19th, 2005 . . 2031 A.13.1 3.3 ANNUITY-DUE (con-
A.7.1 1.9 FORCES OF INTER- tinued) . . . . . . . . 2058
EST AND DISCOUNT 2031 A.13.2 3.4 ANNUITY VALUES
A.7.2 1.10 VARYING INTER- ON ANY DATE . . . 2059
EST . . . . . . . . . . 2032
Information for Students in MATH 329 2005 01

A.14 Supplementary Notes for the Lec- A.18.2 4.4 FURTHER ANALY-
ture of February 4th, 2005 . . 2063 SIS OF ANNUITIES PAYABLE
A.14.1 3.4 ANNUITY VALUES MORE FREQUENTLY
ON ANY DATE (contin- THAN INTEREST IS CON-
ued) . . . . . . . . . . 2063 VERTIBLE . . . . . . 2090
A.14.2 3.5 PERPETUITIES 2066 A.19 Supplementary Notes for the Lec-
A.15 Supplementary Notes for the Lec- ture of February 16th, 2005 . 2093
ture of February 7th, 2005 . . 2067 A.19.1 4.5 CONTINUOUS AN-
A.15.1 3.5 PERPETUITIES (con- NUITIES . . . . . . . 2093
tinued) . . . . . . . . 2067 A.19.2 4.6 BASIC VARYING
A.15.2 3.6 NONSTANDARD TERMS ANNUITIES . . . . . 2093
AND INTEREST RATES 2068 A.20 Supplementary Notes for the Lec-
A.15.3 3.7 UNKNOWN TIME 2068 ture of February 18th, 2005 . 2094
A.16 Supplementary Notes for the Lec- A.20.1 4.6 BASIC VARYING
ture of February 9th, 2005 . . 2070 ANNUITIES (continued) 2094
A.16.1 3.7 UNKNOWN TIME A.20.2 4.7 MORE GENERAL
(continued) . . . . . . 2070 VARYING ANNUITIES 2099
A.16.2 3.8 UNKNOWN RATE A.21 Supplementary Notes for the Lec-
OF INTEREST . . . . 2073 ture of February 28th, 2005 . 2100
A.16.3 3.9 VARYING INTER- A.21.1 4.6 BASIC VARYING
EST . . . . . . . . . . 2081 ANNUITIES (conclusion) 2100
A.16.4 3.10 ANNUITIES NOT A.21.2 4.8 CONTINUOUS VARY-
INVOLVING COMPOUND ING ANNUITIES . . 2101
INTEREST . . . . . . 2081 A.21.3 4.9 SUMMARY OF RE-
A.17 Supplementary Notes for the Lec- SULTS . . . . . . . . 2101
ture of February 11th, 2005 . 2082 A.21.4 5.1 INTRODUCTION 2101
A.17.1 4.1 INTRODUCTION 2082 A.21.5 5.2 DISCOUNTED CASH
A.17.2 4.2 ANNUITIES PAYABLE FLOW ANALYSIS . . 2101
AT A DIFFERENT FRE- A.21.6 5.3 UNIQUENESS OF
QUENCY THAN INTER- THE YIELD RATE . 2101
EST IS CONVERTIBLE 2082 A.22 Supplementary Notes for the Lec-
A.18 Supplementary Notes for the Lec- ture of March 2nd, 2005 . . . 2103
ture of February 14th, 2005 . 2086 A.22.1 5.4 REINVESTMENT
A.18.1 4.3 FURTHER ANALY- RATES . . . . . . . . 2103
SIS OF ANNUITIES PAYABLE A.22.2 5.5 INTEREST MEA-
LESS FREQUENTLY THAN SUREMENT OF A FUND 2106
INTEREST IS CONVERT- A.22.3 5.6 TIME-WEIGHTED
IBLE . . . . . . . . . 2086 RATES OF INTEREST 2106
A.22.4 5.7 PORTFOLIO METH-
ODS AND INVESTMENT
YEAR METHODS . . 2106
Information for Students in MATH 329 2005 01

A.22.5 5.8 CAPITAL BUDGET- A.28.1 7.2 TYPES OF SECU-


ING . . . . . . . . . . 2106 RITIES (conclusion) . 2130
A.22.6 5.9 MORE GENERAL A.28.2 7.3 PRICE OF A BOND 2131
BORROWING/LENDING A.29 Supplementary Notes for the Lec-
MODELS . . . . . . . 2106 ture of March 21st, 2005 . . . 2138
A.23 Supplementary Notes for the Lec- A.29.1 7.2 TYPES OF SECU-
ture of March 4th, 2005 . . . 2107 RITIES (conclusion) . 2138
A.23.1 6.1 INTRODUCTION 2107 A.29.2 7.3 PRICE OF A BOND 2140
A.23.2 6.2 FINDING THE OUT- A.30 Supplementary Notes for the Lec-
STANDING LOAN BAL- ture of March 23rd, 2005 . . . 2144
ANCE . . . . . . . . . 2107 A.31 Supplementary Notes for the Lec-
A.24 Supplementary Notes for the Lec- ture of March 30th, 2005 . . . 2146
ture of March 7th, 2005 . . . 2112 A.31.1 7.4 PREMIUM AND DIS-
A.24.1 6.3 AMORTIZATION SCHED- COUNT . . . . . . . . 2146
ULES . . . . . . . . . 2112 A.31.2 7.7 CALLABLE BONDS 2149
A.25 Supplementary Notes for the Lec- A.32 Supplementary Notes for the Lec-
ture of March 11th, 2005 . . . 2115 ture of April 1st, 2005 . . . . 2153
A.25.1 6.3 AMORTIZATION SCHED- A.32.1 7.6 DETERMINATION
ULES (continued) . . 2115 OF YIELD RATES . 2157
A.25.2 6.4 SINKING FUNDS 2119 A.32.2 7.5 VALUATION BE-
A.26 Supplementary Notes for the Lec- TWEEN COUPON PAY-
ture of March 14th, 2005 . . . 2123 MENT DATES . . . . 2157
A.26.1 6.4 SINKING FUNDS A.32.3 7.8 SERIAL BONDS 2159
(concluded) . . . . . . 2123 A.32.4 7.9 SOME GENERAL-
A.26.2 6.5 DIFFERING PAY- IZATIONS . . . . . . 2159
MENT PERIODS AND A.32.5 7.10 OTHER SECURI-
INTEREST CONVERSION TIES . . . . . . . . . . 2159
PERIODS . . . . . . . 2127 A.32.6 7.11 VALUATION OF
A.26.3 6.6 VARYING SERIES SECURITIES . . . . . 2159
OF PAYMENTS . . . 2127
A.26.4 6.7 AMORTIZATION WITH B Problem Assignments, Tests, and
CONTINUOUS PAYMENTS 2127 Examinations from Previous Years 3001
A.26.5 6.8 STEP-RATE AMOUNTS B.1 2002/2003 . . . . . . . . . . . 3001
OF PRINCIPAL . . . 2127 B.1.1 First 2002/2003 Problem
A.27 Supplementary Notes for the Lec- Assignment, with Solu-
ture of March 16th, 2005 . . . 2128 tions . . . . . . . . . . 3001
A.27.1 7.1 INTRODUCTION 2128 B.1.2 Second 2002/2003 Prob-
A.27.2 7.2 TYPES OF SECU- lem Assignment, with So-
RITIES . . . . . . . . 2128 lutions . . . . . . . . . 3006
A.28 Supplementary Notes for the Lec- B.1.3 Third 2002/2003 Prob-
ture of March 21st, 2005 . . . 2130 lem Assignment, with So-
lutions . . . . . . . . . 3011
Information for Students in MATH 329 2005 01

B.1.4 Fourth 2002/2003 Prob-


lem Assignment, with So-
lutions . . . . . . . . . 3018
B.1.5 Fifth 2002/2003 Problem
Assignment, with Solu-
tions . . . . . . . . . . 3028
B.1.6 2002/2003 Class Tests, with
Solutions . . . . . . . 3036
B.1.7 Final Examination, 2002/2003 3042
B.2 2003/2004 . . . . . . . . . . . 3045
B.2.1 First 2003/2004 Problem
Assignment, with Solu-
tions . . . . . . . . . . 3045
B.2.2 Second 2003/2004 Prob-
lem Assignment, with So-
lutions . . . . . . . . . 3048
B.2.3 Third 2003/2004 Prob-
lem Assignment, with So-
lutions . . . . . . . . . 3058
B.2.4 Fourth 2003/2004 Prob-
lem Assignment, with So-
lutions . . . . . . . . . 3068
B.2.5 Fifth 2003/2004 Problem
Assignment, with Solu-
tions . . . . . . . . . . 3073
B.2.6 2003/2004 Class Test, Ver-
sion 1 . . . . . . . . . 3079
B.2.7 2003/2004 Class Test, Ver-
sion 2 . . . . . . . . . 3080
B.2.8 2003/2004 Class Test, Ver-
sion 3 . . . . . . . . . 3082
B.2.9 2003/2004 Class Test, Ver-
sion 4 . . . . . . . . . 3083
B.2.10 Solutions to Problems on
the 2003/2004 Class Tests 3085
B.2.11 Final Examination, 2003/2004 3094
B.2.12 Supplemental/Deferred Ex-
amination, 2003/2004 3097
Information for Students in MATH 329 2005 01 1

1 General Information
Distribution Date: Friday, January 7th, 2005
subject to correction

(All information is subject to change, either by announcements at lectures,


on WebCT, or in print.)
An updated version may be placed, from time to time, on the Math/Stat
website (cf. 1.4.2 below), and will also be accessible via a link from WebCT.)
The Course Outline for MATH 329 2005 01 can be considered to be pages
1 through 10 of these notes.

1.1 Instructor and Times

INSTRUCTOR: Prof. W. G. Brown


OFFICE: BURN 1224
OFFICE HRS. W 13:2014:15 h.;
(subject to F 1011 h.;
change) and by appointment
TELEPHONE: 3983836
E-MAIL: BROWN@MATH.MCGILL.CA
CLASSROOM: BURN 1B36
(room change effective 7 Jan 05)
CLASS HOURS: MWF 14:3515:25 h.

Table 1: Instructor and Times

1.2 Course Description


1.2.1 Calendar Description
THEORY OF INTEREST. (3 credits) (Prerequisite: MATH 141.) Simple and com-
pound interest, annuities certain, amortization schedules, bonds, depreciation.

1.2.2 Syllabus (in terms of sections of the text-book)


The central part of the course consists of many of the topics in the first nine chapters of
the textbook [1]1 ; section numbers, where shown, refer to that book. In the list below
1
[n] refers to item n in the bibliography, page 901.

UPDATED TO April 12, 2005


Information for Students in MATH 329 2005 01 2

we show the chapters and appendices of the textbook. Following each is a description
as of the date of this revision, of the sections to be excluded. This list will be updated
during the semester, as becomes apparent that certain sections are not appropriate to
the level of the course or the lecture time available.

Chapter 1. The Measurement of Interest 1.1-1.8. Portions of 1.9, will be


omitted. For the present 1.10 will also be omitted.

Chapter 2. Solution of Problems in Interest In 2.6 You may omit the discussion
[1, pp. 45-46] of the method of equated time.

Chapter 3. Elementary Annuities You may omit [1, 3.6 Nonstandard terms and
interest rates], [1, 3.10 Annuities not involving compound interest, pp. 82-88] and cor-
responding exercises. We will also omit [1, 3.8 Unknown rate of interest] and [1, 3.9
Varying interest] for the present (possibly to return).

Chapter 4. More General Annuities In the following section we shall consider the
problems strictly on an ad hoc basis: students are not expected to derive nor to apply
the identities obtained: [1, 4.2 Annuities payable at a different frequency than interest
is convertible; 4.3 Further analysis of annuities payable less frequently than interest is
convertible; 4.4 Further analysis of annuities payable more frequently than interest is
convertible]. Omit [1, 4.5 Continuous annuities] for the present. In [1, 4.6 Nonstandard
terms and interest rates] we shall consider the derivation of formul for (Ia)n , (Is)n ,
(Da)n , (Ds)n , and their due and perpetual variants, also the question of annuities in
geometric progression. Omit [1, 4.7 More general varying annuities, 4.8 Continuous
varying annuities, 4.9 Summary of results] for the present, together with their exercises.

Chapter 5. Yield Rates Omit [1, 5.2 Discounted cash flow analysis], except for the
definition [1, p. 131] of yield rate. Omit [1, 5.3 Uniqueness of the yield rate] except you
should read and understand the example [1, p. 133] of a problem where the yield rate
is not unique. Omit [1, 5.5 5.9] and accompanying exercises; but we will study [1,
5.4 Reinvestment rates] in preparation for Chapter 6.

Chapter 6. Amortization Schedules and Sinking Funds In [1, 6.4] omit pages
178-179, where the function an i&j is introduced. Omit [1, 6.5 6.8] and accompanying
exercises.
Information for Students in MATH 329 2005 01 3

Chapter 7. Bond and Other Securities Omit [1, 7.6 Determination of yield rates],
[1, 7.8 Serial bonds], [1, 7.9 Some generalizations], [1, 7.10 Other securities], [1, 7.11
Valuation of securities].

Chapter 8. Practical Applications Omit this chapter.

Chapter 9. More Advanced Financial Analysis Omit this chapter.

Chapter 10. A Stochastic Approach to Interest Omit this chapter.

Appendix I. Table of compound interest functions While most calculations will


be done using calculators, these tables may prove useful.

Appendix II. Table numbering the days of the year

Appendix III. Basic mathematical review Topics that are beyond the required
prerequisites will be explained if, as, and when they are used.

Appendix IV. Statistical background Omit this section: no background in prob-


ability is prerequisite to Math 329.

Appendix V. Iteration methods

Appendix VI. Further analysis of varying annuities Omit this Appendix, which
is concerned with the formula for Summation by Parts, analogous to integration by
parts for functions of a continuous variable.

Appendix VII. Illustrative mortgage loan amortization schedule

Appendix VIII. Full immunization Omit this Appendix, which is related to [1,
9.9], which is not in the syllabus.

Appendix IX. Derivation of the variance of an annuity Omit this Appendix.

Appendix X. Derivation of the Black-Scholes formula Omit this Appendix.

UPDATED TO April 12, 2005


Information for Students in MATH 329 2005 01 4

1.2.3 Verbal arguments


An essential feature of investment and insurance mathematics is the need to be able to
understand and to formulate verbal arguments; that is, explanations of the truth of
an identity presented verbally i.e., a proof in words, rather then an algebraic proof. In
a verbal argument we seek more than mathematically correctness: we wish to see an
explanation that could be presented to a layman who is not competent in the mathe-
matical bases of this subject, but is still possessed of reason, and needs to be assured
that he is not being exploited. This facet of the course will be seen, at first, to be quite
difficult. When the skill has been mastered it can be used to verify the correctness of
statements proved mathematically. Verbal arguments require some care with the under-
lying language; students who have difficulty with expression in English are reminded that
all students have the right to submit any written materials in either English or French.2

1.3 Evaluation of Your Progress


1.3.1 Term Mark
The Term Mark will be computed one-third from the assignment grades, and two-thirds
from the class test. The Term Mark will count for 30 of the 100 marks in the final
grade, but only if it exceeds 30% of the final examination percentage; otherwise the final
examination will be used exclusively in the computation of the final grade.

1.3.2 Assignments.
A total of about 6 assignments will be worth 10 of the 30 marks assigned to Term Work.

1.3.3 Class Test


A class test, will be held on Wednesday, March 09th, 2005, at the regular class time,
counting for 20 of the 30 marks in the Term Mark. This date may be changed, after
discussion with the class at any scheduled lecture date. Students who dont come to
class should ensure that they are aware of any changes in the date of the test. There
will be no make-up test for persons who miss the test.
2
For a lexicon of actuarial terms in English/French, see The Canadian Institute of Actuaries English-
French lexicon [8], at

http://www.actuaries.ca/publications/lexicon/

UPDATED TO April 12, 2005


Information for Students in MATH 329 2005 01 5

1.3.4 Final Examination


Written examinations form an important part of the tradition of actuarial mathematics.
The final examination in MATH 329 2004 01 will count for either 70% or 100% of the
numerical grade from which the submitted final letter grade will be computed. Where
a students Final Examination percentage is superior to her Term Mark percentage, the
Final Examination grade will replace the Term Mark grade in the calculations.
A 3-hour-long final examination will be scheduled during the regular examination
period for the winter term (April 14th, 2005 through April 29th, 2005). You are advised
not to make any travel arrangements that would prevent you from being present on
campus at any time during this period. Students who have religious or other constraints
that could affect their ability to write examinations at particular times should watch for
the Preliminary Examination Timetable, as their rights to apply for special consideration
at their faculty may have expired by the time the final examination timetable is published.

1.3.5 Supplemental Assessments


Supplemental Examination. For eligible students who obtain a Final Grade of F
or D in the course there will be a supplemental examination. (For information about
Supplemental Examinations, see the McGill Calendar, [3].)

There is No Additional Work Option. Will students with marks of D, F, or J


have the option of doing additional work to upgrade their mark? No. (Additional
Work refers to an option available in certain Arts and Science courses, but not available
in this course.)

1.3.6 Machine Scoring


Will the final examination be machine scored? While there could be Multiple Choice
questions on quizzes, and/or the Final Examination, such questions will not be machine
scored.

1.3.7 Plagiarism
While students are not discouraged from discussing assignment problems with their col-
leagues, the work that you submit whether through homework, the class test, or on
tutorial quizzes or the final examination should be your own. The Handbook on Student
Rights and Responsibilities states in 15(a)3 that
3
http://ww2.mcgill.ca/students-handbook/chapter3secA.html
Information for Students in MATH 329 2005 01 6

No student shall, with intent to deceive, represent the work of another person
as his or her own in any academic writing, essay, thesis, research report,
project or assignment submitted in a course or program of study or represent
as his or her own an entire essay or work of another, whether the material so
represented constitutes a part or the entirety of the work submitted.
You are also referred to the following URL:
http://www.mcgill.ca/integrity/studentguide/

1.4 Published Materials


1.4.1 Required Text-Book
The textbook for the course this semester is [1] Stephen G. Kellison, The Theory of
Interest, Second Edition. Irwin/McGraw-Hill, Boston, etc. (1991), ISBN 0-256-09150-1.

1.4.2 Website
These notes, and other materials distributed to students in this course, will be accessible
at the following URL:
http://www.math.mcgill.ca/brown/math329b.html
The notes will be in pdf (.pdf) form, and can be read using the Adobe Acrobat reader,
which many users have on their computers. This free software may be downloaded from
the following URL:
4
http://www.adobe.com/prodindex/acrobat/readstep.html
Where revisions are made to distributed printed materials for example these informa-
tion sheets it is expected that the last version will be posted on the Web.
The notes will also be available via a link from the WebCT URL:
http://webct.mcgill.ca
but not all features of WebCT will be implemented.

1.4.3 Reference Books


The textbook used for 2001-2003 may be used as a reference: [5] Michael M. Parmen-
tier, Theory of Interest and Life Contingencies, with Pension Applications: A Problem-
Solving Approach, 3rd edition. ACTEX Publications, Winstead, Conn. (1999), ISBN
0-56698-333-9.
4
At the time of this writing the current version is 5.1.
Information for Students in MATH 329 2005 01 7

1.5 Other information


1.5.1 Prerequisites
It is your responsibility as a student to verify that you have the necessary calculus
prerequisites. It would be foolish to attempt to take the course without them.

1.5.2 Calculators
The use of non-programmable, non-graphing calculators only will be permitted in home-
work, tests, or the final examination in this course. Students may be required to convince
examiners and invigilators that all memories have been cleared. The use of calculators
that are either graphing or programmable will not be permitted during test or examina-
tions, in order to level the playing field.

1.5.3 Self-Supervision
This is not a high-school course, and McGill is not a high school. The monitoring of
your progress before the final examination is largely your own responsibility. While the
instructor is available to help you, he cannot do so unless and until you identify the
need for help. While the significance of the homework assignments and class test in the
computation of your grade is minimal, these are important learning experiences, and
can assist you in gauging your progress in the course. This is not a course that can
be crammed for: you must work steadily through the term if you wish to develop the
facilities needed for a strong performance on the final examination.

Working Problems on Your Own. You are advised to work large numbers of prob-
lems from your textbook. The skills you acquire in solving textbook problems could have
much more influence on your final grade than either the homework or the class test.

1.5.4 Escape Routes


At any time, even after the last date for dropping the course, students who are experi-
encing medical or personal difficulties should not hesitate to consult their advisors or the
Student Affairs office of their faculty. Dont allow yourself to be overwhelmed by such
problems; the University has resource persons who may be able to help you.

1.5.5 Showing your work; good mathematical form; simplifying answers


When, in a quiz or examination problem, you are explicitly instructed to show all your
work, failure to do so could result in a substantial loss of marks possibly even all
of the marks; this is the default. The guiding principle should be that you want to be
Information for Students in MATH 329 2005 01 8

able to communicate your precise reasoning to others and to yourself. You are always
expected to simplify any algebraic or numerical expressions that arise in your solutions
or calculations. Verbal proofs are expected to be convincing: it will not be sufficient
to simply describe mathematical expressions verbally.
Information for Students in MATH 329 2005 01 9

2 Timetable
Distribution Date: (original version) Tuesday, January 4th, 2005
this revision as of April 12, 2005
(Subject to correction and change.)
Section numbers refer to the text-book.5

MONDAY WEDNESDAY FRIDAY


JANUARY
04 (TUESDAY!) 1.1 05 1.11.2 07 1.21.3
1.2
10 1.31.5 12 1.6 14 1.7, 1.8
Course changes must be completed on MINERVA by Jan. 16, 2005
17 1.8, 1.9 (part) 19 1.9, 2.1, 2.2 21 2.2, 2.3, 2.4
Deadline for withdrawal with fee refund = Jan. 23, 2005
24 2.5, 2.6 26 2.7, 2.8, 3.1 28 3.2, 3.3
Verification Period: January 31February 04, 2005
31 3.3 1

FEBRUARY
02 3.4 04 3.4, 3.5
Deadline for withdrawal (with W) from course via MINERVA = Feb. 13, 2005
07 3.7 09 3.7 11 4.1, 4.2
14 4.3, 4.4 2 16 4.6 18 4.6
Study Break: February 2125, 2005
No lectures, no regular office hours
21 NO LECTURE 23 NO LECTURE 25 NO LECTURE
28 5.3,5.4

Notation:
n = Assignment #n due today

R = Read Only
X = reserved for eXpansion or review
Information for Students in MATH 329 2005 01 10

MONDAY WEDNESDAY FRIDAY


MARCH
02 5.4 04 6.1, 6.2
07 6.3 3 09 CLASS TEST 11 6.3, 6.4
14 6.4 16 7.1 18 7.2
21 7.3 23 7.3, 7.4 25 NO LECTURE
28 NO LECTURE 30
APRIL
01
04 Discussion of old 06 Discussion of old 08 NO LECTURE
exam exam
11 X 13 X
Information for Students in MATH 329 2005 01 11

3 First Problem Assignment


Mounted on the Web on Sunday, January 16th, 2005
Distributed in hard copy on Wednesday, January 25th, 2005
Full solutions are to be submitted by Monday, January 31st, 2005

These problems are to be solved with full solutions, modelled either on the solutions to
problems in the textbook, or in the notes on the Web for this or previous years. The
essence is that the reader should be able to reconstruct every step of the proof from what
you have written: getting the right answer is never enough. You are not being graded
for elegance, but simply for the proof being logical, without serious gaps.
1. (a) Show that the function 225 (t 10)2 cannot be used as an amount function
for t 10.
(b) For the interval 0 t 10, determine the accumulation function a(t) that
corresponds to A(t) = 225 (t 10)2 .
(c) Determine, for the above functions, I3 , i6 , d7 .
(d) Suppose that A(0) is invested at time t = 0 and accumulates to A(10) at time
t = 10. What is the equivalent effective annual rate of compound interest,
compounded every year, which would yield the same accumulation after 10
years.
(e) Suppose that A(0) is invested at time t = 0 and accumulates to A(10) at time
t = 10. What is the equivalent annual rate of simple interest which would
yield the same accumulation after 10 years.
(f) Suppose that A(0) is invested at time t = 0 and accumulates to A(10) at time
t = 10. What is the equivalent annual rate of simple discount.
(g) Suppose that A(0) is invested at time t = 0 and accumulates to A(10) at time
t = 10. What is the equivalent annual rate of compound discount.
(h) Verify that the rates i and d of compound interest and discount have the
property that (1 + i)(1 d) = 1.

2. Determine the accumulated value of 100 at the end of 2 years if

(a) the nominal annual rate of interest is 8% convertible quarterly


(b) the nominal annual rate of discount is 8% convertible once every 6 years
(c) interest is compounded instantaneously at the rate of 8%
(d) an annual effective compound discount rate of 8% is applied
(e) an annual simple discount rate of 8% is applied
Information for Students in MATH 329 2005 01 12

(f) interest is earned during the first year at a nominal annual rate of 8% con-
vertible quarterly, and during the second year at a nominal annual rate of
discount of 8% convertible quarterly.

3. Today is New Years Day. In return for payments of 1500 at the end of January,
February, and March, and of 3000 at the end of May, July, and September, an
investor agrees to pay now the total value of the 6 payments, and to either make
or receive an additional payment at the end of December. Find the amount of
that additional payment if it is known that the nominal annual interest rate is 6%,
compounded monthly. (First set up an equation of value.)

4. Analogously to the rule of 72, you are asked to develop a rule of n to approximate
how long it takes for money to increase to 1 21 times its initial value. (That is, to
determine a 2-digit integer N = 10n1 + n0 (where n0 , n1 are decimal digits), for
which (0.01) Ni is a good approximation to the number of years required.) Your
approximation should be best around 8%.

5. (a) Find the smallest nominal rate of interest convertible monthly at which the
accumulated value of 15,000 at the end of 3 years is at least 24,000.
(b) Find the smallest6 nominal rate of discount convertible semi-annually at which
the accumulated value of 15,000 at the end of 3 years is at least 24,000.

6. Let x be a positive real number, 0 x < 1. Prove that


1
1+x
1x
for all such x. Conclude that the accumulated value of 100 after 10 years at an
interest rate of x% is always less than or equal to the accumulated value of 100
after 10 years at a discount rate of x%, with equality holding only when x = 0.

6
Added 24 Jan 05: One student observed that the original wording of this problem, with largest, was
clearly not what was intended.

UPDATED TO April 12, 2005


Information for Students in MATH 329 2005 01 13

4 Second Problem Assignment


Mounted on the Web on Sunday, January 30th, 2005
Distributed in hard copy on Wednesday, February 2nd, 2005
Full solutions are to be submitted by Monday, February 14th, 2005.
(subject to correction)

These problems are to be solved with full solutions, modelled either on the solutions to
problems in the textbook, or in the notes on the Web for this or previous years. The
essence is that the reader should be able to reconstruct every step of the proof from what
you have written: getting the right answer is never enough. You are not being graded
for elegance, but simply for the proof being logical, without serious gaps.

1. (a) At a certain rate of compound interest an investment of 1000 will grow to


1500 at the end of 12 years. Determine its value at the end of 5 years.
(b) At a certain rate of compound interest an investment of 1000 will grow to
1500 at the end of 12 years. Determine precisely when its value is exactly
1200.
(c) A debt of 7000 is due at the end of 5 years. If 2000 is paid at the end of
1 year, what single payment should be made at the end of the 2nd year to
liquidate the debt, assuming interest at the rate of 6.5% per year, compounded
quarterly.
(d) George agrees to buy his brothers car for 7000. He makes a down payment of
4000, and agrees to pay two equal payments, one at the end of 6 months, and
the other at the end of a year. If interest is being charged at 5% per annum
effective, how large should each of the equal payments be?
(e) A bill for 1500 is purchased for 1000 15 months before it is due. Determine
the nominal rate of discount, compounded monthly, which the purchaser is
paying.
(f) Bills for 1500 are regularly purchased for 1000 15 months before they are due.
The purchaser knows that, among 10 such bills, he will be unable to collect
anything on one of them, will have to pay his lawyers 500 each to effect
collection on 2 others, and will collect the others without any impediment.
Lumping all of these together, what is the effective annual interest rate earned
by the purchaser on his investment, if it is assumed that the lawyers accounts
are due at the same time as the bills?

2. A government offers savings bonds in multiples of $1000, which mature in 10 years


at $2000, but pay no interest until they are redeemed.
Information for Students in MATH 329 2005 01 14

(a) Assuming interest compounded semi-annually, what nominal annual rate of


interest does the bond holder earn?
(b) Suppose that the bonds earn interest at the nominal rate of i 0.01 for
the first 5 years, compounded semi-annually; and that they earn interest at
the nominal rate of i + 0.01 compounded semi-annually for the last 5 years.
Determine i.
(c) The government has contracted with a bank to market the bonds, at a cost
of $40 per $1000 bond. What interest rate, compounded semi-annually, is the
government paying for the net proceeds it receives for each $1000 bond?

3. The cash price of a new automobile is 18,000 plus 15.025% tax. The purchaser
is prepared to finance the car and taxes at 18% convertible semi-monthly, and to
make payments of 230 at the end of every half-month for 3.5 years, with the first
payment to be made one half-month after delivery. The dealer requires a down
payment upon delivery, both to make up the gap in the financing, and from which
to pay his immediate costs (commission to the salesperson, preparation costs, sales
taxes).

(a) Determine the value of this down payment.


(b) Determine the value of the down payment if the purchaser decides, instead of
semi-monthly payments, to make a payment of 460 at the end of every month,
first payment a month after delivery, last payment to be made 3.5 years after
delivery. The interest rate and compounding period do not change.
(c) Determine the value of the down payment if the original conditions are changed
so that the first payment of 230 is made 1.5 months after delivery, but the
same number of payments of 230 are made as originally planned. The interest
rate and compounding period do not change.

4. An employee aged exactly 40 decides to accumulate a fund for retirement at age 65


by depositing 200 at the beginning of each month for 25 years. When she reaches
age 65, she plans to withdraw a fixed amount at the beginning of each year for 15
years. Assuming that all payments are made, determine the amount of the annual
payments that she will be able to withdraw:

(a) if the interest rate is always taken to be (a nominal annual rate of) 6% per
annum, compounded monthly.
(b) if the interest rate is taken to be 6% per annum compounded monthly during
the time when the fund is being built up, and 8% per annum effective when
she reaches age 65.
Information for Students in MATH 329 2005 01 15

(c) if the interest rate is taken to be 4% per annum compounded monthly for the
coming 5 years, then 6% per annum compounded monthly for the next 20
years while the fund is being built up, then 8% per annum effective when the
fund is paying out annual payments.

5. In his will, a benefactor of McGill contributes a large sum of money to establish


a fund, whose proceeds are to be used to provide annual bursaries of 5000 to
6 actuarial students. The principal of the fund is to remain constant, and the
bursaries are funded by the interest earned, forever.

(a) If the effective annual interest rate is assumed to be 6% forever, determine the
lump sum that the benefactors estate needs to contribute to McGill a year
before the first payment of bursaries.
(b) If the effective annual interest rate is assumed to be 6% forever, determine
the lump sum that the benefactors estate needs to contribute to McGill if
the first bursaries are to be issued immediately.
(c) Suppose that each student receives not a bursary of 5000, but an annual
annuity-due of 1250 for 4 years. Under these changed conditions, determine
the lump sum payment needed now if the first bursaries are to be awarded
immediately or 1 year hence respectively. (The number of awards will remain
the same each year 6 students are awarded a 4-year sequence of bursary
payments of 1250, the first payment to be made immediately.)

6. Let m and n be positive integers. Consider an annuity-immediate which pays 1 at


the end of every period for mn periods. Explain in words why each of the following
formul represents the value of this annuity m + 1 years before the first payment
is made. (If you have doubts about the truth of this claim, you may wish to verify
algebraically that the claim is correct before you attempt to explain it in words.)

(a) v m amn
(b) am(n+1) am
(c) v m am(n+1) v m(n+2) sm
Information for Students in MATH 329 2005 01 16

5 Solutions, First Problem Assignment


Mounted on the Web on Thursday, February 10th, 2005
(Caveat lector! 7 There could be some undetected misprints or errors.)
Full solutions were to be submitted by Monday, January 31st, 2005

Students were advised that These problems are to be solved with full solutions, modelled
either on the solutions to problems in the textbook, or in the notes on the Web for this
or previous years. The essence is that the reader should be able to reconstruct every
step of the proof from what you have written: getting the right answer is never enough.
You are not being graded for elegance, but simply for the proof being logical, without
serious gaps.

1. (a) Show that the function 225 (t 10)2 cannot be used as an amount function
for t 10.
(b) For the interval 0 t 10, determine the accumulation function a(t) that
corresponds to A(t) = 225 (t 10)2 .
(c) Determine, for the above functions, I3 , i6 , d7 .
(d) Suppose that A(0) is invested at time t = 0 and accumulates to A(10) at time
t = 10. What is the equivalent effective annual rate of compound interest,
compounded every year, which would yield the same accumulation after 10
years.
(e) Suppose that A(0) is invested at time t = 0 and accumulates to A(10) at time
t = 10. What is the equivalent annual rate of simple interest which would
yield the same accumulation after 10 years.
(f) Suppose that A(0) is invested at time t = 0 and accumulates to A(10) at time
t = 10. What is the equivalent annual rate of simple discount.
(g) Suppose that A(0) is invested at time t = 0 and accumulates to A(10) at time
t = 10. What is the equivalent annual rate of compound discount.
(h) Verify that the rates i and d of compound interest and discount have the
property that (1 + i)(1 d) = 1.
Solution:

(a) By condition [1, 2, p. 2], an accumulation function must be increasing (or at


least non-decreasing); the same property must hold for an amount function,
7
Let the reader beware!
Information for Students in MATH 329 2005 01 17

since it is a positive multiple of its corresponding accumulation function. But


A(t) = 225 (t 10)2 has the property that
d
A(t) = 2(10 t)
dt
which is increasing only when 20 t 0, i.e., when t 10.
A(t) 125 + 20t t2 (5 + t)(25 t)
(b) a(t) = = = .
A(0) 125 125
(c)

I3 = A(3) A(2) = 15
A(6) A(5) 9
i6 = a(6) a(5) = =
A(5) 200
I7 A(7) A(6) 7
d7 = = =
A(7) A(7) 216

(d) Let i be the equivalent annual rate of compound interest. Since A(10) = 225,
A(0) = 125,
101
10 225
125(1 + i) = 225 1 + i =
125

10
i = 1.8 1 = 6.0540482%.

(e) Let i be the equivalent annual rate of simple interest. Then


225
125(1 + 10i) = 225 1 + 10i =
125
i = 8%.

(f) Let d be the equivalent annual rate of simple discount. Then


2
125 = (1 10d)225 d = = 4.4444%.
45
(g) Let d be the equivalent annual rate of simple discount. Then
r
10 10 5
125 = 225(1 d) d = 1 = 5.70845551%.
9

(h) (1 + 0.060540482)(1 0.0570845551) = 1 .


Information for Students in MATH 329 2005 01 18

2. Determine the accumulated value of 100 at the end of 2 years if

(a) the nominal annual rate of interest is 8% convertible quarterly


(b) the nominal annual rate of discount is 8% convertible once every 6 years
(c) interest is compounded instantaneously at the rate of 8%
(d) an annual effective compound discount rate of 8% is applied
(e) an annual simple discount rate of 8% is applied
(f) interest is earned during the first year at a nominal annual rate of 8% con-
vertible quarterly, and during the second year at a nominal annual rate of
discount of 8% convertible quarterly.

Solution:

(a) A nominal annual interest rate of 8% convertible quarterly is equivalent to a


3-month rate of 84 % = 2%. The accumulated value of 100 after 21 = 8 quarter
4
years is
100(1.02)8 = 117.17 .
(b) We are given that
d( 6 ) = 8% .
1

2 1
The accumulated value of 100 after 6
= 3
6-year periods
2
100 (1 (6 0.08)) 6 = 124.36 .

(c) Interest compounded instantaneously at the rate of 8% yields an accumulation


factor of m
0.08
lim 1 + = e0.08 .
m m
After 2 years 100 accumulates to

100e20.08 = 117.35 .

(d) At an annual effective compound discount rate of 8%, 100 accumulates after
2 years to
100(1 0.08)2 = 118.15 .
(e) At an annual simple discount rate of 8%, 100 accumulates after 2 years to
100
= 119.05 .
1 2(0.08)
Information for Students in MATH 329 2005 01 19

(f) The accumulated value of 100 after the first year will be 100(1.02)4 . During
the second year this amount will accumulate to a final amount of
4 4
4 0.08 1.02
100(1.02) 1 = 100 = 117.36 .
4 0.98
3. Today is New Years Day. In return for payments of 1500 at the end of January,
February, and March, and of 3000 at the end of May, July, and September, an
investor agrees to pay now the total value of the 6 payments, and to either make
or receive an additional payment at the end of December. Find the amount of
that additional payment if it is known that the nominal annual interest rate is 6%,
compounded monthly. (First set up an equation of value.)
Solution: Let us denote the final payment by the investor by X. The effective
6
monthly interest rate is 12 % = 12 %. We will take as the comparison date the end
of December (although any date chosen would yield the same information). The
value on 31 December of the payments paid to the investor is

1500 (1.005)11 + (1.005)10 + (1.005)9

+3000 (1.005)7 + (1.005)5 + (1.005)3 = 13, 957.74 .
The value on 31 December (just after she has paid the last payment) of the pay-
ments paid out by the investor is
(3(1500) + 3(3000))(1.005)12 + X = 14332.65 + X .
Equating the two values yields X = 374.91. Thus the investor is entitled to
receive a final payment of 374.91 at the end of December. (The fact that the final
payment would be received by the investor, rather than paid by her could have
been reasoned without calculation; we didnt need the mathematical calculation to
tell us that from the sign of the answer.)
4. Analogously to the rule of 72, you are asked to develop a rule of n to approximate
how long it takes for money to increase to 1 21 times its initial value. (That is, to
determine a 2-digit integer N = 10n1 + n0 (where n0 , n1 are decimal digits), for
which (0.01) Ni is a good approximation to the number of years required.) Your
approximation should be best around 8%.
Solution: We have to approximate a solution to the equation
(1 + i)n = 1.5,
which is equivalent to
ln 1.5 i
n= .
i ln(1 + i)
We find that
Information for Students in MATH 329 2005 01 20

i
i (ln 1.5)
ln(1 + i)
7.0% 0.419
8.0% 0.421
9.0% 0.423

so we take N = 42.

5. (a) Find the smallest nominal rate of interest convertible monthly at which the
accumulated value of 15,000 at the end of 3 years is at least 24,000.
(b) Find the smallest nominal rate of discount convertible semi-annually at which
the accumulated value of 15,000 at the end of 3 years is at least 24,000.
Solution:

(a) Let i denote the nominal interest rate sought. Then


36
i
15000 1 + 24000
12
36
i
1+ 1.6000
12
i 1
1+ (1.6) 36 = 1.013141254
12
so i = 15.7695048%. The smallest interest rate is 15.8%.
(b) This could be considered a trick question; but the fact is that the word largest
was not intended. I solve it both as written, and with the intended wording.
Let d denote the nominal discount rate.
As written, largest Then
6
d
15000 1 24000
2
6
d 5
1
2 8
r
d 6 5
1 = .9246555971
2 8
implying that d 15.068880%. Thus any discount rate d 15.1% has
the desired property. There is no largest rate, since, as the rate d(2)
approaches 100% from below, the accumulated value approaches . We
Information for Students in MATH 329 2005 01 21

havent attached a meaning to discount rates above 100%. (As for a


discount rate of 100% that does make sense, but would not permit the
accumulation of funds: if we discount a sum X of money back from time
1 to time 0 at an effective discount rate of 100%, we obtain a present
value of 0.)
Corrected to smallest
6
d
15000 1 24000
2
6
d 5
1
2 8
r
d 6 5
1 = .9246555971
2 8
implying that d 15.068880%. Thus the smallest discount rate is 15.1%.

6. Let x be a positive real number, 0 x < 1. Prove that


1
1+x
1x
for all such x. Conclude that the accumulated value of 100 after 10 years at an
interest rate of x% is always less than or equal to the accumulated value of 100
after 10 years at a discount rate of x%, with equality holding only when x = 0.
Solution: Since x2 is a square, it cannot be negative, and can be 0 only when x = 0;
hence 1 x2 1, with equality holding precisely when x = 0. But this inequality
is equivalent to
1 (1 x)(1 + x)
or to
1
1+x
1x
since the inequality is preserved when we divide both sides by the positive number
1 x; again, equality holds when x = 0.

x 10
The accumulated value of 100 at the interest rate of x% is 100 1 + 100 . We
x
apply the preceding argument 10 times, after replacing x by 100 : this cannot exceed

x 10
100 1 100 , which is the accumulated value of 100 at the discount rate of x%.
x
Equality holds when 100 = 0, i.e., when x = 0.
Information for Students in MATH 329 2005 01 22

6 Third Problem Assignment


Mounted on the Web on Sunday, February 13th, 2005
Distributed in hard copy on Wednesday, February 16th, 2005
Full solutions were to be submitted by March 7th, 2005.

These problems are to be solved with full solutions, modelled either on the solutions to
problems in the textbook, or in the notes on the Web for this or previous years. The
essence is that the reader should be able to reconstruct every step of the proof from what
you have written: getting the right answer is never enough. You are not being graded
for elegance, but simply for the proof being logical, without serious gaps.
1. McGill plans to create a scholarship fund that will eternally pay 50 students a
monthly stipend of $200 at the beginning of months September through April,
plus an amount of $300 on the following May 1st.
(a) If interest is assumed to be at a nominal annual rate of 6% per annum, com-
pounded monthly, determine the amount that is needed in this fund on Sep-
tember 1st just before the fund begins making payments.
(b) Determine the amount that will be in the fund just after the December schol-
arship payments in the first year, and on September 1st of the following year,
just before the September payments.
(c) Suppose that at the beginning of September, 8 years after the fund is estab-
lished, it is decided to increase the capital in the fund because the interest
rate has changed to 4% per annum compounded monthly. Determine how
much additional capital needs to be added to the fund.
(d) Suppose that 2 years after the interest rate is changed to 4%, it changes again,
this time to 8%. This time it is decided to leave the capital unchanged, but
to increase the payments to students by a lump sum of $M to each student,
payable on December 1st, together with the regular December payment under
the scholarship. Determine the amount of that lump sum payment.
2. A loan of 10,000 is to be repaid by regular, half-yearly payments of 1,000, the first
to be made at the end of the 3rd year. The loan will be paid off by a final payment
which must be at least 1,000.
(a) If the interest rate is to be 8% per annum, compounded semi-annually, deter-
mine the amount of the final payment, and when it is made.
(b) Suppose that the final payment is now not more than 1,000. and the inter-
est rate remains 8% per annum compounded semi-annually. Determine the
amount of the final payment, and when it is made.
Information for Students in MATH 329 2005 01 23

(c) Suppose that, after signing his original commitment, the borrower decides
that he would like to make the first payment 6 months from the date of
borrowing, and that all the payments under this loan should be exactly equal.
If the interest rate is 6% per annum, compounded semi-annually, determine
that payment level that would be closest to 1000 per half-year, and determine
exactly when the loan will be paid off.

3. (An acknowledgement of source of this problem will be contained in the solutions,


when published.) Katherine, 25 years old, deposits 10,000 at the beginning of
every 4-year period into an RSSP account. The account pays compound interest
annually, at the effective annual rate i. The accumulated amount in the account at
the end of 40 years is X, which is 6 times the accumulated amount in the account
at the end of 20 years. Determine X.

4. For each of the following sequences of payments as of the time stated,

express the value using standard symbols, as simple as possible;


give a formula for the value (in terms of i, v, d, etc.);
evaluate using a calculator or computer not with tables.

(a) a perpetuity-immediate paying 1 per year at effective annual interest rate


4.25%, evaluated 1 year before the first payment;
(b) a perpetuity paying 1 per year at effective annual interest rate 4.25%, evalu-
ated 2 years before the first payment;
(c) a perpetuity-due paying 1 per half-year at nominal interest rate 5%, com-
pounded semi-annually, evaluated just before the first payment;
(d) a 20-year increasing annuity paying 1 the first year, 2 the second year, 3 the
3rd year, etc., at an interest rate of 7% per year effective, evaluated just after
the last payment;
(e) a 20-year increasing annuity paying 1 the first year, 2 the second year, 3 the
3rd year, etc., at an interest rate of 7% per year effective, evaluated at the
time of the 2nd payment;
(f) a 10-year decreasing annuity paying 1 less each quarter-year, evaluated just
after the last payment, where the nominal interest rate is a 8% annual, com-
pounded quarterly;

5. X and Y have sold their home for 400,000, and wish to purchase an annuity-
immediate so that they can spread the proceeds over the next 10 years. The first
payment will be one month from the date of purchase of the annuity.
Information for Students in MATH 329 2005 01 24

(a) Determine the level monthly payments they will receive if the effective annual
interest rate is 6%.
(b) X and Y live frugally, and dont think they need as much income now as
they will as time passes. Accordingly they plan to receive monthly payments
which will gradually increase. If the first payment is 3,000, and the payments
increase each month by the same dollar amount K, determine K. The effective
annual interest rate remains 6%. What is the final monthly payment?
(c) Suppose that the payments remain constant in any year. The first years
monthly payments are all 3000, the next years 3000+L, the next years 3000+
2L, 3000 + 3L, . . . , 3000 + 9L. Determine L if the nominal interest rate is
6%, compounded monthly.
Information for Students in MATH 329 2005 01 25

7 Solutions, Second Problem Assignment


Mounted on the Web on March 3rd, 2005
Full solutions were to be submitted by February 14th, 2005.
(Subject to Correction)

These problems were to be solved with full solutions, modelled either on the solutions
to problems in the textbook, or in the notes on the Web for this or previous years. The
essence is that the reader should be able to reconstruct every step of the proof from what
you have written: getting the right answer is never enough. You are not being graded
for elegance, but simply for the proof being logical, without serious gaps.
1. (a) At a certain rate of compound interest an investment of 1000 will grow to
1500 at the end of 12 years. Determine its value at the end of 5 years.
(b) At a certain rate of compound interest an investment of 1000 will grow to
1500 at the end of 12 years. Determine precisely when its value is exactly
1200.
(c) A debt of 7000 is due at the end of 5 years. If 2000 is paid at the end of
1 year, what single payment should be made at the end of the 2nd year to
liquidate the debt, assuming interest at the rate of 6.5% per year, compounded
quarterly.
(d) George agrees to buy his brothers car for 7000. He makes a down payment of
4000, and agrees to pay two equal payments, one at the end of 6 months, and
the other at the end of a year. If interest is being charged at 5% per annum
effective, how large should each of the equal payments be?
(e) A bill for 1500 is purchased for 1000 15 months before it is due. Determine
the nominal rate of discount, compounded monthly, which the purchaser is
paying.
(f) Bills for 1500 are regularly purchased for 1000 15 months before they are due.
The purchaser knows that, among 10 such bills, he will be unable to collect
anything on one of them, will have to pay his lawyers 500 each to effect
collection on 2 others, and will collect the others without any impediment.
Lumping all of these together, what is the effective annual interest rate earned
by the purchaser on his investment, if it is assumed that the lawyers accounts
are due at the same time as the bills?
Solution:
(a) Let I be the effective annual rate of compound interest. Then
1000(1 + i)12 = 1500 (1)
Information for Students in MATH 329 2005 01 26

implies that
125
5 1500
(1 + i) = = 1.184053587
1000
so the investment is worth 1184.05 after 5 years.
(b) Let t be the time in years when the investment is worth exactly 1200. Then

1000(1 + i)t = 1200 ,

where i is the effective annual rate of compound interest. By equation (1),

ln 1.5
ln(1 + i) = .
12
Hence
ln 1.2 12 ln 1.2
t= = = 5.395923442 :
ln(1 + i) ln 1.5
the investment has the desired value after 5.40 years.
(c) Let x denote the payment that must be made at the end of the 2nd year. The
equation of value at that time is
4 12
0.065 0.065
x + 2000 1 + = 7000 1 +
4 4

implying that
4 12
0.065 0.065
x = 2000 1 + + 7000 1 + = 3635.67
4 4

should be paid at the end of the second year to liquidate the debt.
(d) Let x denote the amount of each of the payments that should be made at the
ends of 6 months and 1 year. Then the equation of value at time 0 is

12 1
x (1.05) + (1.05) = 7000 4000 = 3000 ,

implying that
3000
x= 1 = 1555.79 .
(1.05) 2 + (1.05)1
(e) Let d be the effective monthly rate of discount. Then

1000 = (1 d)15 1500 ,


Information for Students in MATH 329 2005 01 27

implying that
151
1000
d=1 = 2.66689392% .
1500
This is the effective monthly rate. The nominal annual rate of discount,
compounded monthly, is, therefore 12d = 32%.
(f) Of 10 bills which mature in 15 months, the total return that will enure to the
purchaser at that time is

7(1500) + 2(1500 500) + 0(1500) = 12500 .

If we denote the effective annual interest rate by i, then the discounted value
15
of 12500 at time t = 0 will be 12500(1 + i) 12 . The equation of value at time
t = 0 is, therefore,
15
12500(1 + i) 12 = 10(1000) = 10000 ,
5
implying that (1 + i) 4 = 1.25, so i = 1.250.8 1 = 19.5440625%.
2. A government offers savings bonds in multiples of $1000, which mature in 10 years
at $2000, but pay no interest until they are redeemed.
(a) Assuming interest compounded semi-annually, what nominal annual rate of
interest does the bond holder earn?
(b) Suppose that the bonds earn interest at the nominal rate of i 0.01 for
the first 5 years, compounded semi-annually; and that they earn interest at
the nominal rate of i + 0.01 compounded semi-annually for the last 5 years.
Determine i.
(c) The government has contracted with a bank to market the bonds, at a cost
of $40 per $1000 bond. What interest rate, compounded semi-annually, is the
government paying for the net proceeds it receives for each $1000 bond?
Solution:
(a) Let i be the nominal interest rate, compounded semi-annually, earned by the
purchaser. The equation of value at time 10 is then
20
i
1000 1 + = 2000
2
implying that 1
i = 2 2 20 1 = 7.0529848%.
Information for Students in MATH 329 2005 01 28

(b) The equation of value is now


10 10
i1 i+1
1000 1 + 1+ = 2000
2 2
implying that

i 0.01 i + 0.01 1
1+ 1+ = 2 10
2 2
2
i (0.01)2 1
1+ = 2 10
2 4
1
i 1 (0.01)2 2
1+ = 2 10 +
2 4
1
1 (0.01)2 2
i = 2 2 10 + 2
4
= 7.0553996%

(c) Let i be the nominal interest rate, compounded semi-annually. From the
governments point of view the equation of value at maturity is
20
i
960 1 + = 2000
2
implying that !
201
2000
i=2 1 = 7.4760322%
960

3. The cash price of a new automobile is 18,000 plus 15.025% tax. The purchaser
is prepared to finance the car and taxes at 18% convertible semi-monthly, and to
make payments of 230 at the end of every half-month for 3.5 years, with the first
payment to be made one half-month after delivery. The dealer requires a down
payment upon delivery, both to make up the gap in the financing, and from which
to pay his immediate costs (commission to the salesperson, preparation costs, sales
taxes).
(a) Determine the value of this down payment.
(b) Determine the value of the down payment if the purchaser decides, instead of
semi-monthly payments, to make a payment of 460 at the end of every month,
first payment a month after delivery, last payment to be made 3.5 years after
delivery. The interest rate and compounding period do not change.
Information for Students in MATH 329 2005 01 29

(c) Determine the value of the down payment if the original conditions are changed
so that the first payment of 230 is made 1.5 months after delivery, but the
same number of payments of 230 are made as originally planned. The interest
rate and compounding period do not change.
Solution:
(a) The present value of the future payments is

1 (1.0075)84
230 a(3.5)(24) 0.18 = 230
24 0.0075
= 14295.41185
The excess of the purchase price and taxes over the present value of the future
payments is, therefore,
(1.0015025)(18000) 14295.41185 = 6409.08815 .
Thus the down payment will be 6,409.09.
(b) We can still interpret the payments as constituting an annuity-immediate.
But the interval of payments has paid, while the compounding interval has
not. So we need to determine the interest rate that corresponds to one month.
This is
(1.0075)2 1 = 0.01505625 .
The present value of the future payments is
1 (1.01505625)42
460a(3.5)(12) 0.01505625 = 460
0.01505625
= 14242.00433
The excess of the purchase price and taxes over the present value of the future
payments is, therefore,
(1.0015025)(18000) 14242.00433 = 6462.49567 .
Thus the down payment will be 6,462.50.
(c) Since the repayment schedule is delayed by 2 payments, which are added at
the end, the present value of the future payments is

1 (1.0075)86 1 (1.0075)2
230a(3.5)(24)+2 0.18 230a2 0.18 = 230
24 24 0.0075 0.0075
= 230(63.20976257 1.977722907)
= 14083.36912
Information for Students in MATH 329 2005 01 30

The excess of the purchase price and taxes over the present value of the future
payments is now

(1.0015025)(18000) 14083.36912 = 6621.13088 .

Thus the down payment will now be 6,621.13.


4. An employee aged exactly 40 decides to accumulate a fund for retirement at age 65
by depositing 200 at the beginning of each month for 25 years. When she reaches
age 65, she plans to withdraw a fixed amount at the beginning of each year for 15
years. Assuming that all payments are made, determine the amount of the annual
payments that she will be able to withdraw:
(a) if the interest rate is always taken to be (a nominal annual rate of) 6% per
annum, compounded monthly.
(b) if the interest rate is taken to be 6% per annum compounded monthly during
the time when the fund is being built up, and 8% per annum effective when
she reaches age 65.
(c) if the interest rate is taken to be 4% per annum compounded monthly for the
coming 5 years, then 6% per annum compounded monthly for the next 20
years while the fund is being built up, then 8% per annum effective when the
fund is paying out annual payments.
Solution:
(a) The effective interest rate per month is 6%
12
= 12 %. The number of contributions
will be 25 12 = 300. The value of the fund at maturity, when the employee
reaches age 65, will be
!
300
(1.005) 1
200s2512 1 % = 200 1
2 (0.005) 1.005
= 139291.7864

The withdrawals from the fund will be once a year. If we wish to interpret
them as payments under an annuity-due, we need to determine the rate of
interest that corresponds to the period between the payments, i.e. to a 1-year
period; that is, we need to determine i, knowing i(12) . This gives us an annual
rate of (1.005)12 1 = 6.1677812%. If the annual withdrawal from the fund,
at the beginning of each year, be denoted by X, then the equation of value
just before the first withdrawal is

X a15 6.1677812% = 139291.7864


Information for Students in MATH 329 2005 01 31

which we can solve to determine


139291.7864
X = 1(1.061677812)15
0.061677812( 1.061677812
1
)
= 15393.80469
so the withdrawals will be 15393.80, at the beginning of each of 15 years.
(b) Proceeding as in the previous case, we replace the interest rate of 6.1677812%
by 8%. Now we solve
X a15 8% = 139291.7864
to determine
139291.7864
X = 1(1.08)15
0.08( 1.08
1
)
= 15067.95928
so the withdrawals will be 15,067.96, at the beginning of each of 15 years.
(c) We need to recompute the value at the time of maturity of the fund of the
contributions. The last 20 years of payments accumulate at maturity to
!
(1.005)240 1
200s2012 1 % = 200 1
2 (0.005) 1.005
= 92870.21994
The first five years of payments accumulate to
200 s60 4
%
12

at the end of 5 years; at maturity these payments have accumulated to


!
(1.003333333) 60
1
(1.005)2012 200s60 4 % = (1.005)2012 200 1

12 (0.003333333) 1.00333333333
= 44, 038.93886
The sum of the two components of the fund is
44, 038.93886 + 92, 870.21994 = 136, 909.1588
so the annual withdrawal will be
136909.1588 136909.1588(0.08)(1.08)1
=
a15 8% 1 (1.08)15
= 14810.21733
or 14,810.22.
Information for Students in MATH 329 2005 01 32

5. In his will, a benefactor of McGill contributes a large sum of money to establish


a fund, whose proceeds are to be used to provide annual bursaries of 5000 to
6 actuarial students. The principal of the fund is to remain constant, and the
bursaries are funded by the interest earned, forever.
(a) If the effective annual interest rate is assumed to be 6% forever, determine the
lump sum that the benefactors estate needs to contribute to McGill a year
before the first payment of bursaries.
(b) If the effective annual interest rate is assumed to be 6% forever, determine
the lump sum that the benefactors estate needs to contribute to McGill if
the first bursaries are to be issued immediately.
(c) Suppose that each student receives not a bursary of 5000, but an annual
annuity-due of 1250 for 4 years. Under these changed conditions, determine
the lump sum payment needed now if the first bursaries are to be awarded
immediately or 1 year hence respectively. (The number of awards will remain
the same each year 6 students are awarded a 4-year sequence of bursary
payments of 1250, the first payment to be made immediately.)
Solution:
(a) The lump sum payment must equal the present value of the perpetuity-
immediate of annual payments of 30,000, i.e.
30000
6(5000)a 6% = = 500, 000.
0.06
(b) The lump sum payment must equal the present value of the perpetuity-due
of annual payments of 30,000, i.e.
30000
6(5000)a 6% = 1 = 530, 000.
0.06 1.06
Alternatively, this may be viewed as the cost of funding the bursaries of the
preceding question, to which must be added the cost of the bursaries that
must be paid out immediately in the amount of 6 5, 000.
(c) We must replace each payment of 5000 by
1250 a5 = (1250)(1.06) 1 1.064 0.06 = 4591.264936.
i. The lump sum payment when the bursaries are awarded beginning one
year hence is
4591.264936
500, 000 = 459126.4936
5000
or 459,126.49.
Information for Students in MATH 329 2005 01 33

ii. The lump sum payment when the bursaries are awarded beginning im-
mediately is
4591.264936
530, 000 = 486674.0832
5000
or 486,674.08.

6. Let m and n be positive integers. Consider an annuity-immediate which pays 1 at


the end of every period for mn periods. Explain in words why each of the following
formul represents the value of this annuity m + 1 years before the first payment
is made. (If you have doubts about the truth of this claim, you may wish to verify
algebraically that the claim is correct before you attempt to explain it in words.)

(a) v m amn
(b) am(n+1) am
(c) v m am(n+1) v m(n+2) sm

Solution:

(a) One period before the first payment, the value of an annuity of 1 per period
payable for mn periods is amn . At a time m periods earlier the value must be
1
discounted by a factor of 1+i per year, or v m for m years.
(b) am(n+1) is the value of an annuity-immediate of 1 payable for mn + n periods.
At a time m periods ago, this represents the value then of the annuity under
present consideration, augmented by an annuity of m payments of 1, the last
to be made now. If we subtract the value of these m payments as of m periods
ago, we are left with the value of the mn payments under consideration.
(c) Consider an annuity that consists of all the payments under present consider-
ation, extended by an additional m payments. Such a scheme is worth am(n+1)
now, and v m am(n+1) m periods ago. The m payments added at the end are
worth sm at the time of the last of them, which is mn + m periods from
now. We may discount that back to a time m periods ago by multiplying by
v mn+m+m = v m(n+2) .
Information for Students in MATH 329 2005 01 34

8 Class Tests
8.1 Class Test, Version 1
McGILL UNIVERSITY, FACULTY OF SCIENCE
CLASS TEST in MATH 329, THEORY OF INTEREST
EXAMINER: Professor W. G. Brown DATE: Wednesday, 9 March, 2005.
ASSOCIATE EXAMINER: Professor N. Sancho TIME: 45 minutes, 14:3515:20
FAMILY NAME:
GIVEN NAMES:
STUDENT NUMBER:
Instructions
The time available for writing this test is about 45 minutes.
This test booklet consists of this cover, Pages 35 through 38 containing questions together
worth 66 marks; and Page 39, which is blank.
Show all your work. All solutions are to be written in the space provided on the page where
the question is printed. When that space is exhausted, you may write on the facing page, on
the blank page, or on the back cover of the booklet, but you must indicate any continuation
clearly on the page where the question is printed! (Please inform the instructor if you find
that your booklet is defective.)
All your writing even rough work must be handed in.
Calculators. While you are permitted to use a calculator to perform arithmetic and/or ex-
ponential calculations, you must not use the calculator to calculate such actuarial functions
as ani , sni , etc. without first stating a formula for the value of the function in terms of
exponentials and/or polynomials involving n and the interest rate. You must not use your
calculator in any programmed calculations. If your calculator has memories, you are expected
to have cleared them before the test.
In your solutions to problems on this test you are expected to show all your work. You are
expected to simplify algebraic and numerical answers as much as you can.
Your neighbours may be writing a version of this test which is different from yours.
PLEASE DO NOT WRITE INSIDE THIS BOX
1(a) 1(b) 1(c) 2(a) 2(b) 2(c) 3(a) 3(b) 4 Total

/4 /4 /4 /6 /6 /6 /6 /12 /18 /66


Information for Students in MATH 329 2005 01 35

1. Showing your work in detail, determine each of the following; the rates you de-
termine should be accurate to 4 decimal places, or as a percentage accurate to 2
decimal places:

(a) [4 MARKS] the nominal annual interest rate, compounded monthly, corre-
sponding to an effective annual interest rate of i = 6%
(b) [4 MARKS] the effective annual interest rate corresponding to a nominal dis-
count rate, compounded quarterly, of d = 3.6%
(c) [4 MARKS] the effective monthly interest rate corresponding to a force of
interest of = 0.06.
Information for Students in MATH 329 2005 01 36

2. For each of the following sequences of payments, determine, as of the given time,
and for the given interest or discount rate, the value, showing all of your work.
Before determining the numeric value you are expected to express the value using
standard symbols.

(a) [6 MARKS] the value now of 20 payments of 1 at the end of every year starting
one year from now, at an interest rate of 4%
(b) [6 MARKS] the value one year ago of 10 payments of 1 at the end of every
half-year, the first to be paid 5 years from now, at a nominal interest rate of
8% compounded semi-annually
(c) [6 MARKS] 300 payments of 1 at the end of every month, as of the date
of the 100th payment, which has just been made; the interest rate is 12%
compounded monthly
Information for Students in MATH 329 2005 01 37

3. (a) [6 MARKS] An annuity at interest rate i consists of payments of 10 now, 12


at the end of one year, 14 at the end of two years, increasing by a constant
amount until the last payment in the amount of 40, is to be evaluated as of 1
year ago. Express its value in terms of symbols (Ia)n , (Is)n , an , sn , i, v, but
do not evaluate.
(b) [12 MARKS] The accumulated value just after the last payment under a 15-
year annuity of 1000 per year, paying interest at the rate of 8% per annum
effective, is to be used to purchase a perpetuity, first payment to be made
2 years after the last payment under the annuity. Showing all your work,
determine the size of the payments under the perpetuity, assuming that the
interest rate from the time of the last payment under the 15-year annuity is
5%.
Information for Students in MATH 329 2005 01 38

4. [18 MARKS] A loan of 6000 is to be repaid by annual payments of 350 to commence


at the end of the 1st year, and to continue thereafter for as long as necessary. Find
the time and amount of the final payment if the final payment is to be larger than
the regular payments. Assume i = 4%.
Information for Students in MATH 329 2005 01 39

continuation page for problem number

You must refer to this continuation page on the page where the problem is printed!
Information for Students in MATH 329 2005 01 40

8.2 Class Test, Version 2


McGILL UNIVERSITY, FACULTY OF SCIENCE
CLASS TEST in MATH 329, THEORY OF INTEREST

EXAMINER: Professor W. G. Brown DATE: Wednesday, 9 March, 2004.


ASSOCIATE EXAMINER: Professor N. Sancho TIME: 45 minutes, 14:3515:20
FAMILY NAME:
GIVEN NAMES:
STUDENT NUMBER:

Instructions

The time available for writing this test is about 45 minutes.

This test booklet consists of this cover, Pages 41 through 44 containing questions together
worth 66 marks; and Page 45, which is blank.

Show all your work. All solutions are to be written in the space provided on the page where
the question is printed. When that space is exhausted, you may write on the facing page, on
the blank page, or on the back cover of the booklet, but you must indicate any continuation
clearly on the page where the question is printed! (Please inform the instructor if you find
that your booklet is defective.)

All your writing even rough work must be handed in.

Calculators. While you are permitted to use a calculator to perform arithmetic and/or ex-
ponential calculations, you must not use the calculator to calculate such actuarial functions
as ani , sni , etc. without first stating a formula for the value of the function in terms of
exponentials and/or polynomials involving n and the interest rate. You must not use your
calculator in any programmed calculations. If your calculator has memories, you are expected
to have cleared them before the test.

In your solutions to problems on this test you are expected to show all your work. You are
expected to simplify algebraic and numerical answers as much as you can.

Your neighbours may be writing a version of this test which is different from yours.

PLEASE DO NOT WRITE INSIDE THIS BOX


1 2(a) 2(b) 2(c) 3(a) 3(b) 3(c) 4(a) 4(b) Total

/18 /4 /4 /4 /6 /6 /6 /6 /12 /66


Information for Students in MATH 329 2005 01 41

1. [18 MARKS] A loan of 7000 is to be repaid by annual payments of 450 to commence


immediately, and to continue at the beginning of each year for as long as necessary.
Find the time and amount of the final payment if the final payment is to be no
larger than the regular payments. Assume i = 6%.
Information for Students in MATH 329 2005 01 42

2. Showing your work in detail, determine each of the following; the rates you de-
termine should be accurate to 4 decimal places, or as a percentage accurate to 2
decimal places:

(a) [4 MARKS] the effective annual discount rate corresponding to a nominal


interest rate, compounded quarterly, of i = 2.4%
(b) [4 MARKS] the nominal annual interest rate, compounded quarterly, equiva-
lent to an effective semi-annual discount rate of d = 4%
(c) [4 MARKS] the effective semi-annual interest rate corresponding to a force of
interest of = 0.04.
Information for Students in MATH 329 2005 01 43

3. For each of the following sequences of payments, determine, as of the given time,
and for the given interest or discount rate, the value, showing all of your work.
Before determining the numeric value you are expected to express the value using
standard symbols.

(a) [6 MARKS] the value of 25 payments of 1 at the end of every year, the last
one having just been made, at an interest rate of 6%
(b) [6 MARKS] the value 4 months from now of 30 payments of 1 at the end of
every 4 months, the first to be paid 2 years from now, at a nominal interest
rate of 9% compounded 3 times a year
(c) [6 MARKS] 120 payments of 1 at the end of every 2 months, as of the date
of the 90th payment, which has just been made; the interest rate is 6% com-
pounded every 2 months.
Information for Students in MATH 329 2005 01 44

4. (a) [6 MARKS] An annuity at interest rate i consists of payments of 100 now, 95


at the end of 1 year, 90 at the end of 2 years, decreases by a constant amount
until the last payment in the amount of 25, is to be evaluated as of the time
of the payment of 95. Express its value then in terms of symbols (Ia)n , (Is)n ,
an , sn , i, v, but do not evaluate.
(b) [12 MARKS] The accumulated value just after the last payment under a 12-
year annuity of 1000 per year, paying interest at the rate of 6% per annum
effective, is to be used to purchase a 10-year annuity at an interest rate of 7%,
first payment to be made 4 years after the last payment under the annuity.
Showing all your work, determine the size of the payments under the 10-year
annuity. Assume that the 7% rate is in effect from the time of the last payment
under the 12-year annuity.
Information for Students in MATH 329 2005 01 45

continuation page for problem number

You must refer to this continuation page on the page where the problem is printed!
Information for Students in MATH 329 2005 01 46

8.3 Class Test, Version 3


McGILL UNIVERSITY, FACULTY OF SCIENCE
CLASS TEST in MATH 329, THEORY OF INTEREST

EXAMINER: Professor W. G. Brown DATE: Wednesday, 9 March, 2005.


ASSOCIATE EXAMINER: Professor N. Sancho TIME: 45 minutes, 14:3515:20
FAMILY NAME:
GIVEN NAMES:
STUDENT NUMBER:

Instructions

The time available for writing this test is about 45 minutes.

This test booklet consists of this cover, Pages 47 through 50 containing questions together
worth 66 marks; and Page 51, which is blank.

Show all your work. All solutions are to be written in the space provided on the page where
the question is printed. When that space is exhausted, you may write on the facing page, on
the blank page, or on the back cover of the booklet, but you must indicate any continuation
clearly on the page where the question is printed! (Please inform the instructor if you find
that your booklet is defective.)

All your writing even rough work must be handed in.

Calculators. While you are permitted to use a calculator to perform arithmetic and/or ex-
ponential calculations, you must not use the calculator to calculate such actuarial functions
as ani , sni , etc. without first stating a formula for the value of the function in terms of
exponentials and/or polynomials involving n and the interest rate. You must not use your
calculator in any programmed calculations. If your calculator has memories, you are expected
to have cleared them before the test.

In your solutions to problems on this test you are expected to show all your work. You are
expected to simplify algebraic and numerical answers as much as you can.

Your neighbours may be writing a version of this test which is different from yours.

PLEASE DO NOT WRITE INSIDE THIS BOX


1(a) 1(b) 2 3(a) 3(b) 3(c) 4(a) 4(b) 4(c) Total

/6 /12 /18 /4 /4 /4 /6 /6 /6 /66


Information for Students in MATH 329 2005 01 47

1. (a) [6 MARKS] An annuity at interest rate i consists of payments of 10 now, 13


at the end of one year, 16 at the end of twq years, increasing by a constant
amount until the last payment in the amount of 40, and is to be evaluated as
of two years ago. Express its value then in terms of symbols (Ia)n , (Is)n , an ,
sn , i, v, but do not evaluate.
(b) [12 MARKS] The accumulated value just after the last payment under a 15-
year annuity of 1000 per year, paying interest at the rate of 9% per annum
effective, is to be used to purchase a perpetuity at an interest rate of 8%,
first payment to be made at the same time as the last payment under the
annuity. Showing all your work, determine the size of the payments under the
perpetuity.
Information for Students in MATH 329 2005 01 48

2. [18 MARKS] A loan of 8000 is to be repaid by annual payments of 500 to commence


at the end of the 1st year, and to continue thereafter for as long as necessary. Find
the time and amount of the final payment if the final payment is to be smaller than
the regular payments. Assume i = 5%.
Information for Students in MATH 329 2005 01 49

3. Showing your work in detail, determine each of the following; the rates you de-
termine should be accurate to 4 decimal places, or as a percentage accurate to 2
decimal places:

(a) [4 MARKS] the effective annual interest rate, corresponding to an nominal


annual interest rate of i = 6% compounded every 4 months.
(b) [4 MARKS] the effective monthly interest rate corresponding to a force of
interest of = 0.12.
(c) [4 MARKS] the effective annual interest rate corresponding to d(4) = 3.2%
Information for Students in MATH 329 2005 01 50

4. For each of the following sequences of payments, determine, as of the given time,
and for the given interest or discount rate, the value, showing all of your work.
Before determining the numeric value you are expected to express the value using
standard symbols.

(a) [6 MARKS] the value one half-year ago of 12 payments of 1 at the end of
every half-year, the first to be paid 4 years from now, at a nominal interest
rate of 6% compounded semi-annually
(b) [6 MARKS] 180 payments of 1 at the end of every month, as of the date
of the 100th payment, which has just been made; the interest rate is 18%
compounded monthly
(c) [6 MARKS] the value now of 16 payments of 1 at the end of every year starting
1 year from now, at an interest rate of 4%
Information for Students in MATH 329 2005 01 51

continuation page for problem number

You must refer to this continuation page on the page where the problem is printed!
Information for Students in MATH 329 2005 01 52

8.4 Class Test, Version 4


McGILL UNIVERSITY, FACULTY OF SCIENCE
CLASS TEST in MATH 329, THEORY OF INTEREST

EXAMINER: Professor W. G. Brown DATE: Wednesday, 9 March, 2005.


ASSOCIATE EXAMINER: Professor N. Sancho TIME: 45 minutes, 14:3515:20
FAMILY NAME:
GIVEN NAMES:
STUDENT NUMBER:

Instructions

The time available for writing this test is about 45 minutes.

This test booklet consists of this cover, Pages 53 through 56 containing questions together
worth 66 marks; and Page 57, which is blank.

Show all your work. All solutions are to be written in the space provided on the page where
the question is printed. When that space is exhausted, you may write on the facing page, on
the blank page, or on the back cover of the booklet, but you must indicate any continuation
clearly on the page where the question is printed! (Please inform the instructor if you find
that your booklet is defective.)

All your writing even rough work must be handed in.

Calculators. While you are permitted to use a calculator to perform arithmetic and/or ex-
ponential calculations, you must not use the calculator to calculate such actuarial functions
as ani , sni , etc. without first stating a formula for the value of the function in terms of
exponentials and/or polynomials involving n and the interest rate. You must not use your
calculator in any programmed calculations. If your calculator has memories, you are expected
to have cleared them before the test.

In your solutions to problems on this test you are expected to show all your work. You are
expected to simplify algebraic and numerical answers as much as you can.

Your neighbours may be writing a version of this test which is different from yours.

PLEASE DO NOT WRITE INSIDE THIS BOX


1(a) 1(b) 1(c) 2(a) 2(b) 3 4(a) 4(b) 4(c) Total

/6 /6 /6 /6 /12 /18 /4 /4 /4 /66


Information for Students in MATH 329 2005 01 53

1. For each of the following sequences of payments, determine, as of the given time,
and for the given interest or discount rate, the value, showing all of your work.
Before determining the numeric value you are expected to express the value using
standard symbols.

(a) [6 MARKS] 90 payments of 1 at the end of every 2 months, as of the date of the
18th payment, which has just been made; the interest rate is 9% compounded
every 2 months.
(b) [6 MARKS] the value of 20 payments of 1 at the end of every year, the last
one having just been made, at an interest rate of 5%
(c) [6 MARKS] the value 4 months from now of 45 payments of 1 at the end of
every 4 months, the first to be paid 3 years from now, at a nominal interest
rate of 6% compounded 3 times a year
Information for Students in MATH 329 2005 01 54

2. (a) [6 MARKS] An annuity at interest rate i consists of payments of 118 now,


112 at the end of 1 year, 106 at the end of 2 years, decreasing until the last
payment in the amount of 34, the totality to be evaluated as of the time of
the payment of 112. Express its value then in terms of symbols (Ia)n , (Is)n ,
an , sn , i, v, but do not evaluate.
(b) [12 MARKS] The accumulated value just after the last payment under a 14-
year annuity of 1000 per year, paying interest at the rate of 10% per annum
effective, is to be used to purchase a 12-year annuity-immediate at an interest
rate of 7%, first payment to be made 1 year after the last payment under the
14-year annuity. Showing all your work, determine the size of the payments
under the 12-year annuity. Assume that the 7% rate applies from the time of
the last payment under the 10% annuity.
Information for Students in MATH 329 2005 01 55

3. [18 MARKS] A loan of 9000 is to be repaid by annual payments of 800 to commence


immediately, and to continue at the beginning of each year for as long as necessary.
Find the time and amount of the final payment if the final payment is to be no
smaller than the regular payments. Assume i = 8%.
Information for Students in MATH 329 2005 01 56

4. Showing your work in detail, determine each of the following; the rates you de-
termine should be accurate to 4 decimal places, or as a percentage accurate to 2
decimal places:

(a) [4 MARKS] the nominal annual interest rate, compounded quarterly, equiva-
lent to an effective semi-annual discount rate of d = 4%
(b) [4 MARKS] the effective semi-annual interest rate corresponding to a force of
interest of = 0.04.
(c) [4 MARKS] the effective annual discount rate corresponding to a nominal
interest rate, compounded quarterly, of i = 2.5%
Information for Students in MATH 329 2005 01 57

continuation page for problem number

You must refer to this continuation page on the page where the problem is printed!
Information for Students in MATH 329 2005 01 58

9 Solutions, Third Problem Assignment


Mounted on the Web on March 7th, 2005
Full solutions were to be submitted by March 7th, 2005.
(Caveat lector! There could be misprints or other errors!)
These problems were to be solved with full solutions, modelled either on the solutions
to problems in the textbook, or in the notes on the Web for this or previous years. The
essence is that the reader should be able to reconstruct every step of the proof from what
you have written: getting the right answer is never enough. You are not being graded
for elegance, but simply for the proof being logical, without serious gaps.
1. McGill plans to create a scholarship fund that will eternally pay 50 students a
monthly stipend of $200 at the beginning of months September through April,
plus an amount of $300 on the following May 1st.
(a) If interest is assumed to be at a nominal annual rate of 6% per annum, com-
pounded monthly, determine the amount that is needed in this fund on Sep-
tember 1st just before the fund begins making payments.
(b) Determine the amount that will be in the fund just after the December schol-
arship payments in the first year, and on September 1st of the following year,
just before the September payments.
(c) Suppose that at the beginning of September, 8 years after the fund is estab-
lished, it is decided to increase the capital in the fund because the interest
rate has changed to 4% per annum compounded monthly. Determine how
much additional capital needs to be added to the fund.
(d) Suppose that 2 years after the interest rate is changed to 4%, it changes again,
this time to 8%. This time it is decided to leave the capital unchanged, but
to increase the payments to students by a lump sum of $M to each student,
payable on December 1st, together with the regular December payment under
the scholarship. Determine the amount of that lump sum payment.
Solution:
(a) Since monthly payments are not completely regular, we cannot interpret this
fund as a monthly perpetuity with regular payments. We could still develop
its properties from first principles, but it is easier to interpret it as an annual
perpetuity-due. The payments made to each student in an academic year are
worth

8 1 (1.005)8
200a8 0.5% + 300(1.005) = 200(1.005) + 300(1.005)8
0.005
= 1860.680368 .
Information for Students in MATH 329 2005 01 59

Alternatively, we can think of an annuity-due of 9 payments of 200 together


with an additional payment of 100 alongside the last payment:

8 1 (1.005)9
200a9 0.5% + 100(1.005) = 200(1.005) + 100(1.005)8
0.005
= 1860.680368 .

The effective annual interest rate is

(1.005)12 1 = 6.1677812% .

It follows that the amount in the fund at the beginning of September, just
before the September payments, must be

(1.005)12
50(1860.680368)a (1.005)12 1 = 50(1860.680368) = 1, 601, 421.16 .
(1.005)12 1

(b) We could evaluate the fund at the beginning of each month, determining the
growth in principal, and then subtracting the payments that would need to
be made on that day. Instead, we will shift all payments to the dates under
examination. On December 1st, just after the December payments, the fund
will be worth

1, 601, 421.16(1.005)3 50(200)s4 0.005



3 (1.005)4 1
= 1, 601, 421.16(1.005) 50(200)
0.005
= 1, 585, 261.78

On the following September 1st, just before the first payments of the new
academic year, the value of the fund must be

1, 585, 261.78(1.005)9 50(200)s4 0.005 (1.005)5 50(300)(1.005)4



9 (1.005)4 1
= 1, 585, 261.78(1.005) 50(200) (1.005)5 50(300)(1.005)4
0.005
= 1, 601, 421.16

This is no surprise we constructed this fund so that the amount just before
the September 1st payments would be constant.
(c) The amount in the fund on September 1st, just before payments are made,
has been constant. Because of the revised interest rate, the annual costs per
Information for Students in MATH 329 2005 01 60

participant will be, as of the time of the September payment,

200a8 1 % + 300(1.0033333333)8
3

1 (1.0033333333)8
= 200(1.0033333333) + 300(1.0033333333)8
0.00333333333
= 1873.636976 .

The effective annual interest rate is now

(1.0033333333)12 1 = 4.0741539% .

The amount of the fund will, therefore, need to be


1.040741539
50 1873.636976 = 2, 393, 100.36 .
0.040741539
The additional capital required is, therefore,

2, 393, 100.36 1, 601, 421.16 = 791, 679.20 .

(d) Since this is a perpetuity, the amount in the account is periodic. Unless the
interest rate or other conditions change, the amount is always the same on 1
September, and it does not matter how many years have passed. We repeat
the calculations of the preceding part, this time for a nominal rate of 8%.
Because of the revised interest rate, the annual costs per participant will be,
as of the time of the September payment,

200a8 2 % + 300(1.0066666667)8
3

1 (1.0066666667)8
= 200(1.0066666667) + 300(1.0066666667)8
0.0066666667
= 1847.870679 .

The effective annual interest rate is now

(1.0066666667)12 1 = 8.2999511% .

The amount of the fund to support the previous obligations will, therefore,
need to be
1.082999511
50 1847.870679 = 1, 205, 575.19 .
0.082999511
The capital no longer required for the previously defined purposes is, therefore,

2, 393, 100.36 1, 205, 575.19 = 1, 187, 525.17


Information for Students in MATH 329 2005 01 61

on September 1st, prior to the payments due on that date. On December 1st
this excess is worth

1, 187, 525.17(1.0066666667)3 = 1, 211, 434.36

in total, or 24,228.68728 for each of the 50 participants at any time. This can
purchase a perpetuity-due of
24228.68728 24228.68728 0.082999511
M= = = 1856.851435 .
a 8.2999511% 1.082999511
(The problem was somewhat ambiguous: some students thought the intention
was that there would be a single lump sum payment in just one December. If
that had been the intention, the value would have been 24,228.69 determined
above.)

2. A loan of 10,000 is to be repaid by regular, half-yearly payments of 1,000, the first


to be made at the end of the 3rd year. The loan will be paid off by a final payment
which must be at least 1,000.

(a) If the interest rate is to be 8% per annum, compounded semi-annually, deter-


mine the amount of the final payment, and when it is made.
(b) Suppose that the final payment is now not more than 1,000. and the inter-
est rate remains 8% per annum compounded semi-annually. Determine the
amount of the final payment, and when it is made.
(c) Suppose that, after signing his original commitment, the borrower decides
that he would like to make the first payment 6 months from the date of
borrowing, and that all the payments under this loan should be exactly equal.
If the interest rate is 6% per annum, compounded semi-annually, determine
that payment level that would be closest to 1000 per half-year, and determine
exactly when the loan will be paid off.

Solution:

(a) Let n be the number of the last payment (counting in half-years from the date
of the loan). Then n will be the largest integer such that

10000(1.04)n 1000sn5 4%

since the first 5 opportunities for payments are missed. The inequality is
equivalent to
10000(1.04)5 sn5
= an5
1000 (1.04)n5
Information for Students in MATH 329 2005 01 62

and to
(1.04)(n5) 1 0.4(1.04)5
which implies that
ln (1 0.04(1.04)5 )
n5 = 17.00170887
ln(1.04)
so the last payment is made at n = 22. The excess of the amount of the
payment over 1000 will be
10000(1.04)22 1000s17 = 1.67552
rounding to 1.68.
(b) This time the final payment will be #23; the amount of the payment will be
the accumulation of the residue remaining unpaid after the payment at time
n = 22, i.e., (1.04)1.67552 = 1.74254 rounding to 1.74.
(c) If the last payment is n half-years after the date of borrowing, the payments
will be in the amount
10000
.
an
The object is to minimize
10000
1000
an
or to minimize
10
1
an
Since an is an increasing function of n, its reciprocal is decreasing. We need
to find the value of n such that
an 3% 10 < an+1 3%
i.e., the largest n such that an 3% 10, i.e., such that v n 0.7, i.e., such that
ln(0.7)
n = 12.06662371. So the only candidates are n = 12 and n = 13,
ln(1.03)
and the corresponding semiannual payments are
10000
= 1004.620855
a12
and
10000
= 940.2954396
a13
Taking the payment closest to 1000, we find that the last payment will be at
the end of 6 years, and the amount of the regular payments will be 1004.62.
Information for Students in MATH 329 2005 01 63

3. (An acknowledgement of the source of this problem will be contained in the solu-
tions, when published.) Katherine, 25 years old, deposits 10,000 at the beginning
of every 4-year period into an RSSP account. The account pays compound interest
annually, at the effective annual rate i. The accumulated amount in the account at
the end of 40 years is X, which is 6 times the accumulated amount in the account
at the end of 20 years. Determine X.
Solution: This problem was modelled on Problem 8 of Course 2, May, 2003, of the
Society of Actuaries.
There are several ways of attacking a problem like this. One method is to determine
from i the interest rate that would apply to the period of payments, i.e. 4 years;
another is to determine the annual payment in advance that would correspond to
a payment of 10,000 in advance every 4 years.
The problem on which this is modelled was posed as a multiple-choice problem,
with 5 possible numerical answers. None of the answers was expressed in terms of
i; so, in fact, there was no need for the question to even mention i. You may judge
for yourself whether that inclusion made the problem harder or easier.
Denote by Y the annual payment in advance that is equivalent to a payment every
4-years in advance of 10,000. Then
Y a4 i = 10000 ,
10000
so Y = . The values of the account at the ends of 20 years and 40 years are,
a4 i
respectively
(1 + i)40 1
X = Y s40 i = Y (1 + i)
i
(1 + i)20 1
and Y s20 i = Y (1 + i)
i
From the hypothesis that these amounts are in the ratio of 6 : 1, we conclude that
(1+i)40 1
Y (1 + i)
6= i
(1+i)20 1
= (1 + i)20 + 1
Y (1 + i) i

which implies that (1 + i)20 = 5. We were not asked to compute the interest rate,
and dont actually require it. (If you did require it, you could solve the preceding
1
equation to show that 1 + i = 5 20 = 1.083798, so i = 8.38%.) Then
(1+i)40 1
(1 + i) i
X = 10000 4
(1 + i) 1v
i
Information for Students in MATH 329 2005 01 64

(1 + i)40 1 10000(25 1) 240000 240000


= 10000 = 1 = =
1 v4 1 5 5 1 0.72478 0.275220336
= 872, 028.58

4. For each of the following sequences of payments as of the time stated,

express the value using standard symbols, as simple as possible;


give a formula for the value (in terms of i, v, d, etc.);
evaluate using a calculator or computer not with tables.

(a) a perpetuity-immediate paying 1 per year at effective annual interest rate


4.25%, evaluated 1 year before the first payment;
(b) a perpetuity paying 1 per year at effective annual interest rate 4.25%, evalu-
ated 2 years before the first payment;
(c) a perpetuity-due paying 1 per half-year at nominal interest rate 5%, com-
pounded semi-annually, evaluated just before the first payment;
(d) a 20-year increasing annuity paying 1 the first year, 2 the second year, 3 the
3rd year, etc., at an interest rate of 7% per year effective, evaluated just after
the last payment;
(e) a 20-year increasing annuity paying 1 the first year, 2 the second year, 3 the
3rd year, etc., at an interest rate of 7% per year effective, evaluated at the
time of the 2nd payment;
(f) a 10-year decreasing annuity paying 1 less each quarter-year, evaluated just
after the last payment, where the nominal interest rate is a 8% annual, com-
pounded quarterly;

Solution:
1 1
(a) a 4.25% = = = 23.52941176 .
i 0.00425
v 1
(b) v a 4.25% = = = 22.57017915 .
i 1.0425(0.0425)
1 1
(c) a 0.025 = = = 41.
iv 0.025(1.025)1
21
(1.07) 1
s21 0.07 21 0.07
21
(d) (Is)20 0.07 = = = 340.9310971 .
0.07 0.07
(e) v 18 (Is)20 0.07 = (1.07)18 (340.9310971) = 100.8692096 .
Information for Students in MATH 329 2005 01 65

40
n(1 + i)n sn 40(1.02)40 (1.02)
0.02
1
(f) (Ds)40 0.02 = = = 1395.980168.
i 0.02
5. X and Y have sold their home for 400,000, and wish to purchase an annuity-
immediate so that they can spread the proceeds over the next 10 years. The first
payment will be one month from the date of purchase of the annuity.
(a) Determine the level monthly payments they will receive if the effective annual
interest rate is 6%.
(b) X and Y live frugally, and dont think they need as much income now as
they will as time passes. Accordingly they plan to receive monthly payments
which will gradually increase. If the first payment is 3,000, and the payments
increase each month by the same dollar amount K, determine K. The effective
annual interest rate remains 6%. What is the final monthly payment?
(c) Suppose that the payments remain constant in any year. The first years
monthly payments are all 3000, the next years 3000+L, the next years 3000+
2L, 3000 + 3L, . . . , 3000 + 9L. Determine L if the nominal annual interest
rate is 6%, compounded monthly.
Solution:
1
(a) The effective monthly interest rate is (1.06) 12 1 = 0.4867551%, which we
shall denote by j below. The payments will be
1
400000 400000((1.06) 12 1)
= 120
a120 (1.06) 121 1 1 (1.06) 12

1
400000((1.06) 12 1)
= = 4408.961439
1 (1.06)10
(b) We solve for K the equation of value
400000 = K (Ia)120 j + (3000 K) a120 j
!
a120 j 120(1 + j)120
400000 = K + (3000 K) a120j
j
!
a120 j 120(1 + j)120
= K a120j + 3000 a120j
j

Hence

j 400000 3000 a120 j
K =
a120 j 120(1 + j)120 j a120j
Information for Students in MATH 329 2005 01 66


j 400000 3000 a120 j
=
a120 j 120(1 + j)120
= 26.23459805
so the final payment will be
3, 000.00 + 119(26.23459805) = 6, 121.92 .
(c) Consider payments of L at the end of every month for a year. Discounted
back to the beginning of the year, these payments are worth

1 1.00512
L a12 0.005 = L = L(11.61893206) .
0.005
The value of all of the payments as of the date of purchase of the annuity is,
therefore,
3000 a1200.005 + L a12 0.005 (Ia)9 (1.005)12 1 = 400000 .
Now
a9 (1.005)12 1 9(1.005)9(12)
(Ia)9 (1.005)12 1 =
(1.005)12 1
(1.005)12 a9 (1.005)12 1 9(1.005)9(12)
=
(1.005)12 1
a9 (1.005)12 1 9(1.005)10(12)
=
1 (1.005)12
1(1.005)9(12)
(1.005)12 1
9(1.005)10(12)
=
1 (1.005)12
= 31.08041107 .
Solving for L, we obtain
400000 3000 a120 1 %
L = 2

11.61893206(31.08041107)

= 0.002769153521 400000 3000 a120 0.005

1 (1.005)120
= 0.002769153521 400000 3000
0.005
= 359.3797471.
The monthly payments in the first year will be 3,000; those in the last year
will be
3, 000 + 9(359.3797471) = 6, 234.42 .
Information for Students in MATH 329 2005 01 67

10 Fourth Problem Assignment


Mounted on the Web on March 7th, 2005
Distributed in hard copy on Wednesday, March 9th, 2005
Full solutions are to be submitted by March 21st, 2005.

These problems were to be solved with full solutions, modelled either on the solutions
to problems in the textbook, or in the notes on the Web for this or previous years. The
essence is that the reader should be able to reconstruct every step of the proof from what
you have written: getting the right answer is never enough. You are not being graded
for elegance, but simply for the proof being logical, without serious gaps.

1. A loan of 20,000 is to be repaid by 15 annual payments beginning one year after


the loan was received, payments which increase by a factor of 1.03, each over the
preceding. If the effective annual interest rate is 7%, determine

(a) the amount of the first payment


(b) the amount of the loan which is outstanding immediately after the 10th pay-
ment, determined using the Retrospective Method
(c) the amount of the loan which is outstanding immediately after the 10th pay-
ment, determined using the Prospective Method

2. Mary always dreamed of owning a Maserati, and now had the chance. Her employer
was buying a new Ferrari, and was willing to part with his used Maserati for a mere
20,000. Mary agreed to repay the loan by equal monthly payments over 3 years at
4% interest, compounded monthly, first payment one month after the sale. Two
and one-half years after the sale Marys employer declared bankruptcy, and Marys
future payments became one of the assets to be administered by the trustee.

(a) Determine Marys regular monthly payment.


(b) The trustee in bankruptcy needs to know the present value of Marys future
payments. Determine this value just after the payment made on the date of
bankruptcy if
i. the interest rate remains a nominal 4% compounded monthly;
ii. the interest rate is now 6% compounded monthly.
(c) Complete an amortization table beginning on the date of bankruptcy, when
the rate is 4% compounded monthly, under the headings
Information for Students in MATH 329 2005 01 68

Payment Payment Interest Principal Outstanding


Number Amount paid repaid loan balance
30
31
...
(d) Suppose that Marys future payments, discounted at 6%, have been assigned
to one of the employers creditors. You have computed the present value of
these payments. Now set up an amortization table to show how his outstand-
ing debt in this amount will be amortized at 6%.
3. On January 1st, 2005, John receives a loan of 50,000 from his brother, and agrees
to repay it by semi-annual payments: 2,000 every July 1st, and 2,500 every January
1st; the payments begin on July 1st, 2005, and continue until one last, possibly
irregular payment which will be either less than 2,000 if it is on July 1st, or under
2,500 if it is on January 1st. The effective interest rate is 6% per annum.
(a) Determine the outstanding principal just after the payment on January 1st,
2015.
(b) Complete an amortization schedule beginning with January 1st, 2020, under
the following headings
Date Payment Interest Principal Outstanding
Amount paid repaid loan balance
January 1, 2020
etc.
4. A 30-year loan is amortized by level payments at the end of every month, at a
nominal interest rate of 16.8%, compounded monthly. The amount of interest paid
in the 120th payment is 486.72. Determine
(a) the amount of interest in the 300th payment;
(b) the original amount of the loan;
(c) the amount due at the end of 30 years if the lender defaults on8 the last 6
payments.
5. A loan of 100,000 is to be maintained by a semi-annual interest payment, with the
principal being repaid in a single payment at the end of 25 years. The borrowers
total cost is 5,000 at the end of each 6-month period, being made up of the interest
and a contribution to a sinking fund maintained by a 3rd party earning 4% effective:
the proceeds will be paid to the lender upon maturity. Find
8
=fails to pay
Information for Students in MATH 329 2005 01 69

(a) the effective semi-annual interest rate received by the lender of the loan.
(b) the effective interest rate paid out by the borrower. (For this purpose it will
suffice to determine an equation for the interest rate, and to use the tables
in your book to locate the rate between two tabulated values; you are not
expected to interpolate.)

6. Pierre borrows 18,000 for 8 years, and agrees to make level annual payments. The
lender receives 6% on the investment for each of the first 5 years, and 8% for the
last 3 years. Throughout the 8 years, the balance of each payment is invested in a
sinking fund earning 7%.

(a) Determine the amount of the lenders total annual payment.


(b) Complete the following table.

Period Total Interest Interest Earned Contribution to Balance in


Payment to Lender in Sinking Fund Sinking Fund Sinking Fund
0
etc.
Information for Students in MATH 329 2005 01 70

11 Solutions to Problems on the Class Test


Distribution Date: Mounted on the Web on Saturday, March 12th, 2005
Distributed in hard copy on Wednesday, March 16th, 2005
(Subject to correction)

There were 4 versions of this test. The final grades of all were scaled upwards slightly,
by slightly different factors, in an attempt to equalize the difficulty of the versions.

11.1 Problems on rates of interest and discount


1. Problem 1 on Version 1 Showing your work in detail, determine each of the
following; the rates you determine should be accurate to 4 decimal places, or as a
percentage accurate to 2 decimal places:

(a) [4 MARKS] the nominal annual interest rate, compounded monthly, corre-
sponding to an effective annual interest rate of i = 6%
(b) [4 MARKS] the effective annual interest rate corresponding to a nominal dis-
count rate, compounded quarterly, of d = 3.6%
(c) [4 MARKS] the effective monthly interest rate corresponding to a force of
interest of = 0.06.

Solution:
1
(a) The effective interest rate per month is (1.06) 12 1 = 0.4867551%; the nom-
inal annual interest rate, compounded monthly, i(12) , is 12 times this rate,
5.8410612%.
(4)
4
(b) 1d = 1 d 4 = (10.009)4 = 0.9644830906. 1+i = 1d 1
= (0.9644830906)1 =
1.036824813, so the equivalent effective annual interest rate is 3.6824813%.
(c) Since = 0.05 = ln(1 + i), 1 + i = e0.06 .
0.06
i(12) = 12 e 12 1 = 12 0.5012521% = 6.0150252%

i(12)
The effective monthly interest rate is = 0.5012521.
12
2. Problem 2 on Version 2 Showing your work in detail, determine each of the
following; the rates you determine should be accurate to 4 decimal places, or as a
percentage accurate to 2 decimal places:
Information for Students in MATH 329 2005 01 71

(a) [4 MARKS] the effective annual discount rate corresponding to a nominal


interest rate, compounded quarterly, of i = 2.4%
(b) [4 MARKS] the nominal annual interest rate, compounded quarterly, equiva-
lent to an effective semi-annual discount rate of d = 4%
(c) [4 MARKS] the effective semi-annual interest rate corresponding to a force of
interest of = 0.04.

Solution:

(a) The effective interest rate per quarter is 14 2.4% = 0.006. If i and d be the
effective annual interest and discount rates, then
1 1
1d= = ,
1+i (1.006)4
so
1
d=1 = 2.36442751%
(1.006)4
(b) The annual discount factor corresponding to an effective semi-annual discount
rate of 4% is (1 0.04)2 ; the annual accumulation factor will, therefore, be
(0.96)2 . The accumulation factor corresponding to a quarter of a year will be
2 2
(0.96) 4 , and so the equivalent effective quarterly interest rate is (0.96) 4 1;
the nominal annual interest rate, compounded quarterly, is 4 times this, i.e.
2

4 (0.96) 4 1 = 8.2482904%.

(c) If i be the equivalent effective annual interest rate, then ln(1 + i) = 0.04, so
1 + i = e0.04 . The effective semi-annual interest rate corresponding is

1 + i 1 = e0.02 1 = 2.0201340% .

3. Problem 3 on Version 3 Showing your work in detail, determine each of the


following; the rates you determine should be accurate to 4 decimal places, or as a
percentage accurate to 2 decimal places:

(a) [4 MARKS] the effective annual interest rate, corresponding to an nominal


annual interest rate of i = 6% compounded every 4 months.
(b) [4 MARKS] the effective monthly interest rate corresponding to a force of
interest of = 0.12.
(c) [4 MARKS] the effective annual interest rate corresponding to d(4) = 3.2%
Information for Students in MATH 329 2005 01 72

Solution:
(a) A nominal interest rate of 6% compounded every 4 months is equivalent to an
effective interest rate of 2% for a 4-month period. The accumulation factor for
1 year will be (1.02)3 , and the equivalent annual interest rate will, therefore,
be (1.02)3 1 = 6.1208%.
(b) If i be the equivalent annual interest rate, ln(1 + i) = 0.12, so 1 + i = e0.12 .
1
The accumulation factor for 1 month will be (1 + i) 12 = e0.01 , so the effective
interest rate for 1 month will be e0.01 1 = 1.0050167%.
(c) Let i and d denote the effective annual interest and discount rates. Then
1 1 1
1+i= = 4 = = 1.032650385
1d d(4) (1 0.008)4
1 4

so the effective annual interest rate is 3.2650385%.


4. Problem 4 on Version 4 Showing your work in detail, determine each of the
following; the rates you determine should be accurate to 4 decimal places, or as a
percentage accurate to 2 decimal places:
(a) [4 MARKS] the nominal annual interest rate, compounded quarterly, equiva-
lent to an effective semi-annual discount rate of d = 4%
(b) [4 MARKS] the effective semi-annual interest rate corresponding to a force of
interest of = 0.04.
(c) [4 MARKS] the effective annual discount rate corresponding to a nominal
interest rate, compounded quarterly, of i = 2.5%
Solution:
(a) The discount factor for a year is (1 0.04)2 = (0.96)2 . If i is the effective
annual interest rate, then 1 + i = (0.96)2 , and the accumulation factor for
1 2
one quarter of a year is (1 + i) 4 = (0.96) 4 . The effective interest rate for one
2
quarter of a year is, therefore, (0.96) 4 1, and the nominal annual interest
rate, compounded quarterly, is
2

4 (0.96) 4 1 = 4 2.0620726% = 8.2482904%.

(b) If i be the equivalent effective annual interest rate, then ln(1 + i) = 0.04, so
1
1 + i = e0.04 , and (1 + i) 2 = e0.02 is the accumulation factor for half a year.
The effective semi-annual interest rate is, therefore
e0.02 1 = 2.0201340%.
Information for Students in MATH 329 2005 01 73

(c) The accumulation factor for a year is (1+ 0.025


4
)4 = 1.025235353, so the effective
annual interest rate is 2.5235353%. If d is the effective annual discount rate,
then 4
0.025
1+d= 1+ = 0.9753857951
4
so d = 2.46142049%.

11.2 Problems on the values of annuities and perpetuities with


constant payments
1. Problem 2 on Version 1 For each of the following sequences of payments, deter-
mine, as of the given time, and for the given interest or discount rate, the value,
showing all of your work. Before determining the numeric value you are expected
to express the value using standard symbols.

(a) [6 MARKS] the value now of 20 payments of 1 at the end of every year starting
1 year from now, at an interest rate of 4%
(b) [6 MARKS] the value 1 year ago of 10 payments of 1 at the end of every
half-year, the first to be paid 5 years from now, at a nominal interest rate of
8% compounded semi-annually
(c) [6 MARKS] 300 payments of 1 at the end of every month, as of the date
of the 100th payment, which has just been made; the interest rate is 12%
compounded monthly

Solution:

(a) This question was ambiguous. One reading was that the payments would
start one year from now, in which case the value is

1 (1.04)20
a20 4% = = 13.59032635 .
0.04
Another reading was that the first payment would be at the end of the year
that starts one year from now, in which case the preceding value should be
1
multiplied by v = , yielding
1.04
1 (1.04)20
v a20 4% = = 13.06762149 .
(0.04)(1.04)
Information for Students in MATH 329 2005 01 74

(b) The effective half-yearly interest rate is 21 8% = 4%. The value of the payments
4.5 years from now is a10 4% . The value 1 year ago i.e. 5.5 years prior to
the value just given, is

1 (1.04)10
v 11 a10 4% = (1.04)11 = 5.268683237 .
0.04
1
(c) The effective monthly interest rate is 12 12% = 1%. The value of the payments
just after the last of them is s300 1% . Precisely 200 months earlier the value is

(1.01)100 (1.01)200
v 200 s300 1% = = 256.8127449 .
0.01
2. Problem 3 on Version 2 For each of the following sequences of payments, deter-
mine, as of the given time, and for the given interest or discount rate, the value,
showing all of your work. Before determining the numeric value you are expected
to express the value using standard symbols.

(a) [6 MARKS] the value of 25 payments of 1 at the end of every year, the last
one having just been made, at an interest rate of 6%
(b) [6 MARKS] the value 4 months from now of 30 payments of 1 at the end of
every 4 months, the first to be paid 2 years from now, at a nominal interest
rate of 9% compounded 3 times a year
(c) [6 MARKS] 120 payments of 1 at the end of every 2 months, as of the date
of the 90th payment, which has just been made; the interest rate is 6% com-
pounded every 2 months.

Solution:
(1.06)25 1
(a) s25 6% = = 54.86451200 .
0.06
(b) The effective interest rate per 4 months is 31 (9%) = 3%. The first payment is
due 24 months from now; if we wish to interpret the payments as an annuity-
immediate, then the value 20 months from now will be a30 3% . The value 4
months from now, i.e., 16 months, or 43 of a year before the clock starts will
be
1 (1.03)30
v 4 a30 3% = (1.03)4
0.03
(1.03)4 (1.03)34
= = 17.41473827 .
0.03
Information for Students in MATH 329 2005 01 75

2
(c) The effective interest rate for a 2-month period is 12 6% = 1%. At the time
of the 120th payment the value of all the payments is s120 1% . 30 payments
prior to that date, the value is

(1.01)120 1
v 30 s120 1% = (1.01)30 = 170.6709757 .
0.01
3. Problem 4 on Version 3 For each of the following sequences of payments, deter-
mine, as of the given time, and for the given interest or discount rate, the value,
showing all of your work. Before determining the numeric value you are expected
to express the value using standard symbols.

(a) [6 MARKS] the value one half-year ago of 12 payments of 1 at the end of
every half-year, the first to be paid 4 years from now, at a nominal interest
rate of 6% compounded semi-annually
(b) [6 MARKS] 180 payments of 1 at the end of every month, as of the date
of the 100th payment, which has just been made; the interest rate is 18%
compounded monthly
(c) [6 MARKS] the value now of 16 payments of 1 at the end of every year starting
1 year from now, at an interest rate of 4%

Solution:

(a) The effective semi-annual interest rate is 12 (6%) = 3%. Three and one-half
years from now the payments will be worth a12 3% . One half-year ago, they
were worth
v 8 v 20 (1.03)8 (1.03)20
v 8 a12 3% = = = 7.857782670 .
i 0.03
1
(b) The effective monthly interest rate is 12 (18%) = 1.5%. As of the last payment
the payments are worth s180 1.5% ; 80 payments earlier the value is

(1.015)100 (1.015)80
v 80 s180 1.5% = = 275.2103668 .
0.015

(c) This problem is ambiguous. Does the word starting refer to the payments
or to the years? If it refers to the payments, the value is

1 (1.04)16
a16 4% = = 11.65229561 .
0.04
Information for Students in MATH 329 2005 01 76

If the word refers to the years, then one has an annuity-immediate which has
been deferred one year, and the value is
1 (1.04)16
v a16 4% = = 11.20413039 .
(0.04)(1.04)

4. Problem 1 on Version 4 For each of the following sequences of payments, deter-


mine, as of the given time, and for the given interest or discount rate, the value,
showing all of your work. Before determining the numeric value you are expected
to express the value using standard symbols.
(a) [6 MARKS] 90 payments of 1 at the end of every 2 months, as of the date of the
18th payment, which has just been made; the interest rate is 9% compounded
every 2 months.
(b) [6 MARKS] the value of 20 payments of 1 at the end of every year, the last
one having just been made, at an interest rate of 5%
(c) [6 MARKS] the value 4 months from now of 45 payments of 1 at the end of
every 4 months, the first to be paid 3 years from now, at a nominal interest
rate of 6% compounded 3 times a year
Solution:
2
(a) The effective interest rate for a 2-month period is 12 (9%) = 1.5%. Two months
before the first payment the value of all the payments is a90 1.5% . Immediately
after the 18th payment the value of all the payments is
(1.015)18 (1.015)72
(1.015)18 a90 1.5% = = 64.33404251 .
0.015
(1.05)20 1
(b) s20 5% = 0.05
= 33.06595410 .
4
(c) The effective interest rate per 4-month period is 12 (6%) = 2%. Two and two-
thirds years from now the value of the payments is a45 2% . Hence one-third
year from now the value will be
(1.02)7 (1.02)52
(1.02)7 a45 2% = = 25.67295884 .
0.02

11.3 Problems on increasing and decreasing annuities and per-


petuities
1. Problem 3(a) on Version 1 [6 MARKS] An annuity at interest rate i consists of
payments of 10 now, 12 at the end of 1 year, 14 at the end of 2 years, increasing by
Information for Students in MATH 329 2005 01 77

a constant amount until the last payment in the amount of 40, is to be evaluated
as of 1 year ago. Express its value in terms of symbols (Ia)n , (Is)n , an , sn , i, v,
but do not evaluate.
Solution: The total increase of 40 10 = 30 will be spread over 4010
2
= 15 years:
the last payment will be 15 years from now. The value 1 year ago was

8 a16 + 2(Ia)16

2. Problem 4(a) on Version 2 [6 MARKS] An annuity at interest rate i consists of


payments of 100 now, 95 at the end of 1 year, 90 at the end of 2 years, decreasing
by a constant amount until the last payment in the amount of 25, is to be evaluated
as of the time of the payment of 95. Express its value then in terms of symbols
(Ia)n , (Is)n , an , sn , i, v, but do not evaluate.
Solution: The total decrease of the annuity is 75, at 5 per year; so the last payment
will be 75
5
= 15 years from now. The value is

100(1 + i) + 95 1 + a14 5(Ia)14

3. Problem 1(a) on Version 3 [6 MARKS] An annuity at interest rate i consists


of payments of 10 now, 13 at the end of 1 year, 16 at the end of 2 years, increasing
by a constant amount until the last payment in the amount of 40, and is to be
evaluated as of 2 years ago. Express its value in terms of symbols (Ia)n , (Is)n , an ,
sn , i, v, but do not evaluate.
Solution: The increase of 40 10 = 30 will be spread over 30 3
= 10 years. The
value of the annuity today is 10a11 + 3(Ia)10 . As of 2 years ago the value was

10v a11 + 3v 2 (Ia)10

4. Problem 2(a) on Version 4 [6 MARKS] An annuity at interest rate i consists of


payments of 118 now, 112 at the end of 1 year, 106 at the end of 2 years, decreasing
until the last payment in the amount of 34, the totality to be evaluated as of the
time of the payment of 112. Express its value in terms of symbols (Ia)n , (Is)n , an ,
sn , i, v, but do not evaluate.
84
Solution: The total decrease of 118 34 = 84 is to be spread over 6
= 14 years.
The value of the payments at time 1 is

118(1 + i) + 112 + 112a14 6(Ia)13


Information for Students in MATH 329 2005 01 78

11.4 Problems on combinations of annuities and perpetuities


1. Problem 3(b) on Version 1 [12 MARKS] The accumulated value just after the
last payment under a 15-year annuity of 1000 per year, paying interest at the rate
of 8% per annum effective, is to be used to purchase a perpetuity, first payment
to be made 2 years after the last payment under the annuity. Showing all your
work, determine the size of the payments under the perpetuity, assuming that the
interest rate from the time of the last payment under the 15-year annuity is 5%.
Solution: Let the payment under the perpetuity be X. Then the equation of value
at time 15 is
X X X
1000 s15 8% = =
0.05 1.05 (0.05)(1.05)
since the first payment of the perpetuity-immediate whose clock begins to tick at
time 15 is not made. Solving the equation yields X = 1425.49.

2. Problem 4(b) on Version 2 [12 MARKS] The accumulated value just after
the last payment under a 12-year annuity of 1000 per year, paying interest at the
rate of 6% per annum effective, is to be used to purchase a 10-year annuity at an
interest rate of 7%, first payment to be made 4 years after the last payment under
the annuity. Showing all your work, determine the size of the payments under the
10-year annuity. Assume that the 7% rate begins from the time of the last payment
under the 12-year annuity.
Solution: Let X denote the payment under the 10-year annuity. The value of the
payments under the 12-year annuity at the time of the last of them is 1000s12 6% =
(1.07)3 a10 7% . The value
of the payments
under the deferred 10-year annuity, as
of that same date, is X a13 7% a3 7% . Equating these amounts and solving, we
obtain
1000 s12 6% 100(1.07)3 s12 6%
X= = = 2942.43
a13 7% a3 7% a10 7%

3. Problem 1(b) on Version 3 [12 MARKS] The accumulated value just after the
last payment under a 15-year annuity of 1000 per year, paying interest at the rate
of 9% per annum effective, is to be used to purchase a perpetuity at an interest
rate of 8%, first payment to be made at the same time as the last payment under
the annuity. Showing all your work, determine the size of the payments under the
perpetuity.
Solution: Denote the amount of the payments under the perpetuity by X. The
equation of value at time 15 is
1.08
1000 s15 9% = X a 8% = X
0.08
Information for Students in MATH 329 2005 01 79

0.081000((1.09)15 1)
implying that X = 1.080.09
= 2174.88

4. Problem 2(b) on Version 4 [12 MARKS] The accumulated value just after the
last payment under a 14-year annuity of 1000 per year, paying interest at the rate of
10% per annum effective, is to be used to purchase a 12-year annuity-immediate at
an interest rate of 7%, first payment to be made 1 year after the last payment under
the 14-year annuity. Showing all your work, determine the size of the payments
under the 12-year annuity. Assume that the 7% rate applies from the time of the
last payment under the 10% annuity.
Solution: If we denote the amount of the payments under the 12-year annuity by
X, the equation of value at time 14 is

1000 s14 10% = X a12 7%

implying that X = 3522.11.

11.5 Problems on drop and balloon payments


1. Problem 4 on Version 1 [18 MARKS] A loan of 6000 is to be repaid by annual
payments of 350 to commence at the end of the 1st year, and to continue thereafter
for as long as necessary. Find the time and amount of the final payment if the final
payment is to be larger than the regular payments. Assume i = 4%.
Solution: The balloon payment will take place at the time call it n which is
the largest solution of the inequality

350 an 4% < 6000

i.e., of the inequalities

1 (1.04)n 6000
<
0.04 350
110
(1.04)n >
350
ln 350 ln 110
n < = 29.51126321 .
ln 1.04
It follows that the last payment is at the end of the 29th year. The amount of the
payment is

29 (1.04)28 1
6000(1.04) 350(1.04) = 523.70851
0.04
Information for Students in MATH 329 2005 01 80

2. Problem 1 on Version 2 [18 MARKS] A loan of 7000 is to be repaid by annual


payments of 450 to commence immediately, and to continue at the beginning of
each year for as long as necessary. Find the time and amount of the final payment
if the final payment is to be no larger than the regular payments. Assume i = 6%.
Solution: The drop payment will take place at the time call it n which is the
smallest solution of the inequality
450 an+1 6% 7000
i.e., of the inequalities
1 (1.06)(n+1) 7000
(1.06)
0.06) 450
7000 6 19
(1.06)(n+1) 1 =
450 106 159
19
ln 159
n+1 = 36.45967106
ln 1.06
implying that n = 36. The amount of the final drop payment will be
1.06
7000(1.06)36 450s36 6% = 7000(1.06)36 (450) (1.06)36 1
0.06
= 210.11059
so the final payment is in the amount of 210.11.
3. Problem 2 on Version 3 [18 MARKS] A loan of 8000 is to be repaid by annual
payments of 500 to commence at the end of the 1st year, and to continue thereafter
for as long as necessary. Find the time and amount of the final payment if the final
payment is to be smaller than the regular payments. Assume i = 5%.
Solution: Let the final payment be made at time n, which will be the smallest
integer such that
500 an 5% > 8000
1 (1.05)n > 16(0.05) = 0.8
ln 0.2
n> = 32.98693373 ,
ln 1.05
implying that n = 33. If the present value of the deficiency of the last payment
from 500 be denoted by X, then
X = 500 an 5% 8000

1 (1.05)33
= 500 16 = 1.27460500
0.05
Information for Students in MATH 329 2005 01 81

so the final payment is

500.00 (1.27460500)(1.05)33 = 493.6229109

or 493.62.

4. Problem 3 on Version 4 [18 MARKS] A loan of 9000 is to be repaid by annual


payments of 800 to commence immediately, and to continue at the beginning of
each year for as long as necessary. Find the time and amount of the final payment
if the final payment is to be no smaller than the regular payments. Assume i = 8%.
Solution: Let the final payment be at time n, i.e., be the (n + 1)st payment. Then
n will be the largest integer such that

800 an+1 8% 9000


90
an+1 8%
8
90 0.08
1 1.08(n+1)
8 1.08
ln 6
n+1 = 23.28138292
ln 1.08
from which it follows that n = 22. The excess of the final payment over 800 will
be
(1.08)23 1
9000(1.08)22 800 s23 = 9000(1.08)22 800
0.08
= 214.22726 ,

The final payment will, therefore, be 800 + 214.22726 = 1014.23.


Information for Students in MATH 329 2005 01 82

12 Fifth Problem Assignment


Mounted on the Web on March 20th, 2005
Revised on March 21st, 2005
Hard copy distributed on March 23rd, 2005
Full solutions are to be submitted by April 4th, 2005.

1. (a) A 25,000 7% bond matures on June 30th, 2020. Interest is payable semian-
nually on June 30th and December 31st. Determine the price to be paid for
the bond on June 30th, 2005, in order to earn the investor a yield of 6%.
(Remember the convention for bonds interest is normally interpreted as a
nominal (annual) rate, compounded semi-annually, even when not explicitly
stated.)
(b) Repeat problem 1a if the purchase date is December 31st, 2012.
(c) A 10,000 bond has coupon rate 8% payable semiannually, and is redeemable
after a certain number of years at 11,250. The bond is purchased to yield 7%
convertible semiannually. If the present value of the redemption value of the
bond is 4,927 at the given yield rate, determine the purchase price.
(d) A 1,000 bond matures after a m years at par, and has a coupon rate of 10%
convertible semiannually. It is purchased at a price to yield 7% convertible
semi-annually. If the term of the bond is 2m years, the price of the bond will
increase by 107. Determine the price of the m-year bond to the nearest dollar.

2. An investor is considering the purchase of two 1,000 face value bonds which are
redeemable at the end of the same number of years, and are both to be bought to
yield 7.5% convertible semiannually. One bond costs 907, and pays coupons at the
rate of 6% per year, convertible semiannually. The second bond pays coupons at
7% per half-year.

(a) Determine the price he should pay for the second bond so that it is an equiv-
alent investment to the first.
(b) Determine the price the investor should pay for the second bond if the first
bond costs 960.45, and pays a premium of 100 upon redemption, while the
second pays a premium of 150 upon redemption.

3. A loan of 96,000 is being repaid by a monthly payment of interest at a nominal


rate of 6.0% compounded monthly, together with a monthly payment into a sinking
fund which earns interest at the rate of 4.0% per annum, also compounded monthly.
The total monthly payment is constant, at 1,000, with the exception of the very
last payment, which could be less than 1,000, in order to bring the fund up to its

UPDATED TO April 12, 2005


Information for Students in MATH 329 2005 01 83

target level of 96,000; at that time the entire sinking fund will be paid over to the
lender and the loan will have been repaid. Determine when the final deposit into
the sinking fund will be made, and its amount.

4. A 5000 par value 18-year bond has 7% semiannual coupons, and is callable at the
end of the 12th through the 17th years at par.

(a) Find the price to yield 6% convertible semiannually.


(b) Find the price to yield 8% convertible semiannually.
(c) Find the price to yield 8% convertible semiannually, if the bond pays a pre-
mium of 250 if it is called.
(d) Find the price to yield 6% convertible semiannually, if the bond pays a pre-
mium of 250 if it is called at the end of years ##12, 13, 14, and a premium
of 150 if it is called at the end of years 15 or 16. (It may still be called at
the end of year 17 without premium, and otherwise will mature at the end of
year 18.)

5. (a) Construct a bond amortization schedule for a 4 year bond of face amount
5,000, redeemable at 5,250 with semiannual coupons, if the coupon rate is 6%
and the yield rate is 7% both converted semiannually. Use the format
Time Coupon Interest Principal Book
Value Adjustment Value
0
..
.
(b) Construct a bond amortization schedule for a 4 year bond of face amount
5,000, redeemable at 5,250 with semiannual coupons, if the coupon rate is 7%
and the yield rate is 6% both converted semiannually.

6. An 8-year bond of face value 10,000 with semiannual coupons, redeemable at par,
is purchased at a premium to yield 6% convertible semiannually.

(a) If the book value (just after the payment of the coupon) six months before the
redemption date is 10,024.27, find the total amount of premium or discount
in the original purchase price.
(b) Determine the nominal annual coupon rate of the bond, compounded semi-
annually.
(c) Give the amortization table for the last 2 years.
Information for Students in MATH 329 2005 01 84

13 Solutions, Fourth Problem Assignment


Mounted on the Web on March 31st, 2005
Full solutions were to be submitted by March 21st, 2005.

These problems were to be solved with full solutions, modelled either on the solutions
to problems in the textbook, or in the notes on the Web for this or previous years. The
essence is that the reader should be able to reconstruct every step of the proof from what
you have written: getting the right answer is never enough. You are not being graded
for elegance, but simply for the proof being logical, without serious gaps.

1. A loan of 20,000 is to be repaid by 15 annual payments beginning one year after


the loan was received, payments which increase by a factor of 1.03, each over the
preceding. If the effective annual interest rate is 7%, determine

(a) the amount of the first payment


(b) the amount of the loan which is outstanding immediately after the 10th pay-
ment, determined using the Retrospective Method
(c) the amount of the loan which is outstanding immediately after the 10th pay-
ment, determined using the Prospective Method

Solution:

(a) Denote the amount of the first payment by X. The value at time 0 of all
future payments is

(1.07)1 (1.03)0 X + . . . + (1.07)r (1.03)r1 X + . . . + (1.07)15 (1.03)14 X


2 14 !
X 1.03 1.03 1.03
= 1+ + + ... +
1.07 1.07 1.07 1.07
1.03 15
X 1 1.07
=
1.07 1 1.03 1
1.07
15 !
X 1.03
= 1 = 10.8830229X .
0.04 1.07

Equating to 20,000, we find that X = 1837.72472 is the first payment.


(b) By the Retrospective Method, the amount outstanding after the 10th payment
is

10, 000(1.07)10 X (1.03)9 + (1.03)8 (1.07) + . . . + (1.03)0 (1.07)9
Information for Students in MATH 329 2005 01 85

103 10 !
1
= (1.07)10 (20, 000) 1 107
103 15
1 107
= 10, 709.669 .

(c) By the Prospective Method, the amount outstanding after the 10th payment
is
X X X
(1.03)10 + (1.03)11 2
+ . . . + (1.03)14
1.07 (1.07) (1.07)5
2 4 !
(1.03)10 103 103 103
= X 1+ + + ... +
1.07 107 107 107
= 10, 709.669 .

2. Mary always dreamed of owning a Maserati, and now had the chance. Her employer
was buying a new Ferrari, and was willing to part with his used Maserati for a mere
20,000. Mary agreed to repay the loan by equal monthly payments over 3 years at
4% interest, compounded monthly, first payment one month after the sale. Two
and one-half years after the sale Marys employer declared bankruptcy, and Marys
future payments became one of the assets to be administered by the trustee.

(a) Determine Marys regular monthly payment.


(b) The trustee in bankruptcy needs to know the present value of Marys future
payments. Determine this value just after the payment made on the date of
bankruptcy if
i. the interest rate remains a nominal 4% compounded monthly;
ii. the interest rate is now 6% compounded monthly.
(c) Complete an amortization table beginning on the date of bankruptcy, when
the rate is 4% compounded monthly, under the headings
Payment Payment Interest Principal Outstanding
Number Amount paid repaid loan balance
30
31
(d) Suppose that Marys future payments, discounted at 6%, have been assigned
to one of the employers creditors. You have computed the present value of
these payments. Now set up an amortization table to show how his outstand-
ing debt in this amount will be amortized at 6%.

Solution:
Information for Students in MATH 329 2005 01 86

4% 1
(a) The effective interest rate per month is 12
= 3
%. Let X be the regular
monthly payment. Then

X a36 1 % = 20000
3
36
20000 1 20000
X= 1 1 1+ = 300 36 = 590.4797061
300
300 300 1 301

so the monthly payment is 590.48.


(b) i. By the prospective method, the unpaid balance is
a6 1 %
X a6 1 % = 20, 000 3
3 a36 1 %
3
6
1 300301
= 20, 000 300 36 = 3501.909296
1 301

so the unpaid balance is 3,501.91.


ii. By the prospective method, the unpaid balance is
a6 1 %
X a6 1 % = 20, 000 2
3 a36 1 %
3
200 6
1 201 2
= 20, 000 300 36 = 3481.695351
1 301 3

so the unpaid balance is 3,481.70.


(c) The unpaid balance before the 30th payment was

3, 501.91 + 590.48 = 4, 092.39 .


4, 092.39
One month earlier, this was worth = 4, 078.79, so interest earned
1.00333333
in the 30th period was the difference,

4, 092.39 4, 078.79 = 13.60 .

Thus the principal reduction in the 30th payment was

590.48 13.60 = 576.88 .


Information for Students in MATH 329 2005 01 87

Payment Payment Interest Principal Outstanding


Number Amount paid repaid loan balance
30 590.48 13.60 576.88 3,501.91
31 590.48 11.67 578.81 2,923.10
32 590.48 9.74 580.74 2,342.36
33 590.48 7.81 582.67 1,759.69
34 590.48 5.87 584.61 1,175.08
35 590.48 3.92 586.56 585.52
36 590.48 1.96 588.52 0.00
(d) The new interest rate takes effect after the 30th payment.
Payment Payment Interest Principal Outstanding
Number Amount paid repaid loan balance
30 590.48 13.60 576.88 3,481.70
31 590.48 17.41 573.07 2,908.63
32 590.48 14.54 575.94 2,332.69
33 590.48 11.66 578.81 1,753.87
34 590.48 8.77 581.71 1,172.16
35 590.48 5.86 584.62 587.54
36 590.48 2.94 587.54 0.00
3. On January 1st, 2005, John receives a loan of 50,000 from his brother, and agrees
to repay it by semi-annual payments: 2,000 every July 1st, and 2,500 every January
1st; the payments begin on July 1st, 2005, and continue until one last, possibly
irregular payment which will be either less than 2,000 if it is on July 1st, or under
2,500 if it is on January 1st. The effective interest rate is 6% per annum.
(a) Determine the outstanding principal just after the payment on January 1st,
2015.
(b) Complete an amortization schedule beginning with January 1st, 2020, under
the following headings
Date Payment Interest Principal Outstanding
Amount paid repaid loan balance
January 1, 2020
Solution:
(a) Just after a January 1st payment of 2,500, that payment and the payment of
2,000 from the preceding July 1st are together worth

2, 500 + 2, 000 1.06 = 4, 559.26028 .
Information for Students in MATH 329 2005 01 88

By the retrospective method, the outstanding principal just after the payment
of January 1st, 2015, is

50, 000(1.06)10 2, 500 + 2, 000 1.06 s10 6%
(1.06)10 1
= 50, 000(1.06)10 (4559.126028) = 29, 449.47952
0.06
so the outstanding balance is 29,449.48.
(b) We need to begin the table with January 1st, 2020. It is convenient to repeat
the preceding computation: the outstanding balance as of January 1st, 2019,
is
(1.06)14 1
50, 000(1.06)14 (4, 559.126028) = 17, 234.86372 .
0.06
1
or 17,234.86. The effective interest rate per half year is 1.06 2 1 = 2.9563%.
Date Payment Interest Principal Outstanding
Amount paid repaid loan balance
January 1, 2019 2,500.00 17,234.86
July 1, 2019 2,000.00 509.51 1,490.49 15,744.37
January 1, 2020 2,500.00 465.45 2,034.55 13,709.82
July 1, 2020 2,000.00 405.30 1,594.70 12115.12
January 1, 2021 2,500.00 358.16 2,141.84 9,973.28
July 1, 2021 2,000.00 294.84 1,705.16 8,268.12
January 1, 2022 2,500.00 244.43 2,255.57 6,012.55
July 1, 2022 2,000.00 177.75 1,822.26 4,190.29
January 1, 2023 2,500.00 123.88 2,376.12 1,814.17
July 1, 2023 1,867.80 53.63 1,814.17 0.00

4. A 30-year loan is amortized by level payments at the end of every month, at a


nominal interest rate of 16.8%, compounded monthly. The amount of interest paid
in the 120th payment is 486.72. Determine

(a) the amount of interest in the 300th payment;


(b) the original amount of the loan;
(c) the amount due at the end of 30 years if the lender defaults on9 the last 6
payments.

Solution:
9
=fails to pay
Information for Students in MATH 329 2005 01 89

(a) Suppose that the original amount of the loan was X. The level payments
X
are then , which I will denote by Y . By the prospective method, the
a360 1.4%
amount owing just after the rth payment is Y a360r 1.4% , so the interest
component of the r + 1st payment is

(0.014)Y a360r 1.4% = Y 1 (1.014)r360 .

The data tell us that



Y 1 (1.014)119360 = 486.72

so Y = 504.4061464: the monthly payments are 504.41. It follows that the


interest component of the 300th payment is

Y 1 (1.014)299360 = 285.8812253

or 288.40.
(b) The original amount of the loan was
504.41
504.4061464 a360 1.4% = 1 (1.014)360 = 35, 787.47073
0.014
or 35,787.47.
(c) At the due date of the last payment, the value of the last 6 payments is
504.4061464 s6 1.4% = 3, 134.360302 or 3,134.36.

5. A loan of 100,000 is to be maintained by a semi-annual interest payment, with the


principal being repaid in a single payment at the end of 25 years. The borrowers
total cost is 5,000 at the end of each 6-month period, being made up of the interest
and a contribution to a sinking fund maintained by a 3rd party earning 4% effective:
the proceeds will be paid to the lender upon maturity. Find

(a) the effective semi-annual interest rate received by the lender of the loan.
(b) the effective interest rate paid out by the borrower. (For this purpose it will
suffice to determine an equation for the interest rate, and to use the tables
in your book to locate the rate between two tabulated values; you are not
expected to interpolate.)

Solution:
Information for Students in MATH 329 2005 01 90

(a) The sinking fund earns interest at an effective annual rate of 4%; the equivalent
semi-annual rate is
1.04 1 = 1.9803903%.
The semi-annual payments into the sinking fund will each be in the amount
of
100000 100000( 1.04 1)
= = 1188.826454
s50 1.041 (1.04)25 1
or 1,188.83. This leaves a residue of 5,000-1,188.83=3,811.17. The interest
rate per half year is, therefore
3, 811.17
= 3.811% .
100, 000

(b) Let i be the effective semi-annual interest rate being paid by the borrower.
Then
5, 000 a50 i = 100, 000
so a50 i = 20. We see from the tables that

a50 4.5% = 19.7620


a50 4% = 21.4822

(i is approximately 4.427%.)
NOTE TO THE GRADER: STUDENTS WERE NOT EXPECTED TO IN-
TERPOLATE OR ITERATE TO OBTAIN A GOOD APPROXIMATION.

6. Pierre borrows 18,000 for 8 years, and agrees to make level annual payments. The
lender receives 6% on the investment for each of the first 5 years, and 8% for the
last 3 years. Throughout the 8 years, the balance of each payment is invested in a
sinking fund earning 7%.

(a) Determine the amount of the lenders total annual payment.


(b) Complete the following table.

Period Total Interest Interest Earned Contribution to Balance in


Payment to Lender in Sinking Fund Sinking Fund Sinking Fund
0

Solution:
Information for Students in MATH 329 2005 01 91

(a) Let Pierres annual payment be X. The interest component in this payment
will be
.06 18, 000 = 1, 080 during the first 5 years
.08 18, 000 = 1, 440 during the last 3 years
Hence the annual contributions in arrears to the sinking fund will be
X-1,080 during the first 5 years
X-1,440 during the last 3 years
This information enables us to infer an equation that determines X:

(X 1, 080)s8 7% 360 s3 7% = 18, 000

from which it follows that


18, 000 + 360 s3 7%
X = 1, 080 +
s8 7%
1, 260 + 360 ((1.07)3 1)
= 1, 080 +
(1.07)8 1
= 2, 947.2254

(b) Growth of the Sinking Fund:

Period Total Interest Interest Earned Contribution to Balance in


Payment to Lender in Sinking Fund Sinking Fund Sinking Fund
0 0.00
1 2,947.23 1,080.00 0.00 1,867.23 1,867.23
2 2,947.23 1,080.00 130.71 1,867.23 3,865.17
3 2,947.23 1,080.00 270.56 1,867.23 6,002.96
4 2,947.23 1,080.00 420.21 1,867.23 8,290.40
5 2,947.23 1,080.00 580.33 1,867.23 10,737.96
6 2,947.23 1,440.00 751.66 1,507.23 12,996.85
7 2,947.23 1,440.00 909.78 1,507.23 15,413.86
8 2,947.23 1,440.00 1,078.96 1,507.23 18,000.06

Is it surprising that the balance is 18,000? Of course not the repayment


scheme was designed so that Pierre would accumulate in the fund just the
amount needed to repay the loan after 8 years.
Information for Students in MATH 329 2005 01 92

14 Solutions, Fifth Problem Assignment


Mounted on the Web on April 12th, 2005
Full solutions were to be submitted by April 4th, 2005.

1. (a) A 25,000 7% bond matures on June 30th, 2020. Interest is payable semian-
nually on June 30th and December 31st. Determine the price to be paid for
the bond on June 30th, 2005, in order to earn the investor a yield of 6%.
(Remember the convention for bonds interest is normally interpreted as a
nominal (annual) rate, compounded semi-annually, even when not explicitly
stated.)
(b) Repeat problem 1a if the purchase date is December 31st, 2012.
(c) A 10,000 bond has coupon rate 8% payable semiannually, and is redeemable
after a certain number of years at 11,250. The bond is purchased to yield 7%
convertible semiannually. If the present value of the redemption value of the
bond is 4,927 at the given yield rate, determine the purchase price.
(d) A 1,000 bond matures after a m years at par, and has a coupon rate of 10%
convertible semiannually. It is purchased at a price to yield 7% convertible
semi-annually. If the term of the bond is 2m years, the price of the bond will
increase by 107. Determine the price of the m-year bond to the nearest dollar.
Solution:
1
(a) F = C = 25, 000, r = 2
7% = 3.5%, i = 3%, n = 2 15 = 30. By the Basic
Formula

P = (25, 000)(0.035) a15 3% + (25, 000)(1.03)30



1 (1.03)30 30
= 25, 000 (0.035) + (1.03)
0.03
= 27, 450.06

(b) F = C = 25, 000, r = 3.5%, i = 3%, n = 15.

P = (25, 000)(0.035) a15 3% + (25, 000)(1.03)15



1 (1.03)15 15
= 25, 000 (0.035) + (1.03)
0.03
= 26, 492.24
Information for Students in MATH 329 2005 01 93

(c) F = 10, 000, C = 11, 250, r = 4%, i = 3.5%, K = 4927. This last value of K
implies that K = Cv n = 11, 250(1.025)n , so (1.035)n = 11,250
4,927
, where n is the
number of half-years remaining before maturity. By the Basic Formula,
P = F r an 0.035 + K

1 (1.035)n
= 400 + 4, 927
0.035
4,927
!
1 11,250
= 400 + 4, 927 = 11, 350.37
0.035

(d) For the m-year bond, F = C = 1, 000, n = 2m, r = 5%, i = 3.5%. For the
2m-year bond, n = 4m. Denote the price of the m-year bond by P . Then,
using the Premium/Discount formula, we have
P = 1, 000 + (50 35) a2m 3.5% (2)
P + 107 = 1, 000 + (50 35) a4m 3.5% (3)
Subtracting yields
107 = 15 a4m 3.5% a2m 3.5%
which implies that
1.0352m (1.035)4m
0.2496666 =
0.035
yielding, approximately,
(1.035)2m = 0.5182574184 or 0.4817425816
so, by (2),

1 0.5182574184
P = 1, 000 + 15 = 1206.46
0.035
or
1 0.4817425816
P = 1, 000 + 15 = 1, 222.11;
0.035
to the nearest dollar there are two answers, 1,206 and 1,222.
2. An investor is considering the purchase of two 1,000 face value bonds which are
redeemable at the end of the same number of years, and are both to be bought to
yield 7.5% convertible semiannually. One bond costs 907, and pays coupons at the
rate of 6% per year, convertible semiannually. The second bond pays coupons at
7% per half-year.
Information for Students in MATH 329 2005 01 94

(a) Determine the price he should pay for the second bond so that it is an equiv-
alent investment to the first.
(b) Determine the price the investor should pay for the second bond if the first
bond costs 960.45, and pays a premium of 100 upon redemption, while the
second pays a premium of 150 upon redemption.

Solution: The earlier references in the problem to interest rates always stated that
the rate was convertible semiannually. But, in this case, those words were not
mentioned. It appears that the rate of 7% stated is intended to be an effective
semi-annual rate, equivalently that the nominal annual rate is 14%. However, that
would be a much different rate than the others given in the problem. Because of
this ambiguity, I will solve the problem with both interpretations.

Assuming 3.5% effective per half year: (a) For both bonds we know that F =
C = 1000, i = 0.0375. By the Premium/Discount Formula applied to the
first bond we obtain

907 = 1000 + (30 37.5)an0.0375 ,

implying that an 0.0375 = 12.4. Substitution of this value into the same
formula applied to the second bond yields a price for that bond of

1000 + (F r Ci)an 0.0375 = 1000 + (35 37.5)an 0.0375


= 1000 31 = 961.

(b) In this case the Premium/Discount Formula applied to the first bond
gives
960.45 = 1100 + (30 41.25)an 0.0375 ,
which implies that an 0.0375 = 12.404444. Substitution of this value into
the same formula applied to the second bond yields a price for that bond
of

1150 + (F r Ci)an 0.0375 = 1150 + (35 43.125)an 0.035


= 1150 8.125(12.404444) = 1049.21

Assuming 7% effective per half year: (a) For both bonds we know that F =
C = 1000, i = 0.0375. By the Premium/Discount Formula applied to the
first bond we obtain

907 = 1000 + (30 37.5)an0.0375 ,


Information for Students in MATH 329 2005 01 95

implying that an 0.0375 = 12.4. Substitution of this value into the same
formula applied to the second bond yields a price for that bond of
1000 + (F r Ci)an 0.0375 = 1000 + (70 37.5)an 0.0375
= 1000 + 403 = 1403.
(b) In this case the Premium/Discount Formula applied to the first bond
gives
960.45 = 1100 + (30 41.25)an 0.0375 ,
which implies that an 0.0375 = 12.404444. Substitution of this value into
the same formula applied to the second bond yields a price for that bond
of
1150 + (F r Ci)an 0.0375 = 1150 + (70 43.125)an 0.035
= 1150 + 26.875(12.404444) = 1483.37

3. A loan of 96,000 is being repaid by a monthly payment of interest at a nominal


rate of 6.0% compounded monthly, together with a monthly payment into a sinking
fund which earns interest at the rate of 4.0% per annum, also compounded monthly.
The total monthly payment is constant, at 1,000, with the exception of the very
last payment, which could be less than 1,000, in order to bring the fund up to its
target level of 96,000; at that time the entire sinking fund will be paid over to the
lender and the loan will have been repaid. Determine when the final deposit into
the sinking fund will be made, and its amount.
Solution: The monthly interest payment to the lender will be
1
(6%) 96, 000 = 480,
12
so the monthly contribution to the sinking fund will be 1, 000 480 = 520. Regular
contributions of 520 will be made for as long as
520 sn 1 % 96, 000
3
(1.00333333)n 1
520 96, 000
0.00333333
(1.0033333)n 1.615385
n 144.1116636
i.e., until n = 144. Immediately after that payment the balance in the sinking fund
will be 520 s144 1 % = 95, 906.43598. But, by the time for the next payment, this
3
will have grown to
1.0033333 95, 906.44 = 96, 226.12
Information for Students in MATH 329 2005 01 96

so, the amount needed to bring the balance up to the target is 226.12: the lender
will receive a refund! (Note that this refund is from the sinking fund only he
still has to pay interest of 480 to the lender.)

4. A 5000 par value 18-year bond has 7% semiannual coupons, and is callable at the
end of the 12th through the 17th years at par.

(a) Find the price to yield 6% convertible semiannually.


(b) Find the price to yield 8% convertible semiannually.
(c) Find the price to yield 8% convertible semiannually, if the bond pays a pre-
mium of 250 if it is called.
(d) Find the price to yield 6% convertible semiannually, if the bond pays a pre-
mium of 250 if it is called at the end of years ##12, 13, 14, and a premium
of 150 if it is called at the end of years 15 or 16. (It may still be called at
the end of year 17 without premium, and otherwise will mature at the end of
year 18.)

Solution:

(a) F = C = 5, 000, n = 36, r = 3.5%, i = 3%. Let m be the coupon number at


whose date the bond is called or matures. Then, by the Premium/Discount
Formula

P = 5000 + (175 150)am 3% , (n = 24, 26, 28, 30, 32, 34, 36).

Without knowing which will be the date of call if any we take the
worst possible date in order to minimize the price; since am 3% is an increasing
function of m, and is multiplied by a positive number, 25, we minimize by
making m as small as possible, i.e., 2 12 = 24:

P = 5000 + (175 150)a24 3% = 5000.00 + 25 16.9355 = 5, 423.39 .

(b) When the semi-annual yield rate is 4%, the multiplier is negative, 175 200 =
25, and we must choose the largest value of m, i.e., m = 36, for a price of

P = 5000 + (175 200)a36 4% = 5000.00 25 18.9083 = 4, 527.29 .

(c) If the bond should be called at the time of any of coupons ##24, . . . , 34,
C = 5, 250 and
P = 5250 + (175 210)am 4%
Information for Students in MATH 329 2005 01 97

which is minimized when m = 34: P = 4, 605.61. We must compare this with


the price if not called,

5000 + (175 200)a36 4% = 4527.293 ,

so the price will be the latter, 4,527.29.


(d) We will have to determine the minimum of

5250 + (175 157.50)am 3% if m = 24, 26, 28 (4)


5150 + (175 154.50)am 3% if m = 30, 32 (5)
5000 + (175 150)am 4% if m = 34, 36 (6)

i.e., of

5, 250 + (175 157.51)a24 3% = 5, 250 + 17.50 16.9355 = 5546.37


5, 150 + (175 154.50)a30 3% = 5, 150 + 20.50 19.6004 = 5551.81
5, 000 + (175 150)a34 3% = 5, 000 + 25 21.1318 = 5528.30

so the price should be 5528.30.

5. (a) Construct a bond amortization schedule for a 4 year bond of face amount
5000, redeemable at 5250 with semiannual coupons, if the coupon rate is 6%
and the yield rate is 7% both converted semiannually. Use the format
Time Coupon Interest Principal Book
Value Adjustment Value
0
..
.
(b) Construct a bond amortization schedule for a 4 year bond of face amount
5000, redeemable at 5250 with semiannual coupons, if the coupon rate is 7%
and the yield rate is 6% both converted semiannually.
Solution:

(a) By the Basic Formula the purchase price of the bond will be

5, 250(1.035)8 + 150a83.5% = 3, 986.910670 + 150(6.8740) = 5, 018.01


Information for Students in MATH 329 2005 01 98

Time Coupon Interest Principal Book


Value Adjustment Value
0 5,018.01
1 150.00 175.63 -25.63 5,043.64 .
2 150.00 176.53 -26.53 5,070.17
3 150.00 177.46 -27.46 5,097.63
4 150.00 178.42 -28.42 5,126.05
5 150.00 179.41 -29.41 5,155.46
6 150.00 180.44 -30.44 5,185.90
7 150.00 181.51 -31.51 5,217.41
8 150.00 182.61 -32.61 5,250.02
(b) By the Basic Formula the purchase price of the bond will now be

5, 250(1.03)8 + 175a83% = 4, 144.398480 + 175(7.019692190) = 5, 372.84 .

Time Coupon Interest Principal Book


Value Adjustment Value
0 5,372.84
1 175.00 161.19 13.81 5,359.03
2 175.00 160.77 14.23 5,344.80
3 175.00 160.34 14.66 5,330.14
4 175.00 159.90 15.10 5,315.04
5 175.00 159.45 15.55 5,299.49
6 175.00 158.98 16.02 5,283.47
7 175.00 158.50 16.50 5,266.97
8 175.00 158.01 16.99 5,249.98

6. An 8-year bond of face value 10,000 with semiannual coupons, redeemable at par,
is purchased at a premium to yield 6% convertible semiannually.

(a) If the book value (just after the payment of the coupon) six months before the
redemption date is 10024.27, find the total amount of premium or discount in
the original purchase price.
(b) Determine the nominal annual coupon rate of the bond, compounded semi-
annually.
(c) Give the amortization table for the last 2 years.

Solution:
Information for Students in MATH 329 2005 01 99

(a) The book value just after the pnultimate10 coupon is


10, 024.27 = 10, 000v + F r a10.03
10000 + F r
= 10, 000v + F r v =
1.03
so
F r = 1.03 10, 024.27 10000 = 325 .
Knowing the amount of each coupon we can now evaluate the purchase price
of the bond to have been
10, 000(1.03)16 + 325a160.03 = 6, 231.67 + 4, 082.36
= 10, 314.03 .
The bond was purchased at a premium of 10, 314.03 10, 000 = 314.03.
325
(b) The rate per period was 10,000 = 3.25%; hence the nominal rate compounded
semi-annually, is 2 3.25% = 6.5%.
(c) For convenience we could compile this table backwards, beginning with Time=16.
We were given that B15 = 10, 024.27. Hence the Principal Adjustment con-
tained in the 16th coupon is
10, 024.27 10, 000 = 24.27.
The book value at Time=15 will be

(10, 000)(1.03)2 + 325 (1.03)1 + (1.03)2 = 10, 047.84 .
The book value at Time=14 will be
(10, 000)(1.03)3 + 325(1.031 + (1.03)2 + (1.03)3 ) = 10, 070.72 .
etc. Alternatively, we could observe by the Basic Formula that the book value
just after the payment of the 12th coupon is
10, 000(1.03)4 + 325a4 0.03 = 10, 092.93
and so we may construct the table:
Time Coupon Interest Principal Book
Value Adjustment Value
12 325.00 ... ... 10,092.93
13 325.00 302.79 22.21 10,070.72
14 325.00 302.12 22.88 10,047.84
15 325.00 301.44 23.56 10,024.28
16 325.00 300.73 24.27 10,000.01

10
preceding the last, i.e., 2nd last
Information for Students in MATH 329 2005 01 901

References to these sources are often given in the notes for completeness. Students
are not expected to look up sources, but may wish to do so out of curiosity.

The entries in this list may not be in alphabetical order. As the notes are con-
structed, new entries will be added at the end, so as not to upset the earlier
numbering of references.

15 References
[1] S. G. Kellison, The Theory of Interest, Second Edition. Irwin/McGraw Hill, Inc.,
Boston, etc. (1991). ISBN 0-256-04051-1.

[2] S. G. Kellison, The Theory of Interest. Richard D. Irwin, Inc., Homewood, Ill. (1970).
ISBN ???-083841.

[3] McGill Undergraduate Programs Calendar 2004/2005. Also accessible at


http://www.coursecalendar.mcgill.ca/Frameset.html.

[4] R. Muksian, Mathematics of Interest Rates, Insurance, Social Security, and Pen-
sions. Pearson Education, Inc., Upper Saddle River, NJ. (2003). ISBN 0-13-009425-
0.

[5] M. M. Parmenter, Theory of Interest and Life Contingencies, with Pension Appli-
cations. A Problem-Solving Approach, 3rd Edition. ACTEX Publications, Winsted
CT, (1999). ISBN 1-56698-333-9.

[6] J. Stewart, Single Variable Calculus (Early Transcendentals), Fourth Edition.


Brooks/Cole (1999). ISBN 0-534-35563-3.

[7] H. S. Hall and S. R. Knight, Higher Algebra, Fourth Edition, MacMillan & Co.
(London, 1891).

[8] The Canadian Institute of Actuaries English-French lexicon,


http://www.actuaries.ca/publications/lexicon/
Information for Students in MATH 329 2005 01 2001

A Supplementary Lecture Notes


A.1 Supplementary Notes for the Lectures of January 4th and
January 5th, 2005
Distribution Date: Tuesday, January 4th, 2005, revised Friday, January 7th, 2005
(subject to further revision)

A.1.1 These notes


These notes will contain some of the material that I propose to discuss in the lectures, and
also some material that will not make it to the lectures. I will be following the textbook
very closely, sometimes explaining statements that I find require some elaboration. In
the first chapter of the book my notes will be detailed; I do not know yet whether I will
be able to continue this detailed a set of notes for subsequent chapters. Much of the
class time will be spent in discussing problems from the textbook, for which I will be
including sketches of solutions wherever possible.

Textbook Chapter 1. The measurement of interest.

A.1.2 1.1 INTRODUCTION


While some of the concepts in this course may be applied in non-financial contexts, the
language of the course involves growth of amounts of money under the passage of time.
Most of our discussions will concern one or two specific ways in which amounts of money
are, in practice, assumed to grow; but, initially, we will define very general concepts.

Definition A.1 1. Principal is the initial amount of money borrowed, lent, or in-
vested.

2. Interest is the compensation paid by a borrower of capital to the lender for the use
of the capital.

3. Interest is said to be earned by the lender, or to accrue to the lender.

4. A distinction may be made between when and how interest is earned , and when it
is paid or credited to the lender. When the word interest is used, without further
elaboration, payment is normally at the end of each time period. When interest
is paid at the beginning of a time period it is usually called discount; however, the
compensation itself may still be called interest
Information for Students in MATH 329 2005 01 2002

A.1.3 1.2 THE ACCUMULATION AND AMOUNT FUNCTIONS


Definition A.2 The growth of a given, specific amount of principal as a function of
time, t is often denoted by the amount function, denoted by A(t).

We will always assume that the rate of growth does not depend on the amount of money
that is invested.11 This scalability assumption permits us to normalize many of our
discussions, by speaking about the growth of a fund of 1, and then just scaling the
resulting figures by multiplying by the correct initial or terminal value of the fund. We
will take this as our first

Axiom A.1 Invested amounts are normally scalable.

Under this postulate it is often convenient to study amounts that are normalized to
have value 1 at some convenient time. We will sometimes design our notation around
this postulate, by defining pairs of functions with similar names, one for the actual size
of the fund, the other for the normalized account. The first example of this is the
normalized amount function, which we call the accumulation function.

Definition A.3 Corresponding to a specific amount function, which has value A(0) at
time t = 0, we define a normalized amount function or accumulation function a by
A(t) = A(0) a(t) or
A(t)
a(t) = . (7)
A(0)
A simple consequence of this equation is

Theorem A.1 a(0) = 1.

(Most mathematicians dont like to use the word theorem for results as trivial as this;
we would be more likely to call it a proposition or use some other term that reserves
the word theorem for more serious results.)
In practice certain types of amount functions would be absurd: for example, we
usually wouldnt normally want to permit funds to get smaller with increasing time,
although there are situations where such shrinkage could be given some sense. But, for
the present, we will follow the textbook in assuming
11
This is an assumption that is not completely realistic, since, in the real financial world, a person
who has, say $1,000,000 to invest can often obtain a higher interest rate on his investment than the
person who has only $100. But this model is used throughout the textbook, and we will follow it in the
course.
Information for Students in MATH 329 2005 01 2003

Axiom A.2 The function a is non-decreasing; i.e.,


t1 < t2 a(t1 ) a(t2 ) .
(The textbook uses the word increasing, but many mathematicians use that word in a
slightly restrictive way. Here I want to permit the amount to stay unchanged for a time;
I am assuming only that it cannot get smaller as time increases.)
The third axiom of the textbook [1, p. 2], concerning continuity of the function a(t) is
not clear. It appears to say, tautologically, that a(t) is continuous if a(t) is continuous.
I will ignore it until I know what the author means.
Definition A.4 1. The nth period of time is defined to be the period of time between
t = n 1 and t = n. More precisely, the period normally will consist of the time
interval n 1 < t n.
2. Where n is a non-negative integer, the interest earned by the amount A(t) in the
nth period of time is denoted by In , and defined by
In = A(n) A(n 1) . (8)
We may relate this notation to a standard convention in the finite difference calculus,
which would define the increment in A(n) by
4A(n) = A(n + 1) A(n) .
Thus In = 4A(n 1).12
Example A.2 The textbook [1, p. 3] gives graphical examples of 4 kinds of amount
functions.
1. The first has a straight line graph; the graph is sloping upward because the deriv-
ative of the function is positive, in order that the function should be increasing at
a constant rate. In this case the fund is earning a fixed amount which is propor-
tional to the time that has elapsed, even if the time is not an integer number of
time-units; this is the situation we will later call simple interest.
2. The second example is an exponential curve, where A(t) is a function of the form
A(t) = k e`t , (9)
` being a real number. If we set t = 0 in (9), we see that k = A(0). Since the
function must be non-decreasing, we know by the calculus that ` 0. This is the
case that we will be calling compound interest.
12
Note that the name of the function is 2 characters long: 4A. In situations like this, where the
reader might be at risk of not knowing where the name of the function begins or ends, one might use
parentheses, and write (4A)(n 1).
Information for Students in MATH 329 2005 01 2004

3. The third case has a graph which is a horizontal line. This is simply the special
case of the preceding where ` = 0, and A(t) is constant; the amount neither grows
nor shrinks.

4. The fourth case is a step function, whose graph is horizontal line segments of unit
length, each a fixed distance above the preceding one. This function is piecewise
continuous, having points of discontinuity at the integer times. This is a realistic
model, for example, where an amount earns interest only if the amount remains in
the account for a full time period13 , and where the interest is credited at the end
of the period.
To be more precise, the function is continuous from the right everywhere, but it
is discontinuous from the left at integer points; this is because the limit from the
left is not equal to the function value there i.e., because the value you would
infer from the size of the amount preceding an integer time is not equal to the
actual size of the account at integer time.

[1, Exercise 1, p. 29] Consider the amount function A(t) = t2 + 2t + 3.

1. Find the corresponding accumulation function a(t).


2. (modified) Show that a(0) = 1 and that a is a non-decreasing function of time.
3. Find In .

Solution:

1.
A(t) t2 + 2t + 3 1 2
a(t) = = 2 = t2 + t + 1 .
A(0) 0 + 2(0) + 3 3 3
2. (a) a(0) = 13 02 + 23 0 + 1 = 1
(b) If the author intends that we consider this a function of a real variable
that may take any values on the non-negative real axis, then one way to
prove the function is non-decreasing is to show that the derivative is not
negative. (This can be used only for a function that is differentiable, but
the present function is a polynomial, and polynomials have that property.)
1 2 2
a0 (t) = 2 t + 1 = (t + 1) .
3 3 3
We see that a(t) is a non-decreasing function provided t 1.
13
In the language of the textbook [1, p. 6] interest is accrued only for completed periods, with no
credit for fractional periods.
Information for Students in MATH 329 2005 01 2005

This could also have been approached from first principles. Suppose
that t1 < t2 . Then
1 2 2
a(t2 ) a(t1 ) = t2 t21 + (t2 t1 )
3 3
t2 t1 2
= (t2 t1 ) + +
3 3 3
and both factors are positive for non-negative t2 and t1 and t1 < t2 .
3. In = A(n) A(n 1) = (n2 + 2n + 3) ((n 1)2 + 2(n 1) + 3) = 2n + 1.
[1, Exercise 2, p. 30] 1. Prove that A(n) A(0) = I1 + I2 + . . . + In .
2. Verify verbally the result obtained above.
Solution:
1. Summing the equations A(m) A(m 1) = Im for 1 m n, we obtain
n
X
A(n) A(0) = Im .
m=1

2. Verbally, the (amount of) interest earned over the concatenation of n periods
is the sum of the interest earned in each of the periods separately.
[1, Exercise 3, p. 30] Find the amount of interest earned between time t and time
n, where t < n, if
1. Ir = r
2. Ir = 2r .
Solution: In the preceding problem we showed that the amount earned between
time 0 and time n was the sum of the values of I. The interest earned between
time t and time n will be the total interest from time 0 to time n diminished by
the interest earned from time 0 to time t, so it must be
n
X t
X n
X
Im Im = Im
m=1 m=1 m=t+1

When Im = m, this is14


n
X t
X n(n + 1) t(t + 1) (n t)(n + t + 1)
m m= = .
m=1 m=1
2 2 2
14
using the familiar formula for the sum of the first natural numbers
Information for Students in MATH 329 2005 01 2006

When Im = 2m , the sum is


n
X
m t+1 2nt 1
2 =2 = 2n+1 2t+1 .
m=t+1
21
Information for Students in MATH 329 2005 01 2007

A.2 Supplementary Notes for the Lecture of January 7th, 2005


Distribution Date: Friday, January 7th, 2005, subject to further revision

A.2.1 1.2 THE ACCUMULATION AND AMOUNT FUNCTIONS (con-


clusion)
Summation of Geometric Progressions and Series (These results were applied
in the solution of [1, Exercise 3, p. 30], shown in the notes for the preceding lecture.)
The reader is reminded of the easily derived formula [1, Appendix III, A.2, p. 394]
n1
X
k rn 1 1 rn
ar = a =a (r 6= 1) . (10)
k=0
r1 1r

The textbook overlooks the essential restriction that r 6= 1 for the use of this formula;
when r = 1 the sum is
n1
X
a1k = a n (r 6= 1) . (11)
k=0

We shall be using these formul repeatedly. We shall also be interested in the behavior
of (10) in the limit as n :

X
k 1
ar = a (|r| < 1) . (12)
k=0
1r

(When |r| 1, the limit does not exist.)

[1, Exercise 4, p. 30] It is known that a(t) is of the form ct2 + b. If $100 invested
at time 0 accumulates to $172 at time 3, find the accumulated value at time 10 of
$100 invested at time 5.
Solution: Lets begin by normalizing the function: a(0) = 1 c02 +b = 1, so b = 1.
72
The given growth condition implies that 100(c32 + 1) = 172, so c = 900 = 0.08.
Consequently 100 invested at time 5 grows to 100((0.08) 52 + 1) =300.

A.2.2 1.3 THE EFFECTIVE RATE OF INTEREST


When we speak of a rate of interest, we mean the amount of interest per unit investment.
As this rate is normalized, we follow the convention we began with the pair A and a, by
using a lower case letter to represent it.
Information for Students in MATH 329 2005 01 2008

Definition A.5 The effective rate of interest during the nth period, in , is the ratio of
the amount of interest earned during, and paid at the end of the period to the amount
of principal invested at the beginning of the period. When the effective rate is constant
over all unit time intervals, we may suppress the subscript, and simply write i.
We may use the term effective for periods of lengths other than one unit; but the
most common use will be for terms of one year, where we will speak of the effective
annual rate.

Note that, when we use the unqualified word interest alone, the listener may not know
whether we are referring to the amount of interest, the rate of interest, or the general
concept of payment for the use of capital. There may also be doubts about the accrual
period.

Theorem A.3 In symbols,

In A(n) A(n 1) a(n) a(n 1)


in = = = (13)
A(n 1) A(n 1) a(n 1)

for integer n 1. And, when the rate is constant, i, recalling that a(0) = 1, we have

a(n) = (1 + i)a(n 1) = (1 + i)2 a(n 2) = . . . = (1 + i)n a(0) = (1 + i)n . (14)

In all of this discussion the interest rate is associated with one time interval; this will be
contrasted later with rates called nominal that are stated for one time interval,
but need to be applied to another.
Information for Students in MATH 329 2005 01 2009

A.3 Supplementary Notes for the Lecture of January 10th,


2005
Distribution Date: Monday, January 10th, 2005, subject to further revision

A.3.1 1.3 THE EFFECTIVE RATE OF INTEREST (conclusion)


[1, Exercise 5, p. 30] Assume that A(t) = 100 + 5t. Find i5 and i10 .
Solution:
A(5) A(4) 51 1
i5 = = = .
A(4) 100 + 5 4 24
A(10) A(9) 51 1
i10 = = = .
A(9) 100 + 5 9 29

[1, Exercise 6, p. 30] Assume A(t) = 100(1.1)t . Find i5 and i10 .


Solution:
A(5) A(4) 100(1.1)4 (0.1)
i5 = = = 0.1 .
A(4) 100(1.1)4
A(10) A(9) 100(1.1)9 (0.1)
i10 = = = 0.1 .
A(9) 100(1.1)9

[1, Exercise 7, p. 30] Show that A(n) = (1 + in ) A(n 1).



Solution: (1 + in )A(n 1) = 1 + A(n)A(n1)
A(n1)
A(n 1) = A(n)
A(n1)
A(n 1) =
A(n).
A verbal solution is also feasible.

[1, Exercise 8, p. 30] If A(4) = 1000 and in = 0.01n, find A(7).


Solution: I(n) = in A(n 1) = (0.01)nA(n 1). It follows that

A(n) = In + A(n 1) = (0.01(n) + 1)A(n 1)


A(7) = (0.01(7) + 1)A(6)
= (0.01(7) + 1)(0.01(6) + 1)A(5)
= (0.01(7) + 1)(0.01(6) + 1)(0.01(5) + 1)A(4) = (1.07)(1.06)(1.05)1000
= 1190.91 .
Information for Students in MATH 329 2005 01 2010

A.3.2 1.4 SIMPLE INTEREST


We saw in the preceding discussion that a(0) = 1 and a(1) = 1 + i1 . Under simple
interest we assume the linear function we saw in the first graphical example above: the
amount of interest earned in a time interval of length t and starting at a fixed time, like
t = 0, is assumed to be proportional to t; we obtain the formula
a(t) = 1 + i1 t
over the 1st time interval, or any longer interval where the so-called simple interest rate
remains equal to i1 . Suppose that the rate of interest is i, and that this rate is to remain
in effect over an interval of n or more successive time periods without adjusting the base
principal from which it is computed. We can determine the rate of interest during the
nth period under this assumption; it is
a(n) a(n 1) [1 + i1 n] [1 + i1 (n 1)] i1
in = = = (15)
a(n) 1 + i1 (n 1) 1 + i1 (n 1)
where we see that, even though the amount of interest earned in any period of time is
a constant multiple of the length of the period, the rate of interest per period is not
constant it is decreasing from each period to the next.15 Thus a constant rate of
simple interest implies that the effective rates of interest for the successive time intervals
form a decreasing sequence. What is constant for simple interest is the absolute amount
of interest earned in each time interval; under compound interest which we shall meet
in the next section, it is the relative amount of interest that is constant that is, the
ratio of interest earned to principal.
Note that, when we speak of simple interest, we must specify not only the rate, but
also give enough information so that the reader will know when the principal amount is
adjusted to incorporate the accrued interest.

A rigorous mathematical derivation of simple interest I modify here a deriva-


tion in the textbook. The object is to show that the formula assumed above for simple
interest is a consequence of a reasonable, general assumption.
Axiom A.3 (Defining postulate of simple interest) Under simple interest the rate
of interest in all time periods of length t + s is the same; and it is equal to the sum of
the interest rates for periods of lengths t and s. Symbolically,
a(t + s) a(0) = [a(t) a(0)] + [a(s) a(0)] , (16)
or, more compactly,
a(t + s) a(0) = a(t) + a(s) 2a(0) , (17)
for all non-negative real numbers t and s.
15
But the derivative of the amount function will be constant.
Information for Students in MATH 329 2005 01 2011

Note that the axiom assumes the rule is to hold for periods of any non-negative length,
not just of integer length. The textbook then attempts16 to show that this hypothesis
implies that
d d
a(t) = a(0) .
dt dt
Thus the time derivative of a(t) is shown to be constant. We know from elementary
calculus17 that a(t) must have the form

a(t) = a0 (0) t + C

where C is a constant; and we can determine that constant by assigning to t the particular
value 0, so that
C = a(0) = 1
and
a(t) = 1 + a0 (0) t .
If, in this last equation, we assign t = 1, we find that

a0 (0) = a(1) a(0)

which is what was defined to be i1 , so

a(t) = 1 + i1 t for t 0 . (18)

The textbook writes i in place of i1 in the preceding equation, but this is confusing,
since we saw in equation (15) above that the rate of interest is not constant under simple
interest.

[1, Exercise 9, p. 30] 1. At what rate of simple interest will $500 accumulate to
$615 in 2 12 years?
2. In how many years will $500 accumulate to $630 at 7.8% simple interest?
Solution:

1. Under simple interest at a rate i, 615 = 500a(2.5) = 500(1 + i2.5); solving for
615
1
i, we obtain i = 5002.5 = 0.092 = 9.2%.
16
To a mathematician the proof appears slightly defective, although it can be repaired: the definition
of derivative requires a 2-sided limit, and the textbook only considers a right derivative. The same
idea used in the proof can be used to supply the needed result for the limit from the left. You are not
expected to be concerned with this degree of subtlety in this course.
17
The Mean Value Theorem
Information for Students in MATH 329 2005 01 2012

2. Under a rate of 7.8% simple interest for a period of t years, 630 = 500a(t) =
630
1
500(1 + 0.078t); solving for t, we obtain t = 500
0.078
= 10
3
years.

[1, Exercise 10, p. 30] If ik is the rate of simple interest for period k, where k =
1, 2, . . . , n, show that a(n) a(0) = i1 + i2 + . . . + in .
Solution: Note that there is an implicit assumption that the earned interest is not
incorporated into the principle during the period 0 t n. What is denoted by
it in this problem is not the effective rate of interest for the tth period, shown in
equation (1.6). Rather, it is the total amount earned during the tth period by the
amount that began as 1 at time t = 0.
Under simple interest an initial investment of a(0) = 1 grows by the prescribed
amounts in each of the following years: a(n) = a(0)+i1 +i2 +. . .+in , so a(n)a(0) =
i1 + i2 + . . . + in .

[1, Exercise 11, p. 30] At a certain rate of simple interest $1000 will accumulate to
$1100 after a certain period of time. Find the accumulated value of $500 at a rate
of simple interest three fourths as great over twice as long a period of time.
Solution: It is known that, at the rate of interest i, and for a prescribed
length
of
3i 3
time t, 1000(1 + it) = 1110, so it = 0.11. Then 500 1 + 4 2t = 500 1 + 2 it =
500(1 + 0.165) = 582.50.

[1, Exercise 12, p. 30] Simple interest of i = 4% is being credited to a fund. In


which period is this equivalent to an effective rate of 2 21 %?
Solution: We have to solve for n the equation
0.04
0.025 = in =
1 + (0.04)(n 1)
and find that n = 16.

A.3.3 1.5 COMPOUND INTEREST


Under compound interest, the earned interest is incorporated into the principal after each
accrual period, following which the interest in the next period is based on the updated
principal. Thus the earned interest is reinvested together with the original principal. This
is what we observed in equation (14) above, where the interest rate remained constant
over successive time periods.
The growth of the principal can be analyzed in various ways. Thus, for example, the
value a(2) = (1 + i)2 a(0) = (1 + i)2 may be interpreted
as (1 + i)a(1) = (1 + i) (1 + i) as we did in the discussion preceding (14)
Information for Students in MATH 329 2005 01 2013

as 1+2i+i2 , where the principal of 1 is augmented by 2i, representing the simple


interest earned by the principal over two successive time periods, augmented by
the interest at rate i earned by the interest i from the first period and credited at
the end of that period.

Under compound interest at an effective interest rate of i we assume that an amount of 1


grows to (1+i)t during a time interval of length t, where t is not necessarily a non-negative
integer . In most of the sequel it is compound interest that will be assumed to be acting.
Where we assume simple interest, it will often be for fractions of a time period. In some
ways this assumption is obsolete now a relic of the days when exact computing was
difficult because of the unavailability of reliable and inexpensive calculators. However,
some of the practices of using simple interest have become entrenched in the financial
system, and so we are obliged to continue using them.

A rigorous mathematical derivation of compound interest (This is parallel


to the derivation given earlier for simple interest; again I modify here a derivation in the
textbook.)

Axiom A.4 ((Defining postulate of compound interest) Under compound interest


a(t + s)
the ratio is constant for all s, as a function of t; and the product of these ratios
a(t)
for times t1 and t2 is equal to the ratio for time t1 + t2 . Symbolically,

a(t1 + t2 ) a(t1 ) a(t2 )


= (19)
a(0) a(0) a(0)

or, more compactly,


a(t1 + t2 ) = a(t1 ) a(t2 ) , (20)
for all non-negative real numbers t1 and t2 .

The textbook then infers that


a0 (t)
= a0 (0),
a(t)
again a constant for all t. This equation may be solved in a standard way. The fraction
on the left is the derivative of ln a(t), so

ln a(t) = a0 (0) t + C for all t. (21)

Since a(0) = 1, setting t = 0 yields C = 0, so


0
a(t) = ea (t) . (22)
Information for Students in MATH 329 2005 01 2014

Setting t = 1, and recalling that

a(1) a(0) = i1 ,

yields
a0 (0) = ln(1 + i1 ) (23)
so
a(t) = (1 + i1 )t . (24)
Here we can show that in is constant for all n, so we can legitimately suppress the
subscripts and write simply i:
a(t) = (1 + i)t . (25)
We thus see the graphical distinction between simple and compound interest: the graph
of an amount function under simple interest is a straight line a linear function;
the graph of an amount function under compound interest is an exponential function.
Some of the variations we will consider will be piecewise linear, where the graph is
made up of line segments attached end-to-end, where the points of attachment lie on an
exponential curve.

[1, Exercise 13, p. 30] (This solution is not examination material, but you should
remember the result.)
Assuming that 0 < i < 1, show that:

1. (1 + i)t < 1 + it if 0 < t < 1;


2. (1 + i)t = 1 + it if t = 1;
3. (1 + i)t > 1 + it if t > 1.

This exercise verifies the relative magnitudes of accumulated values at simple and
compound interest over various periods of time.
Solution: This problem supplies the proof of a statement made in the text: that
compound interest provides a greater return than simple interest for periods greater
than 1; it also proves the complementary statement, that compound interest pro-
vides a lower return than simple interest for periods shorter than 1. Observe, for
the function f (t) = (1 + i)t (1 + it),

f 0 (t) = (1 + i)t ln(1 + i) i (26)


f 00 (t) = (1 + i)t (ln(1 + i))2 (27)
f (0) = f (1) = 0 (28)
Information for Students in MATH 329 2005 01 2015

From (27) we see that, at any critical point t,

f 00 (t) > 0 : (29)

thus any critical point is a local minimum for f . Hence, on the closed interval
0 t 1, f attains its global maximum only at one or both of the end points, at
both of which, by (28), it is 0. This proves that

(1 + i)t < 1 + it if 0 < t < 1

which is part (a). Equation (28) proves part (b).


Finally, consider the case t > 1. Define g(x) = (1 + x) ln(1 + x) x, observing that

g 0 (x) = ln(1 + x) (30)


g(0) = 0 , (31)
f 0 (t) > f 0 (1) = g(i) . (32)

Suppose that there existed a point a > 0 where g(a) 0. Then, by the Mean
Value Theorem, there would exist a point c such that 0 < c < a and
g(a) g(0) 0
0 < ln(1 + c) = g 0 (c) = = 0,
a a
which is a contradiction. It follows that g(x) > 0 for x > 0, and f 0 (t) > 0 for all
t > 1. Suppose that it happened that f (b) 0 for b > 1, and apply the Mean
Value Theorem to f (t) on the interval 1 t b. There would exist a point k,
1 < k < b, such that
f (b) f (1)
f 0 (k) = < 0,
b1
contradicting (32). We conclude that f (t) > 0 for t > 1, which implies (c).

[1, Exercise 14, p. 30] It is known that $600 invested for two years will earn $264
in interest. Find the accumulated value of $2000 invested at the same rate of
compound interest for three years.
Solution: What is known is that, at the given interest rate i, 600(1+i)2 = 600+264,
so (1 + i)2 = 1.44 and (1 + i)1 = 1.2 (i.e. i = 20%). At the same rate, 2000(1 + i)3 =
2000(1.2)3 = 2000 1.728 = 3456.

[1, Exercise 15, p. 31] Show that the ratio of the accumulated value of 1 invested at
rate i for n periods, to the accumulated value of 1 invested at rate j for n periods,
i > j, is equal to the accumulated value of 1 invested for n periods at rate r. Find
an expression for r as a function of i and j.
Information for Students in MATH 329 2005 01 2016

Solution: n
1(1 + i)n ij
= 1+ .
1(1 + j)n 1+j
ij
Thus r = .
1+j
[1, Exercise 16, p. 31] At a certain rate of compound interest, 1 will increase to 2 in
a years, 2 will increase to 3 in b years, and 3 will increase to 14 in c years. If 6 will
increase to 10 in n years, express n as a function of a, b, and c.
Solution: If the common rate is i, the hypotheses are that

1(1 + i)a = 2
2(1 + i)b = 3
3(1 + i)c = 15
6(1 + i)n = 6.

Taking logarithms and solving, we obtain


ln 2
a = (33)
ln(1 + i)
ln 3 ln 2
b = (34)
ln(1 + i)
ln 5
c = (35)
ln(1 + i)
ln 5 ln 3
n =
ln(1 + i)
= cab (36)

Note that this answer is overdetermined, in the sense that a, b and c are not
independent: by equations (33), (34), (35), a : b : c :: ln 2 : ln 32 : ln 5, that is, a, b,
and c are related by proportionality equations of the form
a b c
= 3 = .
ln 2 ln 2 ln 5

Put another way, we didnt need all 3 of these equations to solve the problem
any 2 of them would have been sufficient!

[1, Exercise 17 p. 31] An amount of money is invested for one year at a rate of
interest of 3% per quarter. Let D(i) be the difference between the amount of
Information for Students in MATH 329 2005 01 2017

interest earned on a compound interest basis, and on a simple interest basis for
quarter k, where k = 1, 2, 3, 4. Find the ratio of D(4) to D(3).
Solution: Let the amount of money invested be x. The interest rate is 3% per
quarter . We have

D(1) = x (1.03)0 ((1.03) 1) 0.03

D(2) = x (1.03)1 ((1.03) 1) 0.03

D(3) = x (1.03)2 ((1.03) 1) 0.03

D(4) = x (1.03)3 ((1.03) 1) 0.03

from which it follows that


D(4) (1.03)3 ((1.03) 1) 0.03 1.033 1
= = = 1.522610837 .
D(3) (1.03)2 ((1.03) 1) 0.03 1.032 1
Information for Students in MATH 329 2005 01 2018

A.4 Supplementary Notes for the Lecture of January 12th,


2005
Distribution Date: Thursday, January 13th, 2005, subject to revision

A.4.1 1.6 PRESENT VALUE


Definition A.6 1. Under an interest rate of i the sum 1+i is called the accumulation
factor which, when applied to an amount, yields the value of the amount at the
end of 1 time period.

2. When we view an amount from the past, asking what should be todays value of
an amount receivable some time in the future, we call the value today the present
or current value or the value discounted (back) to the present.

3. To obtain the amount which, at the beginning of a time period, would yield, at the
interest rate i, an amount of 1 at the end of the period, we may divide by 1 + i or
1
multiply by the discount factor , which is often denoted by v.
1+i
1
The textbook denotes the reciprocal by a1 (t). This is an unfortunate notation,
a(t)
since it appears to involve an inverse function, and that is not the authors intention.
1
You are advised not to use this notation: write or (a(t))1 .
a(t)

Definition A.7 Analogous to the accumulation function a(t), we can call (a(t))1 the
discount function. It represents the amount that needs to be invested today to yield an
amount of 1 at the end of t time periods.

In the following problems there will be a gradual transition from the general formulation
of the preceding definitions to the specific case of compound interest.

[1, Exercise 18 p. 31] Find an expression for the discount factor during the nth pe-
riod from the date of investment, i.e., (1 + in )1 , in terms of the amount function.
1 A(n1)
Solution: 1+in
= A(n)
.

[1, Exercise 19 p. 31] The sum of the present value of 1 paid at the end of n periods
and 1 paid at the end of 2n periods is 1. Find (1 + i)2n .
Solution:
2
(1 + i)n + (1 + i)2n = 1 (1 + i)n + (1 + i)n 1 = 0
Information for Students in MATH 329 2005 01 2019

2
n1 5
(1 + i) + =
2 4
(completing the square)

n 1 5
(1 + i) =
2
But the negative sign would be associated with a negative value of (1 + i)n , which
is impossible. Hence

n 1 + 5
(1 + i) =
2
3 5
(1 + i)2n = 1 (1 + i)n =
2
2n 2 2 3+ 5
(1 + i) = =
3 5 3 5 3+ 5

2(3 + 5) 3+ 5
= =
95 2

[1, Exercise 20, p. 31] Show that the current value of a payment of 1 made n periods
ago and a payment of 1 to be made n periods in the future is greater than 2, if
i > 0.
Solution: The payment from the past is currently worth (1 + i)n , while the future
payment is currently (=presently) worth v n = (1 + i)n . The excess of this sum
over 2 is n n 2
(1 + i)n + (1 + i)n 2 = (1 + i) 2 (1 + i) 2 , (37)
which, being a square, cannot be negative. Hence

(1 + i)n + (1 + i)n 2 ,
n n
which equality holding precisely when (1+i) 2 = (1+i) 2 , i.e., when (1+i)n 1 = 0,
i.e. if either i = 0 or n = 0.

[1, Exercise 21 p. 31] It is known that an investment of $500 will increase to $4000
at the end of 30 years. Find the sum of the present values of three payments of
$10,000 each which will occur at the end of 20, 40, and 60 years.
Solution: At the rate i, 500(1 + i)30 = 4000, so (1 + i)30 = 8, and (1 + i)10 = 2.
Hence the present value of payments of 10,000 to occur at the ends of 20, 40, and
Information for Students in MATH 329 2005 01 2020

60 years is

20 40 60
1 1 1
10000 v +v +v = 10000 + +
4 16 64
210000
= = 3281.25.
64
Information for Students in MATH 329 2005 01 2021

A.5 Supplementary Notes for the Lecture of January 14th,


2005
Distribution Date: Friday, January 14th, 2005, subject to revision

A.5.1 1.7 THE EFFECTIVE RATE OF DISCOUNT


We will try to develop, in parallel to the interpretation of amount and accumulation
functions from the point of view of the addition of interest, paid at the ends of time
periods, a discount interpretation under which the amount and accumulation functions
are discounted by subtracting an amount of discount. This will lead to several symbols
analogous to those in the interest model. This parallel development will be incomplete,
and most of the time we will be working only with the interest model. For example,
we could interpret In given in equation (8) as an amount of discount. Then we could
parallel equation (13) with
In A(n) A(n 1) a(n) a(n 1)
= = (38)
A(n) A(n) a(n)
Definition A.8 The common amount of the members of equation (38)is denoted by dn ,
and called the effective rate of discount.
Note the differences between the two models:
Under the interest model, the payment for the use of the money is made at the
end of the period, based on the balance at the beginning of the period.
Under the discount model, the payment is deducted at the beginning of the
period from the final amount which will be present at the end of the period.
We will use the word equivalent to describe schemes of interest and/or discount that
produce the same ultimate accumulated value from an initially invested amount. In the
present application, which is only the first where we will meet this concept, we can, for
example, equate
1
=1d
1+i
from which we obtain various other relationships, e.g.,
i
d = (39)
1+i
d
i = (40)
1d
d = 1v (41)
i d = id (42)
Information for Students in MATH 329 2005 01 2022

You should try to explain each of these identities verbally, i.e., in words, in a way that
a person lacking your technical knowledge can find the equation plausible.
If a discount rate d is applied t times to a final amount of 1, the current value will be
1
= v t = (1 d)t .
a(t)
The textbook makes several observations about the use of the discount model, and about
the term discount. For these and other reasons, we shall not be devoting equal time to
the discount model in this course. But you should be aware that most of what we do
could be redeveloped from the discount point of view.
Definition A.9 The accumulation function for simple discount at a discount rate d 0
1 1
is given by a(t) = , for t < .
1 dt d
Note that, unlike the situation for simple interest, we must restrict the length of time
over which we propose to apply simple discount.
Definition A.10 The accumulation function for compound discount at a discount rate
1
d is given by a(t) = .
(1 d)t
While we need not restrict t in the compound discount case, we must restrict the effective
discount rate d, requiring that 0 d < 1.
[1, Exercise 22 p. 31] The amount of interest earned on A for one year is $336, while
the equivalent amount of discount is $300. Find A.
Solution: Denote the equivalent rates of interest and discount by i and d respec-
tively. The hypotheses are that

iA = 336
dA = 300

subject to the relationship between i and d, which may be expressed in various


ways. We wish to obtain A, not i or d. So let us solve the preceding equations for
i and d in terms of A,
336 330
i= d=
A A
and substitute into one of the equations relating i and d, e.g., into i d = id:
336 300 336 300
= 36A = 336(300)
A A A2
A = 2800
Information for Students in MATH 329 2005 01 2023

[1, Exercise 23 p. 31] 1. Find d5 if the rate of simple interest is 10%.


2. Find d5 if the rate of simple discount is 10%.
Solution:

1. Under simple interest at 10%, In = 0.1 A(0) for all non-negative integers n.
So
I5 (0.1)A(0) 0.1 1
d5 = = = = .
A(5) (1 + 5(0.1))A(0) 1.5 15
A(0)
2. But, if the rate of simple discount is 10%, then A(n) = .
1 0.01n
1 1
A(5) A(4) 1
d5 = = 0.05 0.06 = .
A(5) 1 6
0.5
[1, Exercise 24 p. 31] 1. Assuming compound discount, show that dn is constant
for all n.
2. Assuming simple discount, show that dn is increasing for increasing n if 0 <
1
n 1 < .
d
Solution:

1. Under compound discount at a rate d of discount, A(n 1) = (1 d)A(n), so


A(n) A(n 1)
dn = = d for all n. (This result is the discount analogue of
A(n)
the equation at [1, p. 8, l. 5].)
2. Under simple discount at a rate d of discount, A(0) = (1 dn) A(n). Hence
1 1

A(n) A(n 1) 1 dn 1 d(n 1) 1
dn = = = .
A(n) 1 1
(n 1)
1 dn d
So long as d1 > n 1; thus dn is an increasing function of d in this interval. (If
the inequality were to fail the principal would have been depleted as discount.
This result is the discount analogue of equation [1, (1.6), p. 5].)

[1, Exercise 25 p. 31] (As I stated in [1, Exercise 13, p. 30], this solution also is not
examination material, but you should remember the result.) Assuming that 0 <
d < 1, show that:
Information for Students in MATH 329 2005 01 2024

1. (1 d)t < 1 dt if 0 < t < 1;


2. (1 d)t = 1 dt if t = 1;
3. (1 d)t > 1 dt if t > 1.
This exercise verifies the relative magnitudes of present values at simple and
compound discount over various periods of time.
Solution: (We adapt the solution given above to [1, Exercise 16, p. 31] on 2016 of
these notes.) Observe, for the function f (t) = (1 d)t (1 dt),
f 0 (t) = (1 d)t ln(1 d) + d (43)
f 00 (t) = (1 d)t (ln(1 d))2 (44)
f (0) = f (1) = 0 (45)
From (44) we see that, at any critical point t,
f 00 (t) > 0 : (46)
thus any critical point is a local minimum for f . Hence, on the closed interval
0 t 1, f attains its global maximum at one or both of the end points, at both
of which, by (45) it is 0. This proves that
(1 d)t < 1 dt if 0 < t < 1
which is part (a). Equation (45) proves part (b).
Finally, consider the case t > 1. Define g(x) = (1 x) ln(1 x) + x, observing that
g 0 (x) = ln(1 x) (47)
g(0) = 0 , (48)
f 0 (t) > f 0 (1) = g(d) . (49)
Suppose that there existed a point a > 0 where g(a) 0. Then, by the Mean
Value Theorem, there would exist a point c such that 0 < c < a and
g(a) g(0) 0
0 < ln(1 c) = g 0 (c) = = 0,
a a
which is a contradiction. It follows that g(x) > 0 for x > 0, and f 0 (t) > 0 for all
t > 1. Suppose that it happened that f (b) 0 for b > 1, and apply the Mean
Value Theorem to f (t) on the interval 1 t b. There would exist a point k,
1 < k < b, such that
f (b) f (1)
f 0 (k) = < 0,
b1
contradicting (32). We conclude that f (t) > 0 for t > 1, which implies (c).
Information for Students in MATH 329 2005 01 2025

[1, Exercise 26 p. 32] If i and d are equivalent rates of simple interest and simple
discount over t periods, show that i d = idt.
Solution: The discounted value (at simple interest) at time 0 of 1 at time t is 1dt.
At simple interest this accumulates to (1 dt)(1 + it) = 1. Expanding and dividing
by t yields
i d = idt .

[1, Exercise 27 p. 32] Show that

d3 (i d)2
= .
(1 d)2 1v

Solution:

1. Algebraic proof: In an algebraic proof of an identity we can, in the most


uninspired proof, prove both members of the alleged equation are equal to
the same quantity. Usually many possible proofs exist, but some may be
more interesting than others, because they may be easier to explain in words,
i.e., verbally. Such proofs are more satisfying if we can start with the quantity
on one side of the equal sign, and, by applying familiar results, transform it
into the quantity on the other side. For example
2
d3 1
=d
(1 d)2 1d
= di2 [1, (1.14), p. 14]
(id)2 (i d)2
= =
d 1v
[1, (1.17), p. 15] .

2. Verbal proof: HOPEFULLY TO BE SUPPLIED

A.5.2 1.8 NOMINAL RATES OF INTEREST AND DISCOUNT (barely


begun
Information for Students in MATH 329 2005 01 2026

A.6 Supplementary Notes for the Lecture of January 17th,


2005
Distribution Date: Monday, January 17th, 2005, subject to revision

A.6.1 1.8 NOMINAL RATES OF INTEREST AND DISCOUNT (contin-


ued)
Definition A.11 [1, p. 14] Two rates of interest or discount are equivalent if an amount
of principal invested for the same length of time under either of the rates accumulates
to the same value.
We have already applied this concept in the preceding section, when we related equiv-
alent rates of interest and discount over the same time interval. In this section we will
relate equivalent rates of interest and/or discount over different time intervals, where one
interval is a multiple of the other.
When we speak of a nominal rate of interest, we will normally describe a payment
scheme that is more or less frequent than the usual payment interval; the interest rate
that we will describe will represent the total of all payments made over that usual
interval. Summing the payments does not take into account the interest or discount to
shift all payments to one specific time. We will determine relationships between nominal
rates and effective rates. The definitions are particularly unintuitive, so they must be
memorized.
Definition A.12 Let m be either a positive integer or the reciprocal of a positive integer.
1. A nominal rate of interest i(m) payable m times per period represents m times the
amount of compound interest that will have to be paid at the end of each m1 th of
a period which is equivalent to an effective interest rate of i per period.
2. A nominal rate of discount d(m) charged m times per period represents m times
the amount of compound discount that will have to be charged at the beginning of
each m1 th of a period which is equivalent to d, the amount charged at the beginning
of the period under an effective discount rate of d per period.
Theorem A.4 1. A nominal rate of interest of i(m) payable and compounded at the
(m)
end of every m1 th of a year is equivalent to18 an effective rate of interest of i m
payable every mth mof a year. Thus a principal of 1 accumulates at the end of a full
i(m)
year to 1 + m . We have, therefore, the relationship
m
i(m)
1+ = 1 + i, (50)
m
18
Indeed, it has been defined to be
Information for Students in MATH 329 2005 01 2027

or, equivalently,
1
i(m) = m (1 + i) m 1 . (51)

2. A nominal rate of discount of d(m) payable and compounded at the beginning of


(m)
every m1 th of a year is equivalent to an effective rate of discount of d m payable
in advance, every
mth(m)ofma year. Thus an accumulation of 1 at the end of a full
year has value 1 d m at the beginning of the year. We have, therefore, the
relationship m
d(m)
1 = 1 d, (52)
m
or, equivalently,
1
1

(m)
d = m 1 (1 d) m =m 1v m . (53)

We can combine equations (50) and (52) into


m p
i(m) d(p)
1+ =1+i= 1 ; (54)
m p
when m = p, this yields the equation
i(m) d(m) i(m) d(m)
= (55)
m m m m
of which (42) is the special case m = 1.
[1, Exercise 28 p. 32] 1. Express d(4) as a function of i(3) .
2. Express i(6) as a function of d(2) .
Solution:
1. Since
3
i(3)
1+ = 1+i
3
4
d(4)
and 1 = 1 d,
4
34
d(4) i(3)
(1 + i)(1 d) = 1 1 = 1+
4 3
34
d(4) i(3)
=4 1 .
4 3
Information for Students in MATH 329 2005 01 2028

6 2
i(6) 1 d(2)
2. 1+ = 1 + i = (1 d) = 1
6 2
13 !
(2)
d
i(6) = 6 1 1 .
2

[1, Exercise 29 p. 32] On occasion, interest is convertible less frequently than once a
year. Define i( m ) and d( m ) to be the nominal annual rates of interest and discount
1 1

convertible once every m years. Find a formula analogous to formula (1.22a) for
this situation.
Solution:
m1 p1
1 + i( m ) m
1
(
= 1d p
1
p )

[1, Exercise 30 p. 32] Find the accumulated value of $100 at the end of two years:

1. if the nominal annual rate of interest is 6%, convertible quarterly; and


2. if the nominal annual rate of discount is 6%, convertible once every four
years.

0.06 24
1. 100 1 + 4
= 100(1.015)8
2 1
2. 100 (1 4 0.06) 4 = 100(1 0.24) 2

[1, Exercise 31 p. 32] Derive the formula

i(m) d(m) i(m) d(m)


= . (56)
m m m m

Solution: For m1 th of a year, the effective interest and discount rates are, respec-
(m) (m)
tively i m and d m . We apply the formula

i d = id [1, (1.17), p. 15]


1
for m
th of a year.
Another derivation of this equation would be from (18) with p = m.
1
[1, Exercise 32 p. 32] 1. Show that i(m) = d(m) (1 + i) m .
2. Verbally interpret the result obtained above.
Solution:

UPDATED TO April 12, 2005


Information for Students in MATH 329 2005 01 2029

1.
1
1
1
d(m) (1 + i) m = m 1 v m (1 + i) m
1 1 1

= m (1 + i) v (1 + i)
m m m

1 1

= m (1 + i) m (v(1 + i)) m
1 1

= m (1 + i) m 1 m
1
1
= m (1 + i) 1 = i m
m

1
2. i(m) =interest payable on loan of 1 at the end of m
period. If prepaid, d(m)
1
accumulates for m1 period at (1 + i) m .
[1, Exercise 33 p. 32] Given that i(m) = 0.1844144, and d(m) = 0.1802608, find m.
Solution:
i(m) d(m)
1+ 1 =1
m m
i(m) d(m) 0.1844144 0.1802608
m= (m) (m)
= = 8.0033434...
d + i 0.0041536
i(4)
i(n) 1+ 4
[1, Exercise 34 p. 32] It is known that 1 + = . Find n.
n 1+ i(5)
5
Solution:
i(4)
i(n) 1+ 4
1+ =
n 1+ i(5)
5
1 1 1 1
(1 + i) n = (1 + i) 4 5 = (1 + i) 20
n = 20 .
i(4)
[1, Exercise 35 p. 32] If r = d(4)
, express v in terms of r.
Solution:
1

(4)
i = 4 (1 + i) 1 4

1

d(4) = 4 1 (1 d) 4
i(4) 1 14
r = = 1 = v
d(4) (1 d) 4
Hence v = r4 .
Information for Students in MATH 329 2005 01 2030

A.6.2 1.9 FORCES OF INTEREST AND DISCOUNT (barely begun)


Information for Students in MATH 329 2005 01 2031

A.7 Supplementary Notes for the Lecture of January 19th,


2005
Distribution Date: Wednesday, January 19th, 2005, subject to revision

A.7.1 1.9 FORCES OF INTEREST AND DISCOUNT


This section of the textbook begins with the definition of the force of interest, t at time
t, defined as
A0 (t) a0 (t)
t = = .
A(t) a(t)
A definition in this generality is appropriate if we are considering very general interest
schemes. However, in the context of compound interest we can simply define
Definition A.13 = lim i(m) = ln(1 + i)
m

and we can then prove that


Theorem A.5 = lim d(m) = ln(1 + i)
m

[1, Exercise 36 p. 32] Derive the formula lim d(m) = .


m
Solution:
1

lim i(m) = lim m (1 + i) m 1
m m
1
(1 + i) m 1
= lim 1
m
m
x
(1 + i) 1
= lim+
x0 x
d
= ((1 + i)x )
dx x=0
x
= (1 + i) ln(1 + i)|x=0 = ln(1 + i)

Analogously,
1

(m)
lim d = lim m 1 (1 d) m
m m
1
1 (1 d) m
= lim 1
m
m
1 (1 d)x
= lim+
x0 x
Information for Students in MATH 329 2005 01 2032


d
x
= ((1 d) )
dx x=0
= ln(1 d) = ln(1 + i)

[1, Exercise 37 p. 32] Use relationship19 (1.23) to give a third proof of the result that
0 = .
Solution: By the given equation,

i(m)
d(m) = =
1+ i(m) 1+0
m

as m .

A.7.2 1.10 VARYING INTEREST


Omit this section for the present.

A.7.3 1.11 SUMMARY OF RESULTS


Table A.7.3, page 2033 is reproduced from the textbook [1, Table 1.1, p. 29].

19
these notes, equation (56)
Information for Students in MATH 329 2005 01 2033

Rate of interest The accumulated value The present value of 1


1
or discount of 1 at time t = a(t) at time t =
a(t)
Compound interest
i (1 + i)t v t = (1 + i)t
mt mt
i(m) i(m)
i(m) 1+ 1+
m m
d (1 d)t (1 d)t
mt mt
d(m) d(m)
d(m) 1 1
m m
et et

Simple interest
i 1 + it (1 + it)1

Simple discount
d (1 dt)1 1 dt

Table 2: Summary of Relationships in Chapter 1

Textbook Chapter 2. Solution of problems in interest

A.7.4 2.1 INTRODUCTION


Chapter 2 discusses general principles to be followed in the solution of problems in
interest. The purpose of this chapter is to develop a systematic approach by which the
basic principles from Chapter 1 can be applied to more complex financial transactions.
Successive chapters have two main purposes:
1. to familiarize the reader with more complex types of financial transactions, in-
cluding definitions of terms, which occur in practice;
2. to provide a systematic analysis of these financial transactions, which will often
lead to a more efficient handling of the problem than resorting to basic principles.

A.7.5 2.2 OBTAINING NUMERICAL RESULTS


Numerical results will be obtained by
direct calculation
Information for Students in MATH 329 2005 01 2034

use of compound interest tables, as in [1, Appendix I]; in this case we may inter-
polate between entries in the tables; more information will be provided you are
not expected to have any prior knowledge about interpolation;

direct calculation by hand, typically using series expansions. Students in MATH


329 to not often have background from Calculus 3, where the computation using
series is normally introduced. Thus any applications of this type will have to be
justified to students whose background cannot be assumed to include more than
Calculus 1 and Calculus 2. Examples of series of this type are

k(k 1) 2 k(k 1)(k 2) 3


(1 + i)k = 1 + ki + i + i + ... (57)
2! 3!
x2 x3
ex = 1 + x + + + ... (58)
2! 3!
x2 x3
ln(1 + x) = x + ... (59)
2 3
which are MacLaurin series (a special case of Taylor series), and have restric-
tions on the numbers for which they are valid. (The first is always valid if |i| < 1,
and also for any i if k is a non-negative integer; the second is valid for all x; and
the third is valid for 1 < x 2.) The error that occurs when one truncates a
series i.e., when one stops adding at a particular term can be estimated.
Using series expansions for calculation purposes is cumbersome and should be
unnecessary except in unusual circumstances.

[1, Exercise 1, p. 53] 10000 is invested for 4 months at 12.6%, where interest is com-
puted using a quadratic to approximate an exact calculation. Find the accumulated
value.
Solution: The exact value using compound interest is
4
10000(1.126) 12 = 10403.50 .

Alternatively, when we approximate by a quadratic, we truncate the MacLaurin


series after the 2nd degree term (cf. (57))

1 1 1 2 1 2 1 2 5 1
(1 + i) 3 = 1 + i + i + i3 + . . .
3 3 3 2! 3 3 3 3!

1 1 2 1
1+ i+ i2
3 3 3 2!
= 1.040236
Information for Students in MATH 329 2005 01 2035

hence the accumulated value is approximately 10402.36.


Note that a linear approximation, which would be equivalent to simple interest,
would give 10420: as the period of time is less than one unit, a linear approximation
is higher than the value given by compound interest equivalently, the line joining
the points (x, y) = (0, (1.043)0 ) and (x, y) = (1, (1.043)1 ), passes over the graph of
y = (1.043)x .
Information for Students in MATH 329 2005 01 2036

A.8 Supplementary Notes for the Lecture of January 21st, 2005


Distribution Date: Friday, January 21st, 2005, subject to further revision

A.8.1 2.2 OBTAINING NUMERICAL RESULTS (continued)


[1, Exercise 2, p. 53] Find the present value of 5000, to be paid at the end of 25
months, at a rate of discount of 8% convertible quarterly:

1. assuming compound discount throughout;


2. assuming simple discount during the final fractional period.

Solution: The nominal discount rate compounded quarterly corresponds to an


(4)
effective discount rate of d 4 = 2% per quarter.
25
1. Under compound discount the present value of 5000 is (1 0.02) 3 = 4225.27.
(Twenty-five months is 25
3
quarter-years.)
2. We discount through 8 full quarter-years by multiplying by (1 0.02)8 : this
brings the payment to one month from the present. For that month we are
instructed to discount by simple discount; as the time is one-third of a period
of 3 months, the discount factor is 1 0.02
3
= 0.9933333. We obtain a present
value of
5000(0.98)8 (0.9933333) = 4225.46.

[1, Exercise 3, 2.2, p. 53] Find an expression for the fraction of a period at which
the excess of present values computed at simple discount over compound discount
is a maximum.
Solution: Define f (x) = (1 xd) (1 d)x , where 0 x 1. Then f 0 (x) =
d (1 d)x ln(1 d), and f 00 (x) = (1 d)x (ln(1 d))2 < 0: the graph of
f is concave downward, and so the point where f 0 (x) = 0, i.e. x = ln ln

d
, is the
global maximum point on the given interval. This result is analogous to that of [1,
Exercise 44 1.9, p. 33].

[1, Exercise 4, 2.2, p. 53] (not discussed in the lectures) Method A assumes simple
interest over final fractional periods, while Method B assumes simple discount over
final fractional periods. The annual effective rate of interest is 20%. Find the ratio
of the present value of a payment to be made in 1.5 years computed under Method
A to that computed under Method B.
Solution: Under Method A, the present value of a principal of 1 after 1.5 years at
1 1
20% effective interest is (1.2)(1.1) = 1.32 . Under Method B, the corresponding rate
Information for Students in MATH 329 2005 01 2037

1
of discount is 1 1.2 = 61 ; the present value will be 1
= 11 1
1.2 . The ratio
(1 12 16 )(1.2) 12
120
of values under Method A to Method B is, therefore, 121 .

A.8.2 2.3 DETERMINING TIME PERIODS


This section is part of the course, but was not discussed in the lectures. Students are
asked to read the text, and to remember the definitions. There is nothing particularly
logical about the definitions they just have to be memorized. You should be able to
solve problems of the following types, for which (unchecked) solutions are included.
A number of schemes are in common use for calculating interest for fractions of a
period:
Under exact simple interest or actual/actual one counts the exact number of
days, and assumes the year has 365 days.

Under ordinary simple interest or 30/360 one assumes that each calendar month
has 30 days, and the year has 360 days.

Under Bankers Rule or actual/360 one uses the exact number of days but treats
a year as having 360 days. The treatment of February 29th in leap years (like the
present) is not completely standardized. These calculation bases can be used for
either simple or compound interest.

Example A.6 Here is a statement I found last year at the bottom of a bank statement
that I received today from a large Canadian bank: (name of bank) calculates interest
daily using a 365-day year, including leap years. The interest rate charged in a leap year
will be equal to the Annual Interest Rate in effect on each day in that year multiplied by
366 and divided by 365. Although this results in slightly more interest being charged,
the effective annual rate is the same when rounded to the nearest 1/8th of 1%.

[1, Exercise 5, p. 53] If an investment was made on the date the United States en-
tered World War II, i.e., December 7, 1941, and was terminated at the end of the
war on August 8, 1945, for how many days was the money invested:

1. on the actual/actual basis?


2. on the 30/360 basis?

Solution:

1. From December 7, 1941 to December 7, 1944, 3 years passed, the last of


which was a leap year. The number of days is 3(365) + 1. From December 7,
1944 to August 7, 1945, the numbers of days in the months December through

UPDATED TO April 12, 2005


Information for Students in MATH 329 2005 01 2038

July were 31 + 31 + 28 + 31 + 30 + 31 + 30 + 31. Add one day, from 7th to


8th August. The total is 1340.
2. From December 7, 1941 to December 7, 1944, 3 years passed, the last of which
was a leap year. The number of days is 3(360). From December 7, 1944 to
August 7, 1945, the numbers of days in the months December through July
were 8(30). Add one day, from 7th to 8th August. The total is 1321.

[1, Exercise 6, p. 54] A sum of 10,000 is invested for the months of July and August
at 6% simple interest. Find the amount of interest earned:

1. Assuming exact simple interest.


2. Assuming ordinary simple interest.
3. Assuming the Bankers Rule.

Solution:
621
1. The number of days is 31+31 = 62; exact simple interest is 365
10000(0.06) =
101.92.
60
2. Ordinary simple interest is 360
10000 (0.06) = 100.
62
3. Interest under the Bankers Rule is 360
10000 (0.06) = 103.33.

[1, Exercise 7, p. 54] 1. Show that the Bankers Rule is always more favourable
to the lender than is exact simple interest.
2. Show that the Bankers Rule is usually more favourable to the lender than
is ordinary simple interest.
3. Find a counterexample to (the preceding) for which the opposite relationship
holds.
Solution:

1. Under the Bankers Rule the denominator of the fraction is decreased, so the
fraction is increased.
2. Suppose that k months have passed. The Bankers Rule counts the days as
30k. How does this compare with the actual number of days? To answer this
question correctly, we need to compare all terms of lengths under a year. But
the question is not precisely stated, and its not clear how we would have to
weight the results. As the computations are extensive, we will not bother.
3. A loan between February 1 and March 1 provides a counterexample.
Information for Students in MATH 329 2005 01 2039

A.8.3 2.4 THE BASIC PROBLEM


The author states that ...an interest problem involves four basic quantities:

1. the principal originally invested

2. the length of the investment period

3. the rate of interest

4. the accumulated value of the principal at the end of the investment period,

and observes that 3 of these variables can be used to determine the 4th.
Read the authors comments on language. He observes that many problems have two
point of view the borrowers point of view, and the lenders; you should be comfortable
with both, and with the terminology of both. He also observes that some of the accepted
terminology of the industry is ambiguous, or not intuitive, or even counter-intuitive. If a
statement doesnt make sense it could be that you are interpreting a definition literally,
instead of using its definition, which may be misleading.
Read paragraph [1, d), p. 42], where the author discusses the reality that language
in the subject is not always completely unambiguous. As you work problems in the
textbook, and see solutions which will be given, you will gradually become familiar with
the usual conventions.

A.8.4 2.5 EQUATIONS OF VALUE


We will often express the relationship between different sums in a problem in an equation
of value, which is simply a statement obtained by bringing all amounts to a particular
time (by accumulating or discounting according to whatever accumulation functions are
appropriate). Sometimes we will have an inequality rather than an equation. The time
where the various amounts are compared is the comparison date. Under compound inter-
est equations of value for the same payments, made a different comparison dates, should
be equivalent: you should be able to extract one from the other simply by multiplying
or dividing by the appropriate accumulation factors.

Time diagrams A time diagram is a one-dimensional diagram where the only variable
is time, shown on a single coordinate axis. We may show above or below the coordinate
of a point on the time-axis values of money intended to be associated with different
funds. As in any mathematical exercises, the diagram is not a formal part of a solution,
but may be very helpful in visualizing the solution. Some authors use variants of time
diagrams where there may be several parallel horizontal axes, representing several funds;
alternatively, you could use the usual type of graph that you have seen in calculus,
Information for Students in MATH 329 2005 01 2040

where the values of the fund will be shown as points in the plane, with the horizontal
axis representing time, and the second coordinate giving the function value. (The system
I am using for these notes accommodates figures with great difficulty; I will usually not
attempt to show time diagrams for that reason.)

[1, Exercise 8, p. 55] In return for payments of 2000 at the end of 4 years and 5000
at the end of ten years, an investor agrees to pay 3000 immediately and to make an
additional payment at the end of three years. Find the amount of the additional
payment if i(4) = 0.06.
Solution: The equation of value at time t = 0 is
43 44 410
0.06 0.06 0.06
3000 + x 1 + = 2000 1 + + 5000 1 +
4 4 4

implying that

2000(1.015)16 + 5000(1.015)40 3000


x =
(1.015)12
= 2000(1.015)4 + 5000(1.015)28 3000(1.015)12
= 1593.01 .
Information for Students in MATH 329 2005 01 2041

A.9 Supplementary Notes for the Lecture of January 24th,


2005
Distribution Date: Monday, January 24th, 2005
(subject to revision)
Assignment 1 is due next Monday. No problems will be added to the list that
was posted on the Web. A hard copy of the assignment should be distributed on
Wednesday, January 26th.

A.9.1 2.5 EQUATIONS OF VALUE (continued)


[1, Exercise 9, p. 54] At a certain interest rate the present values of the following
two payment patterns are equal:
(i) 200 at the end of 5 years plus 500 at the end of 10 years;
(ii) 400.94 at the end of 5 years.
At the same interest rate 100 invested now plus 120 invested at the end of 5 years
will accumulate to P at the end of 10 years. Calculate P .
Solution: Payment schemes (i) and (ii), both with comparison date t = 0, give rise
to the following equations of value:
200(1 + i)5 + 500(1 + i)10 = 400.94(1 + i)5 (60)
100 + 120(1 + i)5 = P (1 + i)10 (61)
Equation (60) implies that
(1 + i)5 = 0.40188 . (62)
Substituting in (61) yields
100 120
P = + = 917.76.
(0.40188)2 0.40188

[1, Exercise 10, p. 54] An investor makes three deposits into a fund, at the end of
1, 3, and 5 years. The amount of the deposit at time t is 100(1.025)t . Find the
size of the fund at the end of 7 years, if the nominal rate of discount convertible
4
quarterly is 41 .
4
Solution: The nominal rate of discount of d(4) = 41 convertible quarterly produces

1 4
an annual accumulation factor of 1 41 = (1.025)4 . At the end of 7 years the
3 payments accumulate to
100(1.025)1+(46) + 100(1.025)3+(46) + 100(1.025)5+(42) = 483.11 .
Information for Students in MATH 329 2005 01 2042

[1, Exercise 11, p. 54] Whereas the choice of a comparison date has no effect on
the answer obtained with compound interest, the same cannot be said of simple
interest. Find the amount to be paid at the end of 10 years which is equivalent
to two payments of 100 each, the first to be paid immediately, and the second to
be paid at the end of 5 years. Assume 5% simple interest is earned from the date
each payment is made, and us a comparison date of
1. The end of 10 years.
2. The end of 15 years.
Solution:
1. With a comparison date at t = 10, the payment at the end of 10 years will be
100(1 + 10 0.05) + 100(1 + 5 0.05) = 275.
2. With a comparison date at t = 15, the amount P that must be paid at the
end of 10 years satisfies the equation of value
100(1 + 15 0.05) + 100(1 + 10 0.05) = P (1 + 5 0.05) ,
which implies that P = 260.

A.9.2 2.6 UNKNOWN TIME


You may omit the discussion [1, pp. 45-46] of the method of equated time.
Read the discussion of the Rule of 72 . The textbook shows that, at a given rate i of
compound interest, money will double in
ln 2 ln 2 i
n= =
ln(1 + i) i ln(1 + i)
measurement periods. He then approximates the last ratio when i = 8%, so
ln 2 0.8 0.72
time for money to double
i ln(1.08) i
and observes that the approximation is surprisingly accurate over a wide range of in-
i
terest rates. To justify this rule we could investigate the behavior of ln(1+i) , whose
i i2
MacLaurin expansion begins 1 + 2
12
+ . . ., but that investigation is beyond this
course.
[1, Exercise 12, p. 55] Solve the following equation for t:
n ! n
X X
t
sr v = sr v tr
r=1 r=1
Information for Students in MATH 329 2005 01 2043

Solution: Taking logarithms of both sides yields


n ! n !
X X
ln sr + t ln v = ln sr v tr .
r=1 r=1

Solving this equation for t yields


n ! n !
X X
ln sr v tr ln sr
r=1 r=1
t =
n ln v
! !
X n
X
ln sr v tr ln sr
r=1 r=1
=

[1, Exercise 13, p. 55] Find how long 1000 should be left to accumulate at 6% effec-
tive in order that it will amount to twice the accumulated value of another 1000
deposited at the same time at 4% effective.
P
n
(sk vtk )
t
Solution: Equation (??) implies that v = k=1
P
n , so
sk
k=1

n n
P
n P P
n P
ln (sk v tk ) ln sk ln (sk v tk ) ln sk
k=1 k=1 k=1 k=1
t= = .
ln v

If we let t denote the number of years, the equation of value at t = n is

1000(1.06)t = 2000(1.04)t

so
2
n= = 36.39 .
ln 106 ln 104
[1, Exercise 14, p. 55] The present value of two payments of 100 each to be made at
the end of n years and 2n years is 100. If i = 0.08, find n.
Solution:
Solving the equation of value, 100v 2n + 100v n = 100, we obtain v n =
1 5
2
, in which only the + sign is acceptable, since v n > 0. Taking logarithms
gives
ln 1+2 5
n= = 6.2527 years.
ln(1.08)
Information for Students in MATH 329 2005 01 2044

[1, Exercise 16, p. 55] You are asked to develop a rule of n to approximate how long
it takes money to triple. Find n.
Solution: On page 2042 of these notes we have considered the Rule of 72. Let us
consider the rationale of that rule:

(1 + i)n = 2
ln 2
n =
ln(1 + i)
ln 2 i
=
i ln(1 + i)
ln 2 i
=
i i 2 + 3 i44 + . . .
i2 i3

ln 2 1
=
i 1 2 + 3 i43 + . . .
i i 2

ln 2 1
= i i2 i3
i 1 2 3 + 4 ...
2 !
ln 2 i i2 i3 i i2 i3
= 1+ + ... + + ... + ...
i 2 3 4 2 3 4

ln 2 i i2
= 1+ + ...
i 2 12

i2
(ln 2) 1 + 2i 12 + ...

i
The only change if we wish to obtain a rule for the time for money to triple is that
we obtain
i2
(ln 3) 1 + 2i 12 + ...
n .
i
If we approximate i in the numerator by 8%, we obtain a numerator of approxi-
mately 1.14; the rule could be called the Rule of 114.
Information for Students in MATH 329 2005 01 2045

A.10 Supplementary Notes for the Lectures of January 26th,


2005
Distribution Date: Wednesday, January 26th, 2005
(subject to revision)
(Some parts of these notes were not discussed in the lecture, which soon
moved from Chapter 2 to an introduction to Chapter 3.)

A.10.1 2.7 UNKNOWN RATE OF INTEREST


The textbook describes 4 methods for determining an unknown rate of interest:
Sometimes particularly if only one payment is involved it may be possible to
solve the equation of value by using the fact that the logarithm and exponential
functions are mutually inverse.
Sometimes the equation may be solved by algebraic techniques, for example when
the equation is equivalent to a polynomial equation that factorizes.
Sometimes the equation can be solved by interpolation in interest tables.
When all else fails, it may be necessary to solve by successive approximation or
iteration. There are various methods, and some converge much faster than others.
The justification of the better methods is beyond this course, but is the subject
matter of courses like MATH 317 (Numerical Analysis).
[1, Exercise 19, p. 55] Find the nominal rate of interest convertible semiannually at
which the accumulated value of 1000 at the end of 15 years is 3000.

i 2(15)
Solution: The
1 equation
of value at time t = 15 is 1000 1 + 2
= 3000, implying
that i = 2 3 30 1 = 7.4598394% .

[1, Exercise 20, p. 55] Find an expression for the exact effective rate of interest at
which payments of 300 at the present, 200 at the end of one year, and 100 at the
end of two years will accumulate to 700 at the end of two years.
Solution: For the unknown interest rate i, which we assume to be non-negative,
the equation of value at time 2 is
300(1 + i)2 + 200(1 + i)1 + 100 = 700 ,
yielding a quadratic equation in 1 + i:
3(1 + i)2 + 2(1 + i) 6 = 0 ,

1+ 19
whose only positive solution is 1 + i = 3
= 1.119632981, so i = 11.9632981%.
Information for Students in MATH 329 2005 01 2046

[1, Exercise 21, p. 55] It is known that an investment of 1000 will accumulate to
1825 at the end of 10 years. If it is assumed that the investment earns simple
interest at rate i during the 1st year, 2i during the second year, . . . , 10i during the
10th year, find i.
Solution: A misreading of the information given was interpreted as stating that

(1 + i)(1 + 2i)...(1 + 10i) = 1.825 .

This is a polynomial equation of degree 10. We shall apply successive approxima-


tions. We begin by ignoring powers greater than the 1st, and approximating
10
X
1+i k 1.825 ,
k=1

yielding a first approximation i0 = 0.825 55


= 0.015. Lets define f (i) = (1 + i)(1 +
2i)...(1 + 10i) 1.825. Then f (i0 ) = 0.37. We compute f (0.016) = 0.48, f (0.014) =
0.26, f (0.013) = 0.16, f (0.012) = 0.06, f (0.011) = 0.03. This suggests try-
ing f (0.0113) = 0.006, f (0.0114) = 0.003, f (0.01136) = 0.00044, f (0.01137) =
0.00048, f (0.011365) = 0.000024, f (0.011364) = 0.000068, f (0.0113647) = 0.000000589,
f (0.01136473) = 7.97 107 , etc.
But this was not the intended interpretation. In fact, we have read the problem
as though interest was compounded at varying rates, which was not intended. The
intended interpretation was that the equation of value is

1 + i + 2i + 3i + ... + 10i = 1.825 ,

which implies that i = 0.15 exactly.

[1, Exercise 23, p. 55] The sum of the accumulated value of 1 at the end of three
years at a certain effective rate of interest, and the present value of 1 to be paid
at the end of three years at an effective rate of discount numerically equal to i is
2.0096. Find the rate.
Solution: The equation of value at time t = 3 of a payable at time 0 and 1 payable
at time 6 is (1 + i)3 + (1 i)3 = 2.0096, which implies that i = 0.04 (the only
positive solution).

A.10.2 2.8 PRACTICAL EXAMPLES


[1, Exercise 25, p. 56] A bill for 100 is purchased for 96 three months before it is
due. Find
Information for Students in MATH 329 2005 01 2047

1. The nominal rate of discount convertible quarterly earned by the purchaser.


2. The annual effective rate of interest earned by the purchaser.

Solution:

d(4)
1. 100 1 4
= 96 d(4) = 16%.
1 4
2. 96(1 + i) 4 = 100 i = 100
96
1 = 17.7375701%.

[1, Exercise 26 p. 56] A two-year certificate of deposit pays an annual effective rate
of 9%. The purchaser is offered two options for prepayment penalties in the event
of early withdrawal:

A - a reduction in the rate of interest to 7%


B - loss of three months interest.

In order to assist the purchaser in deciding which option to select, compute the
ratio of the proceeds under Option A to those under Option B if the certificate of
deposit is surrendered:

1. at the end of 6 months


2. at the end of 18 months

Solution:

1. Under Option A the effective annual interest rate is now 7%, so a principal of 1
1 1 1
returns (1.07) 2 . Under Option B a principal of 1 returns (1.09) 2 4 . The ratio
1
(1.07) 2
of returns under Option A to those under option B is 1 1 = 1.012360669.
(1.09) 2 4

2. Under Option A the effective annual interest rate is now 7%, so a principal of 1
3 3 1
returns (1.07) 2 . Under Option B a principal of 1 returns (1.09) 2 4 . The ratio
3
(1.07) 2
of returns under Option A to those under option B is 3 1 = .9937852442.
(1.09) 2 4

[1, Exercise 27, p. 56] A savings and loan association pays 7% effective on deposits
at the end of each year. At the end of every 3 years a 2% bonus is paid on the
balance at that time. Find the effective rate of interest earned by an investor if
the money is left on deposit

1. Two years.
2. Three years.
3. Four years.
Information for Students in MATH 329 2005 01 2048

Solution: Let i denote the effective rate of interest in each case.

1. There being no bonus, i = 7%.


1
2. Here (1 + i)3 = (1.07)3 (1.02), so i = (1.07)(1.02) 3 1 = 7.7086300%.
1
3. This time i = (1.07)(1.02) 4 1 = 7.5410377%.

[1, Exercise 28 p. 56] A bank offers the following certificates of deposit:

Nominal annual interest rate


Term in years (convertible semiannually)
1 5%
2 6%
3 7%
4 8%

The bank does not permit early withdrawal. The certificates mature at the end of
the term. During the next six years the bank will continue to offer these certificates
of deposit. An investor deposits 1000 in the bank. Calculate the maximum amount
that can be withdrawn at the end of six years.
Solution: It is instructive to begin first with an interpretation that resulted from
a misreading of the problem. In this misreading, I overlooked the fact that the
rates were convertible semi-annually. the terms are correct, but, where I used
factors 1.05, 1.06, 1.07, 1.08, I should have used (1.025)2 , (1.03)2 , (1.035)2 , and
(1.04)2 respectively; but fractional years were still not permitted. We consider
the partitions of 6 into sums of integers: 6 = 4 + 2 = 4 + 1 + 1 = 3 + 3 = 3 + 2 + 1 =
3+1+1+1 = 2+2+2 = 2+2+1+1 = 2+1+1+1+1 = 1+1+1+
1 + 1 + 1. Because the rates increase for periods of increased length, there is no
advantage to partitioning a summand further; so we need not consider any partition
containing 1 + 1, since 2 will always be better, etc. This leaves the partitions
6 = 4 + 2 = 3 + 3 = 3 + 2 + 1 to consider. These three partitions correspond to
2
accumulations of 1000(1.08)4 (1.06)2 , 1000 ((1.07)3 ) , and 1000(1.07)3 (1.06)2 (1.05)1 ,
which are respectively equal to 1528.645395, 1500.730352, and 1445.281231, so the
best partition is into 4 + 2 years, and the best return is 1528.645395.
Now I repeat the computations, this time correctly. I must compare 1000(1.04)8 (1.03)4 ,
2
1000 ((1.035)6 ) , and 1000(1.035)6 (1.03)4 (1.025)2 , which are respectively equal to
1540.336523, 1511.068657, 1453.579295; thus the best partition is again into 4 + 2
years, and the best return is 1540.336523.
Information for Students in MATH 329 2005 01 2049

A.10.3 2.9 MISCELLANEOUS PROBLEMS


[1, Exercise 29, p. 56] A manufacturer sells a product to a retailer who has the op-
tion of paying 30% below the retail price immediately, or 25% below the retail price
in six months. Find the annual effective rate of interest at which the retailer would
be indifferent between the two options.
1 2
Solution: We solve the equation of value, 70% = 75% (1 + i) 2 . i = 75 70
1 =
14.7959183%.

[1, Exercise 30, p. 56] If an investment will be doubled in 8 years at a force of interest
, in how many years will an investment be tripled at a nominal rate of interest
numerically equal to and convertible once every three years?

Solution: The given information implies that e = 2, so = ln82 . We need
n
to determine the number of years n with the property that (1 + 3) 3 = 3, so
3 ln 3
n = ln(1+3) = 14.26421450 years.

[1, Exercise 31, p. 56] Fund A accumulates at 6% effective, and Fund B accumulates
at 8% effective. At the end of 20 years the total of the two funds is 2000. At the
end of 10 years the amount in Fund A is half that in Fund B. What is the total of
the two funds at the end of 5 years.?
Solution: Denote the principals in funds A and B by A and B respectively. Then
we know that

A(1.06)20 + B(1.08)20 = 2000


1
A(1.06)10 = B(1.08)10
2
and wish to determine A(1.06)5 + B(1.08)5 . Solving the equations yields

2000(1.06)10
A = = 182.8195812
(1.06)10 + 2(1.08)10
2000(1.08)10
B = 1 10 + (1.08)10
= 303.3009728 .
2
(1.06)

Hence
2000(1.06)5 2000(1.08)5
A(1.06)5 + B(1.06)5 = + 1
(1.06)10 + 2(1.08)10 2
(1.06)10 + (1.08)10
= 182.8195812(1.06)5 + 303.3009728(1.08)5 = 690.3024748.
Information for Students in MATH 329 2005 01 2050

[1, Exercise 32, p. 57] An investor deposits 10,000 in a bank. During the first year
the bank credits an annual effective rate of interest i. During the second year the
bank credits an annual effective interest i 0.05%. At the end of two years the
account balance is 12,093.75. What would the account balance have been at the
end of three years if the annual effective rate of interest were i + 0.09 for each of
the three years?
Solution: The equation of value is

10000(1 + i)(1 + i 0.05) = 12093.75 ,

which we interpret as a quadratic equation in 1 + i:

(1 + i)2 0.05(1 + i) 1.209375 = 0

whose only positive solution is


p
0.05 + (0.05)2 + 4(1.209375)
1+i= = 1.125000000
2
from which we conclude that i = 12.5%, and that the account balance after 3 years
would be 10000(1.125 + 0.09)3 = 10000(1.215)3 = 17936.13.
[1, Exercise 33, p. 57] A signs a one-year note for 1000 and receives 920 from the
bank. At the end of six months, A makes a payment of 288. Assuming simple
discount, to what amount does this reduce the face amount of the note?
80
Solution: The rate of simple discount is 1000 = 8%. At the due date, 1 year from
288
now, the 288 accumulates to 10.04 = 300, which is the reduction in the face value
of the note, reducing the value to 700.

Textbook Chapter 3. Basic Annuities

A.10.4 3.1 INTRODUCTION


Definition A.14 1. An annuity is a series of payments, usually made at equal inter-
vals of time.
2. The total period during which payments will be made is called the term.
3. The interval between payments is called the payment period ; the name annuity
suggests a default period of 1 year, but, in some specific applications, the default
period may have some other length; there can, in fact, be two distinct periods
involved one associated with the interest calculations, and one associated with
the payments under the annuity.
Information for Students in MATH 329 2005 01 2051

4. An annuity-certain consists of payments that are all certain to be made. Otherwise


an annuity is a contingent annuity. An annuity whose payments are contingent on
whether a given individual is alive is a life annuity; the study of such annuities
is concerned with life contingencies, and is outside of this course; following the
textbook, we will usually suppress the suffix -certain.
Information for Students in MATH 329 2005 01 2052

A.11 Supplementary Notes for the Lecture of January 28rd,


2005
Distribution Date: Friday, January 28rd, 2005, subject to further revision

A.11.1 3.2 ANNUITY-IMMEDIATE


In an annuity-immediate the payments are made at the end of each period. The present
value of an annuity-certain for n periods is denoted by an , or, if the interest rate needs
to be stated explicitly an i . The author uses a diagram to denote annuities, with 2
arrows, one sometimes labelled t1 and the other sometimes labelled t2 . The first shows
the beginning of the first period, at the end of which a payment is due under the annuity-
certain. The second arrow, labelled t2 , indicates the last payment date more precisely,
just after the payment has been made. I shall usually not include diagrams in these notes,
as they are very time-consuming to arrange using the text-preparation software that I
am using.
The accumulated value of the payments as of time t2 is denoted by sn , or, if the
interest rate needs to be stated explicitly sn i . We can prove the following basic formul:

an = v + v 2 + . . . + v n1 + v n
1 vn
= v when i 6= 0
1v
1 vn
= (63)
i
sn = 1 + (1 + i) + (1 + i)2 + . . . + (1 + i)n1
(1 + i)n 1
= when i 6= 0
(1 + i) 1
(1 + i)n 1
= (64)
i
Note that we have assumed as is usually the case that i 6= 0.20 We can also prove
the following identities, both algebraically and verbally:

1 = ian + v n (65)
sn = an (1 + i)n (66)
1 1
= +i (67)
sn an
Remember to sketch a time diagram as you read each of the following problems.
(Not all of these problems was discussed in the lecture.)
20
When i = 0 an = n = sn .
Information for Students in MATH 329 2005 01 2053

[1, Exercise 1, p. 88] A family wishes to accumulate 50,000 in a college education


fund (by) the end of 20 years. If they deposit 1000 into the fund at the end of each
of the first 10 years, and 1000 + X at the end of each of the second 10 years, find
X to the nearest unit if the fund earns 7% effective.
Solution: The equation of value at time t2 = 20 is

1000s20 + X s10 = 50000 ,

which we may solve to yield


50000 s
X = 1000 20 0.07
s10 0.07 s10 0.07
50000 (1.07)20 1
= 1000
s10 0.07 (1.07)10 1
= 50000s1
10 0.07
1000((1.07)10 + 1)
= 50000(0.072378) 1000(2.96715) from the tables
= 651.75.

A more precise computation would yield 651.724; the difference is due to rounding
errors and/or the limited precision of the tables.

[1, Exercise 2, p. 88] The cash price of a new automobile is 10,000. The purchaser
is willing to finance the car at 18% convertible monthly and to make payments of
250 at the end of each month for 4 years. Find the down payment which will be
necessary.
Solution: Let X be the down payment. Then the equation of value at time t = 0
is X + 250a48 1.5% = 10000, from which we determine the down payment, X =
1489.361590. (Here again, the answer obtained using the tables in the text-book
is slightly incorrect, at 1489.35.)

[1, Exercise 3, p. 88] An annuity provides a payment of n at the end of each year
for n years. The annual effective interest rate is n1 . What is the present value of
the annuity?
1 n
1(1+ n ) n n
Solution: n an 1 = n 1 = n2 1 n+1 .
n n

[1, Exercise 4, p. 88] If an = x and a2n = y, express d as a function of x and y.


Solution:
an = x 1 ix = v n ,
Information for Students in MATH 329 2005 01 2054

a2n = y 1 iy = (v n )2 .
2xy
Eliminating v n between the equations yields i = x2
. Hence

i 2x y
d = iv = = 2 .
1+i x + 2x y

[1, Exercise 5, p. 88] 1. Show that

am+n = am + v m an = v n am + an

2. Show that
sm+n = sm + (1 + i)m sn = (1 + i)n sm + sn
3. Interpret the results in (a) and (b) verbally.
Solution:

1. (supply an algebraic proof)


2. (supply an algebraic proof)
3. (a) The present value of the first m payments of an (m + n)-year annuity-
immediate of 1 is am . The remaining n payments have value an at time
t = m; discounted to the present, these are worth v m an at time t = 0.
Since m + n = n + m, the same argument shows that am+n = v n am + an .
(b) We can obtain similar results for the s functions by analogous arguments;
or by multiplying both sides of the previous equations by (1 + i)m+n .

[1, Exercise 7, p. 88] You are given the following annuity values. Find i:

a7 i = 5.153 a11 i = 7.036 a18 i = 9.180.

Solution: Lets first prove a useful identity:

am+n = am + an iam an (68)

This identity follows from


1 vm vn 1 (1 iam )(1 ian )
am+n = = .
i i
For a verbal explanation, consider an annuity-immediate of 1 per year for m + n
years, the first payment at the end of year #1. The present value of the first m
payments is am . The value of the remaining payments at time t = m is v m an , which
Information for Students in MATH 329 2005 01 2055

may be interpreted as the value of an n-year annuity-immediate whose payments


are each v m . The proof may be completed by replacing v m by 1 iam , for which
a verbal proof is given in [1, p. 60]
The identity implies that
am + an am+n
i= .
am an
With m = 7 and n = 11, we obtain
5.153 + 7.036 9.180
i =
(5.153)(7.036)
= .08299199691

or approximately 8.3%.
Note that we didnt need three equations to find i we could have determined it
from any one of the equations. For example, since a7 i is a continuous, decreasing
function of i, and since it is 7 when i = 0 and it approaches 0 as i becomes large,
the function must take on all intermediate values, so a7 i 5.153 takes on the value
0 for just one positive number, which may be found by Successive Bisection [1, p.
399].

[1, Exercise 8, p. 88] Show that



1 1 1
10
= s10 +
1v s10 i

Solution:
1 (1 + i)n
=
1 vn (1 + i)n 1
((1 + i)n 1) + 1
=
(1 + i)n 1
i ((1 + i)n 1) + 1
=
(1 + i)n 1 i
n

1 (1 + i) 1 1
= i +
(1+i)n 1
i i

1 1
= sn +
sn i

Information for Students in MATH 329 2005 01 2056

A.12 Supplementary Notes for the Lecture of January 31st,


2005
Distribution Date: Monday, January 31st, 2005, subject to further revision

A.12.1 3.3 ANNUITY-DUE


In an annuity-due the payments are made at the beginning of each period. The present
value of an annuity-certain for n periods is denoted by an , or, if the interest rate needs
to be stated explicitly an i . Again the author uses a diagram to denote annuities, with
2 arrows, one labelled t1 and the other labelled t2 . The first shows the beginning of the
first period, just before a payment is due under the annuity-due. The second arrow,
labelled t2 , indicates the end of the last period at the beginning of which a payment was
made. The accumulated value of the payments as of time t2 is denoted by sn , or, if the
interest rate needs to be stated explicitly sn i . We can prove the following basic formul:

an = 1 + v + v 2 + . . . + v n2 + v n1
1 vn
= when i 6= 0
1v
1 vn
= (69)
d
sn = (1 + i) + (1 + i)2 + . . . + (1 + i)n
(1 + i)n 1
= when i 6= 0 . (70)
d
Note that we have assumed as is usually the case that i 6= 0. We can also prove
the following identities, both algebraically and verbally:

an = (1 + i)an (71)
sn = (1 + i)sn (72)
an = 1 + an1 (73)
sn = 1 + sn+1 (74)
sn = (1 + i)n an (75)

There will not be time to discuss all of the following problems in the lecture:
[1, Exercise 9, p. 89] Find the present value of payments of 200 every six months
starting immediately and continuing through four years from the present, and 100
every six months thereafter through ten years from the present, if i(2) = 0.06.
Solution: The reader is likely to make assumptions in a casual reading that were
not intended by the author. Reading carefully, one might see that he intends that,
Information for Students in MATH 329 2005 01 2057

though the payments start immediately, they continue to the end of 4 years from
now that is, that there be 9 payments of 200, not 8. Similarly, it is the payments
that end after 10 years, not the years for which they are prepaid so there are
payments over 21 half-years in all. With this interpretation, at the effective rate
of 12 i(2) = 3% per half-year,

Present value = 100a21 3% + 100a9 3%



= 100(1.03) 100a21 3% + 100a9 3%
= 2389.716705

[1, Exercise 10, p. 89] A worker aged 40 wishes to accumulate a fund for retirement
by depositing 1000 at the beginning of each year for 25 years. Starting at age 65
the worker plans to make 15 annual withdrawals at the beginning of each year.
Assuming that all payments are certain to be made, find the amount of each with-
drawal starting at age 65 to the nearest dollar, if the effective rate of interest is 8%
during the first 25 years, but only 7% thereafter.
Solution: Let the constant amount of the withdrawals beginning at age 65 be X.
The equation of value at age 65 is

1000 s25 8% = X a15 7%


1000 s25 8%
X =
a15 7%
1.08 0.07 (1.08)25 1
= 1000
1.07 0.08 1 (1.07)15
= 8101.654558

so, to the nearest dollar, the annual withdrawals will be 8102.

[1, Exercise 11, p. 89] Find a8 if the effective rate of discount is 10%.
Solution: Since (1 d)(1 + i) = 1, v = 1 d = 0.9 when d = 0.1.

1 (0.9)8
a8 = = 5.695327900.
0.1
Information for Students in MATH 329 2005 01 2058

A.13 Supplementary Notes for the Lecture of February 2nd,


2005
Distribution Date: Wednesday, February 2nd, 2005, subject to further revision

A.13.1 3.3 ANNUITY-DUE (continued)


[1, Exercise 12, p. 89] Prove that
1 1
= + d.
an sn

Solution: I suspect that the textbook was looking only for an algebraic proof. That
is straightforward, and I leave that to the student.
Here is a verbal proof: We can consider two different payment schemes that are
worth 1 today. One of these is an annuity-due of n payments, each in the amount
of a1 . The second scheme is more complicated. Consider an annuity-immediate of
n
n payments, each in the amount of s1 . These payments will be worth 1 at the end
n
of n years not today. But we can account for the n years delay in the repayment
if we pay for the use of that 1 by an annual payment i of interest at the end of the
year; or, alternatively, by the annual prepayment of d of discount at the beginning
of each year. It is the latter that we propose to combine with the n payments of
1
s
. Thus the annual payments are to be the sum
n

1
+d
sn
1
and this annual payment must, therefore, be equal to the annual level payment an
.

[1, Exercise 13, p. 89]] (not worked in the lecture) If ap = x and sq = y, show that
vx + y
ap+q = .
1 + iy
Solution:
1 vp
ap = x = x v p = 1 dx
d
(1 + i)q 1 1
sq = y = y vq =
i 1 + iy
1 dx
v p+q =
1 + iy
Information for Students in MATH 329 2005 01 2059

iy + dx iy + ivx
1 v p+q = =
1 + iy 1 + iy
y + vx
ap+q =
1 + iy

A.13.2 3.4 ANNUITY VALUES ON ANY DATE


Three cases are considered: the date is always an integral number of periods from each
payment date.

1. present values more than one period before the first payment date

2. accumulated values more than one period after the last payment date;

3. current values at a date strictly between the first and last payment dates.

Present values more than one period before the first payment date An annuity
is said to be deferred by m time units if the first payment is m time units later than
the type of annuity in question would normally be paid, and the subsequent payments
occur at the expected intervals. The notation of the earlier edition of the textbook was
standard: the value of an annuity that is deferred through k periods is denoted by the
earlier symbol, prefixed by k |. Thus k |an is the present value of an annuity-immediate
whose first payment has been deferred through k time periods; analogous symbols can
be used for an . It can be seen that

k |an = v k an
= an+k ak ,
k |an = v k an
= an+k ak .

Accumulated values more than 1 period after the last payment date Here
we can express the current values by multiplying by the appropriate power of 1 + i; or,
alternatively, by taking the difference of two annuity values.

Summary You are urged to follow the textbooks suggestion, The reader should not
try to work problems by memorizing formulas... Remember the reasoning that was used
to derive the formul, and apply that reasoning from first principles in each case.
Information for Students in MATH 329 2005 01 2060

Some exercises from the textbook The textbook asks you to prove several formul.
There are two levels at which such exercises should be approached:
an algebraic proof
a verbal justification
Usually an algebraic proof should not be difficult; I will not normally include proofs in
these notes, but you can see me if you have difficulty working through a proof. The
issue is not to find an elegant proof just to show that the two sides of the equation
are equal. As for a verbal proof, that will be much harder, and I will spend increasing
amounts of time at the lectures discussing problems of this type.
[1, Exercise 17, p. 89] Payments of 100 per quarter are made from June 7, year
Z through December 7, year Z + 11, inclusive. If the nominal rate of interest,
convertible quarterly, is 6%:
1. find the present value on September 7, year Z 1;
2. find the current value on March 7, year Z + 8;
3. find the accumulated value on June 7, year Z + 12.
Solution:
1. As of September 7, year Z 1, no payments have yet been made. The present
value is21
(1.015)2 100a44+3 1.5% = 3256.879998 .
2. As of March 7, year Z + 8, the value as of March 7, year Z will accumulate
by a factor (1.015)(84) , for an accumulated value of 5403.152103.
3. As of June 7, year Z + 12, the originally computed value will accumulate by
a factor (1.015)(134)1 , for an accumulated value of 6959.369761.
The next 3 problems were not discussed in the lecture:
[1, Exercise 18, p. 89] To prove that
15
X
st st = s16 s10 6 .
t=10

21
Why is the exponent 2 and not 3 even though the the evaluation is being made 9 months before
the first payment? Because I am viewing the payments as an annuity-immediate: the clock starts
ticking one period before the first payment. And why have 3 periods been added, even though June
and December are only 2 quarter-years apart? Again because the first payment is associated with the
3-month period ending with the payment, so the actual length of the annuity period is 3 months longer.

UPDATED TO April 12, 2005


Information for Students in MATH 329 2005 01 2061

Solution: Here is a verbal proof: st st can be interpreted as the excess over 1 of


the value after t + 1 years of a payment of 1 at time t = 0, which is (1 + i)t+1 1;
for the present purposes, let us interpret this as the excess over 1 of the present
P
15
value of a payment t + 1 years ago. The sum is, therefore, the excess over 6 of
t=10
the value accumulated to the present of an annuity of 6 annual payments of 1, the
first 16 years ago, the last 11 years ago; hence the sum is equal to

s16 s10 6 .

[1, Exercise 19, p. 89] Annuities X and Y provide the following payments:

End of Year Annuity X Annuity Y


110 1 K
1120 2 0
2130 1 K

Annuities X and Y have equal present values at an annual effective interest rate
i such that v 10 = 12 . Determine K.
Solution: As of today,
the present values
of annuities
X and Y are respectively
1 a30 + 1 a20 a10 and K a30 K a20 a10 . Setting these amounts equal and
solving, we obtain
a30 + a20 a10
K =
a30 a20 + a10
1 v 30 v 20 + v 10
=
1 v 30 + v 20 v 10
1 18 14 + 12 9
= 1 1 1 = = 1.8.
1 8 + 4 2 5

[1, Exercise 20, p. 90] At an annual effective interest rate i it is known that

1. The present value of 2 at the end of each year for 2n years, plus an additional
1 at the end of each of the first n years, is 36.
2. The present value of an n-year deferred annuity-immediate paying 2 per year
for n years is 6.

Find i.
Solution: It is convenient, because of a situation that will develop later in the
proof, to distinguish two cases.
Information for Students in MATH 329 2005 01 2062

Case i 6= 0: From hypotheses 1 we have an equation of value

36 = 2 a2n + 1 an ; (76)

from (ii) we have the equation of value



6 = v n 2 an = 2 a2n an . (77)

Solving these equations, we obtain a2n = 13, an = 10, implying that

1 v 2n 1.3
n
=
1v 1
n 2
(v ) 1.3v n + 0.3 = 0
1.3 0.7
vn = = 0.3 or 1 .
2
The value v n = 1 corresponds to i = 0, which we shall treat in the next case.
Since v n = 0.3, we can substitute in the second equation of value:
1 0.3
6 = (0.3) 2
i
and solve for i, obtaining i = 7%.
Case i = 0: Here Equations (76) and (77) become

36 = 2(2n) + n
6 = 2n = 2(2n n)

which are inconsistent. Thus this case is impossible.


Information for Students in MATH 329 2005 01 2063

A.14 Supplementary Notes for the Lecture of February 4th,


2005
Distribution Date: Friday, February 4th, 2005, subject to further revision

A.14.1 3.4 ANNUITY VALUES ON ANY DATE (continued)


[1, Exercise 21, p. 90] It is known that
a7 a + sx
= 3 . (78)
a11 ay + sz

Find x, y, and z.
Solution: Intuitively we usually expect that the determination of 3 variables re-
quires 3 constraints. While this is not always the case, the fact that only one
equation has been presented here should ring an alarm bell. The variable i has
not been mentioned, and we might also wish to know whether the solution we are
asked to find should be dependent on i.22 It appears from the textbook that the
values given above are the only solution the textbook was seeking but there
exist others!.
At time t = 0 the present values of annuities-due of 1 per year for respectively 7
a
and 11 years have value in the ratio 7 . Both of these values should increase by
a11
a factor of 1 + i per year under compound interest. Hence, 4 years later, the ratio
a + s4
will not have changed. But, at that time, it can be interpreted as 3 . Thus
a7 + s4
one solution to the problem is

(x, y, z) = (4, 7, 4), (79)

and this solution is valid for all i. I would not expect students to be able to generate
the rest of this solution!
Having found one solution, which we can see from the solutions [1, p. 418] to
be the solution the author is seeking, we might be expected to stop. But, to a
mathematician, an instruction like Find x, y, and z means, implicitly,

Find all possible sets of values for x, y, and z.


22
This can explain the apparent paucity of equations: to assume that the solution holds for all i is
equivalent to assuming an equation for every value of i infinitely many equations, for the 3 unknowns
we are trying to determine. In such a situation we should not be surprised if there is no solution at all.
Information for Students in MATH 329 2005 01 2064

So lets investigate whether we have all solutions. This raises new issues. The
solution we found above in (79) is valid for all i. We have two subquestions:

Could there be other solutions for all i?


Could there be solutions that hold for specific values of i, but not for all i?

Could there be other solutions for all i? When i = 0, equation


a7 a + sx
= 3 (80)
a11 ay + sz

becomes
7 3+x
= ,
11 y+z
implying that
11x 7y 7z = 33 . (81)
As i , v 0, and the left side of equation (80) approaches

1 (1 + i)7
lim = 1;
i 1 (1 + i)11

the limit of the right side will be 1 only if

x = z. (82)

Then (81) becomes


4x 7y = 33 . (83)
Our earlier solution (79) satisfies this last diophantine equation; some other
solutions are
(x, y, z) = (11, 11, 11), (18, 15, 18), (3, 3, 3) .
If we define
y3 x+3
w= = ,
4 7
the conditions we have determined on x, y, z have the general solution

(x, y, z) = (7w 3, 4w + 3, 7w 3)

and the case (4, 7, 4) corresponds to w = 1. What about the other solutions? All
we have shown is that, if there are any solutions valid for all i, then they have to be
Information for Students in MATH 329 2005 01 2065

of the preceding form. But, when we substitute these general values into equation
(78), we may reduce the equation to the following condition on w:
1 v7 1 v 7w
= .
1 v 11 1 v 11w
All values of w other than w = 1 produce a polynomial equation which constrains
v, so the equality will not hold for all interest rates. Thus solution (79) is the only
solution valid for all i.
You are not expected to be able to reproduce this detailed argument.
[1, Exercise 22, p. 90] Simplify a15 (1 + v 15 + v 30 ) to one symbol.
Solution: Here is an algebraic solution:
15 30
1 v 15
a15 1 + v + v = 1 + v 15 + v 30
i
1 v 45
= = a45
i
A verbal proof could be based on the equation

a15 1 + v 15 + v 30 = a15 + v 15 a15 + v 30 a15

= a15 + 15 a15 + 30 a15
In the last member the first summand is the present value of an annuity of 15
annual payments of 1, the first one year from now and the last 15 years from now;
the second summand is the present value of a sequence of 15 payments which begin
one year after the last represented by the first summand and the last being paid
30 years from now; and the last summand is the present value of a final subseries
of 15 payments of 1, the first to be paid one year after the last of the 30 payments
mentioned above, and the last to be paid 45 years from now. We know that the
present value of such a series of 45 payments of 1 is a45 .
[1, Exercise 23, p. 90] (not discussed in the lecture) Find the present value to the
nearest dollar on January 1 of an annuity which pays 2000 every six months for
five years. The first payment is due on the next April 1, and the rate of interest is
9% convertible semiannually.
Solution: On April 1st the annuity is worth 2000a10 4.5% . Discounting back to
January 1st, i.e. through half of a half-year period, we obtain a value of
1 1
(1.045) 2 2000a10 4.5% = (1.045) 2 2000a10 4.5%
1 1 (1.045)10
= (1.045) 2 2000 = 16177.59053
0.045
or 16178 to the nearest dollar.
Information for Students in MATH 329 2005 01 2066

A.14.2 3.5 PERPETUITIES


The symbol a is used for the present value of a perpetual annuity an infinite sequence
of payments, the first one period from now; where the interest rate is not clear from the
context, we may write a i . When the interest rate i is 0 the present value would be
infinite. Otherwise we sum an infinite geometric series, obtaining
v v 1
v + v2 + v3 + . . . = = = .
1v iv i
It can be seen that this is the limit of the usual formula for an as n . An infinite
annuity of this type is called a perpetuity-immediate, or usually just a perpetuity.
Analogously we may define a perpetuity-due to be an infinite sequence of equal pay-
ments, where the first is made immediately. We use the symbol a , and can prove that
its present value is d1 . While perpetuities do exist in the real world, for example in the
bond market, they are also useful in providing verbal explanations of identities. For
n
example, we can explain the formula an = 1v i
by expressing the quotient on the right
1 n1
as a difference, i v i and interpreting the n-payment annuity as the difference of 2
perpetuities, one beginning a year from now, and the other n + 1 years from now; i.e.,

an = a (n |a )
Information for Students in MATH 329 2005 01 2067

A.15 Supplementary Notes for the Lecture of February 7th,


2005
Distribution Date: Monday, February 7th, 2005, subject to further revision

A.15.1 3.5 PERPETUITIES (continued)


[1, Exercise 24, p. 90] A sum P is used to buy a deferred perpetuity-due of 1 payable
annually. The annual effective rate of interest is i > 0. Find an expression for the
deferred period.
Solution: Denote the deferral period by n. Then P = v n d1 n = 1 + ln P +ln i
ln v
=
1 ln Pln+ln

i
= 1 lnlnPi .

[1, Exercise 25, p. 90] Deposits of 1000 are placed into a fund at the beginning of
each year for the next 20 years. After 30 years, annual payments commence, and
continue forever, with the first payment at the end of the 30th year. Find an
expression for the amount of each payment.
Solution: The deposits accumulate to a fund worth 1000s20 a year after the last
payment, and 1000(1 + i)10 s20 10 years after the last payment. A perpetuity-due
bought with this amount will have annual payments of

(1 + i)21 (1 + i)
1000s20 (1 + i)10 d = 1000(1 + i)10 iv
i
= 1000 (1 + i)30 (1 + i)10 .

Alternatively, we could set up an equation of value at any other time, for example
at time t = 0. The value of the deposits will be 1000a20 . Suppose that the level
amount of the withdrawals is X. These withdrawals may be interpreted as either
an perpetuity-due for which the clock starts at the time of the first payment, 30
years from now; or as a perpetuity-immediate for which the clock starts 29 years
from now in order that the first payment occur at the end of that year. This leads
to equations, either
1000a20 = Xv 30 a
or
1000a20 = Xv 29 a
which may be solved for the same value of X as determined earlier.

[1, Exercise 26, p. 90] (not discussed in detail in the lecture) A benefactor leaves an
inheritance to 4 charities, A, B, C, and D. The total inheritance is a series of level
payments at the end of each year forever. During the first n years, A, B, and C
Information for Students in MATH 329 2005 01 2068

share each payment equally. All payments after n years revert to D. If the present
values of the shares of A, B, C, and D are all equal, find (1 + i)n .
Solution: Imposing the condition that the sum of the first n payments is equal to 3
times the present value of payments ##n + 1, n + 2, . . ., we obtain an = 3v n a
v n = 41 (1 + i)n = 4.
[1, Exercise 27, p. 90] A level perpetuity-immediate is to be shared by A, B, C,
and D. A receives the first n payments, B the second n payments, C the third n
payments, and D the payments thereafter. It is known that the ratio of the present
value of Cs share to As share is 0.49. Find the ratio of the present value of Bs
share to Ds share.
Solution: The present values of the shares of A, B, C, D are, respectively, an , v n an ,
v 2n an , and v 3n a . The fact that Cs share, divided by As share is to equal 0.49
implies that v n = 0.7. The ratio of Bs share to Ds is then seen to be
v n an 0.7 0.3
3n
=
v a (0.7)3
30
= .
49
A.15.2 3.6 NONSTANDARD TERMS AND INTEREST RATES
Omit this section.

A.15.3 3.7 UNKNOWN TIME


In this section the textbook considers situations where n, the number of annuity pay-
ments, is not known. Rather than solving such problems for some non-integer n
which is not practical because there would not likely be universal agreement about the
interpretation such problems are usually resolved by having a final payment in an
otherwise regular sequence of payments cover the amount necessary to make up the dif-
ference. Where the final payment needed to meet a goal is larger than a regular payment,
it is called a balloon payment; where it is smaller, it is called a drop payment. Solving
problems in this section typically involves two phases:
Determination of the number of regular payments.
Determination of the value of the final payment, subject to given constraints (e.g.
that it be a balloon or drop payment).
The author uses tables in the first phase, but we can solve such problems without ta-
bles, by first solving an inequality. Read [1, Example 3.7, pp. 74-75], where the author
considers a problem where a planned final payment turns out to be negative.
Information for Students in MATH 329 2005 01 2069

[1, Exercise 32, p. 91] A loan of 1000 is to be repaid by annual payments of 100 to
commence at the end of the 5th year, and to continue thereafter for as long as
necessary. Find the time and amount of the final payment if the final payment is
to be larger than the regular payments. Assume i = 4.5%.
Solution: Let the time of the last balloon payment be n, and let the amount
of the last payment be X. Then n is the largest integer solution to the inequality

1 (1.045)(n4)
1000 100(1.045)4 an4 = 100(1.045)4
0.045
4
1000 0.045 (1.045)
(1.045)(n4) 1
100
(n 4) ln 1.045 ln 1 10 0.045 (1.045)4
ln (1 0.45(1.045)4 )
(n 4)
ln 1.045
ln (1 0.45(1.045)4 )
n4 = 21.47594530.
ln 1.045
Thus we conclude that the balloon payment is made at time t = 21. The equation
of value at time t = 21 is

1000(1.045)21 = 100s17 + (X 100)

implying that
100
X = 100 + 1000(1.045)21 (1.045)17 1 = 146.070467 .
0.045

UPDATED TO April 12, 2005


Information for Students in MATH 329 2005 01 2070

A.16 Supplementary Notes for the Lecture of February 9th,


2005
Distribution Date: Wednesday, February 9th, 2005, subject to further revision

A.16.1 3.7 UNKNOWN TIME (continued)


[1, Exercise 33, p. 91] A fund of 2000 is to be accumulated by n annual payments
of 50, followed by n annual payments of 100, plus a smaller final payment made 1
year after the last regular payment. if the effective rate of interest is 4.5%, find n
and the amount of the final irregular payment.
Solution: We shall interpret the payments to be made under two annuities-due:
the first, for 2n years, consists of an annual deposit of 50 in advance; the second,
for n years, deferred n years after the first, also consists of an annual deposit of 50
in advance. It is at the end of year 2n that the final, drop payment is to be made,
and it is to be under 100. (Note that this is the type of problem where the drop
payment could turn out to be negative. We seek the smallest n for which

50s2n + 50sn > 2000 100


(1.045)2n + (1.045)n 2
50(1.045) > 1900
0.045
1900 0.045
(1.045)2n + (1.045)n 2 >
50 1.045
2
1 1900 0.045
(1.045)n + > + 2.25 = 3.886363636
2 50 1.045
Since the exponential is positive, the preceding inequality is equivalent to (1.045)n >
1.471386222, and, in turn, to
ln 1.471386222
n> = 8.774018446 .
ln 1.045
Thus the drop payment will be when t = 2 9, i.e., 18 years after the first pay-
ment under the annuity with payments of 50. Just before the drop payment the
accumulated value of all previous payments is
(1.045)18 + (1.045)9 2
50 s9 + s18 = 50(1.045) = 1967.588591
0.045
so the drop payment at time t = 18 is 2000 1967.588591 = 32.411409.
Note that there is an error in the answers in the textbook: while n = 9 is correct,
the payment of 32.41 is not at time n = 9.
Information for Students in MATH 329 2005 01 2071

(If tables like those in the textbook were available, one could determine the value
of n by inspecting the value of s2n + sn . We observe from the 4.5% tables the
following values:
n s2n + sn
8 32.0993
9 37.6572
10 43.6596
We seek the smallest n such that

50s2n + 50sn > 2000 100

i.e., such that


s2n + sn > 38 ,
equivalently,
38
s2n + sn > = 36.37 ,
1.045
and so can conclude that n = 9.)

[1, Exercise 34, p. 91] One annuity pays 4 at the end of each year for 36 years.
Another annuity pays 5 at the end of each year for 18 years. The present values
of both annuities are equal at effective rate of interest i. If an amount of money
invested at the same rate i will double in n years, find n.
Solution: If i 6= 0, the equation of value for the two annuities may be solved as
follows:

4a36 = 5a18 4 1 v 36 = 5 1 v 18
2
4 v 18 5 v 18 + 1 = 0
1
v 18 = 1 or v 18 =
4
Of these equations, the first is inadmissible, since it corresponds to the excluded
case i = 0; the second equation is the only valid conclusion, and it implies that
(1 + i)18 = 4 = 22 , so money doubles in 18
2
= 9 years.
As for the case i = 0, the equation of value can be transformed as follows:

4a36 = 5a18 4 36 = 5 18 (84)

which is a contradiction; hence i 6= 0, and the preceding conclusion is the only


valid one.
Information for Students in MATH 329 2005 01 2072

[1, Exercise 35, p. 91] A fund earning 8% effective is being accumulated with pay-
ments of 500 at the beginning of each year for 20 years. Find the maximum number
of withdrawals of 1000 which can be made at the ends of years under the condition
that once withdrawals start they must continue through the end of the 20-year
period.
Solution: Suppose that the first withdrawal occurs at time t = n. We must deter-
mine the smallest n satisfying the following sequence of equivalent inequalities:

500s20 1000s21n

(1.08) (1.08)20 1 2 (1.08)21n 1
(1.08)21
(1.08)21 1.08 2 2
(1.08)n
2
(1.08)n 0.92
1 + (1.08)21

n 6.825449633 ,

hence n = 7. The maximum number of withdrawals of 1000 is, therefore, 21 7 =


14.

[1, Exercise 36, p. 91] A borrower has the following two options for repaying a loan:

(i) Sixty monthly payments of 100 at the end of each month.


(ii) A single payment of 6000 at the end of K months.

Interest is at the nominal annual rate of 12% convertible monthly. The two options
have the same present value. Find K.
Solution: The equation of value at the present time t = 0 leads to the following
sequence of equivalent equations:

100a60 1% = 6000(1.01)K
1 (1.01)60 60
=
0.01 (1.01)K
0.01 60
(1.01)K =
1 (1.01)60
0.0160
ln 1(1.01)60
K= = 29.01227333 .
ln 1.01
To the precision of the problem, K = 29.
Information for Students in MATH 329 2005 01 2073

A.16.2 3.8 UNKNOWN RATE OF INTEREST


The details of this section can be omitted for examination purposes.

Approximating compound interest for fractions of a measurement period


The textbook observes in [1, 2.2] that a method that is often used when a fraction
of a measurement period is involved is to approximate compound interest by simple
interest. This amounts to using series (57) above, but stopping after the 1st degree
term:
(1 + i)k 1 + ki .
It can also be interpreted as linear interpolation between values of (1 + i)x for integer
x. Suppose that we are interested in the accumulation factor for compound interest
between t = n and t = n + k, where 0 k 1. If we think of a line joining the points
(n, (1 + i)n ) and (n + 1, (1 + i)n+1 ), and take as a value approximating (1 + i)n+k , the
ordinate of the point where this line meets the line x = n + k, we have similar triangles,
leading to the equation
k approximation (1 + i)n
=
1 (1 + i)n+1 (1 + i)n
from which we determine that the approximation is

k(1 + i)n+1 + (1 k)(1 + i)n = (1 + i)n (k(1 + i) + (1 k)) = (1 + i)n (1 + ki).

The textbook remarks that linear approximation between successive integer values of v x
is equivalent to simple discount. We can obtain a better approximation by truncating
the MacLaurin expansion after a higher term than the first degree term. In the following
example the accumulation factor is approximated by a quadratic not a linear
function.

Unknown rate of interest Where it is the interest rate that is not known, there can
be several different approaches:

Algebraic methods:

Where possible, one tries to find a solution using algebraic methods. Since
the various formul we work with are usually polynomial in i or ratios of
polynomials in i, it may be possible to find the particular solution(s) we seek
by algebraic means.
Algebraic means may still be available where solving for i fails, if we can find
another convenient intermediary variable.
Information for Students in MATH 329 2005 01 2074

It may be possible to solve by interpolation on tables, provided the functions we


are interested in are tabulated.

The favoured method is by successive approximation, to obtain a solution to any


desired accuracy.

Linear Interpolation Suppose that we know the values of f at distinct points x = x1


and at x = x2 , and that f (x1 ) 6= f (x2 ). If |x1 x2 | is small, it may be reasonable to
assume that the graph of f is approximately linear between x1 and x2 ; or, more generally,
near x1 and x2 . That is equivalent to assuming that
f (x2 ) f (x1 )
f (x) = f (x1 ) + (x x1 )
x2 x1
f (x2 ) f (x1 ) x2 f (x1 ) x1 f (x2 )
= x+ .
x2 x1 x2 x1
If now we wish to determine an approximate value of x where f (x) has a specific value
y0 , we may use this equation for that purpose provided f (x2 ) f (x1 ) is not too small:

y0 f (x1 )
x = x1 + (x2 x1 ) .
f (x2 ) f (x1 )

In particular, if we wish to find a zero of f near the given points, and if f (x1 ) 6= f (x2 ),
a good approximation could be
0 f (x1 )
x x1 + (x2 x1 )
f (x2 ) f (x1 )
x1 f (x2 ) x2 f (x1 )
= .
f (x2 ) f (x1 )
When we apply these formul for points between two points x1 and x2 , we speak of linear
inter polation; otherwise linear extrapolation. Interpolation may be used to improve the
efficiency of the Method of Successive Bisection [1, p. 349, Appendix V], or to provide a
first approximation prior to the application of other approximation methods.

Newton-Raphson iteration method In [1, Appendix V, pp. 399-400] the author


considers several Iteration methods. In all of these methods one is interested in solving
an equation
f (x) = 0 , (85)
i.e. in finding the zeroes of f ; usually there are several solutions, and one is interested
in a zero in a certain interval. For example, if x is to represent an interest rate, or
Information for Students in MATH 329 2005 01 2075

an accumulation factor, we would not normally be interested in a solution that was


negative. One of the iteration methods discussed is the method of Successive bisection
[1, B, pp. 399-400] which you have already seen in an assignment problem. Another is
the Newton-Raphson method [1, D, p. 400].
The general idea we are applying to (85) is to find a fixed point for a mapping
x h(x). It can be shown that if, in an interval where f 0 is continuous and less than
1 in magnitude, we start with an approximation call it x0 and replace it by a
succession of approximations [1, p. 76]:
x1 = h(x0 )
x2 = h(x1 )
x3 = h(x2 )

xn = h(xn1 )
xn+1 = h(xn )

this iteration algorithm will converge in the interval within which x0 is chosen; that is,
the sequence x0 , x1 , . . . will approach a limit x which satisfies equation
x = h(x) . (86)
In applying the Newton-Raphson method to solve (85) for a function f , we take
f (x)
h(x) = x .
f 0 (x)
The condition |h0 (x)| < 1 that must be satisfied is equivalent to

f (x) f 00 (x)

(f 0 (x))2 < 1 . (87)

Any solution to (85) will be a fixed point x for the transformation h. Here the successive
approximations are:
x0
f (x0 )
x1 = x0
f 0 (x0 )
f (x1 )
x2 = x1 0
f (x1 )
...
f (xn )
xn+1 = xn 0
f (xn )
...
Information for Students in MATH 329 2005 01 2076

and the sequence converges to a point x with the property that

f (x)
x=x ,
f 0 (x)

so f (x) = 0. This method can be shown to produce an extremely fast rate of conver-
gence, called second-order convergence23 . While the theory of approximations of this
type is beyond this course, we shall use the method when it is useful.

Newton-Raphson iteration to solve an i = k for i The given equation must be


recast as an equivalent equation in the form f (i) = 0. Our usual formula for an i gives

1 (1 i)n
=k
i
so we might express this as an equivalent constraint on a function

1 (1 i)n
f (i) = k,
i
asking that
1 (1 i)n
f (i) = k = 0.
i
It can be shown that condition (87) is satisfied for this function. The iteration is given
by [1, (3.28), p. 77]

1 (1 + ir )n kir
ir+1 = ir 1 + (88)
1 (1 + ir )(n+1) (1 + (n + 1)ir )

Newton-Raphson iteration to solve sn i = k for i Analogously to the preceding,


it may be shown that the iteration is given by [1, (3.28), p. 78]

(1 + ir )n 1 kir
ir+1 = ir 1 + (89)
(1 + ir )n1 (1 ir (n 1)) 1

Some remaining exercises from [1, Chapter 3] (not all were discussed in the
lecture):

[1, Exercise 38, p. 92] If a2 = 1.75, find an exact expression for i.


23
cf., http://mathworld.wolfram.com/NewtonsMethod.html
Information for Students in MATH 329 2005 01 2077

Solution:
(1 + i)2 1
a2 = 1.75 = 1.75(1 + i)2
i
1.75i2 + 2.5i 0.25 = 0 (90)
7i2 + 10i 1 = 0

10 + 100 + 28 5 4 2 5 + 4 2
i= = or . (91)
14 7 7
Since i > 0, the second solution is inadmissible, and i = 0.09383632129.

[1, Exercise 41, p. 92] A fund of 17,000 is to be accumulated at the end of 5 years
with payments at the end of each half-year. The first 5 payments are 1,000 each,
while the second 5 payments are 2,000 each. Find the nominal rate of interest
convertible semiannually earned on the fund.
Solution: The equation of value at time 5, just after the last payment, is

1000 s10 + s5 = 17000 (92)
2
(1 + i)5 + (1 + i)5 2 = 17i. (93)

There is a unique solution to this problem, since the sum on the left side of equation
(92) is an increasing function of positive i.
Define
f (x) = (1 + i)10 + (1 + i)5 2 17i .
We can approximate with successive bisection, beginning with f (0.03) = 0.006809547
and f (0.04) = 0.016897187.

f (0.035) = 0.016897187
f (0.0325) = 0.002194300
f (0.03375) = 0.000436424
f (0.033125) = 0.000906060
f (0.0334375) = 0.000241612
f (0.03359375) = 0.000095705
f (0.033515625) = 0.000073378
f (0.033554687) = 0.000011056
f (0.033535156) = 0.000031188
f (0.033544922) = 0.000010071
f (0.033549805) = 0.000000491
Information for Students in MATH 329 2005 01 2078

f (0.033547364) = 0.000004790
f (0.033548585) = 0.000002148
f (0.033549195) = 0.0000008286
f (0.033549500) = 0.000000169
f (0.033549653) = 0.000000162
f (0.033549577) = 0.000000001
f (0.033549615) = 0.000000080
f (0.033549587) = 0.000000020
f (0.033549582) = 0.000000009
f (0.033549580) = 0.000000005
f (0.033549579) = 0.000000002
f (0.033549578) = 0.000000000

This is the effective interest rate for a half-year. The nominal annual rate, com-
pounded semi-annually will be 2 0.033549578 = 6.7099156%.

[1, Exercise 42, p. 92] A beneficiary receives a 10,000 life insurance benefit. If the
beneficiary uses the proceeds to buy a 10-year annuity-immediate, the annual pay-
out will be 1,538. If a 20-year annuity-immediate is purchased, the annual payout
will be 1,072. Both calculations are based on an annual effective interest rate of i.
Find i.
Solution: The equations of value at time 0 are

10000 = 1538 a10 = 1072 a20

The last equation alone implies that 1 + v 10 = 1.434701492, which implies that
i = 8.6878222%. I dont see why the amount of the insurance benefit was given
(although it is consistent with the other other information.)

[1, Exercise 50, p. 93] 1. Show that

n(n 1) n(n 1)(n 2) 2


sn = n + i+ i + ...
2! 3!
2. Show that
1 1 n1 n2 1) 2
= 1 i+ i + ...
sn n 2 12

Solution:

UPDATED TO April 12, 2005


Information for Students in MATH 329 2005 01 2079

1. The sum on the right a finite sum is obtained from the expansion of
the 10th power of the binomial 1 + i by
deleting the term for i0 , which is 1;
dividing by i.
2. Unlike the preceding expansion, this one is infinite. It can be obtained from
the preceding result as follows:
1 i
=
sn (1 + i)n 1
1
1 n 1 (n 1)(n 2) (n 1)(n 2)(n 3) 2
= 1+i + i+ i + ...
n 2 6 24

1 n 1 (n 1)(n 2) (n 1)(n 2)(n 3) 2
= 1 + i+ i + ... i
n 2 6 24
2 #
n 1 (n 1)(n 2) (n 1)(n 2)(n 3) 2
+ + i+ i + . . . i2 + . . .
2 6 24

1 n1 n2 1 2 (n 1)(n2 n 1) 3
= 1 i+ i + i + ...
n 2 12 24

[1, Exercise 51, p. 93] A loan of 1,000 is to be repaid with annual payments at the
end of each year for the next 20 years. For the next 5 years the payments are k
per year; the second 5 years, 2k per year; the third 5 years, 3k per year; and the
fourth 5 years, 4k per year. Find an expression for k.
Solution: The equation of value at time 0 is

kv 20 s20 + s15 + s10 + s5 = 1000 ,

implying that
1000
k =
s20 + s15 + s10 + s5
1000i a5
=
s20

[1, Exercise 52, p. 93] The present value of an annuity-immediate which pays 200
every 6 months during the next 10 years and 100 every 6 months during the follow-
ing 10 years is 4,000. The present value of a 10-year deferred annuity-immediate
which pays 250 every 6 months for 10 years is 2,500. Find the present value of an
annuity-immediate which pays 200 every 6 months during the next 10 years and
300 every 6 months during the following 10 years. (Hint: Payments made during
Information for Students in MATH 329 2005 01 2080

the first 10 years are discounted at a different rate than payments made during the
second 10 years.)
Solution: The Hint is objectionable, as it reveals a datum that should have been
stated in the problem.
Denote the effective semi-annual interest rates during the first and second 10 years
by i and j respectively. The equations of value at time 0 are
200 a20 i + 100 a20 j (1 + i)20 = 4000
250 a20 j (1 + i)20 = 2500
Solving these equations, we infer that
a20 i = 15
a20 j (1 + i)20 = 10
200 a20 i + 300 a20 j (1 + i)20 = 6000
where the last sum is the present value of the desired annuity-immediate.
[1, Exercise 53, p. 94] A depositor puts 10,000 into a bank account that pays an
annual effective interest rate of 4% for 10 years. If a withdrawal is made during
the first 5 12 years, a penalty of 5% of the withdrawal amount is made. The depositor
withdraws K at the end of each of years 4, 5, 6, and 7. The balance in the account
at the end of year 10 is 10,000. Find K.
Solution: We shall assume that the account is debited by each penalty after the
withdrawal that triggers it. An equation of value at time 10 is

10000(1.04)10 K(1.05) (1.04)6 + (1.04)5 K (1.04)4 + (1.04)3 = 10000 .
Hence
(1.04)10 1
K =
(1.04)3 (2.04) ((1.05)(1.04)2 + 1)
= 979.9317732 .
40
X
[1, Exercise 54, p. 94] Simplify sn .
n=15
Solution:
40 40
!
1 X X 1 (1 + i)41 (1 + i)15
n
(1 + i) 1 = 26
i n=15 n=15
i i
1
= s41 s15 26
i
Information for Students in MATH 329 2005 01 2081

[1, Exercise 55, p. 94] Show that sn an > n2 if i > 0 and n > 1.
Solution:
2
(1 + i)n 1 1
sn an = n1
= (sn )2 (1 + i)n1 .
i (1 + i)

But sn is a strictly increasing function of i; its minimum value is when i = 0, where


it is equal to n. And 1 + i > 1 (1 + i)n1 > 1.

A.16.3 3.9 VARYING INTEREST


Omit this section for now.

A.16.4 3.10 ANNUITIES NOT INVOLVING COMPOUND INTEREST


Omit this section.
Information for Students in MATH 329 2005 01 2082

A.17 Supplementary Notes for the Lecture of February 11th,


2005
Distribution Date: Friday, February 11th, 2005, subject to further revision

Textbook Chapter 4. More General Annuities.

A.17.1 4.1 INTRODUCTION


The textbook considers

annuities whose payments do not have the same frequency as the interest conversion
periods;

annuities whose payments are not constant.

For the first of these topics, contained in the following three sections of the textbook, we
shall not study all the material carefully, but shall consider ad hoc solutions to problems.

A.17.2 4.2 ANNUITIES PAYABLE AT A DIFFERENT FREQUENCY


THAN INTEREST IS CONVERTIBLE
A typographical error. The following example, given in the textbook [1, Example
4.1, p. 96] contains a serious error. Example 4.1 Find the accumulated value at the
end of 45 years of an investment fund in which 100 is deposited at the beginning of each
quarter for the first 2 years, and 200 is deposited at the beginning of each quarter for
the second 2 years, if the fund earns 12% convertible quarterly.
Solution: If the interest rate were as stated the problem would be of a type studied in
Chapter 2. A reading of the solution shows that the author intended the last word to be
monthly.

Some Exercises

[1, Exercise 1, p. 122] Find the accumulated value 18 years after the first payment
is made of an annuity on which there are 8 payments of 2000 each made at 2-year
intervals. The nominal rate of interest convertible semiannually is 7%. Answer to
the nearest dollar.
Solution:
Information for Students in MATH 329 2005 01 2083

1. First Solution: Working with 2-year periods. The nominal biennial


interest rate i( 2 ) is given by
1

! 12 2
i( 2 )
1
i(2)
1+ 1 = 1+i= 1+
2
2
2
0.07
= 1+
2

i( 2 ) = 2 (1.035)4 1
1

and the effective biennial rate call it j is

i( 2 )
1

j= 1 = (1.035)4 1 .
2

Using this interest rate and a scale with time unit of a 2 year interval, the
payments 2 intervals after the last are worth
2000
2000 s10 s2 = (1 + j)10 (1 + j)8
j

(1.035)40 (1.035)32
= 2000
(1.035)4 1
= 35824.25354
which is 35824 to the nearest unit. (Why, then, does the textbook give the
value as 35825? Because the author is using his tables. If we work with the
3.5% tables [1, p. 383], we obtain

s40 0.035 s8 0.035
2000 s10 s2 = 2000 (94)
s4 0.035

84.5503 9.0517
= 2000 (95)
4.2149
= 35824.62217 . (96)
The tables also contain values of the inverse of s4 0.035 :

s40 0.035 s8 0.035
2000 = 2000s1 s40 0.035 s8 0.035
s4 0.035 4 0.035

= 2000(0.237251)(84.5503 9.0517)
= 35824.23670 ,
which gives an answer which is closer to the correct one.)
Information for Students in MATH 329 2005 01 2084

2. Second Solution: Working with half-year periods. The effective inter-


est rate for half-year periods is 12 (7%) = 3.5%. A payment of 2000 at the end
of a 2-year period could be replaced by 4 equal payments at the ends of the
first, second, third, and 4th half year (which is the time of the payment of
2000). If we denote the amount of these equal payments by X, we have the
equation of value at the end of the 2-year period,

X s4 3.5% = 2000

2000
so X = . There will be 4 8 = 32 such payments of X, the last of them
s4 3.5%
8 half-years before the date at which we require the accumulated value. We
can, instead think of a series of 32 + 8 = 40 payments, and then subtract the
accumulated value of the last 8 payments that we have added. We obtain as
the accumulated value

X s40 3.5% s8 3.5%
2000
= s40 3.5% s8 3.5%
s4 3.5%

which agrees with equation (94) above.

[1, Exercise 2, p. 122] (not discussed in the lecture) Find the present value of a 10-
year annuity which pays 400 at the beginning of each quarter for the first 5 years,
increasing to 600 per quarter thereafter. The annual effective rate of interest is
12%. Answer to the nearest dollar.
1
Solution: With 1 + i = (1.12) 4 ,

Present Value = 600a40 200a20



1 v 39 1 v 19
= (600 200) + 600 200
i i
= 11466.12687

[1, Exercise 3, p. 122] (not discussed in the lecture) A sum of 100 is placed into a
fund at the beginning of every other year for 8 years. If the fund balance at the
end of 8 years is 520, find the rate of simple interest earned by the fund.
Solution: The wording of this problem is not as precise as it could be while it
is clear that the payments into the fund are 2 years apart, it is not clear whether
they are at the beginnings of years ##0, 2, 4, 6 or the beginnings of years ##1, 3,
5, 7. From the authors answer we see that he intended the former interpretation.
Information for Students in MATH 329 2005 01 2085

Let i be the annual rate of simple interest. Then the equation of value at time
t = 8 is

100[(1 + 8i) + (1 + 6i) + (1 + 4i) + (1 + 2i)] = 520 i = 6%

(The other interpretation mentioned would have given

100[(1 + 7i) + (1 + 5i) + (1 + 3i) + (1 + 1i)] = 520 i = 7.5% .)


Information for Students in MATH 329 2005 01 2086

A.18 Supplementary Notes for the Lecture of February 14th,


2005
Distribution Date: Monday, February 14th, 2005, subject to further revision

A.18.1 4.3 FURTHER ANALYSIS OF ANNUITIES PAYABLE LESS FRE-


QUENTLY THAN INTEREST IS CONVERTIBLE
Some Exercises

[1, Exercise 5, p. 122] Give an expression in terms of functions assuming a rate of


interest per month for the present value, 3 years before the first payment is made,
of an annuity on which there are payments of 200 every 4 months for 12 years:

1. expressed as an annuity-immediate;
2. expressed as an annuity-due.

Solution: In order to use these functions, we need to replace the regular payment
every 4 months by an equivalent monthly payment. Let i represent the effec-
tive monthly interest rate, and A and B respectively be the monthly payments
at the end of the month/in advance equivalent to a payment under an annuity-
immediate/annuity-due of 200 every 4 months. Then

As4 i = 200
Ba4 i = 200

Both replacement annuities will pay 12 12 = 144 monthly payments.

1. The annuity is deferred 3 years, i.e. 3 12 = 36 months, before the first


payment is made that means 32 months before the beginning of annuity-
immediate at 4-month intervals, and also 32 months before the beginning of
the replacement annuity-immediate with monthly payments of A.

Value = A 32 a144i
v 32 a144
= 200
s4
a176 a32
= 200
s4
Information for Students in MATH 329 2005 01 2087

2. This annuity is deferred 36 months.



Value = B
36 a144i
v 36 a144
= 200
a4
a a36
= 200 180
a4
a a36
= 200 180
a4

[1, Exercise 6, p. 122] Show that the present value at time 0 of 1 payable at times
7, 11, 15, 19, 23, and 27 is
a28 a4
.
s3 + a1

Solution: s3 + a1 is the present value just after the third payment of a 4-


payment annuity of 1 for which the first payment was 1 unit ago. Hence a single
1
payment of 1 at time 3 is equivalent to payments of at times 1, 2, 3, 4.
s3 + a1
Hence an n-payment annuity that pays 1 every 4 years, starting in 2 years with first
payment in 3 years, may be replaced by a 4n-payment annuity-immediate starting
now, with first payment in 4 years, whose present value is
a4n
.
s3 + a1

In the problem the first payment of the given annuity is excluded; this corresponds
to the first 4 payments of the replacement annuity. Thus the present value of the
annuity described is
a28 a4
.
s3 + a1

[1, Exercise 7, p. 122] A perpetuity of 750 payable at the end of every year, and a
perpetuity of 750 payable at the end of every 20 years are to be replaced by an
annuity of R payable at the end of every year for 30 years. If i(2) = 0.04, show that

1 v 40 s
R = 37500 + 2
s2 a40 a60

where all functions are evaluated at 2% interest.


Information for Students in MATH 329 2005 01 2088

Solution: The given nominal annual interest rate of 4%, compounded semi-annually,
is equivalent to an effective semi-annual rate of 2%.
750
1. Each payment of 750 at the end of a year is equivalent to 2 payments of s
,
2 2%
one at the end of 6 months, the other at the end of the year. Thus the present
750
value of the first perpetuity is a 2% .
s2 2%
2. The perpetuity of 750 payable at the end of 20-year intervals, is, analogously,
750
worth at present a 2% .
s40 2%
3. Hence the amount available to purchase the annuity is

1 1 1 1 v 40
750 a 2% + = 750 +
s2 2% s40 2% 0.02 s2 2% v 40 s40 2%

1 v 40
= 37500 + (97)
s2 2% a40 2%

R
4. A payment of R at the end of a year is equivalent to a payment of s
at the
2
end of every 6 months. The present value of the 30-year annuity is, therefore,
a60
R (98)
s2

since we are replacing it by a 60-half-year annuity-immediate.


5. The equation of value equates amounts (97) and (98), implying that

1 v 40
37500 +
s2 2% a40 2%
R = a 60
s2

1 v 40 s2
= 37500 +
s2 2% a40 2% a60

[1, Exercise 8, p. 122] Find an expression for the present value of an annuity-due of
600 per annum payable semiannually for 10 years, if d(12) = 0.09.
Solution: The effective discount rate per month is 0.09
12
= 34 %. The effective discount
(2)
rate per 6 months is, therefore, d0 = d 2 = 1 (1 0.0075)6 . The textbook wishes
Information for Students in MATH 329 2005 01 2089

us to interpret the statement ...600 per annum payable semiannually to mean


...300 paid per half-year. With this interpretation the present value of the 20-
half-year annuity-due is, therefore,
1 (1 d0 )20
300 a20 = 600
d0
1 (1 0.0075)120
= 300
1 (1 0.0075)6

[1, Exercise 9, p. 122] The present value of a perpetuity paying 1 at the end of every
125
3 years is . Find i.
91
Solution: A payment of 1 at the end of the year for 3 years is equivalent to a
payment of s31 at the end of every year. The equation of value is

125 1
= s31
91 i
125 1
=
91 (1 + i)3 1
3
3 6
(1 + i) =
5
i = 20%.

[1, Exercise 10, p. 123] Find an expression for the present value of an annuity on
which payments are 100 per quarter for 5 years, just before the first payment is
made, if = 0.08.
0.08
Solution: The effective interest rate per quarter is i = e 4 1, so v = e0.02 .
Accordingly, the present value of the annuity-due is
(1 + i) (1 v 20 )
100a20 i = 100
i
1 e0.4
= 100 0.02
e (1 + e0.02 )

[1, Exercise 11, p. 123] A perpetuity paying 1 at the beginning of each year has a
present value of 20. If this perpetuity is exchanged for another perpetuity paying R
at the beginning of every 2 years, find R so that the values of the two perpetuities
are equal.
Solution: An equation of value now for the perpetuity is d1 = 20, implying that
1
i = 19 . At an effective annual discount rate of d, a payment of 1 now is equivalent to
Information for Students in MATH 329 2005 01 2090

1
a 2-year annuity-due paying 1+v = 1+i
2+i
each year, in advance. The perpetuity-due
paying R at the beginning of each 2-year period is equivalent to a perpetuity-
due paying R 1+i
2+i
at the beginning of every year. Equating this to 20 yields
39
R = 20 = 1.95.
[1, Exercise 12, p. 123] Find an expression for the present value of an annuity on
which payments are 1 at the beginning of each 4-month period for 12 years, as-
suming a rate of interest per 3-month period.
Solution: If the effective interest rate per 3-month period is i, then the equivalent
4
effective rate per 4-month period will be j = (1 + i) 3 1. The present value of the
annuity-due at this rate for 12 3 4-month periods will be
(1 + j) (1 + j)35
a36 j =
j
4 140
(1 + i) 3 (1 + i) 3
= 4
(1 + i) 3 1
1 v 48
= 4
1 v3
A.18.2 4.4 FURTHER ANALYSIS OF ANNUITIES PAYABLE MORE
FREQUENTLY THAN INTEREST IS CONVERTIBLE
In 4.4 we have a new notation. A right-superscribed (m) indicates that the annuity is
payable m times in a time unit for compounding of interest. More than that, we will
assume that the unit previously associated with one payment is divided into m parts;
(m)
this usage is analogous to the use of the upper right parentheses in i(m) . Thus an i
represents the present value of an annuity of m1 payable at m times regularly spaced
through an interest period. There will be immediate and due versions of the symbol,
analogous symbols for perpetuities, and a corresponding symbol for s.

Some Exercises
[1, Exercise 14, p. 123]
(m) 1 1 2 1

an = 1 + v m + v m + . . . + v n m
m
1 1 vn
=
m d(m)
[1, Exercise 20, p. 123] A sum of 10000 is used to buy a deferred perpetuity-due
paying 500 every 6 months forever. Find an expression for the deferred period
expressed as a function of d.
Information for Students in MATH 329 2005 01 2091

1
Solution: The effective discount rate per 6 months is 1 v 2 . If the perpetuity-due
has been deferred for n full years, the equation of value is
1
10000 = 500 v n
1 v
from which it follows that

ln(20(1 v))
n =
ln v
ln(20(1 1 d))
=
ln(1 d)

(2) (2) (2)


[1, Exercise 21, p. 124] If 3 an = 2 a2n = 45 s1 , find i.
Solution:
(2) (2) (2)
3 an = 2 a2n = 45 s1
3 2 45
(1 v n ) = 1 v 2n = i
2 2 2
3
The first equation implies that 1 + v n = 2
(1 + i)n = 2. Substitution in the
1
equations yields i = 30 .

[1, Exercise 22, p. 124] Find an expression for the present value of an annuity which
pays 1 at the beginning of each 3-month period for 12 years, assuming a rate of
interest per 4-month period.
Solution: If i be the effective interest rate per 4-month period, the effective rate
3
per 3-month period will be j = (1 + i) 4 1. Accordingly the value of the desired
annuity is

1 (1 + j)48
a48 j = (1 + j)
j
3 1 (1 + i)36
= (1 + i) 4 3
(1 + i) 4 1
1 (1 + i)36
= 3
1 (1 + i) 4

An alternative approach to this problem would be to define A to be the amount


of payment that would have to be made under an annuity-due every four months
for 12 years to give the same present value as the annuity-due described in the
Information for Students in MATH 329 2005 01 2092

problem. We can look at a single year in order to determine A. The present value
of payments of A now, 4 months from now, and 8 months from now, is
1 (1 + i)3
A 1 + (1 + i)1 + (1 + i)2 = A (1 + i)
i
and the value of the 4-payment annuity-due at 3 month intervals is

1 (1 + j)4
1(1 + (1 + j) + (1 + j 2 + (1 + j)3 = (1 + j) .
j

i(1 + j)
Equating the two, and recalling that (1 + i)3 = (1 + j)4 , gives A = . The
j(1 + i)
value of the annuity will then be

a48 j = A a36 i
i(1 + j) 1 (1 + i)36
= (1 + i)
j(1 + i) i
36
1 (1 + i)
= 3
1 (1 + i) 4

as found previously.
Information for Students in MATH 329 2005 01 2093

A.19 Supplementary Notes for the Lecture of February 16th,


2005
Distribution Date: Wednesday, February 16th, 2005, subject to further revision

A.19.1 4.5 CONTINUOUS ANNUITIES


Omit this section for the present.

A.19.2 4.6 BASIC VARYING ANNUITIES


Payments varying in arithmetic progression. A general formula will be developed
for an annuity-immediate with a term of n periods, with payments beginning at P and
increasing by Q per period thereafter, at interest rate i, showing that its present value
A is
an nv n
A = P an + Q
i
and the accumulated value just after the last payment is
sn n
S = P sn + Q .
i
When P = Q = 1, a specific symbol is used:
an nv n
(Ia)n = (99)
i
sn n sn+1 (n + 1)
(Is)n = = (100)
i i
Similarly, when P = n and Q = 1, we have a decreasing annuity for which the present
and accumulated values are given by
n an
(Da)n = ,
i
n(1 + i)n sn
(Ds)n = .
i
Other symbols of interest are (Ia)n , (I s)n , (Da)n , (Ds)n , (Ia) , (Ia) .
Information for Students in MATH 329 2005 01 2094

A.20 Supplementary Notes for the Lecture of February 18th,


2005
Distribution Date: Friday, February 18th, 2005, subject to further revision

A.20.1 4.6 BASIC VARYING ANNUITIES (continued)


You may omit the discussion of Fn , Gn , Hn on [1, p. 113], and any exercises based on
these functions. However, you should be able to derive formul
an nv n 1+i
(Ia) = lim = 2
n i i
(1 + i)2
(Ia) = (1 + i)(Ia) =
i2
using the calculus.

Verbal Interpretations A number of the identities we can develop in connection with


increasing and decreasing annuities admit interesting verbal interpretations.

1.
an = i(Ia)n + nv n (101)
This equation can be obtained from (99): that provides an algebraic proof. One
verbal interpretation is as follows. The annual payments of 1 under an n-payment
annuity-due of present value an may be each invested for repayment n years after
the first of the payments. The interest earned by these investments will be multiples
of i: 1i at the end of year 1, 2i at the end of year 2, . . . , ni at the end of year n;
the principal of n repaid after n years is worth nv n at time 0.

2. Equation (99) can be interpreted directly as describing a decomposition of the


increasing annuity. Think of a perpetuity paying an nv n at the end of every year.
That annual payment may be interpreted as the value of an n-payment annuity-due
of regular payments of 1, the first payment at that time, followed by a charge of n
the year after the last payment. These payments, when summed, yield 0 for years
n + 1, . . ., and yield an increasing annuity with payments 1, 2, . . . , n for the first
n payments.

Payments varying in geometric progression. We shall work some problems to


illustrate that problems of this type are not hard to solve, since the effect of the geometric
progression is equivalent to altering the interest rate.
Information for Students in MATH 329 2005 01 2095

Some Exercises

[1, Exercise 29, p. 124] In [1, Example 4.13, p. 116] it is shown that the present value
of an annuity-immediate such that payments start at 1, increase by annual amounts
of 1 to a payment of n, and then decrease by annual amounts of 1 to a final payment
of 1, is an an . The present exercise is to justify this value verbally.
Solution: Consider a sequence of n annuities-due, each of them consisting of n
payments of 1. The first of these annuities is to make its first payment 1 year
from now, the second 2 years from now, ..., the nth n years from now. The total
payments made will increase from 1 to n, then decrease to a payment of 1, 2n 1
years from now. The value of the payments under each of these annuities-due is an
just before the first payment. These values, when discounted to the present, have
value an an .

[1, Exercise 31, p. 124] Show algebraically, and by means of a time diagram, the
following relationship between (Ia)n and (Da)n :

(Da)n = (n + 1) an (Ia)n .

Solution: In this method of presenting notes it is difficult to present a time diagram;


instead, I give a verbal explanation. To simplify, I shall move the subtracted term
from the right side to an added term on the left side of the identity. Then we are
summing two variable annuities: as the amount of one decreases by 1 unit, the
amount of the other takes up the slack and increases by one unit, so the sum of
the two remains constant for n payments. And that sum begins with value n + 1,
so that is the amount that remains constant, giving an annuity-immediate of that
constant payment for n payments.
Algebraically, we have
1 v n an nv n
(n + 1) an (Ia)n = (n + 1)
i i
(n + 1) v n an
=
i
(n + 1) v n 1 an1
=
i
(n + 1 1) an1 + v n
=
i
n an
=
i
= (Da)n
Information for Students in MATH 329 2005 01 2096

[1, Exercise 32, p. 124] The following payments are made under an annuity: 10 at
the end of the 5th year, 9 at the end of the 6th year, decreasing by 1 each year
until nothing is paid. Show that the present value is

10 a14 + a4 (1 10i)
.
i

Solution: The payments decrease until a payment of 1 at the end of the 14th year.
We can think of a decreasing annuity starting with a payment of 14 at the end of
the 1st year, and then make corrections. We can subtract a 4-payment decreasing
annuity-immediate beginning with a payment of 4 at the end of the year, and a
4-payment annuity-immediate with a constant payment of 10. Thus we have

14 a14 4 a4 10i a4
(Da)14 (Da)4 10a4 =
i
10 a14 + a4 (1 10i)
=
i

[1, Exercise 33, p. 124] Find the present value of a perpetuity under which a pay-
ment of 1 is made at the end of the 1st year, 2 at the end of the 2nd year, increasing
until a payment of n is made at the end of the nth year, and thereafter payments
are level at n per year forever.
Solution: We can begin with a perpetuity-immediate of n per year, and subtract
from it a decreasing annuity-immediate which begins with a payment of n 1 and
decreases by 1 unit per year.

n (n 1) an1
n a (Da)n1 =
i
1 + an1
=
i
an
=
i
(1 + i)an
=
i
an an
= =
vi d

[1, Exercise 34, p. 124] A perpetuity-immediate has annual payments of 1, 3, 5, 7,


. . . . If the present value(s) of the 6th and 7th payments are equal, find the present
value of the perpetuity.
Information for Students in MATH 329 2005 01 2097

Solution: We equate the values of the 6th and 7th payments:

(1 + (6 1)2)v 6 = (1 + (7 1)2)v 7
13
1+i=
11
2
i=
11
The perpetuity can be viewed as the sum of an increasing perpetuity with pay-
ments increasing by 2 each year, i.e., 2(Ia) diminished by a constant perpetuity-
immediate a :

n
lim an (1+i)n 1
2 (Ia) a = 2 n
i i
2a 1
=
i
by lHopitals Rule
= 66

[1, Exercise 35, p. 124] If X is the present value of a perpetuity of 1 per year with
the first payment at the end of the 2nd year and 20X is the present value of a
series of annual payments 1, 2, 3, . . . with the first payment at the end of the 3rd
year, find d.
Solution: The constraints are:
1 v
X =v =
i i
20X = (Ia) 2a + v
v
= 2,
i
1 1
implying that i = 20
, d= 21
.
[1, Exercise 36, p. 125] An annuity-immediate has semiannual payments of 800, 750,
700, . . . , 350, at i(2) = 0.16. If a10 0.08 = A, find the present value of the annuity in
terms of A.
Solution: We will be working with an effective semi-annual interest rate of j =
i(2)
2
= 8%. The truncated decreasing annuity-immediate that we wish to evaluate
has present value

10 16 a16 v 10 6 a6
50(Da)10 50v (Da)6 = 50
i
Information for Students in MATH 329 2005 01 2098

50
= 16 6v 10 a16 v 10 a6
i
50
= 10 + 6ia10 a10
i
50
= 10 + (6i 1)a10
i
= 625(10 0.52A) = 6250 325A .

Alternatively we could express the payments as a constant annuity of payment size


300, superimposed on a decreasing annuity of payments 500, 450, 400, . . . , 50. This
has value
10 A
300 a10 8% + 50(Da)10 8% = 300A + 50
0.08
= 300A + 6250 625A = 6250 325A.

[1, Exercise 58, p. 127] There are two perpetuities. The first has level payments of
p at the end of each year. The second is increasing such that the payments are q,
2q, 3q, . . . . Find the rate of interest that will make the difference in present value
between these perpetuities
a) Zero;
b) A maximum.
Solution: I assume that the second perpetuity is a perpetuity-immediate, like
p
the first. The present value of the first perpetuity-immediate is . The second
i
perpetuity-immediate is worth
a q
q(Ia) = q = .
i di
a) For the present values to be equal,
p q
=
i id
p q(1 + i)
=
i i2
q
i=
pq

b) The difference is
2
p q (p q)2 1 pq
= q
i id 4q i 2q
Information for Students in MATH 329 2005 01 2099

which is maximized when


1 pq
=0
i 2q
2q
i.e., when i = .
pq

A.20.2 4.7 MORE GENERAL VARYING ANNUITIES


(m)
We will not formally study this section and its interesting generalizations, e.g., I (m) a n
is an increasing annuity with payments of m1 per interest conversion period at the end of
the first mth of an interest conversion period, m2 per interest conversion period at the end
of the second mth of an interest conversion period, etc. Thus the first payment is m12 ,
the second is m22 , etc. If you meet any problems of these types, they should be solvable
by first principles.
Information for Students in MATH 329 2005 01 2100

A.21 Supplementary Notes for the Lecture of February 28th,


2005
Distribution Date: Monday, February 28th, 2005, subject to further revision

A.21.1 4.6 BASIC VARYING ANNUITIES (conclusion)


[1, Exercise 37, p. 125] Annual deposits are made into a fund at the beginning of
each year for 10 years. The first 5 deposits are 1,000 each, and deposits increase by
5% per year thereafter. If the fund earns 8% effective, find the accumulated value
at the end of 10 years.
Solution: The first 5 deposits are today worth

1000a5 8% = 4312.12684 .

I see 2 ways of interpreting the words increase by 5% per year thereafter: either
the increase is geometric, by a factor of 1.05 applied repeatedly; or the deposits
increase in arithmetic progression (before discounting). If the deposits increase in
geometric progression, then the present value will be

1000 v 5 (1.05) + v 6 (1.05)2 + . . . + v 9 (1.05)5

5 1 (v(1.05))5
= 1000v (1.05)
1 v(1.05)
5 !
1.05
= 36000(1.05)(1.08)5 1
1.08
= 3379.996182.

The present value will then be 4312.1268+3379.9962 = 7692.1230; the accumulated


value at the end of 10 years will be 16606.72. This is the answer given by the
textbook, so, presumably, our interpretation is the one the author intended.
But the language is ambiguous, and the other interpretation is plausible also. If
the deposits increase in arithmetic progression, then the present value will be

1000(1.08)5 a5 8% + 50(1.08)5 (Ia)5 8% = 3510.09347

so the present value is 7822.2203, and the value after 10 years is 16,887.59.

[1, Exercise 38, p. 125] Find the present value of a 20-year annuity with annual
payments which pays 600 immediately and each subsequent payment is 5% greater
than the preceding payment. The annual effective rate of interest is 10.25%.
Information for Students in MATH 329 2005 01 2101

Solution:
19
X
Present Value = 600v n (1.05)n
n=0
1.05 20
1 1.1025
= 600 1.05
1 1.1025
= 7851.1926 .

A.21.2 4.8 CONTINUOUS VARYING ANNUITIES


Omit this section.

A.21.3 4.9 SUMMARY OF RESULTS


Omit this section.
Textbook Chapter 5. Yield Rates.

A.21.4 5.1 INTRODUCTION


A.21.5 5.2 DISCOUNTED CASH FLOW ANALYSIS
Definition A.15 The yield rate is that rate of interest at which the present value of
returns from the investment is equal to the present value of contributions into the in-
vestment.

Finding the yield rate may require the use of various approximation methods, since the
equations that have to be solved may be polynomial of high degree.

A.21.6 5.3 UNIQUENESS OF THE YIELD RATE


Example A.7 [1, p. 133] A person makes payments of 100 immediately and 132 at the
end of 2 years, in exchange for a payment in return of 230 at the end of 1 year. The
yield rate i can be shown to satisfy the equation

((1 + i) 1.1)((1 + i) 1.2) = 0

which has two distinct solutions.

Example A.8 [1, Example 5.3, p. 136] A is able to borrow 1000 from B for 1 year at
8% effective, and to lend it to C for 1 year at 10% effective. What is As yield rate on
this transaction.
Information for Students in MATH 329 2005 01 2102

Solution: The equation of value at time 0 is

1000 v(1080) = 1000 v(1100) ,

which has no finite solution. We can say that the yield rate is infinite.

Read [1, Example 5.4, p. 136], in which the borrower cannot possibly earn sufficient
interest to cover her payments; in this case we might wish to speak of an imaginary
yield rate.
Information for Students in MATH 329 2005 01 2103

A.22 Supplementary Notes for the Lecture of March 2nd, 2005


Distribution Date: Monday, March 2nd, 2005, subject to further revision

A.22.1 5.4 REINVESTMENT RATES


Single payment with interest reinvested Suppose 1 is invested at time 0, with
interest being paid at rate i at the ends of n years, but where the interest can be
reinvested only at rate j. At the end of n years the total accumulated value of the
investment is
1 + isn j

Annuity-immediate at rate i, with reinvestment at rate j If an annuity-immediate


of 1 per period for n periods pays interest at rate i, but the interest can be reinvested
only at rate j, the accumulated value at time n is
i
n + i (Is)n1 j = n + (sn j n)
j

Some exercises on reinvestment rates Some of these problems were not discussed
in the lecture.

[1, Exercise 10, p. 161] It is desired to accumulate a fund of 1,000 at the end of 10
years by equal deposits at the beginning of each year. If the deposits earn interest
at 8% effective, but the interest can only be reinvested at only 4% effective, show
that the deposit necessary is
1000
.
2s11 0.04 12

Solution: Let x denote the necessary deposit. We will sum the principal payments
(x10) and treat the interest payments as forming an increasing annuity-immediate
with increments of 0.08x.

x 10 + 0.08 (Is)10 4% = 1000

s10 4% 10
x 10 + 0.08 = 1000
0.04
1000
x= .
2s11 0.04 12

UPDATED TO April 12, 2005


Information for Students in MATH 329 2005 01 2104

[1, Exercise 11, p. 161] A loan of 10,000 is being repaid with payments of 1,000 at
the end of each year for 20 years. If each payment is immediately reinvested at 5%
effective, find the effective annual rate of interest earned over the 20-year period.
Solution: Let the effective yield rate be i. The payments do not become available
until the maturity date, after 20 years. Until that time they are locked into a
payment-scheme that accumulates to value 1000s20 5% . We are asked for the interest
rate that was earned. There are thus just two transactions: the loan at time 0, in
the amount of 10,000, and the repayment at time 20, in the amount given above.
The equation of value at time t = 0 is

1000s20 5% (1 + i)20 = 10000


10(0.05)
(1 + i)20 =
(1.05)20 1
i = 6.1619905%.

[1, Exercise 12, p. 161] An investor purchases a 5-year financial instrument having
the following features:
(i) The investor receives payments of 1000 at the end of each year for 5 years.
(ii) These payments earn interest at an effective rate of 4% per annum. At the end
of the year, this interest is reinvested at the effective rate of 3% per annum.

Find the purchase price to the investor to produce a yield rate of 4%.
Solution: To determine the yield we need to consider when the payment is finally
released to the investor. The payments of 1000 are to be invested at 4%; they will
generate an increasing annuity whose payments start at 40 at the end of year 2,
up to 160 at the end of year 5. But they are not released to the investor; rather,
they earn interest at 3%. At the end of 5 years and only then the investor
receives
5(1000) + 40(Is)4 3%
and these amounts have to be discounted to the present at 4%, giving a present
value of
40
(1.04)5 5(1000) + 40(Is)4 3% = 5000 + s5 3% 5
0.03

5 40 (1.03)5 1
= (1.04) 5000 + 5
0.03 0.03
= 4448.418326

if the yield is to be 4%.


Information for Students in MATH 329 2005 01 2105

[1, Exercise 13, p. 161] An investor deposits 1,000 at the beginning of each year
for five years in a fund earning 5% effective. The interest from this fund can be
reinvested at only 4% effective. Show that the total accumulated value at the end
of ten years is
1250 s11 0.04 s6 0.04 1 .

Solution: The 5 deposits of 1000 would be worth 1000 s10 4% s5 4% at the end of
10 years if they were earning interest together with the reinvested annual interest
payments. But they are locked into a fund where they earn 5%, and are not released
until time 10, still worth 5,000. The interest payments constitute an increasing
annuity to the investor, beginning with 50 at time 1, increasing to 250 at time 5,
and then remaining constant until time 10. They are available to the investor as
they
are paid, but shereinvests them at 4%. Their value at time 10 is, therefore
50 (Is)10 4% (Is)5 4% . Summing yields
50
5000 + 50 (Is)10 4% (Is)5 4% = 5000 + s11 4% 11 s6 4% + 6
0.04
= (5000 6250) + 1250 s11 4% s6 4%

= 1250 s11 0.04 s6 0.04 1
= 7316.719914

Using the tables in the textbook, we would find the answer to be



1250 s11 0.04 s6 0.04 1 = 1250(13.4864 6.6330 1)
= 7316.7500 .

[1, Exercise 14, p. 161] A invests 2,000 at an effective interest rate of 17% for 10
years. Interest is payable annually and is reinvested at an effective rate of 11%. At
the end of 10 years the accumulated interest is 5,685.48. B invests 150 at the end
of each year for 20 years at an effective interest rate of 14%. Interest is payable
annually and is reinvested at an effective rate of 11%. Find Bs accumulated interest
at the end of 20 years.
Solution: As interest payments begin at the end of year 1, in the amount of
17% 2000 = 340 and continue at the same level for 10 years. The equation of
value at the end of year 10 is

340s10 11% = 5685.48 ,

implying that
5685.48
s10 11% = = 16.722.
340
UPDATED TO April 12, 2005
Information for Students in MATH 329 2005 01 2106

It follows that (1.11)10 = (16.722)(0.11) + 1 = 2.83942, and that

(1.11)20 1
s20 11% =
0.11
= s10 11% (1.11)10 + 1

= s10 11% s10 11% + 2
= 16.722 3.83942 = 64.20278124.

Bs interest payments constitute an increasing annuity-immediate whose first pay-


ment is at the end of year 2, in the amount of 21. The accumulated value of Bs
accumulated interest is
s20 11% 20
21(Is)19 11% = 21
0.11
64.20278124 20
= 21
0.11
= 8438.712782.

A.22.2 5.5 INTEREST MEASUREMENT OF A FUND


Omit this section.

A.22.3 5.6 TIME-WEIGHTED RATES OF INTEREST


Omit this section.

A.22.4 5.7 PORTFOLIO METHODS AND INVESTMENT YEAR METH-


ODS
Omit this section.

A.22.5 5.8 CAPITAL BUDGETING


Omit this section.

A.22.6 5.9 MORE GENERAL BORROWING/LENDING MODELS


Omit this section.
Information for Students in MATH 329 2005 01 2107

A.23 Supplementary Notes for the Lecture of March 4th, 2005


Distribution Date: Friday, March 4th, 2005, subject to further revision

Textbook Chapter 6. Amortization schedules and sinking


funds.

A.23.1 6.1 INTRODUCTION


We consider methods of repaying a loan, in particular

The Amortization Method: In this method the borrower makes instalment payments
to the lender. Usually these payments are a regularly spaced periodic intervals; the
progressive reduction of the amount owed is described as the amortization of the
loan.

The Sinking Fund Method: In this method the loan will be repaid by a single lump
sum payment at the end of the term of the loan. However the borrow may prepare
himself for the repayment by making deposits to a fund called a sinking fund to
accumulate the repayment amount. (Sometimes the lender may be aware of the
existence of the sinking fund; for example, an institutional borrower that issues a
series of bonds may let the public know that the accumulation of funds to redeem
the bonds may be disciplined by a sinking fund.)

A.23.2 6.2 FINDING THE OUTSTANDING LOAN BALANCE


When a loan is being amortized the outstanding balance is being reduced by the amor-
tization payments. Each payment may be analyzed an interpreted as consisting of an
interest component and a component for reduction of principal . An equation of value
can be set up at any time during the amortization, equating
Current value of payments = Accumulated value of Loan

where Payments consists of both past and future payments. Decomposing the term
and rearranging the equation gives
Present Value of Future Payments
=Accumulated Value of Loan Accumulated Value of Past Payments
Synonymous terms:

outstanding loan balance

outstanding principal
Information for Students in MATH 329 2005 01 2108

unpaid balance

remaining loan indebtedness

Not all of the following examples were discussed at the lecture.

[1, Exercise 1, p. 195] A loan of 1,000 is being repaid with quarterly payments at the
end of each quarter for 5 years, at 6% convertible quarterly. Find the outstanding
loan balance at the end of the 2nd year.
1000
Solution: The level payments under this annuity-immediate will be .
a20 1.5%
Retrospective method: The value of the payments already made is
1000
s = 491.1769.
a20 1.5% 8 0.015

Subtracting this from 1000(1.015)8 yields 635.3157.


Prospective method: The value of the 12 remaining payments is
1000
a = 635.3157 .
a20 1.5% 12 1.5%

[1, Exercise 2, p. 195] A loan of 10,000 is being repaid by instalments of 2,000 at the
end of each year, and a smaller final payment made one year after the last regular
payment. Interest is at the effective rate of 12%. Find the amount of outstanding
loan balance remaining when the borrower has made payments equal to the amount
of the loan.
Solution: The problem asks for the outstanding loan balance just after payments
totalling 10,000 have been made; this will be immediately after the 5th payment.
We shall use the retrospective method only here. The accumulated value of the
loan at time t = 5 is 10000(1.12)5 . The accumulated value of the payments made
is 2000s5 . The outstanding loan balance will, therefore be

2000
10000(1.12)5 2000s5 = 10000(1.12)5 (1.12)5 1
0.12
= 4917.72212

Were we to use the prospective method, we would need to determine the value
of the last drop payment. This is interesting information, and we could have been
asked for it. But it has not been requested, and so we shall not bother finding it.
(But you should know how to do that if it is necessary.)

UPDATED TO April 12, 2005


Information for Students in MATH 329 2005 01 2109

[1, Exercise 3, p. 195] A loan is being repaid by quarterly instalments of 1,500 at


the end of each quarter, at 10% convertible quarterly. If the loan balance at the
end of the first year is 12,000, find the original loan balance.
Solution: Denote the original loan balance by L. Here the retrospective method is
the most appropriate, since we dont know how many future payments have to be
made. The equation of value at time 1 year is
L(1.025)4 1500s4 2.5% = 12000
which we solve to yield

L = (1.025)4 1200 + 1500 (1.025)4 1
= 48000(1.025)4 + 60000 = 16514.36905.

[1, Exercise 4, p. 195] A loan is being repaid by annual payments at the ends of 15
successive years. The first 5 instalments are 4,000 each, the next 5 are 3,000 each,
and the final 5 are 2,000 each. Find expressions for the outstanding loan balance
immediately after the second 3,000 instalment.
1. prospectively;
2. retrospectively.
Solution:
1. Prospective Method. The value at time 7 of the payments yet to be made
is 2000a8 + 1000a3 .
2. Retrospective Method. We have to use the prospective method at some
time to obtain the initial value of the loan. This will be the value of all
payments at time t = 0, i.e., 2000a15 + 1000a10 + 1000a5 . The value of all
payments made before and at time t = 7 is 4000s 2s2 . The outstanding loan
balance is, therefore

2000a15 + 1000a10 + 1000a5 (1 + i)7 4000s7 + 1000s2

[1, Exercise 5, p. 196] A loan is to be repaid with level instalments payable at the
end of each half-year for 3 12 years, at a nominal rate of interest of 8% convertible
semiannually. After the fourth payment the outstanding loan balance is 5,000.
Find the initial amount of the loan.
Solution: The effective semiannual rate is 4%. There are to be 7 level payments in
all. The amount of the payments is, by the prospective method
5000 5000 0.04
= .
a3 4% 1 (1.04)3
Information for Students in MATH 329 2005 01 2110

It follows that the amount of the loan, now by the prospective method, is

5000 1 (1.04)7
a = 5000 = 10814.15817 .
a3 2% 7 2% 1 (1.04)3

[1, Exercise 6, p. 196] A 20,000 loan is to be repaid with annual payments at the end
of each year for 12 years. If (1 + i)4 = 2, find the outstanding balance immediately
after the fourth payment.
20000
Solution: The annual payment is, by the prospective method, . Again by
a12 i
the prospective method, the outstanding balance after the 4th payment is

20000 1 (1 + i)8 6
a8 i = 20000 12
= 20000 = 17142.85714.
a12 i 1 (1 + i) 7

[1, Exercise 7, p. 196] A 20,000 mortgage is being repaid with 20 annual instalments
at the end of each year. The borrower makes 5 payments, and then is temporarily
unable to make payments for the next 2 years. Find an expression for the revised
payment to start at the end of the 8th year if the loan is still to be repaid at the
end of the original 20 years.
20000
Solution: The original payments are (by the prospective method) . The
a20
outstanding balance at the end of the 7th year (with no payment then or at the
end of the previous year) is the value then of all unpaid payments, i.e.
20000
a13 + s2 .
a20

It follows that the level payment needed to repay the loan in 13 payments (under
an annuity-immediate) is

20000 1 20000 (1 + i)2 a15


a13 + s2 = .
a20 a13 a20 a13

[1, Exercise 8, p. 196] A loan of 1 was originally scheduled to be repaid by 25 equal


annual payments at the end of each year. An extra payment K with each of the 6th
through the 10th scheduled payments will be sufficient to repay the loan 5 years
earlier than under the original schedule. Show that
a20 a15
K= .
a25 a5

UPDATED TO April 12, 2005


Information for Students in MATH 329 2005 01 2111

1
Solution: The level payment of this loan of 1 is . The extra payments are equal
a25
in value to the value of the last 5 payments, so, at time t = 5,
a20 a15
K a5 =
a25

which yields the desired value for K.

[1, Exercise 9, p. 196] A loan is being repaid with level payments. If Bt , Bt+1 , Bt+2
and Bt+3 are four successive outstanding loan balances, show that

1. (Bt Bt+1 ) (Bt+2 Bt+3 ) = (Bt+1 Bt+2 )2


2. Bt + Bt+3 < Bt+1 + Bt+2

Solution:

1. Let P denote the level payment. By the prospective method, the unpaid
balance after payment #t, Bt = P ant . It follows that

(Bt Bt+1 ) (Bt+2 Bt+3 ) = P ant ant1 ant2 ant3
= v nt P v nt2 P
2
= v nt1 P 1
2
= P ant1 P ant2
= (Bt+1 Bt+2 )2

2.

Bt + Bt+3 Bt+1 Bt+2 = ant ant1 ant2 ant3
= P v nt P v nt2
= P (v 2 1) v nt2

in which product P , v nt2 are positive, and v 2 1 is negative.

UPDATED TO April 12, 2005


Information for Students in MATH 329 2005 01 2112

A.24 Supplementary Notes for the Lecture of March 7th, 2005


Distribution Date: Monday, March 7th, 2005, subject to further revision

A.24.1 6.3 AMORTIZATION SCHEDULES


An amortization schedule is a chart showing, for each payment date, the information
shown in the columns of the following beginning of such a schedule with n payments of
size x:
Year Payment Interest Principal Outstanding
amount paid repaid loan balance
0 x an i
1 x x(1 (1 + i)n ) x(1 + i)n x an1 i
2 x x(1 (1 + i)n+1 ) x(1 + i)1n x an2 i

In practice there will be rounding errors as the table is generated line by line, and the
last line may not quite balance. Standard practice is to adjust the last payment so that
it is exactly equal to the amount of interest for the final period plus the outstanding
loan balance at the beginning of the final period, in order to bring the outstanding loan
balance to 0. In order to determine the entries in one row of the table, we do not need to
generate the table line by line; the table is useful when a number of rows are of interest.

Some worked examples Few of the exercises in the textbook require extensive com-
putation of schedules. Students should, however, be capable of completing a full table
for a loan when all the information needed is available.
[1, Exercise 10, p. 196] A loan is being repaid with quarterly instalments of 1,000 at
the end of each quarter for 5 years at 12% convertible quarterly. Find the amount
of principal in the 6th instalment.
Solution: The principal is
1000
1000a20 3% = 1 (1.03)20 = 14877.47486 .
0.03
After the 5th payment, the outstanding principal is (by the prospective method)
1000a205 3% , so the interest component of the next payment is
30
30a205 3% = 1 (1.03)15 = 358.1380526 ,
0.03
and the amount of principal reduction is
1000 358.1380526 = 641.8619474 .
Information for Students in MATH 329 2005 01 2113

[1, Exercise 11, p. 196] Consider a loan which is being repaid with instalments of 1
at the end of each period for n periods. Find an expression at issue for the present
value of the interest which will be paid over the life of the loan.
Solution: We can use the information in [1, Table 6.1, p. 170]. The sum of the
n
X
interest portions of the payments is, as shown in the table, (1 v r ) = n an .
r=1
This, however, is not what the problem requests. The present value of the interest
payments presumably as of time t = 0, is
n
X
v nr+1 (1 v r ) = an nv n+1 .
r=1

[1, Exercise 12, p. 196] A loan of 10,000 is being repaid with 20 instalments at the
end of each year, at 10% effective. Show that the amount of interest in the 11th
instalment is
1000
.
1 + v 10
Solution: The amount of each payment is, by the prospective method,
10000
.
a20 10%

By the prospective method, the unpaid balance after the 10th payment is
10000 10000
a10 10% =
a20 10% 1 + (1.1)10

so the interest component of the 11th payment is



10000 100
0.01 10
= .
1 + (1.1) 1 + (1.1)10

[1, Exercise 13, p. 196] A loan is being repaid with 20 instalments at the end of each
year at 9% effective. In what instalment are the principal and interest portions
most nearly equal to each other?
Solution: Without limiting generality, assume the payments are all of size 1, so that
the loan is for a20 9% , and [1, Table 6.1, p. 170] applies. In that table we see that the
repayments of principal range between v 20 in the first payment to v 1 in the last. The
two portions under consideration sum to a constant, so they cannot both exceed 12
at any time. Both sequences interest payments and reductions of principal
Information for Students in MATH 329 2005 01 2114

are monotone: there will be only one pair of values t = a, t = a + 1, where they
reverse their positions from being greater than/less than 12 to the reverse. But it
is not clear which of those two will have the closest values! We see from [1, Table
6.1, p. 170] that the difference between principal and interest components in the
tth payment is |1 2v nt+1 |. We shall begin by determining the largest value of t
if any for which
1
v 20t+1 ,
2
equivalently, the largest value of t such that

(1 + i)21t 2 ,

equivalently, the largest value of t such that


ln 2.
t 21 = 12.95676827 ,
ln 1.09
i.e., t = 12. This is the best choice where the difference 1 2v nt+1 is positive.
We need also to consider the best choice when the difference is negative, i.e.,
t = 13. We compute the differences:

t = 12 1 2(1.09)9 = 0.0791444410
t = 13 1 2(1.09)8 = 0.003732559

and see that the values are closest when t = 13.


Information for Students in MATH 329 2005 01 2115

A.25 Supplementary Notes for the Lecture of March 11th, 2005


Distribution Date: Friday, March 11th, 2005, subject to further revision

A.25.1 6.3 AMORTIZATION SCHEDULES (continued)


[1, Exercise 14, p. 197] A loan is being repaid with a series of payments at the end
of each quarter, for 5 years. If the amount of principal in the 3rd payment is 100,
find the amount of principal in the last 5 payments. Interest is at the rate of 10%
convertible quarterly.
Solution: The loan is being repaid in 5 4 = 20 quarterly payments, and the
effective interest rate per quarter is 14 (10%) = 2.5%. Lets assume that the amount
of each level payment is x, and begin by compiling the first part of the amortization
table.

Payment Payment Interest Principal Outstanding


amount paid repaid loan balance
0 x a20 2.5%
1 x x(1 (1.025)20 ) x(1.025)20 x a19 2.5%
2 x x(1 (1.025)19 ) x(1.025)19 x a18 2.5%
3 x x(1 (1.025)18 ) x(1.025)18 x a17 2.5%

from which we see that x(1.025)18 = 100, so

x = 100(1.025)18 = 155.9658718 .

(We didnt need to use the schedule here. By the Prospective Method, the unpaid
balance just after the 2nd payment is x s18 , so the interest component of the 3rd
payment is ix s18 and the residue for reduction of principal is

x 1 1 v 18 = xv 18 .)

We could now compile the last lines of the amortization table backwards. Alter-
natively, if the author is requesting the total amount of principal in the last 5
payments, that is

x(v + v 2 + v 3 + v 4 + v 5 ) = 155.9658718 a5 0.025 = 724.5906916 .

[1, Exercise 15, p. 197] A loan is being repaid with instalments of 1 at the end of
each year for 20 years. Interest is at effective rate i for the first 10 years, and
effective rate j for the second 10 years. Find expressions for
Information for Students in MATH 329 2005 01 2116

a) the amount of interest paid in the 5th instalment;


b) the amount of principal repaid in the 15th instalment.
Solution: The present value of the last 10 payments is (1 + i)10 a10 j ; the principal
of the loan is, therefore,

P = a10 i + (1 + i)10 a10 j .

We compile the first lines of the amortization table:

Year Payment Interest Principal Outstanding


amount paid repaid loan balance
0 P
1 1 iP 1 iP P (1 + i) s1 i 1
2 1 i(P(1 + i)
1) (1 + i)(1 iP ) P (1 + i)2 s2 i
2
3 1 i(1 + i) P a2 i s2 i i(1 + i)2 P P (1 + i)3 s3 i
4 1 i(1 + i)3 P a3 i s3 i i(1 + i)3 P P (1 + i)4 s4 i
5 1 i(1 + i)4 P a4 i s4 i i(1 + i)4 P P (1 + i)5 s5 i

Thus the amount of interest in the 5th instalment is



i(1 + i)4 ( a10 i + (1 + i)10 a10 j a4 i = i a6 i + (1 + i)6 a10 j .

More simply, we can observe (using the Prospective Method) that the outstanding
balance just after the 4th instalment is

(1 + i)6 a10 j + a6 i .

The interest component of the 5th payment is obtained by multiplying this amount
by i.
After the 10th instalment has been paid, we shift to the second interest rate. The
outstanding balance after the 14th payment is, again by the Prospective Method,
a6 j ; interest one payment later will be

j a6 j = 1 v 6

so the payment will reduce principal by 1 (1 v 6 ) = v 6 . the 5th of the j-portion


of the loan is (1 + j)115 .
[1, Exercise 16, p. 197] (not discussed in the lectures) A mortgage with original
principal A is being repaid with level payments of K at the end of each year
for as long as necessary, plus a smaller final payment. The effective rate of interest
is i.
Information for Students in MATH 329 2005 01 2117

a) Find the amount of principal in the tth instalment.


b) Is the principal repaid column in the amortization schedule in geometric pro-
gression (excluding the irregular final payment)?

Solution: Just after the (t 1)th instalment the outstanding principal is, by the
retrospective method,
A(1 + i)t1 K st1
and the interest earned during the next interval will be the product with i; hence
the amount of principal in the tth instalment will be

K i A (1 + i)t1 K st1 = K st i(1 + i)t1 A
= (K Ai)(1 + i)t1

which shows that the column entries are, indeed, in geometric progression, with
common ratio 1 + i.

[1, Exercise 17, p. 197] (not discussed in the lectures) A borrower has a mortgage
which calls for level annual payments of 1 at the end of each year for 20 years. At
the time of the 7th regular payment an additional payment is made equal to the
amount of principal that, according to the original amortization schedule, would
have been repaid by the 8th regular payment. If payments of 1 continue to be made
at the end of the 8th and succeeding years until the mortgage is fully repaid, show
that the amount saved in interest payments over the full term of the mortgage is
1 v 13 .
Solution: At time 0 the amount owing is a20 . By the 8th regular payment on the
original schedule, the principal repaid would have been

A A(1 + i)8 + s8 = (Ai 1)s8 .

It is intended that this amount is added to the 7th payment. Immediately after
the original 7th payment the principal owing would have been

A(1 + i)7 s7 .

The additional payment at time 7 reduces this amount to



A(1 + i)7 s7 Ai 1)s8 = (1 + i)7 + A 1 i(1 + i)7

[1, Exercise 18, p. 197] (not discussed in the lectures) A loan of L is being amortized
with payments at the end of each year for 10 years. If v 5 = 32 , find the following:
Information for Students in MATH 329 2005 01 2118

a) The amount of principal repaid in the first 5 payments.


b) The amount due at the end of 10 years if the final 5 payments are not made
as scheduled.

Solution:

1. The annual level payments constitute an annuity-immediate with annual pay-


ment of
L Li 9iL
= 2 2 = .
a10 1 5
3
By the prospective method, the amount of principal remaining to be paid
immediately after the 5th annual payment is

9iL 9L 2 3L
a5 = 1 = .
5 5 3 5

Hence
the
amount of principal that has already been paid at that time is
3 2L
1 L= .
5 5
2. If no further payments are made, the amount repayable at the end of 10 years
is
3L 3 3L 9L
(1 + i)5 = = .
5 2 5 10
[1, Exercise 19, p. 197] (not discussed in the lectures) A 35-year loan is to be repaid
with equal instalments at the end of each year. The amount of interest paid in the
8th instalment is 135. The amount of interest paid in the 22nd instalment is 108.
Calculate the amount of interest paid in the 29th instalment.
Solution: Suppose the amount of the loan at time 0 was L. The annual instalments
L
are each . By the prospective method the amount of principal outstanding just
a35
after the 7th instalment is aL a28 . This will incur an interest payment of i aL a28
35 35
in the 8th instalment; we thus have the equation

1 v 28
iL = 135 .
1 v 35
A similar computation involving instalments 21 and 22 gives

1 v 14
iL = 108 .
1 v 35
Information for Students in MATH 329 2005 01 2119

Taking the ratio of the two equations, we obtain

1 v 28 135 5
1 + v 14 = 14
= =
1v 108 4
279
so v 7 = 12 ; substitution in either equation above yields iL = . The amount of
2
interest in the 29th instalment is
a7 1 v7 16iL
iL = iL 35
= = 72 .
a35 1v 31

A.25.2 6.4 SINKING FUNDS


Differences between the Amortization and Sinking Fund Methods of repay-
ment In the Amortization Method for repaying a loan, the borrower makes regular
payments often level payments directly to the lender. The Outstanding Loan Bal-
ance at any time is then the net amount owing on the lenders books either the excess
of the accumulated value of the loan minus the accumulated value of the payments made
Retrospective Method or the present value of the payments yet to be made Prospective
Method .
Where a loan is repaid by a Sinking Fund , portions of the borrowers payments are is
not transmitted to the lender until a later date, usually when they have accumulated in
fund the Sinking Fund often at an interest rate different from the rate associated
with the loan. Where the borrower is required or permitted to make regular payments
directly to the lender in addition to those paid into the fund, those are described as
service on the loan; where those regular payments cover the interest costs on the loan,
the principal outstanding will remain constant. This should not be confused with the
Net Amount of the loan, which will be the excess of the outstanding principal over the
accumulated value of the sinking fund.
Where the interest rate associated with the Sinking Fund is the same rate as is being
paid on the loan, the Sinking Fund Method is equivalent to the Amortization Method.

1 1
The identity = + i. While the identity can easily be proved algebraically,
an sn
it admits an interesting verbal proof in terms of a loan of 1 being repaid over n periods.
1 1
is the regular payment necessary under an annuity-immediate. is the portion of
an sn
that regular payment that will accumulate in a sinking fund to a value of 1 just after
the nth payment; the complement i is the amount necessary to service the loan
annually until the time that the sinking fund matures.
Omit pages 178-179
Information for Students in MATH 329 2005 01 2120

[1, Exercise 20, p. 197] A has borrowed 10,000 on which interest is charged at 10%
effective. A is accumulating a sinking fund at 8% effective to repay the loan. At
the end of 10 years the balance in the sinking fund is 5000. At the end of the 11th
year A makes a total payment of 1500.

a) How much of the 1500 pays interest currently on the loan?


b) How much of the 1500 goes into the sinking fund?
c) How much of the 1500 should be considered as interest?
d) How much of the 1500 should be considered as principal?
e) What is the sinking fund balance at the end of the 11th year?

Solution: This problem requires attention to the terminology used.

1. Presumably we are to assume that the borrower is servicing the loan so that
the outstanding balance remains constant. The interest payment necessary at
the end of the 11th year will, therefore, be 0.10 10000 = 1000.
2. The remainder of the payment of 1500 is a contribution of 500 to the sinking
fund.
3. The net interest paid is the excess of the interest paid 1000 over the
8% 5000 = 400 interest earned by the sinking fund, or 600.
4. The excess of the contribution over the net interest payment can be assigned
to reducing the principal of the loan. The amount is 1500 600 = 900. This
can be considered as made up of two components: 500 which is paid into
the sinking fund, and 8% of 5000, which is 400 the interest earned by the
sinking fund, and which will ultimately be paid to the lender to retire the
loan.
5. At the end of the 11th year, the sinking fund balance is

5000(1.08) + 500 = 5900 .

[1, Exercise 21, p. 198] A loan of 1000 is being repaid with level annual payments of
120 plus a smaller final payment made one year after the last regular payment. The
effective rate of interest is 8%. Show algebraically and verbally that the outstanding
loan balance after the 5th payment has been made is:

a) 1000(1.08)5 120 s5
b) 1000 40s5 .

Solution: I give only the verbal explanations.


Information for Students in MATH 329 2005 01 2121

1. Interpret the loan as being amortized by the regular payments and the final
drop payment. The amount of 1000(1.08)5 120 s5 is that given by the
retrospective method.
2. Now interpret the loan as being repaid by the sinking fund method. As there
is no formal sinking fund, one may simply view a portion of the payments
as being contributed to a sinking fund in the hands of the lender, where the
sinking fund earns interest at the same rate as the principal. Each annual
payment of 120 may be interpreted as being the sum of the service cost of
80, plus a contribution of 40 to the sinking fund which will repay the loan at
maturity. After the 5th payment the value of the sinking fund is 40 s5 . The
outstanding loan balance will be the excess of the face value of the loan
(which has been serviced annually, so there is no additional accumulation of
interest) over the value of the sinking fund.

[1, Exercise 23, p. 198] On a loan of 10,000, interest at 9% effective must be paid
at the end of each year. The borrower also deposits X at the beginning of each
year into a sinking fund earning 7% effective. At the end of 10 years the sinking
fund is exactly sufficient to pay off the loan. Calculate X.
Solution: An equation of value at time t = 10 is X s10 7% = 10000, so

10000 0.07
X= = 676.4252593 .
((1.07)10 1) 1.07

The interest rate of 9% is totally irrelevant.

[1, Exercise 24, p. 198] A borrower is repaying a loan with 10 annual payments of
1,000. Half of the loan is repaid by the amortization method at 5% effective. The
other half of the loan is repaid by the sinking fund method, in which the lender
receives 5% effective on the investment and the sinking fund accumulates at 4%
effective. Find the amount of the loan.
Solution: Let the amount of the loan be L. The amortization of the loan of L2
L 0.025L
entails an annual payment of = . For the other half of the
2a10 5% 1 (1.05)10
loan the borrower must pay interest annually in the amount of 0.05 L2 = 0.025L.
These two expenses the amortization of half the loan, and the servicing of the
other half leave from his annual payment a balance of
0.025L
1000 0.025L
1 (1.05)10
Information for Students in MATH 329 2005 01 2122

L
which must accumulate at 4% in the sinking fund to produce a balance of 2
at
maturity. We have the equation of value
0.025L L 0.02L
1000 0.025L = =
1 (1.05)10 2s10 4% (1.04)10 1
40000
L= 1 0.8 = 7610.479836
1(1.05)10
+ 1 + (1.04)10 1
Information for Students in MATH 329 2005 01 2123

A.26 Supplementary Notes for the Lecture of March 14th, 2005


Distribution Date: Monday, March 14th, 2005, subject to further revision

The start of the lecture was delayed 15 minutes because of the expected
administration of a course evaluation. (However, the person who was to
administer it did not arrive.)

A.26.1 6.4 SINKING FUNDS (concluded)


Negative final payment? I begin this lecture with an example from an earlier chapter
which can be relevant to sinking fund problems. In [1, Example 3.7, pp. 7475] there is
a fund that has a designated target value, where the payments are not designed so that
an integer number of them will exactly bring the fund to its desired value. This means
that there can be a final payment which is less than a prescribed maximum value. But it
can also happen that, at the time of a payment, the outstanding balance is just slightly
larger than a prescribed maximum payment, so that, after the payment is made, a very
small balance remains. One waits for the next payment, and, during that time, the value
of the fund grows by an accumulation factor, and may exceed the target value. So the
next payment is negative! Read the example. This type of situation can occur in other
ways, so you should be prepared for it, even though it is not the most likely outcome.

[1, Exercise 25, p. 198] A borrows 12,000 for 10 years, and agrees to make semian-
nual payments of 1,000. The lender receives 12% convertible semiannually on the
investment each year for the first 5 years and 10% convertible semiannually for the
second 5 years. The balance of each payment is invested in a sinking fund earning
8% convertible semiannually. Find the amount by which the sinking fund is short
of repaying the loan at the end of the 10 years.
Solution: The interest payments for the first 10 half-years are 6% of 12,000, i.e.
720 per half-year; and, for the second 10 half-years, 600 per half-year. This leaves
280 at the end of each of the first 10 half-years, and 400 at the end of each of the
second 10 half-years to accumulate in the sinking fund, which earns 4% effective
every half year. The accumulated balance in the sinking fund at maturity will be
1
120s10 4% + 280s20 4% = 120 (1.04)10 1 + 280 (1.04)20 1
0.04

= 25 120(1.04)10 + 280(1.04)20 400
= 9778.594855

implying that the shortfall to repay the loan will be 12, 000 9778.59 = 2221.41.
Information for Students in MATH 329 2005 01 2124

[1, Exercise 26, p. 198] 1. A borrower takes out a loan of 3000 for 10 years at 8%
convertible semiannually. The borrower replaces one-third of the principal
in a sinking fund earning 5% convertible semiannually, and the other two-
thirds in a sinking fund earning 7% convertible semiannually. Find the total
semiannual payment.
2. Rework (a) if the borrower each year puts one-third of the total sinking fund
deposit into the 5% sinking fund and the other two-thirds into the 7% sinking
fund.
3. Justify from general reasoning the relative magnitude of the answers to (a)
and (b).
Solution:

1. The semiannual contribution to the sinking funds is


1000 2000
+
s20 2.5% s20 3.5%

and the semiannual interest payment is 4% of 3, 000, or 120. Hence the total
semiannual payment is
1000 2000 25 70
+ + 120 = + + 120
s20 2.5% s20 3.5% (1.025) 1 (1.035)20 1
20

= 229.8692824

2. Let the total sinking fund deposit be D. Then the equation of value at ma-
turity is
D 2D
s20 2.5% + s20 3.5% = 3000 ,
3 3
implying that
9000
D =
s20 2.5% + 2s20 3.5%
9000
= (1.025)20 1 (1.035)20 1
+2
0.025 0.035
= 109.6170427 ,

so the total semi-annual payment is 109.6170427+120=229.6170427.


3. In the original repayment scheme the portion of the payment contributed to
the 5% sinking fund grows more slowly than that to the 7% fund. Thus, while
the final accumulations in the funds will be in the ratio of 1:2, the proportion
Information for Students in MATH 329 2005 01 2125

of the contribution to the 5% fund would have been more than 13 . By reducing
that proportion to 31 we increased the interest earned by the fund, so a smaller
total contribution was required for the sinking fund.

[1, Exercise 27, p. 198] A payment of 36,000 is made at the end of each year for 31
years to repay a loan of 400,000. If the borrower replaces the capital by means of a
sinking fund earning 3% effective, find the effective rate paid to the lender on the
loan.
Solution: The annual contribution to the sinking fund is
400000 12000
= = 7999.571516.
s31 3% (1.03)31 1

Hence the annual interest payment is 36, 000 7, 999.57 = 28000.43, i.e., 7% of the
principal of 400,000.

[1, Exercise 28, p. 198] A 20-year annuity-immediate has a present value of 10,000,
where interest is 8% effective for the first 10 years, and 7% effective for the second
10 years. An investor buys this annuity at a price which, over the entire period,
yields 9% on the purchase price; and, further, allows the replacement of capital by
means of a sinking fund earning 6% for the first 10 years and 5% for the second 10
years. Find an expression for the amount that is placed in the sinking fund each
year.
Solution: The level annual payments under the annuity will be
10000
.
a10 8% + (1.08)10 a10 7%

It appears to be intended that the sinking fund payments be level also. If their
value is S, and the purchase price is P , then

S (1.05)10 s10 6% + s10 5% = P .

The purchase price satisfies the following equation of value at time 0:



10000
10
S a20 9% + (1.09)20 P = P ,
a10 8% + (1.08) a10 7%

which implies that



10000
S = 0.09P .
a10 8% + (1.08)10 a10 7%
Information for Students in MATH 329 2005 01 2126

Solving with the earlier equation yields


10000
S= .
a10 8% + (1.08)10 a 10 7% 1 + 0.09 (1.05)10 s10 6% + s10 5%

[1, Exercise 29, p. 199] A loan of 1 yields the lender rate i per period for n periods,
while the borrower replaces the capital in a sinking fund earning rate j per period.
Find expressions for the following if 1 t n:
1. Periodic interest paid to the lender.
2. Period sinking fund deposit.
3. Interest earned on sinking fund during the tth period.
4. Amount in sinking fund at end of the tth period.
5. Net amount of loan at the end of the tth period.
6. Net interest paid in period t.
7. Principal repaid in period t.
Solution:
1. The yield rate is i, so the lender must be receiving an amount of i each period.
2. The capital is 1, so the borrower is depositing sn1j regularly into the sinking
fund. (I understand that interest will be paid regularly, and the capital will
be repaid at maturity.)
s
t1 j
3. At the beginning of the tth period the balance in the sinking fund is sn j
;
s
t1 j
during the period it earns interest in the amount of j sn j
, payable at the
end of the period, i.e., at time t.
s
tj
4. The amount in sinking fund at end of the tth period is sn j
.
5. The net amount of loan at the end of the tth period is the excess of 1 over
s
the balance in the sinking fund, i.e., 1 s t j .
nj
6. The net interest paid in the tth period is the excess of interest paid over
s
interest earned, i.e. i j t1
s
j

nj
7. By 5. above, the change in the amount of the loan between the t 1th and
the tth payment is

st j st1 j st j st1 j (1 + i)t1
1 1 = =
sn j sn j sn j sn j
.
Information for Students in MATH 329 2005 01 2127

A.26.2 6.5 DIFFERING PAYMENT PERIODS AND INTEREST CON-


VERSION PERIODS
Omit this section.

A.26.3 6.6 VARYING SERIES OF PAYMENTS


Omit this section.

A.26.4 6.7 AMORTIZATION WITH CONTINUOUS PAYMENTS


Omit this section.

A.26.5 6.8 STEP-RATE AMOUNTS OF PRINCIPAL


Omit this section.
Information for Students in MATH 329 2005 01 2128

A.27 Supplementary Notes for the Lecture of March 16th, 2005


Distribution Date: Wednesday, March 16th, 2005, subject to further revision

Textbook Chapter 7. Bonds and other securities.

A.27.1 7.1 INTRODUCTION


The chapter is concerned with relations between the price of a security and its yield rate,
and with the value of a security at any time after it has been purchased, even at a time
that is not an interest compounding date.

A.27.2 7.2 TYPES OF SECURITIES


We shall confine our study to bonds, which are a commitment by the issuer to repay
a loan at a particular time, with interest payments according to a prescribed rate and
schedule. Many variations are possible, and this type of instrument is still evolving.
Read the book and become familiar with the following terms concerning bonds:

Definition A.16 The word bond originally had a much more general meaning: we
are using the word in the sense of a security that commits a borrower to pay one
or more specific sums at specific times, subject to detailed requirements of interest
and/or bonuses.

The term of the bond is the length of time from the date of issue until the date of
final payment, which date is the maturity date.

The detailed conditions of the bond may permit the bond to be called at some date
prior to the maturity date, at which time the issuer (=the lender) will repay the
commitment, possibly with some additional amounts. Such a bond is callable.

While the calling of a bond is at the initiative of the borrower, the lender (=the
purchaser) may possibly have the right to redeem the bond prior to the date of
maturity. Or, he may have some other type of right, e.g., to exchange the bond for
shares of the stock of the issuing company; this is a convertible bond.

A bond may be supplied with coupons portions of the paper bond that are to be
cut (=coupes) from the bond and exchanged for interest payments. Nowadays the
coupons may no longer be printed, but the purchaser may receive regular payments
from the lender. This can be the case if the bond is fully registered , so that the
issuer has the coordinates of the purchaser. (A bond could also be registered only
as to principal , in which case the coupons are of the traditional type.)
Information for Students in MATH 329 2005 01 2129

A coupon bond may be stripped, separating the coupons from the commitment
to repay the face value of the bond, following which the two parts may be sold to
separate purchasers.

The preceding is just a brief introduction: this is not the course to learn about the variety
of investment vehicles available in todays financial markets.
Information for Students in MATH 329 2005 01 2130

A.28 Supplementary Notes for the Lecture of March 21st, 2005


Distribution Date: Monday, March 21st, 2005, subject to further revision

A.28.1 7.2 TYPES OF SECURITIES (conclusion)


Unlike the problems we have been considering in earlier chapters, those here sometimes
involve technical definitions that are not necessarily intuitive, and need simply to be
memorized. I will eventually discuss with the class what definitions need to be absorbed
for examination purposes.

[1, Exercise 1, p. 240] Find the price which should be paid for a zero coupon bond
which matures for 1000 in 10 years to yield:

1. 10% effective
2. 9% effective
3. Thus a 10% reduction in the yield rate causes the price to increase by what
percentage?

Solution:

1. The bond is now worth 1000(1.10)10 = 385.5432894.


2. When the interest rate is reduced to 9%, the present value of the bond in-
creases to 422.4108069.
3. The 10% decrease in the interest rate thereby increases the price by
422.4108069
1 = 9.5624846%.
385.5432894

[1, Exercise 2, p. 240] A 10-year accumulation bond with an initial par value of 1000
earns interest of 8% compounded semiannually. Find the price to yield an investor
10% effective.
Solution:

Definition A.17 [1, p. 205] An accumulation bond is one in which the redemption
price includes the original loan plus all accumulated interest.

Solution: The only return payment is at maturity. The price to yield 10% interest
will therefore be
(1.10)10 1000(1.04)20 = 844.7728240.
Information for Students in MATH 329 2005 01 2131

[1, Exercise 3, p. 240] A 26-week (U.S.) T(reasury)-bill is bought for 9,600 at issue,
and will mature for 10,000. Find the yield rate computed as:

1. A discount24 rate, using the typical method for counting days on a T-bill25 .
2. An annual effective rate of interest, assuming the investment period is exactly
half a year.

Solution: Here is an example of a problem requiring some technical preparation.


While we have encountered the use of discount rather than interest in isolated
problems, this is the first time we have met it in a complex transaction.

1. The time is 26 weeks, i.e., 26 7 = 182 days, or, under the actual/360 system,
182
of a year. The discount rate will, therefore, be
360
360 400
= 7.912087912% .
182 10000
2. The effective interest rate for half a year is
400 1
= .
9600 24
The effective rate for a full year will be
2
1 1 1
1+ 1= + = 8.506944444% .
24 12 576

A.28.2 7.3 PRICE OF A BOND


Remember that a bond is essentially a contract to pay a large amount at maturity,
and smaller amounts of interest periodically on coupon dates until maturity. It differs
from the types of loans we have studied hitherto in that the contract is often (but not
always)26 transferable from one owner to another, and so it is reasonable to investigate
the value of the entire contract under conditions of varying yield.
24
simple discount
25
i.e., actual/360 [1, p. 39], using the exact number of days, but assuming 360 days in the year.
26
Of course, the issuer of a security cannot prevent the purchaser from arranging privately to transfer
the proceeds to another person, and to receive payment for that. What the issuer may be able to do is to
restrict the establishment of a secondary market at which the security may be routinely bought and sold,
and which would create a market value for the security. Some securities have restricted transferability
conditions, e.g., only on the death of the owner. And no security is immune from a court order.
Information for Students in MATH 329 2005 01 2132

Further definitions Familiarize yourself with the following terms, defined in the text-
book, and with the symbols usually used for them:
Definition A.18 1. The price P paid for a bond. In practice bond prices are usually
quoted in terms of a bond with face value (see next item) of 100.
2. The par value or face value or face amount. This amount is usually printed in
the bond contract, but may not be the amount paid at maturity. Its function
is to determine, once the coupon rate r has been specified, the magnitude of the
coupons.
3. The redemption value C is the amount paid when the bond is redeemed. When
a bond is redeemable at par, C = F . Where the redemption value exceeds the
face value, the word premium may be used for the excess; this word premium is
also used to denote the excess of the price paid for a bond over what would have
been the value if the yield rate was the same as the coupon rate.
4. The coupon rate r is the effective rate per coupon payment period, based on which
the amount of the coupon is calculated. The default payment period is a half-year.
5. The amount of a coupon is the product F r.
Fr
6. The modified coupon rate g = is the coupon rate per unit of redemption value,
C
rather than per unit of par value.
7. The yield rate or yield to maturity i is the actual interest rate earned by the
investor.
8. The number of coupon payment periods from the date of calculation until maturity
is denoted by n.
9. The present value of the redemption value, discounted back to the present by the
yield rate, is denoted by K; so K = C(1 + i)n .
Fr
10. The base amount G is : the amount which, if invested at the yield rate i, would
i
produce periodic interest payments equal to the coupons.
11. A callable bond is one where the lender has the right to declare that interest pay-
ments will stop and a bond may be redeemed at certain dates before the maturity
date; there could be a premium paid in addition to the redemption value, to en-
courage lenders to cash in the bond.
12. The word discount is often used where we have been using the word premium if
the premium is negative: the discount is the negative of the premium.
Information for Students in MATH 329 2005 01 2133

Four formul for price. We shall consider four different ways of determining the
present value, or price of a bond. The formulae are derivable one from the other: there
are situations where one may be more useful than another for specific applications; as
with many of the other formula we have met, the relative advantages were often linked to
the number of times that tables had to be consulted in computing the bond value; such
distinctions may no longer be significant. The most basic of the formul computes the
price of a bond as the present value of the coupons, interpreted as an annuity-immediate,
to which is added the present value K of the redemption value C of the bond, the face
value plus any premium that is payable upon redemption. This formula may be applied
either at the maturity date, or, if the bond can be called (by the issuer) or redeemed (by
the purchaser), at some earlier date.
Theorem A.9 (The Basic Formula) P = F r an i + Cv n = F r an i + K

Theorem A.10 (The Premium/Discount Formula) P = C + (F r Ci) an

Proof: This formula may be derived from the preceding by recalling that 1 = v n + i
an i .
Fr
The Base Amount G was defined above to be .
i
Theorem A.11 (The Base Amount Formula) P = G + (C G)v n
g
Theorem A.12 (Makehams Formula) P = K + (C K)
i
There are interesting verbal explanations of the preceding formul, to which we may
return.
[1, Exercise 5, p. 241] A 10-year 100 par value bond bearing a 10% coupon rate
payable semi-annually, and redeemable at 105, is bought to yield 8% convertible
semiannually. Find the price. Verify that all four formul produce the same
answer.
Solution:
F 100
C 105
r 5%
Fr 5
g = FCr 1
21
i 4%
n 20
K = C(1 + i)n 105(1.04)20 = 47.92
G = Fir 125
Information for Students in MATH 329 2005 01 2134

Basic Formula P = 100(0.05)a20 4% +105(1.04)10 = 100(0.05)a10 +105(1.04)20 =


115.8722611.
Premium/Discount Formula P = 105+(100(0.05)105(0.04))a20 4% = 115.8722611.
Base Amount Formula P = 125 + (105 125)(1.04)20 = 115.8722611.
1
Makehams Formula P = 105(1.04)20 + 0.04
21
(105 105(1.04)20 ) = 115.8722611.

[1, Exercise 6, p. 241] For the bond in [1, Example 7.3, pp. 212-213] determine the
following:

1. nominal yield based on the par value


2. nominal yield, based on the redemption value
3. current yield
4. yield to maturity.

(The bond is described as a 1000 par value 10-year bond with coupons at 8.4%
convertible semiannually, which will be redeemed at 1050, purchased to yield 10%
convertible semiannually..)
Solution:

1. The nominal yield is simply the annualized coupon rate on the bond. Since
the coupon rate is 8.4% (convertible...), that is the nominal yield.
2. This usage of the term nominal differs slightly from that given in the textbook.
The intention is that we should interpret the coupons as percentages of, not
the face value, but the redemption value, i.e, we should use the modified
coupon rate, which here is FCr = 1050
42
= 4%. But that is the effective rate
per half-year, so the corresponding nominal annual rate compounded semi-
annually is 8%.
3. The current yield is the ratio of the annualized coupon to the original price
of the bond [1, p. 221]. Here the coupon is 84, and the price has been shown
84
in the example to be 919.15, so the current yield is 919.15 = 9.139%.
4. The purchase price was calculated to yield 10% (convertible semi-annually).

[1, Exercise 7, p. 241] Two 100 par value bonds, both with 8% coupon rates payable
semi-annually are currently selling at par. Bond A matures in 5 years at par, while
Bond B matures in 10 years at par. If prevailing market rates of interest suddenly
go to 10% convertible semiannually, find the percentage change in the price of

1. Bond A
Information for Students in MATH 329 2005 01 2135

2. Bond B
3. Justify from general reasoning the relative magnitudes of the answers to (a)
and (b).

Solution:

1. For both of the bonds F = C = 100, r = 4%; for bond A n = 10. Using the
Premium/Discount formula we find that

P = C + (F r Ci)an 4%
1 (1.05)10
= 100 + 100(0.04 0.05)
0.05
= 92.27826507
7.72173493
The percentage change in the price of this bond is 100
= 7.72173493%.
2. For bond B n = 20. Using the Premium/Discount formula we find that

P = C + (F r Ci)an 4%
1 (1.05)20
= 100 + 100(0.04 0.05)
0.05
= 87.53778966
12.46221034
The percentage change in the price of this bond is 100
= 12.46221034%.
3. the last coupon payments under bond B are more affected by the rate change
than the first.

[1, Exercise 8, p. 241] Two 1000 bonds redeemable at par at the end of the same
period are bought to yield 4% convertible semiannually. One bond costs 1136.78,
and has a coupon rate of 5% payable semiannually. The other bond has a coupon
rate of 2 12 % payable semiannually. Find the price of the second bond.
Solution: For the first bond we have F1 = C1 = 1000, i1 = 2%, P1 = 1136.78,
r1 = 2.5%. By the Premium/Discount formula,

1136.78 = 1000 + (25 20) an 2% ,

implying that an = 27.356. For the second bond we have F2 = C2 = 1000, i2 = 2%,
r2 = 1.25%, n2 = n1 ,

P2 = 1000 + (12.5 20) an


= 794.83 .
Information for Students in MATH 329 2005 01 2136

[1, Exercise 9, p. 241] A 1000 bond with a coupon rate of 9% payable semiannually
is redeemable after an unspecified number of years at 1125. The bond is bought to
yield 10% convertible semiannually. If the present value of the redemption value is
225 at this yield rate, find the purchase price.
Solution: We have F = 1000, C = 1125, r = 4.5%, i = 5%, K = 225. From this
1 1
last fact we have 225 = 1125(1.05)n , so (1.05)n = 5, and an i = 0.055 = 16. By the
Basic Formula, the price is F r an + K = 45(16) + 225 = 945.

[1, Exercise 10, p. 241] A 1000 par value n-year bond maturing at par with annual
coupons of 100 is purchased for 1110. If K = 450, find the base amount G.
Solution: We have F = C = 1000, F r = 100 (so r = 10%), P = 1110, K = 450.
By Makehams formula,
Fr 100
1110 = 450 + (C K) = 450 + (1000 450)
Ci 1000i
1
so i = 12
, and
Fr
G= = 1200 .
i
[1, Exercise 11, p. 241] An investor owns a 1000 par value 10% bond with semian-
nual coupons. The bond will mature at par at the end of 10 years. The investor
decides that an 8-year bond would be preferable. Current yield rates are 7% con-
vertible semiannually. The investor uses the proceeds from the sale of the 10%
bond to purchase a 6% bond with semiannual coupons, maturing at par at the end
of 8 years. Find the par value of the 8-year bond.
Solution: We have C = F = 1000, r = 5%, n1 = 20, i = 3.5%. The price of the
bond presently owned is, by the premium/discount formula,

1 (1.035)20
P1 = C + (F r Ci)a20 3.5% = 1000 + (50 35) = 1213.186050 .
0.035
With these proceeds the investor buys a bond with par value F = C, n = 16,
r = 3%; by the premium/discount formula,

1 (1.035)16
1213.186050 = F + F (0.03 0.035) ,
0.035
implying that F = 1291.269895.

[1, Exercise 12, p. 241] An n-year 1000 par value bond matures at par and has a
coupon rate of 12% convertible semiannually. It is bought at a price to yield 10%
Information for Students in MATH 329 2005 01 2137

convertible semiannually. If the term of the bound is doubled, the price will increase
by 50. Find the price of the n-year bond.
Solution: We have F = C = 1000, r = 6%, i = 5%. Since we know that
2
a4n = 2 a2n i a2n ,

we can eliminate P between the two equations

P = 1000 + (60 50)a2n ,


P + 50 = 1000 + (60 50)a4n ,

and deduce that 2


a2n 20a2n + 100 = 0
so a2n = 10, and P = 1000 + 10(10) = 1100.
Information for Students in MATH 329 2005 01 2138

A.29 Supplementary Notes for the Lecture of March 21st, 2005


Distribution Date: Monday, March 21st, 2005, subject to further revision

A.29.1 7.2 TYPES OF SECURITIES (conclusion)


[1, Exercise 1, p. 240] Find the price which should be paid for a zero coupon bond
which matures for 1000 in 10 years to yield:
1. 10% effective
2. 9% effective
3. Thus a 10% reduction in the yield rate causes the price to increase by what
percentage?
Solution:
1. The bond is now worth 1000(1.10)10 = 385.5432894.
2. When the interest rate is reduced to 9%, the present value of the bond in-
creases to 422.4108069.
3. The 10% decrease in the interest rate thereby increases the price by
422.4108069
1 = 9.5624846%.
385.5432894
[1, Exercise 2, p. 240] A 10-year accumulation bond with an initial par value of 1000
earns interest of 8% compounded semiannually. Find the price to yield an investor
10% effective.
Solution:

Definition A.19 [1, p. 205] An accumulation bond is one in which the redemption
price includes the original loan plus all accumulated interest.

Solution: The only return payment is at maturity. The price to yield 10% interest
will therefore be
(1.10)10 1000(1.04)20 = 844.7728240.

[1, Exercise 3, p. 240] A 26-week (U.S.) T(reasury)-bill is bought for 9,600 at issue,
and will mature for 10,000. Find the yield rate computed as:
1. A discount27 rate, using the typical method for counting days on a T-bill28 .
27
simple discount
28
i.e., actual/360 [1, p. 39], using the exact number of days, but assuming 360 days in the year.
Information for Students in MATH 329 2005 01 2139

2. An annual effective rate of interest, assuming the investment period is exactly


half a year.
Solution:
1. The time is 26 weeks, i.e. 26 7 = 182 days, or, under the actual/360 system,
182
of a year. The discount rate will, therefore, be
360
360 400
= 7.912087912% .
182 10000
2. The effective interest rate for half a year is
400 1
= .
9600 24
The effective rate for a full year will be
2
1 1 1
1+ 1= + = 8.506944444% .
24 12 576

[1, Exercise 4, p. 241] T-bills of all maturities yield 8% compounded on a discount


basis. Find the ratio of the annual effective rate of interest earned on a 52-week
T-bill to that earned on a 13-week T-bill. Use an approach which does not involve
the counting of days.
Solution: The 52-week T-bill will earn interest at the rate
d 0.08
= = 8.695652174%.
1d 0.92
The 13-week T-bill earns interest quarterly at the rate
d 0.02
= = 2.173913043% ,
1d 0.92
so the effective annual rate is
(1.02173913043)4 1 = 8.9833377% ,
8.695652174
and the ratio to the 52-week rate to this is = 0.9679756527. THIS IS
8.9833377
1
NOT THE AUTHORS ANSWER. HE GIVES 1.0332. Note that =
0.9679756527
1.033083836. The effective rate for a shorter period should be higher than that for
a longer period because of the compounding, since the rates of 8% given are simple
discount rates.
Information for Students in MATH 329 2005 01 2140

A.29.2 7.3 PRICE OF A BOND


Remember that a bond is essentially a contract to pay a large amount at maturity,
and smaller amounts of interest periodically on coupon dates until maturity. It differs
from the types of loans we have studied hitherto in that the contract is often (but not
always) transferable from one owner to another, and so it is reasonable to investigate the
value of the entire contract under conditions of varying yield.
Familiarize yourself with the following terms, defined in the textbook, and with the
symbols usually used for them:
The price P paid for a bond. In practice bond prices are usually quoted in terms
of a bond with face value (see next item) of 100.
The par value or face value or face amount. This amount is usually printed in
the bond contract, but may not be the amount paid at maturity. Its function
is to determine, once the coupon rate r has been specified, the magnitude of the
coupons.
The redemption value C is the amount paid when the bond is redeemed. When
a bond is redeemable at par, C = F . Where the redemption value exceeds the
face value, the word premium may be used for the excess; this word premium is
also used to denote the excess of the price paid for a bond over what would have
been the value if the yield rate was the same as the coupon rate.
The coupon rate r is the effective rate per coupon payment period, based on which
the amount of the coupon is calculated. The default payment period is a half-year.
The amount of a coupon is the product F r.
Fr
The modified coupon rate g = is the coupon rate per unit of redemption value,
C
rather than per unit of par value.
The yield rate or yield to maturity i is the actual interest rate earned by the
investor.
The number of coupon payment periods from the date of calculation until maturity
is denoted by n.
The present value of the redemption value, discounted back to the present by the
yield rate, is denoted by K; so K = C(1 + i)n .
Fr
The base amount G is : the amount which, if invested at the yield rate i, would
i
produce periodic interest payments equal to the coupons.
Information for Students in MATH 329 2005 01 2141

A callable bond is one where the lender has the right to declare that interest pay-
ments will stop and a bond may be redeemed at certain dates before the maturity
date; there could be a premium paid in addition to the redemption value, to en-
courage lenders to cash in the bond.

The word discount is often used where we have been using the word premium if
the premium is negative: the discount is the negative of the premium.

Four formul for price.

Theorem A.13 (The Basic Formula) P = F r an + Cv n = F r an + K

Theorem A.14 (The Premium/Discount Formula) P = C + (F r Ci) an

Theorem A.15 (The Base Amount Formula) P = G + (C G)v n


g
Theorem A.16 (Makehams Formula) P = K + (C K)
i
[1, Exercise 5, p. 241] A 10-year 100 par value bond bearing a 10% coupon rate
payable semi-annually, and redeemable at 105, is bought to yield 8% convertible
semiannually. Find the price. Verify that all four formul produce the same
answer.
Solution:
F 100
C 105
r 5%
Fr 5
Fr 1
g= C 21
i 4%
n 20
n
K = C(1 + i) 105(1.04)20 = 47.92
Fr
G= i 125

Basic Formula P = 100(0.05)a20 4% +105(1.04)10 = 100(0.05)a10 +105(1.04)20 =


115.8722611.
Premium/Discount Formula P = 105+(100(0.05)105(0.04))a20 4% = 115.8722611.
Base Amount Formula P = 125 + (105 125)(1.04)20 = 115.8722611.
1
Makehams Formula P = 105(1.04)20 + 0.04
21
(105 105(1.04)20 ) = 115.8722611.
Information for Students in MATH 329 2005 01 2142

[1, Exercise 6, p. 241] For the bond in [1, Example 7.3, pp. 212-213] determine the
following:

1. nominal yield based on the par value


2. nominal yield, based on the redemption value
3. current yield
4. yield to maturity.

(The bond is described as a 1000 par value 10-year bond with coupons at 8.4%
convertible semiannually, which will be redeemed at 1050, purchased to yield 10%
convertible semiannually..)
Solution:

1. The nominal yield is simply the annualized coupon rate on the bond. Since
the coupon rate is 8.4% (convertible...), that is the nominal yield.
2. This usage of the term nominal differs slightly from that given in the textbook.
The intention is that we should interpret the coupons as percentages of, not
the face value, but the redemption value, i.e, we should use the modified
coupon rate, which here is FCr = 1050
42
= 4%. But that is the effective rate
per half-year, so the corresponding nominal annual rate compounded semi-
annually is 8%.
3. The current yield is the ratio of the annualized coupon to the original price
of the bond [1, p. 221]. Here the coupon is 84, and the price has been shown
84
in the example to be 919.15, so the current yield is 919.15 = 9.139%.
4. The purchase price was calculated to yield 10% (convertible semi-annually).

[1, Exercise 7, p. 241] Two 100 par value bonds, both with 8% coupon rates payable
semi-annually are currently selling at par. Bond A matures in 5 years at par, while
Bond B matures in 10 years at par. If prevailing market rates of interest suddenly
go to 10% convertible semiannually, find the percentage change in the price of

1. Bond A
2. Bond B
3. Justify from general reasoning the relative magnitudes of the answers to (a)
and (b).

Solution:
Information for Students in MATH 329 2005 01 2143

1. For both of the bonds F = C = 100, r = 4%; for bond A n = 10. Using the
Premium/Discount formula we find that

P = C + (F r Ci)an 4%
1 (1.05)10
= 100 + 100(0.04 0.05)
0.05
= 92.27826507
7.72173493
The percentage change in the price of this bond is 100
= 7.72173493%.
2. For bond B n = 20. Using the Premium/Discount formula we find that

P = C + (F r Ci)an 4%
1 (1.05)20
= 100 + 100(0.04 0.05)
0.05
= 87.53778966
12.46221034
The percentage change in the price of this bond is 100
= 12.46221034%.
3. the last coupon payments under bond B are more affected by the rate change
than the first.

[1, Exercise 8, p. 241] Two 1000 bonds redeemable at par at the end of the same
period are bought to yield 4% convertible semiannually. One bond costs 1136.78,
and has a coupon rate of 5% payable semiannually. The other bond has a coupon
rate of 2 12 % payable semiannually. Find the price of the second bond.
Solution: For the first bond we have F1 = C1 = 1000, i1 = 2%, P1 = 1136.78,
r1 = 2.5%. By the Premium/Discount formula,

1136.78 = 1000 + (25 20) an 2% ,

implying that an = 27.356. For the second bond we have F2 = C2 = 1000, i2 = 2%,
r2 = 1.25%, n2 = n1 ,

P2 = 1000 + (12.5 20) an


= 794.83 .
Information for Students in MATH 329 2005 01 2144

A.30 Supplementary Notes for the Lecture of March 23rd, 2005


Distribution Date: Wednesday, March 23rd, 2005, subject to further revision
The following problems are from the textbook, but were not discussed explicitly in the
lecture.
[1, Exercise 9, p. 241] A 1000 bond with a coupon rate of 9% payable semiannually
is redeemable after an unspecified number of years at 1125. The bond is bought to
yield 10% convertible semiannually. If the present value of the redemption value is
225 at this yield rate, find the purchase price.
Solution: We have F = 1000, C = 1125, r = 4.5%, i = 5%, K = 225. From this
1 1
last fact we have 225 = 1125(1.05)n , so (1.05)n = 5, and an i = 0.055 = 16. By the
Basic Formula, the price is F r an + K = 45(16) + 225 = 945.
[1, Exercise 10, p. 241] A 1000 par value n-year bond maturing at par with annual
coupons of 100 is purchased for 1110. If K = 450, find the base amount G.
Solution: We have F = C = 1000, F r = 100 (so r = 10%), P = 1110, K = 450.
By Makehams formula,
Fr 100
1110 = 450 + (C K) = 450 + (1000 450)
Ci 1000i
1
so i = 12 , and
Fr
G= = 1200 .
i
[1, Exercise 11, p. 241] An investor owns a 1000 par value 10% bond with semian-
nual coupons. The bond will mature at par at the end of 10 years. The investor
decides that an 8-year bond would be preferable. Current yield rates are 7% con-
vertible semiannually. The investor uses the proceeds from the sale of the 10%
bond to purchase a 6% bond with semiannual coupons, maturing at par at the end
of 8 years. Find the par value of the 8-year bond.
Solution: We have C = F = 1000, r = 5%, n1 = 20, i = 3.5%. The price of the
bond presently owned is, by the premium/discount formula,
1 (1.035)20
P1 = C + (F r Ci)a20 3.5% = 1000 + (50 35) = 1213.186050 .
0.035
With these proceeds the investor buys a bond with par value F = C, n = 16,
r = 3%; by the premium/discount formula,
1 (1.035)16
1213.186050 = F + F (0.03 0.035) ,
0.035
implying that F = 1291.269895.
Information for Students in MATH 329 2005 01 2145

[1, Exercise 12, p. 241] An n-year 1000 par value bond matures at par and has a
coupon rate of 12% convertible semiannually. It is bought at a price to yield 10%
convertible semiannually. If the term of the bound is doubled, the price will increase
by 50. Find the price of the n-year bond.
Solution: We have F = C = 1000, r = 6%, i = 5%. Since we know that
2
a4n = 2 a2n i a2n ,

we can eliminate P between the two equations

P = 1000 + (60 50)a2n ,


P + 50 = 1000 + (60 50)a4n ,

and deduce that 2


a2n 20a2n + 100 = 0
so a2n = 10, and P = 1000 + 10(10) = 1100.

Most of the lecture was devoted to a discussion of [5, Example 5.9, pp. 99-101]. In
that example the bond with r = 4% was purchased at a premium because the yield rate
was i = 3%. Then we considered the same problem with the two rates reversed (not
discussed explicitly in the cited source).
Information for Students in MATH 329 2005 01 2146

A.31 Supplementary Notes for the Lecture of March 30th, 2005


Distribution Date: Thursday, March 31st, 2005
corrected Friday, April 1st, 2005,
subject to further revision

A.31.1 7.4 PREMIUM AND DISCOUNT

Premium = P C = (F r Ci)an i
Discount = C P = (F r Ci)an i

In practice the word premium is used when P C > 0, and discount when P C < 0;
when P = C we speak of a purchase at par .

Book Value For accounting purposes it is necessary to show a gradual progression of


the value of a bond from purchase to maturity that is in some reasonable relationship
with its market value. There are a number of possible methods for doing this in practice,
but we shall adhere to the method whereby the value is shown to be what would be the
price if the yield rate does not change after the purchase of the bond. Of course, the yield
rate could very well change after the purchase, and so this is not a completely realistic
assumption. There are other methods for assigning a value to the bond. In this section
we shall consider the value only at coupon payment dates; generalization to other dates
will be considered in the next section.
When a bond is purchased at a premium29 the value of the redemption value and
the unpaid coupons just after the payment of the tth coupon i.e. of the book value
Bt will decrease until the maturity date. Thus the coupons may be interpreted as
consisting of the interest which is earned at the yield rate i and an amount to amortize
the premium. The redemption value C is C = Bn , and the price is P = B0 . We will use
this notation even when the bond is purchase with some of its coupons already paid (to
the vendor). The writing down of the premium may be shown on a schedule like

Coupon Coupon Interest Amount for Amortization Book


Number amount earned of Premium value
0 ... ...
1 Fr
... Fr ... ... ...
29
mutatis mutandis, at a discount
Information for Students in MATH 329 2005 01 2147

[1, Exercise 16, p. 242] For a bond of face value 1 the coupon rate is 150% of the
yield rate, and the premium is p. For another bond of 1 with the same number of
coupons and the same yield rate, the coupon rate is 75% of the yield rate. Find
the price of the second bond.
Solution: We are assuming that C = F for both bonds, and that the common
value is 1. For the first bond we have

1 + p = C + (F r Ci)an i = 1 + (1.5 1)ian i

so ian i = 2p. The price of the second bond is then, again by the Premium/Discount
formula,
p
1 + (0.75 1)ian i = 1 0.25ian i = 1 .
2
[1, Exercise 17, p. 242] For a certain period a bond amortization schedule shows
that the amount for amortization of premium is 5, and that the required interest
is 75% of the coupon. Find the amount of the coupon.
Solution: The bond was purchased at a premium, because the coupon rate exceeds
the yield rate. In the question, the author means by the amount of the coupon,
the total amount of all the coupons under discussion. We have

F rn F in = 5

and
F in
= 0.75 .
F rn
implying that F r = 4(5) = 20.

[1, Exercise 18, p. 242] A 10-year bond with semi-annual coupons is bought at a
discount to yield 9% convertible semiannually. If the amount for accumulation of
discount in the next-to-last coupon is 8, find the total amount for accumulation of
discount during the first four years in the bond amortization schedule.
Solution: We have i = 4.5%, n = 20,

2 Fr ri
C(g i)v = C i v2 = C = 8,
C (1 + i)2
so
C(g i) = 8(1 + i)2 .
Information for Students in MATH 329 2005 01 2148

The total amount for accumulation of discount during the first four years in the
schedule is

C(g i)(v 20 + v 19 + . . . + v 13 ) = 8(1 + i)2 (v 20 + v 19 + . . . + v 13 )


= 8v 14 a4 i
(1.045)10 (1.045)18
= 8 = 33.98
0.045

[1, Exercise 19, p. 242] A 1000 par value 5-year bond with a coupon rate of 10%
payable semi-annually and redeemable at par is bought to yield 12% convertible
semiannually. Find the total of the interest paid column in the bond amortization
schedule.
Solution: We have C = F = 1000, n = 10, r = 5%, i = 6%. The premium paid for
the bond is
1000(0.05 0.06)a10 6% = 73.60
(i.e., a discount of 73.60.) The total of the coupons is 10F r = 500., so the sum
of the coupons and the discount is 500.00 + 73.60 = 573.60, and this is the true
interest paid. While the problem did not ask us to actually set up the amortization
table, lets do it anyhow:

Coupon Coupon Interest Amount for Amortization Book


Number amount earned of Premium value
0 926.40
1 50.00 55.58 -5.58 931.98
2 50.00 55.92 -5.92 937.90
3 50.00 56.27 -6.27 944.17
4 50.00 56.65 -6.65 950.82
5 50.00 57.05 -7.05 957.87
6 50.00 57.47 -7.47 965.34
7 50.00 57.92 -7.92 973.26
8 50.00 58.39 -8.39 981.65
9 50.00 58.90 -8.90 990.55
10 50.00 59.43 -9.43 999.98
TOTAL 500.00 573.58 -73.58

The following example illustrates a problem where the price P of a bond is not determined
explicitly until an equation for P is determined and solved.
Example A.17 [1, Example 7.4, p. 219] Find the price of a 1,000 par value 2-year
8% bond with semi-annual coupons bought to yield 6% convertible semiannually, if the
Information for Students in MATH 329 2005 01 2149

investor plans to replace the premium by means of a sinking fund earning 5% convert-
ible semi-annually. (Note: The intention is that this plan for a sinking fund has been
considered in calculating the yield rate of the bond.)
Solution: Note that, following the usual convention, the 8% rate is interpreted as a
nominal interest rate compounded twice a year.
The unknown is the price, P . The coupons each have value 0.04 1000 = 40, but
the interest earned is 3% of the price P , which remains to be determined. The sinking
fund is to mature at value P 1000 after 2 years, i.e., after 4 contributions. Thus an
equation of value is
(40 .03P )s4 2.5% = P 1000 . (102)
This can be solved for P :
1000 + 40s4 2.5%
P =
1 + 0.03s4 2.5%
= 1, 036.93 .
Note the assumption used in equation (102): all of the coupon that exceeds the interest
earned at the yield rate is contributed to the sinking fund; this was not explicitly stated
in the problem, but is the authors interpretation. If, for example, the purchaser had
decided that he would allocate only half of the excess to his sinking fund contributions,
then the price of the bond would be about 1082.43.

A.31.2 7.7 CALLABLE BONDS


[1, Exercise 31, p. 243] A 1000 par value bond has 8% semiannual coupons, and is
callable at the end of the 10th through the 15th years at par.
1. Find the price to yield 6% convertible semiannually.
2. Find the price to yield 10% convertible semiannually.
Solution:
1. Let n be the coupon number at whose date the bond is called matures. Then,
by the Premium/Discount Formula
P = 1000 + (40 30)an 3% , (n = 20, 22, 24, 26, 28, 30).
Without knowing which will be the date of call, we take the worst possible
date in order to minimize the price; since an 3% is an increasing function of n,
and is multiplied by a positive number, 10, we minimize by making n as small
as possible, i.e., 2 10 = 20:
P = 1000 + (40 30)a20 3% = 1000.00 + 148.78 = 1148.78 .
Information for Students in MATH 329 2005 01 2150

2. When the semi-annual yield rate is 5%, the multiplier is negative, 40 50 =


10, and we must choose the largest value of n, i.e., n = 30, for a price of

P = 1000 + (40 50)a30 5% = 1000.00 153.72 = 846.28 .

[1, Exercise 32, p. 243] If the bond in Exercise 31(b) were actually called at the end
of 10 years, find the yield rate to the owner of the bond.
Solution: The yield rate i satisfies the equation

846.28 = 1000 + 1000(0.04 i)a20 i ,

and can be found by iteration to be 10.52%%.


[1, Exercise 33, p. 243] A 1,000 par value 8% bond with quarterly coupons is callable
five years after issue. The bond matures for 1,000 at the end of 10 years, and is
sold to yield a nominal rate of 6% convertible quarterly, under the assumption that
the bond will not be called. Find the redemption value at the end of 5 years that
will provide the purchaser the same yield rate.
Solution: If the bond is certain not to be called, its present value (i.e., its price) is

1000 + (20 15)a40 1.5% = 1000 + 5(29.9258) = 1149.63.

Let x denote the redemption value after 5 years that will provide the same yield
rate. Then
1149.63 = x + (20 0.015(x))a20 1.5%
which implies that
1149.63 20a20 1.5%
x= = 1085.91 .
1 0.05a20 1.5%

[1, Exercise 34, p. 243] A 1000 par value 4% bond with semiannual coupons matures
at the end of 10 years. The bond is callable at 1050 at the ends of years 4 through
6, at 1025 at the ends of years 7 through 9, and at 1000 at the end of year 10. Find
the maximum price that an investor can pay and still be certain of a yield rate of
5% convertible semiannually.
Solution: We have to find the minimum of the following prices based on the given
call dates and premiums:

1050.00 + (20.000 26.250)an 2.5% (n = 8, 10, 12) (103)


1025.00 + (20.000 25.625)an 2.5% (n = 14, 16, 18) (104)
1000.00 + (20.000 25.000)a20 2.5% . (105)
Information for Students in MATH 329 2005 01 2151

In each case the coefficient of an 2.5% is negative, so the lowest value will be when
n is as large as possible; that is, we have to compare the following three amounts

1050.00 + (20.000 26.250)a12 2.5% = 1050 (6.250)10.2578 = 985.89


1025.00 + (20.000 25.625)a18 2.5% = 1025 (5.625)14.3534 = 944.26
1000.00 + (20.000 25.000)a20 2.5% = 1000 (5.000)15.5892 = 922.05 ,

whose minimum is the last, the price of the bond if not called before maturity.
That is the highest price the investor may pay if she wishes to be sure that the
yield will not be less than 5% convertible semiannually.

[1, Exercise 35, p. 243] A 1,000 par value 6% bond with semiannual coupons is
callable at par 5 years after issue. It is sold to yield 7% under the assumption
that the bond will be called. The bond is not called, and it matures at the end
of 10 years. The bond issuer redeems the bond for 1000 + X without altering the
buyers yield rate of 7% convertible semiannually. Find X.
Solution: Under the assumption that the bond will be called at par 5 years after
issue, its price, when yielding 3.5% effective semi-annually, would be

1000 + (30 35)a10 3.5% = 958.42 .

The premium upon maturity will be given by the equation

958.42 = 1000 + X + (30 (0.035)(1000 + X))a20 3.5% .

which implies that


958.42 30a20 3.5%
X= 1000 = 58.66 .
1 0.035a20 3.5%

Example A.18 We discussed in class the following problem: [1, Example 7.9, p. 231]
Consider a $100 par value 4% bond with semiannual coupons callable at $109 on any
coupon date starting 5 years after issue for the next 5 years, at $104.50 starting 10 years
after issue for the next 5 years, and maturing at $100 at the end of 15 years. What is
the highest price which an investor can pay and still be certain of a yield of: (1) 5%
convertible semiannually, and (2) 3% convertible semiannually?
Solution: The solution was discussed in class. I recommended that you use the Pre-
mium/Discount Formula, and remember that the function an i is an increasing function
of n; this means that, in a problem of this type, the values least favourable to the in-
vestor will always be at the end of an interval of payment periods: at the lower end when
F r Ci is positive, and at the upper end when F r Ci is negative. The intervals to be
Information for Students in MATH 329 2005 01 2152

considered will be consecutive payment dates when the call premium has the same value.
Each of these intervals need to be treated separately, and then the least favourable from
each interval taken together and the least favourable of them determined.
The textbook states, in connection with the part of the problem where the yield
rate is 3%, that it is immediately clear that the latest possible redemption date is less
favourable to an investor, since the bond will be selling at a discount. This statement
may not be clear. The factor that determines which end of the interval is least favourable
is F r Ci = C(g i), where g is the modified coupon rate (cf. Definition A.18.4, p.
2132). The given data lead to finding the minimum of the following three values:

100 + (2 2.50)a30 0.025 = 89.53


109 + (2 2.725)a29 0.025 = 94.17
104.50 + (2 2.6125)a19 0.025 = 95.33

And, indeed, the minimum will be when n = 30. The situation is best viewed through
the Base amount formula:
P = G + (C G)v n
where G = Fir (cf. Definition A.18.10). Since v n is a decreasing function of n, what is
critical is the sign of
Fr
C G=C .
i
Information for Students in MATH 329 2005 01 2153

A.32 Supplementary Notes for the Lecture of April 1st, 2005


Distribution Date: Friday, April 1st, 2005, subject to further revision

Following is an example of a type of problem we have seen once on an assignment. This


example was not discussed in the lecture.

An amortization problem

Example A.19 [5, Exercise 4-9, p. 87] A loan is being repaid with 30 equal annual
installments, at i = 0.17. In what installment are the principal and interest portions
most nearly equal to each other?
Solution: Let A be the amount of the loan, received at time n = 0, and let X be the
annual payment. An equation of value at time n = 0 is Xa30 = A, yielding the value of
the annual payment to be
A
X= .
a30
By the prospective method we see that the unpaid principal immediately after the (r
1)st payment is
a31r 1 v 31r
Xa31r = A =A . (106)
a30 1 v 30
The unpaid principal is a decreasing function of r, and the interest earned on it during
the next period and included in the next regular payment is its product with i, which will
also be a decreasing function of r. Since fixed payments are being made, and the portion
of the payment that is interest will be decreasing, the balance of the rth payment, which
is applied to reduction of principal, is an increasing function of r, namely

A 1 a31r
1 i a31r = A ; (107)
a30 a30

note that this amount is always positive until the loan is repaid. If, in the rth payment,
the amount of interest is less than or equal to the amount for reduction of principal,
then the same property will hold for all subsequent payments; if, in the rth payment,
the amount of interest is greater than or equal to the amount for reduction of principal,
then the same property will hold for all prior payments. The observations just made
about functions increasing and decreasing hold even for non-integral values of r. So one
way of solving the present problem is to equate the interest and principal formul, and
solve for r. If r is not an integer which will usually be the case we know that the
Information for Students in MATH 329 2005 01 2154

payment for which the difference between interest and principal is minimal will be one
of payments ##brc and dre (the integers which are closest to r from below and above).
a31r v 31r
iA =A
a30 a30
1
v 31r =
2
31r
(1.17) =2
(31 r) ln 1.17 = ln 2
ln 2
r = 31 = 31 4.41 = 26.58
ln 1.17
so the candidates for closest interest and principal are payments ##26 and 27. The
difference between the interest and principal in payments ##26 and 27 are, respectively,

1 2v 3126 0.087777695
A = = 0.0886
1 v 30 0.990996235

1 2v 3127 0.067300096
A = = 0.0679
1 v 30 0.990996235

so the payment where these are closest is #27.


The student should note that the formul we have derived above in (106) and (107)
are generally true when a loan is amortized in equal payments, and are found in [5, pp.
78, 79]. But be cautioned these formul are not necessary applicable under other
conditions, e.g. where payments are not level.

30
A sinking fund problem

Example A.20 A borrower takes out a loan of 20000 for three years. Construct a
sinking fund schedule where the lender receives a nominal rate of 10% effective semi-
annually and paid semi-annually on the loan, and the borrower replaces the amount of
the loan with semi-annual deposits into a sinking fund earning 8% convertible quarterly.
Use the following headings
Duration Contribution Interest Interest Earned Balance of
(Months) to Sinking Fund on Loan in Sinking Fund Sinking Fund
0 0 0 0 0
3 ... ... ... ...
6 ... ... ... ...
30
Source of problem: modified from Final Examination in Math 329, April, 2000
Information for Students in MATH 329 2005 01 2155

Solution: Every six months the borrower pays the interest that has accrued on the loan
at the rate of 10
2
% = 5% per half-year; i.e. the amount of 5% of 20000, or 1000. In
order that the sinking fund accumulate an amount of 8% convertible quarterly, i.e. 2%
quarterly, the borrower must make 6 semi-annual payments X which satisfy the equation
X s6(1.02)2 1 = 20000
whence
20000
X =
s61.022 1
808
= = 3012.21
(1.02)12 1
Duration Contribution Interest Interest Earned Balance of
(Months) to Sinking Fund on Loan in Sinking Fund Sinking Fund
0 0.00 0.00 0.00 0.00
3 0.00 0.00 0.00 0.00
6 3012.21 1000.00 0.00 3012.21
9 0.00 0.00 60.24 3072.45
12 3012.21 1000.00 61.45 6146.11
15 0.00 0.00 122.92 6269.03
18 3012.21 1000.00 125.38 9406.62
21 0.00 0.00 188.13 9594.75
24 3012.21 1000.00 191.90 12798.86
27 0.00 0.00 255.98 13054.84
30 3012.21 1000.00 261.10 16328.15
33 0.00 0.00 326.56 16654.71
36 3012.20 1000.00 333.09 20000.00
Note that the last semi-annual payment has been reduced by 1 cent. An equivalent
table could have been compiled with 6 months intervals, with semi-annual interest rate
(1.02)2 1 = .0404:
Duration Contribution Interest Interest Earned Balance of
(Months) to Sinking Fund on Loan in Sinking Fund Sinking Fund
0 0.00 0.00 0.00 0.00
6 3012.21 1000.00 0.00 3012.21
12 3012.21 1000.00 121.69 6146.11
18 3012.21 1000.00 248.30 9406.62
24 3012.21 1000.00 380.03 12798.86
30 3012.21 1000.00 517.07 16328.14
36 3012.20 1000.00 659.66 20000.00

UPDATED TO April 12, 2005


Information for Students in MATH 329 2005 01 2156

31
Another final examination problem

Example A.21 An investor plans to buy a twenty-year mortgage whose amount is


150,000, having equal monthly payments; the nominal interest rate is 6%, compounded
monthly. The investor plans to invest the monthly payments she receives in a savings ac-
count earning a nominal annual rate of 3%, compounded monthly, in order to accumulate
a retirement fund after 20 years.

1. What is the monthly payment on the mortgage?

2. What should the investor pay for this mortgage, in order to receive an effective
annual yield rate of 8%?

3. How much is in the retirement fund after 10 years.

4. How much is in the retirement fund after 20 years?

Solution:

1. The mortgage is amortized over 20 years, i.e., 240 months. The monthly payment
X has the property that X a240 6 % = 150000, hence
12

150000
X= = 1074.65
a2400.5%

2. (a) Suppose that the intention of the problem is that the mortgage
payments are still to be placed in the sinking fund. The sinking fund
becomes available only at the end of 20 years. Discounting back to the present
at 8%, we find its present value, i.e. the price to be paid, to be

(1.08)20 1074.65 s2400.025%


s
= (1.08)20 150, 000 2400.025%
a2400.05%
= 75, 694.46.

(b) Suppose that the intention of the problem is that the mortgage
payments are no longer to be placed in a sinking fund. Let i denote
the monthly interest rate that, when compounded 12 times, is equivalent to
31
Source of problem: modified from Final Examination in Math 329, April, 1998.
Information for Students in MATH 329 2005 01 2157

1
an annual rate of 8%; i.e. i = (1.08) 12 1. The present value of all future
payments is
1 (1 + i)240
X a240i = X 1
(1.08) 12 1
1 (1.08)20
= X 1
(1.08) 12 1
= 1074.65 122.0777 = 131190.83

3. After 10 years the payments have accumulated to X s1200.0025 = 1074.65139.74 =


150171.59.
4. After 20 years the payments have accumulated to X s2400.0025 = 1074.65328.30 =
352807.60.
The following is not examination material.

A.32.1 7.6 DETERMINATION OF YIELD RATES


Omit this section.

A.32.2 7.5 VALUATION BETWEEN COUPON PAYMENT DATES


In the preceding section we have considered the price of a bond on coupon payment
dates. We have chosen to use the term book value for the adjusted value of the bond
on coupon payment dates after a coupon has been paid: the book value is the residual
value of the instrument, where the yield rate is taken to be the rate in effect when the
bond was purchased. These rates are related by the recurrence
Bt+1 = (1 + i)Bt F r . (108)
Do not confuse this with the equations of value that we computed earlier in the course,
where we found that, under compound interest, sums of money could be moved forward
and backwards on in time by multiplication by the appropriate power of 1 + i. That
principle is still in effect, but the sums we are considering the book values at different
times are not one single value, but a whole sequence of values; we normally are
interested in Bt precisely at time t, and it is not going to be the same as the book value
Bt+1 at time t + 1 for 2 reasons:
maturity is one year further away;
the instrument guarantees one more coupon payment.
One way to prove equation (108) is by considering the value of Bt at time t + 1.
Information for Students in MATH 329 2005 01 2158

Market price and Flat price. The market price on a coupon payment date is what
we earlier called simply the price, and will have an associated yield rate. It will be
the book value of a previously purchased bond when the yield rate has not changed since
purchase, and we will normally make such an assumption. But bonds are not always
purchased on a coupon payment date; and, even when they are, there is normally a delay
between the date of the formal order and the delivery of the bond. So we need to have
a method for computing a reasonable price at times between coupon payment dates.
The Flat Price is the actual sum of money that changes hands at the time of formal
purchase. It can be interpreted as being made up of two elements:

the Market Price; and

the Accrued Coupon

The Flat Price at time t + k between coupon dates ##t, t + 1 is obtained from the Flat
Price just after the payment of the coupon #t by one of two methods:

Theoretical Method: Compound interest, producing (1 + i)k Bt ;

Practical Method: Simple interest, producing (1 + ki)Bt .

The Accrued Coupon can also be calculated by both a Theoretical and a Practical
method:

(1+i)k 1
Theoretical Method: Compound interest, producing F r i
;

Practical Method: Simple interest, producing kF r.

(Even here there are still some conventions needed, in the way in which we determine
the time interval k as a fraction of a period.) In the Semi-theoretical Method the Flat
Price is calculated by the theoretical method, but the accrued coupon is calculated by
simple interest, i.e., by the Practical Method.
Note that it is the Flat Price that is being calculated by the choice of methods,
not the Market Price. The Market Price can be obtained from the two elements by
subtraction:

Market Price = Flat Price - Accrued Coupon

In practice prices are quoted in the form

Flat Price = Market Price + Accrued Coupon


Information for Students in MATH 329 2005 01 2159

The graph of the Book Value (which we take to be the same as Market Price) will be
monotonely increasing when i > r, constant when i = r, and monotonely decreasing
when i < r. Between the values at integer points the graph will be linear in the case
of the Practical Method, and exponential in the case of the theoretical method, but
the differences between the two are very small; in either case the function is continuous.
However, the Flat Price will be discontinuous at coupon payment dates, because its value
drops each time a coupon is paid, by the value of the coupon.

A.32.3 7.8 SERIAL BONDS


Omit this section

A.32.4 7.9 SOME GENERALIZATIONS


Omit this section.

A.32.5 7.10 OTHER SECURITIES


Omit this section.

A.32.6 7.11 VALUATION OF SECURITIES


Omit this section.
Information for Students in MATH 329 2005 01 3001

B Problem Assignments, Tests, and Examinations


from Previous Years
B.1 2002/2003
B.1.1 First 2002/2003 Problem Assignment, with Solutions
In all of the following problems students were are expected to show your work.

1. (a) What principal will earn interest of 100 in 7 years at a simple interest rate of
6%?
(b) What simple interest rate is necessary for 10,000 to earn 100 interest in 15
months?
(c) How long will it take for money to double at a simple interest rate of 8%?
(d) For the rate stated and the period of time computed in the previous part of
the question, what would 1 grow to if interest were compounded annually?
Solution:

(a) Let P denote the unknown principal. Equating the interest earned, P (0.06)7
100
to 100 and solving for P , we obtain that P = 70.06 = 238.10.
(b) If the interest rate is i, the amount of interest earned will be i 15
12
10000 = 100.
4 100
Solving yields i = 5 10000 = 0.008 or 0.8%.
(c) Let the number of years for money to double be denoted by t. We solve
21
1 + (0.08)t = 2: t = 0.08 = 12.5. Money doubles in 12 12 years.
(d) (1.08)12.5 = 2.62.

2. The total amount of a loan to which interest has been added is 20,000. The term
of the loan was four and one-half years.

(a) If money accumulated at simple interest at a rate of 6%, what was the amount
of the loan?
(b) If the nominal annual rate of interest was 6% and interest was compounded
semi-annually, what was the amount of the loan?
(c) If the rate of interest was 6%, interest was compounded annually for full years,
but simple interest was paid for the last half-year, what was the amount of
the loan?
(d) If the rate of interest was 6%, interest was compounded annually for full and
part years, what was the amount of the loan?
Information for Students in MATH 329 2005 01 3002

(e) If the effective annual rate of interest was 6%, but interest was compounded
semiannually, what was the amount of the loan?
(f) If the nominal annual rate of interest was 6%, but interest was compounded
continuously, what was the amount of the loan?
(g) If interest was compounded continuously, and the force of interest was 6%,
what was the amount of the loan?

Solution:

(a) If the amount of the loan is P , then 20000 = (1 + 0.06 4.5)P = 1.27P , so
P = 20000
1.27
= 15, 748.03.
0.06 24.5
(b) If the amount of the loan is P , then 20000 = (1 + 2
) P,

P = 20000(1.03)9 = 15, 328.33 .


0.06

(c) If the amount of the loan is P , then 20000 = (1 + 0.06)4 1 + 2
P,

P = 20000(1.06)4 (1.03)1 = 15, 380.46 . (109)

(d) If the amount of the loan is P , then 20000 = (1+0.06)4.5 P , P = 20000(1.06)4.5 =


15, 386.99.
(e) If the semi-annual rate of interest is denoted by i, then (1 + i)2 = 1.06, so
1
1 + i = (1.06) 2 . If the amount of the loan is P , then 20000 = (1 + i)9 P =
(1.06)4.5 P , so P = 20000 (1.06)4.5 = 15, 386.99.
Note that this is exactly the same principal as in the preceding version of the
problem, since it is precisely the same problem! Note also that the amount
of the principal is slightly more than in the version in part 2c, since simple
interest for a fraction of a year in that case will have a higher yield than
compound interest in this case.
(f) We are told that the nominal rate of interest, compounded instantaneously,
is 6%; thus, if i is the effective annual rate, 0.06 = = ln(1 + i); equivalently,
e0.06 = 1 + i, so i = 6.18365%. In a full year an amount of 11 will grow by a
factor 1.0618365; in a half year, by a factor 1.0618365. In 4 2 years we have
P (1.0618365)4.5 = 20000, so P = 20000(1.0618365)4.5 = 20000(0.763379) =
15267.59.
(g) The force of interest is the nominal rate of interest which is convertible contin-
uously [5, p. 17]. We are told that interest was compounded continuously. The
effective annual rate of interest, which we shall denote by i, has the property
Information for Students in MATH 329 2005 01 3003

that 0.06 = ln(1 + i), or that 1 + i = e0.06 . If the amount of the loan is P ,
then 20000 = (1 + i)4.5 P = e4.50.06 = e0.27 P , so P = 20000e0.27 = 15267.59,
the same result as in the preceding part.
3. (cf. [5, Exercise 1-13, p. 24]) Henry plans to have an investment of 10,000 on
January 1, 2006, at a compound annual rate of discount d = 0.11.
(a) Find the value that he would have to invest on January 1, 2003.
(b) Find the value of i corresponding to d.
(c) Using your answer to part (b), rework part (a) using i instead of d. Do you
get the same answer?
Solution:
(a) The accumulation of 10,000 will have to be discounted by a factor of (10.11)
three times to reduce it by compound discount to January 1, 2003. The
amount to be invested is, accordingly, 10000(1 0.11)3 = 7049.69.
(b) The relationship between d and i is given, for example, by (1 + i)(1 d) = 1,
d
which implies that i = 1d . Here
0.11 11
i= = = 0.1233596.. = 12.36..%.
1 0.11 89
1 1
(c) When i = 12.36%, v = 1+i = 1.1236 = 0.89. The value on January 1, 2003 of
the 10,000 expected on January 1, 2006 will then be 10000(0.89)3 = 7049.69,
as before.
4. (cf. [5, Exercise 1-24, p. 26]) Recall that (cf. [5, (1.21)])
m m
i(m) 1 d(m)
1+ =1+i= = 1 . (110)
m 1d m
(a) Determine whether there is an integer n such that
i(2)
i(n) 1+ 2
1+ = (111)
n 1+ i(3)
3
and, if there is such an integer, find it.
(b) Replace the right member of (111) by a product

i(2) d(3)
1+ 1
2 3
and then interpret this product verbally to show that it must be equal to
(n)
1 + i n if a suitable n exists.
Information for Students in MATH 329 2005 01 3004

Solution:
(a) i(2) is the nominal annual interest rate which, when compounded semi-annually,
(2) (3)
yields an effective annual rate of i. Thus 1 + i 2 = 1 + i; similarly, 1 + i 3 =
(2)
1+ i 2 1 1 1
3
1 + i. The ratio i(3) is, therefore, equal to (1 + i) 2 3 = (1 + i) 6 , which is
1+ 3
1
the accumulation of 1 after a period of 6
of a year; this is, by definition, equal
(6)
to 1 + i 6 .
(2)

(b) Under an effective annual interest rate of i, the factor 1 + i2 is the value
12
of 1 after = 6 months. If this amount
2 is discounted
back 12
= 4 months,
3
(3)
it decreases by a reduction factor of 1 d 3 . The result is equivalent to a
1
net accumulation period of 6 4 = 2 months, i.e. 6
of a year, under which it
(6)
would grow by a factor 1 + i 6 .
5. (cf. [5, Exercise 1-30, p. 27]) Show that f (t) = (1 + i)t (1 + it) is minimized at
t = ln iln


.
Solution:
If only elementary calculus is used, this problem is more difficult than
it looks. Students were accorded a full grade for showing that the point
claimed is, indeed, a local minimum; the proof that it is a global =
absolute minimum, is more difficult; one possible solution is given below.
No attempt has been made to produce a compact solution.
Applying elementary calculus, we find that
f 0 = (1 + i)t ln(1 + i) i = (1 + i)t i
f 00 = (1 + i)t 2
To find the critical points of the function, we solve for t the equation f 0 (t) = 0.
Taking natural logarithms yields
t ln(1 + i) + ln = ln i
which is satisfied only for
ln i ln ln i
t0 = = . (112)

The second derivative, f 00 (t0 ) is positive everywhere, since it is the product of an
exponential always positive and the square of a real number; this tells us
that the point t = t0 (112) is a local minimum.
Information for Students in MATH 329 2005 01 3005

Does this completely solve the problem? Not yet! To solve an extremum problem
we need to interpret local extremum information with reference to the domain of
the function. For example, if the domain is infinite, then the function might not
even have a global or absolute minimum, even though it has a local minimum.32
And, if the domain of the function is a closed interval, we need to investigate the
behavior at the end points of that interval. The function f is meaningful for all real
values of t. One interpretation would be to take the domain to be t 0; another
interpretation would be to take the domain to be t +.
What follows is just one possible way of completing this problem. We observe that
f (0) = f (1) = 0. Could f (t) = 0 for t different from 0, 1? Rolles theorem implies
the existence of a point with zero slope between any two zeros of the function; as we
have seen that there is only one such point with zero slope, there cannot exist more
than two zeros of the function: and thus the point t0 is the only local extremum.
Thus, by the Intermediate Value Theorem, f has the same sign throughout each
of the intervals < t < 0, 0 < t < 1, 1 < t. As t , lim f (t) ; hence
f (t) > 0 for all t > 1; as t , lim f (t) ; so the function is positive
in the interval < t < 0 also. Thus the global minimum is in the interval
0 t 1; and, from our investigation of the critical point, we know that the
minimum is attained at one (or more) of t = 0, t = 1 or t = t0 . We can complete
this investigation if we can argue that f (t0 ) < 0. As we know the sign of the
function will be the same throughout the interval 0 < t < 1, we can take any
convenient value of t in that interval.

1 i
f = 1+i 1+
2 2

1 + i (1 + 2i ) 1 + i + (1 + 2i )
=
1 + i + (1 + 2i )
2

(1 + i) 1 + i + i4
=
1 + i + 1 + 2i
i2 1
= <0
4 1 + i + 1 + 2i

Thus the global minimum is attained at t0 .

6. (cf. [5, Exercise 1-33, p. 27]) Find the accumulation function a(t) if it is known
that t = 0.04(1 + t)1 for t > 0.
32
Consider, for example, the function t3 t, which has a local minimum at t = 1, a local maximum at
t = 1, but has neither a global maximum nor a global minimum over its entire domain < t < +.
Information for Students in MATH 329 2005 01 3006

Solution: Applying definition [5, (1.28), p. 19], we solve the differential equation
d
ln(a(t)) = 0.04(1 + t)1 :
dt
Integration gives
Z
0.04
ln(a(t)) = dt = 0.04 ln(1 + t) + C
1+t
where C is the constant of integration. Setting t = 0, where we know, by definition,
that a(0) = 1, we have
0 = ln 1 = 0.04 ln 1 + C
so C = 0, and
0.04
a(t) = e0.04 ln(1+t) = eln(1+t) = (1 + t)0.04 .

7. Let () denote the value of 1 at the end of 3 years, accumulated at an effective


rate of interest ; let () denote the present value of 1, to be paid at the end of 3
years at an effective rate of discount numerically equal to . Suppose it is known
that () + () = 2.0294. Determine .
Solution: (cf. [2, Exercise 52, p. 30]) () = (1 + )3 ; () = (1 )3 . Summing
2
yields ()
q+ () = 2 + 6 , which we equate to 2.0294, and from which we infer
0.0294
that = 6
= .07 = 7%.

8. Showing your work, determine a formula in terms of the force of interest, , for
the number of years that are needed for a sum of money to double itself. Verify
your answer by determining the value of when the annual interest rate is 100%.
Solution: Let the number of years needed be t, the interest rate be i, and the force
of interest . We solve the equation (1 + i)t = 2 by taking logarithms of both sides:
t ln(1 + i) = ln 2, so
ln 2 ln 2
t= = .
ln(1 + i)
When the interest rate is 100% money doubles in one year; here = ln 2.

B.1.2 Second 2002/2003 Problem Assignment, with Solutions


1. (cf. Exercise 2-5, p. 36) A vendor has three offers for a house:

(a) three equal payments one now, one 1 year from now, and the other 2 years
from now;
Information for Students in MATH 329 2005 01 3007

(b) a single cash payment now of 120,000;


(c) two payments, 45,000 a year from now, and 90,000 two years from now.
He makes the remark that one offer is just as good as another. Determine the
interest rate and the sizes of the equal payments that will make this statement
correct.
Solution: Let the interest rate be i, and the equal payments be k. Equating the
present value of the payments of 45,000 and 90,000 to 120,000 yields
45000v + 90000v 2 = 120000
which we solve for v, obtaining
q
12 1
4
+ 16
3
v =
2
1 22.33333
=
4
The lower sign yields a negative value of v, and so that solution is extraneous. We
obtain v = 0.931454, so i = 0.0736 = 7.36%.
The present value of the equal payments is then k(1+v+v 2 ) = 2.79906k = 120, 000,
so the equal payments will each be 42,871.53.
2. (cf. [5, Exercise 2-9, p. 37]) Fund A accumulates at 9% effective, and Fund B at
8% effective. At the end of 12 years the total of the two funds is 50,000. At the
end of 6 years the amount in Fund B is 4 times that in Fund A. How much is in
Fund A after 15 years?
Solution: Let a and b denote the initial amounts in funds A and B. We have two
constraints relating a and b:
a(1.09)12 + b(1.08)12 = 50000
b(1.08)6 = 4a(1.09)6
From the second of these we can determine the relative sizes of a and b; substituting
in the first equation and solving gives
50000
a = 6
(1.09)12 + 4 1.09
1.08
(1.08)12
50000
= = 3, 715.25.
(1.09) ((1.09)6 + 4(1.08)6 )
6

6
Hence b = 4 109108
a = 15, 705.96. The value of Fund A after 15 years is, therefore,
15
3, 715.25(1.09) = 13, 532.73.
Information for Students in MATH 329 2005 01 3008

3. The initial balance in an investment fund was 100,000. At the end of 3 months it
had increased to 105,000; at that time 25,000 was added to the fund. Six months
later the fund had increased to 143,000, and this time 30,000 was removed. Finally,
at the end of a year, the fund had a balance of 120,000. What was the time-weighted
rate of return?
Solution: [4, Example 2.3.2, p. 39] The balances and withdrawals are respectively
B0 = 100, 000, B1 = 105, 000, B2 = 143, 000, B3 = 120, 000; W0 = 0, W1 = 25, 000,
W2 = 30, 000. Hence the rates of interest in the successive time periods are given
by
B1 105, 000
1 + i1 = = = 1.05
B0 + W 0 100, 000 + 0
B2 143, 000
1 + i2 = = = 1.10
B1 + W 1 105, 000 + 25, 000
B3 120, 000
1 + i3 = = = 1.062
B2 + W 2 143, 000 30, 000
The time-weighted rate of return i is, by definition, given by the product

1 + i = (1.05)(1.10)(1.062) = 1.227

so i = 22.7%.

4. (cf. [5, Exercise 2-15, p. 38]) A trust company pays 5% effective on deposits at the
end of each year. At the end of every 3 years a 2% bonus is paid on the balance at
the time. Find the effective rate of interest earned by an investor if she leaves her
money on deposit

(a) for 2 years;


(b) for 3 years (until after the bonus payment is made);
(c) for 4 years;
(d) forever take a limit!

Solution:

(a) Since there are no bonus payments, the effective rate of interest is 5%.
(b) A deposit of 1 grows to 1.05 at the end of the first year, (1.05)2 at the end
of the second year, and, after the bonus, (1.05)3 (1.02) at the end of the 3rd
year. If the effective rate of interest is i, then we must solve the equation

(1 + i)3 = (1.05)3 (1.02) .


Information for Students in MATH 329 2005 01 3009

Taking logarithms, we obtain 3 ln(1 + i) = 3 ln(1.05) + ln(1.02), so


1
ln(1 + i) = (3 ln(1.05) + ln(1.02))
3
1
1 + i = e 3 (3 ln(1.05)+ln(1.02))
1
i = e 3 (3 ln(1.05)+ln(1.02)) 1
3
= 1.05 1.02 1 = .056953846 = 5.70%.

(c) Analogously to the preceding,


1
i = e 4 (4 ln(1.05)+ln(1.02)) 1
4
= 1.05 1.02 1 = .0552 = 5.52%.

(d) The effects of the bonuses depend on whether the remainder of the number of
years is, upon division by 3, 0, 1, or 2. We have, for any non-negative integer
n,

(1 + i)3n = (1.05)3n (1.02)n


(1 + i)3n+1 = (1.05)3n+1 (1.02)n
(1 + i)3n+2 = (1.05)3n+2 (1.02)n

By taking logarithms and dividing, or, equivalently, by taking the appropriate


(positive) root of both sides of the equation, we obtain

1 + i = (1.05)(1.02)1/3
n
1 + i = (1.05)(1.02) 3n+1
n
1 + i = (1.05)(1.02) 3n+2

As n , the exponent of 1.02 approaches 31 , and so the interest rate


approaches the value of 5.70% we obtained in case of 3 years.

5. Alice borrows 5000 from The Friendly Finance Company, at an annual rate of
interest of 18% per year, where the company compounds interest annually, but
charges simple interest for fractions of a year.

(a) She plans to pay the company 5000 at the end of 2 years.
i. How much will she continue to owe the company at that time?
ii. What is the present value of that residual amount, assuming the same
18% interest rate?
Information for Students in MATH 329 2005 01 3010

(b) Alice discovers that she doesnt need the loan, so she offers to lend the money
to her brother, at an effective annual rate of 18%. When her brother pays off
his loan, Alice will pay off hers. How much will Alice still owe Friendly if
i. Her brother pays off his loan exactly 3 years from now?
ii. Her brother pays off his loan 3.5 years from now?

Solution:

(a) i. The residual amount immediately after the payment will be



5000 (1.18)2 1 = 1962 .

ii. The present value of the residual amount is 5000 ((1.18)2 1) (1.18)2 =
5000 (1 (1.18)2 ) = 1409.08.
(b) i. If her brother pays off his loan after an integer number of years, and Alice
immediately repays her loan, she will owe nothing to Friendly.
ii. After 3.5 years Alice will receive 5000(1.18)3.5 , but will be owing
5000(1.18)3 (1.09); after making her payment, she will continue to owe

5000(1.18)3.5 + 5000(1.18)3 (1.09) = 5000(1.18)3 (1.09 1.18) = 30.58 .

6. (cf. [5, Exercise 2-12, p. 37])

(a) Find an equation that gives information about the effective rate of interest i
if payments of 200 at the present, 300 at the end of 1 year, and 400 at the
end of 3 years, are to accumulate to 1000 at the end of 4 years.
(b) Use the Intermediate Value Theorem to argue that there exists a positive rate
of interest less than 100% which can solve this problem. Then use the Mean
Value Theorem to show that there is just one solution to the problem (by
showing that a certain derivative is positive).
(c) While there exist more efficient algorithms for solving problems like this, a
naive solution could be found by successively subdividing an interval at whose
ends a certain function would have values with opposite signs. Apply this idea
to find the interest rate i to within an error of 0.1%.

Solution:
Information for Students in MATH 329 2005 01 3011

(a) The equation of value at the end of 4 years33 is

200(1 + i)4 + 300(1 + i)3 + 400(1 + i) = 1000

Define f (x) = 200x4 + 300x3 + 400x 1000. Then f (1) = 100 < 0 < 5400 =
3200 + 2400 + 800 1000 = f (2). By the Intermediate Value Theorem func-
tion f , which, being a polynomial, is continuous, has a zero somewhere in
1 < x < 2. If there were 2 or more zeroes, then, by Rolles Theorem, (since f ,
being a polynomial, is differentiable), there would be a point between them
where f 0 would be 0. But f 0 (x) = 800x3 +900x2 +400 > 0 for 1 < x < 2. From
this contradiction we know that there is at most one zero for f , hence exactly
one solution i for our effective interest rate. We can apply the Intermediate
Value Theorem between any two points in the domain. The most naive solu-
tion would be to repeatedly halve the interval. We find that f (1.5) = 1625, so
we may confine ourselves to the interval 1 < x < 1.5; then f (1.25) = 574.22,
f (1.125) = 197.51, f (1.0625) = 39.72, f (1.03125) = 32.29. We evalu-
ate f at the midpoint of the interval [1.03125, 1.0625]: f (1.046875) = 3.17,
so we next use the interval [1.046875, 1.0625], whose midpoint is 1.0546875,
where f (1.0546875) = 21.30. As there will is a sign change in the interval
[1.03125, 1.0546875], we next evaluate f at its mid-point: f (1.042968750) =
5.80.
In the course of these calculations we have not bothered to round the decimal
expansions of the midpoints. There is nothing to be gained by this persistence,
as the procedure will work even if we do not take the precise midpoints.
Having now confined
1.043+1.049 the root to the interval [1.043, 1.055].
1.043+1.046 f (1.049) = 8.07,
we try f 2
= f (1.046) = 1.15, f 2
= f (1.0445) = 2.29,
f (1.045) = 1.15, f (1.0455) = 0.002, f (1.04549913) = 0.0000013. Thus
the rate is approximately 4.55%.

B.1.3 Third 2002/2003 Problem Assignment, with Solutions


1. (a) Find the sum of the positive integers 1, 2, . . . , N .
(b) Find the sum of the odd integers 1, 3, 5, . . . , 2N + 1.
(c) In an arithmetic progression x1 , x2 , . . ., xn , . . . the third term is 4 times the
first term, and the sixth term is 17. Find the general term xn .
33
Non-trivial equations are never unique; also, we could have found an equation of value at another
time. For example, an equation of value at the present could be

200 + 300v + 400v 3 = 1000v 4 .


Information for Students in MATH 329 2005 01 3012

(d) The sum of n terms of the arithmetic series 2, 5, 8, . . . is 950. Find n.


(e) The sum of the first 6 terms of a geometric progression is equal to 9 times the
sum of the first 3 terms. Find the common ratio. Is it possible to determine
the sequence from this information?
(f) Use your knowledge of the sum of geometric series to determine a vulgar
fraction of integers m
n
which is equal to the repeating decimal number
3.157157157157...
Do not use a calculator for this problem.
Solution: These topics were once part of the standard high school curriculum.
These problems were adapted from [7].
(a) The common difference is 1 and the first term is also 1; the sum of N terms
is, therefore, N2 (1 + N ) = N (N2+1) .34
(b) The common difference is 2, the first term is 1, and the N +1st term is 2N +1.
The sum of N + 1 terms is N2+1 (2 1 + N 2) = (N + 1)2 .
[Many students failed to notice that the number of summands was N + 1
not N . An error of this type might have been detected by checking ones
computations for small values of N , e.g. N = 0 or N = 1. Carry out the
summation mechanically, then compare the sum that you obtain with the
value of the formula you have derived; if the values are different, you need to
check every step of your work carefully.]
(c) Let the first term be a and the common difference be d. We have to solve the
equations:
x3 = 4x1 a + 2d = 4a
x6 = 17 a + 5d = 17
which yield a = 2, d = 3. Hence xn = 2 + 3(n 1) = 3n 1.
(d) We solve the equation n2 (2 2 + (n 1) 3) = 950, which reduces to 3n2 + n
1900 = 0, whose only positive solution is n = 25.
(e) If the first term is a and the common ratio is r, the given information implies
that
r6 1 r3 1
a = 9a if r 6= 1 , (113)
r1 r1
6a = 9a if r = 1 . (114)
34
This expression is known, from other considerations, to be the number of ways of choosing 2 objects
from a set of N distinct objects; it is often denoted by N2 .
Information for Students in MATH 329 2005 01 3013

When a = 0, both equations are satisfied: the sequence is 0, 0, 0, . . . ; the


common ratio is indeterminate. When r 6= 1, (113) yields r6 1 = 9(r3 1),
so r3 = 1 (which contradicts the hypothesis) or r3 = 8, hence r = 2 and the
sequence is then
a, 2a, 4a, ..., 2n1 a, ...
But the sequence is not completely determined, since any value of a is ac-
ceptable including the value 0 which we already saw as the solution to
(114).
(f)

1 5 7 1 1 5 7
3.157157157157... = 3 + + + + + +
10 100 1000 1000 10 100 1000

1 1 5 7
+ + + + ...
10002 10 100 1000
157 1 157 1 157
= 3+ + + 2
+ ...
1000 1000 1000 1000 1000
157 1 157 3154
= 3+ 1 =3+ = .
1000 1 1000 999 999
2. (cf. [5, Exercise 3-6, p. 66] An annuity pays 1000 per year for 8 years. If i = 0.05,
find each of the following
(a) The value of the annuity one year before the first payment.
(b) The value of the annuity one year after the last payment.
(c) The value of the annuity at the time of the 4th payment.
(d) If possible, the number of years an annuity-immediate would have to run in
order that its value, viewed one year before the first payment should be twice
that of the 8-payment annuity whose value at the same time was determined
above.
(e) If possible, the number of years an annuity-immediate would have to run
in order that its value, viewed one year before the first payment, should be
three times that of the 8-payment annuity whose value at the same time was
determined above.
(f) If possible, the number of years an annuity-immediate would have to run
in order that its value, viewed one year before the first payment, should be
four times that of the 8-payment annuity whose value at the same time was
determined above.35
35
Note that the wording of the cited questions in the textbook required a number of assumptions that
have been made more explicit in the present questions.
Information for Students in MATH 329 2005 01 3014

(g) (cf. [5, Exercise 3-58, p. 73]) Redo part (a), assuming now that the annuity
is continuous. (The effective annual interest rate remains 5%, and the time is
still 8 years.)
Solution:
1(1.05)8
(a) 1000a8 5% = 1000 0.05
= 20000 (1 (1.05)8 ) = 6463.21.
(1.058 1)(1.05)
(b) 1000s8 5% = 1000 0.05
= 10026.56.
(c)

1000s4 5% + 1000a4 5% = 1000(1.05)4 a45%


= (1.05)4 (6463.21) = 7856.07 .

(d) We have to solve for n:

an 5% = 2a8 5%
1 v n = 2 2v 8
v n = 2v 8 1
ln(2v 8 1)
n= = 21.30 years.
ln v
(e) We have to solve for n:

an 5% = 3a8 5%
1 v n = 3 3v 8
v n = 3v 8 2
ln(3v 8 2)
n= = 71.52 years.
ln v
(f) In this case we observe that the value of a perpetuity of 1000 per year at
5% is only 1000
0.05
= 20000 < 4(6463.21); so the problem will have no solution.
If we attempt to solve as in the preceding case, we will obtain the equation
1 v n = 4 4v 8 v n = 4v 8 3 = 0.29, which has no solution.
(g)
Z8
1000a8 5% = 1000 v t dt
0
1 (1.05)8
= 1000 = 6623.48.
ln(1.05)
Information for Students in MATH 329 2005 01 3015

3. [5, Exercise 3.15] Prove each of the following identities

algebraically; and
verbally

(a) an = an + 1 v n
(b) sn = sn 1 + (1 + i)n

Solution:

(a) an differs from an in that it has an immediate payment of 1 but lacks the final
payment of 1 n years hence, whose value now is v n .
Algebraically,

an = an (1 + i)
= an + ian
1 vn
= an + i = an + (1 v n )
i
(b) sn differs from sn in that is lacks a payment of 1 at time t = 0, but has a
payment of 1 that has accumulated interest over n years, so that its present
value is (1 + i)n .
Algebraically,

sn = sn (1 + i)
= sn + isn
(1 + i)n 1
= sn + i = sn + ((1 + i)n 1)
i
4. [5, Exercise 3-45, p. 71] Wilbur leaves an inheritance to four charities: A, B, C,
D. The total inheritance is a series of level payments at the end of each year,
payable forever. During the first 20 years, A, B, C share each payment equally. All
payments after 20 years are to revert to charity D. The present value of the shares
of A, B, C, and D are all equal. Showing all your work , prove that i = 0.07177.
Solution: It does not limit generality to assume that the level payments are all of
1. The present value of the payments to each of A, B, C is 31 a20 i% . The payments
to D constitute a perpetuity-immediate of 1 deferred 20 years; its value is v 20 1i .
Accordingly we have to solve the following equation for i:
1 1
a20 i% = v 20
3 i
Information for Students in MATH 329 2005 01 3016

1 v 20
= v 20
3
1
v 20 =
4
1
1 + i = 4 20 = 1.0717735
so i = 7.177%.
5. Find the present value at i effective of a perpetuity whose annual payments of 1000
begin with a payment of 1000 after one year, with the property that each payment
thereafter is reduced by 10% from the preceding payment. In particular, determine
the present value when i = 2.5%.
Solution: [STUDENTS WERE ASKED NOT TO SUBMIT A SOLUTION TO
THIS PROBLEM.] The present value is
1000(v + 0.9v 2 + 0.92 v 3 + 0.93 v 4 + ... + 0.9n1 v n + ...)
X n
0.9 1000v
= 1000v =
n=0
1+i 1 0.9v
1000 1000
= =
(1 + i) 0.9 0.1 + i
1000
When i = 2.5%, the present value is 0.125
= 8000.
6. A fund of 10,000 is to be accumulated by means of deposits of 1000 made at the
end of every year, as long as necessary. If the fund earns an effective rate of interest
of 2 12 %, find how many regular deposits will be necessary, and the size of a final
deposit to be made one year after the last regular deposit.
Solution: [2, Example 6.5, pp. 59-60] There are often tacit assumptions in inter-
est problems; usually there is an obvious intended interpretation, while other
interpretations might be justified by some unusual reading of the wording. In the
present problem one is to assume that the deposits are not permitted to exceed
1000, and that all deposits (the regular deposits) but the last are to be exactly
1000. The last deposit can be smaller, but not larger.
Let n be the number of regular deposits required. Then n is the largest integer
that satisfies the inequality of value
1000sn 2.5% 10000 ;
equivalently, sn 2.5% 10 . Computing the values of sn 2.5% , we find that s8 2.5% =
8.73612, s9 2.5% = 9.95452, s10 2.5% = 11.20338. Hence n = 9. The final partial de-
posit will have to be the excess of 10,000 over the accumulated value of s9 2.5% after
Information for Students in MATH 329 2005 01 3017

one year, i.e., the excess of 10,000 over (1.025)s9 2.5% = 10, 000(1.025)(9954.52) =
203.38. So rather than a final deposit, there will be a final refund of 203.38. This
situation could also have been seen from the value of 10,000s10 2.5% = 11203.38,
which would be the value after a 10th deposit; since this exceeds 11,000, no 10th
deposit would be required.
[ADDED March 10th, 2003] Another possible interpretation of the instructions in
this problem is to treat the 8th as the last regular deposit, and to reduce the 9th
deposit so that, when the time arrives for a possible 10th deposit, the balance in
the fund is exactly 10000. If we define the value of the 9th deposit to be x, then

(1 + i) (1 + i)s8 + x = 1000

10000
x = 1025s8
1.025
= 9756.0976 8954.5188 = 801.58.
We can verify the correctness of this computation by observing that the excess
payment of 1000801.58 = 198.42 accumulated at 2.5% to 1.025198.42 = 203.38,
which was computed earlier as the amount refunded one later.
[This assignment was intended as a learning exercise, rather than a testing exercise.
Students were not expected to have seen an example of this type before.]
7. A deferred annuity is one that begins its payments later than might otherwise have
been expected. We define
m |an = v m an (115)
m
m |an = v an (116)
Prove, both algebraically and verbally, that, for non-negative integers m and n,
m|an = am+n am (117)
m
(1 + i) sn = sm+n sm (118)
1 |an = an (119)

Solution:
(a)
m |an = v m (v + v 2 + . . . + v n )
= (v m+1 + v m+2 + . . . + v m+n
= (v 1 + v 2 + . . . + v m+n (v 1 + v 2 + . . . + v m
= am+n am

UPDATED TO April 12, 2005


Information for Students in MATH 329 2005 01 3018

In deferring an n-payment annuity-immediate by m years we are planning for


the first payment to be made m + 1 years from now, and the last m + n years
from now. These can be viewed as the last n payments of an m + n-payment
annuity-immediate whose first payment begins one year hence; thus we obtain
the value of m |an by subtracting am from am+n .
(b)

(1 + i)m sn = (1 + i)m 1 + (1 + i)1 + . . . + (1 + i)n1
= (1 + i)m + (1 + i)m+1 + . . . + (1 + i)m+n1

= 1 + (1 + i)1 + . . . + (1 + i)m+n1

1 + (1 + i)1 + . . . + (1 + i)m1
= sm+n sm
The payments associated with (1 + i)m sn can be interpreted as the first m
payments of an m + n-payment annuity whose last payment has just been
made. If we subtract from sm+n the value of the last n payments as viewed
from the day of the last payment, we obtain the value of those first m pay-
ments.
(c)

1 |an = v 1 + v + v 2 + . . . + v n1
= v + v 2 + . . . + v n = an
When we defer an annuity-due one year it becomes an annuity-immediate.

B.1.4 Fourth 2002/2003 Problem Assignment, with Solutions


1. (cf. [5, Exercise 3-60, p. 73]) A student attending engineering school has increasing
amounts of income as she advances through her programme. Accordingly she agrees
to borrow a decreasing annual amount from her parents during her 5 training years,
and to repay the loan with increasing amounts for 15 years after graduation. She
receives amounts 5X, 4X, 3X, 2X and X at the beginning of each of 5 years,
where the last payment is paid at the beginning of her final year. At the end of
her first year after graduation she pays 500, and then increases the amount by 200
each year until a final payment of 3300.36 If the interest rate is 5%, determine X.
Solution: An equation of value at the time of graduation is
(1 + i)X(Ds)5 = 300a15 + 200(Ia)15
36
The original version of this problem gave the final payment as 3500, which would have required 16
years of payments. A correction was announced at the lecture of March 3rd, 2003.
Information for Students in MATH 329 2005 01 3019

implying that

300a15 + 200(Ia)15
X =
(1 + i)(Ds)5
300(1 v 15 ) + 200((1 + i)a15 15v 15 )
=
5(1 + i)6 (1 + i)6 a5
= 993.11.

For students who corrected the error in the problem by increasing the number of
years by 1, here is a solution:
Solution: An equation of value at the time of graduation is

(1 + i)X(Ds)5 = 300a16 + 200(Ia)16

implying that

300a16 + 200(Ia)16
X =
(1 + i)(Ds)5
300(1 v 16 ) + 200((1 + i)a16 16v 16 )
=
5(1 + i)6 (1 + i)6 a5
= 1082.33.

2. (cf. [5, Exercise 4-2, p. 85]) A loan is being repaid by 36 monthly payments. The
first 12 installments are 250 each; the next 18 are 300 each; and the last 6 are 500
each. Assuming a nominal annual interest rate of 12% compounded monthly,

(a) Find the principal, A(0), of the loan.


(b) Using the Prospective Method, find the loan balance immediately after the
6th payment.
(c) Using the Retrospective Method, find the loan balance immediately after the
6th payment.
(d) Divide the 7th and 8th payments into principal and interest.

Solution:

(a) Using the Prospective Method, the principal is seen to be

A(0) = 250a12 + 300v 12 a18 + 500v 30 a6


= 9, 329.46
Information for Students in MATH 329 2005 01 3020

(b) Immediately after the 6th payment, the value of the remaining 30 payments
(at an interest rate of 1% per period) is

250a6 + 300v 6 a18 + 500v 24 a6


1 v6 1 v 18 1 v6
= 250 + 300v 6 + 500v 24
i i i
1
= (250 + 500v 24 )(1 v 6 ) + 300v 6 (1 v 18 )
i
1
= 250 + 50v 6 + 200v 24 500v 30
i
= 8365.40

(c) The principal of the loan has been determined above. The outstanding prin-
cipal is the accumulated value of this principal decreased by the accumulated
values of the payments that have been made, i.e.
(1.01)6 1
(1.01)6 A(0) 250s6 = (1.016 )(9, 329.46) 250
0.01
= 9, 805.38 1, 538.00 = 8, 365.40.

(d) The principal owing immediately after the 6th payment is known to be 8,365.40.
At the time of the 7th payment, this will have accumulated interest of 1%, or
83.65; the balance of the payment, i.e. 250 83.65 = 166.35, will be applied
to reduction of principal. The reduced balance of 8365.40 166.35 = 8199.05
will accumulate interest in the amount of 0.01 8199.05 = 81.99 in the 8th
month. The 8th payment will include, in addition to this amount of interest,
an amount of 250 81.99 = 168.01 for the reduction of principle; the out-
standing principal after the 8th payment will be 8199.05 168.01 = 8031.04.

3. (a) [5, Exercise 4-16, p. 87] Harriet is repaying a car loan with payments of 2,000
every three months and a final payment 3 months after the last full payment
of 2,000. If the amount of interest in the 4th installment (paid at the end of
the first year) is 1,100, find the principal of the loan, the time and amount
of the final payment, and the amounts of principal and interest in that final
payment. Assume that interest is compounded monthly, at a nominal annual
rate of 18%.
(b) Construct an amortization schedule for the first year of this loan.
Solution:

(a) The interest rate being charged monthly is 0.18/12 = 1.5%. Let the principal
of the loan be A. The Retrospective Method shows that the amount owing
Information for Students in MATH 329 2005 01 3021

immediately after the 3rd installment (paid at 9 months) is



2000 (1.015)6 + (1.015)3 + (1.015)0 + (1.015)9 A .
This unpaid balance will, in 3 months, earn the lender interest in the amount
of

1100 = (1.015)3 1 2000 (1.015)6 + (1.015)3 + (1.015)0 + (1.015)9 A .
Thus
1100
A =
(1.015)12
(1.015)9

+2000 (1.015)9 + (1.015)6 + (1.015)3 (120)
= 26552.32
We can consider the payments as constituting an annuity, with time interval 3
months, and interest rate per 3 months of (1.015)3 1 = 0.045678375. Using
the Prospective Method, we see that the value of n installments, as of the day
of the loan, is
1 (1.015)3n
2000an0.045678375 = 2000 .
0.045678375
We seek the smallest n such that
26552.32 0.045678375
1 (1.015)3n = 0.6064334
2000
i.e., such that
1.0153n 0.3935666
or
ln 0.3935666
n = 20.88
3 ln 1.015
Thus there will be 20 full payments of 2,000, the last full payment being made
5 years after the beginning of the loan. At that time the amount outstanding
will be
26552.32(1.015)60 2000s200.045678375
20
(1.0153 ) 1
= 64873.15 2000
(1.015)3 1
= 64873.15 63190.50 = 1682.65
The last payment will be 1682.65(1.015)3 = 1759.51; of this, the interest
component will be 1682.65 ((1.015)3 1) = 76.86, and the balance will be the
outstanding principal of 1682.65.
Information for Students in MATH 329 2005 01 3022

(b)
Duration Payment Interest Principal Repaid Outstanding Principal
(Months)
0 26552.32
3 2000.00 1212.87 787.13 25765.19
6 2000.00 1176.91 823.09 24942.10
9 2000.00 1139.31 860.69 24081.41
12 2000.00 1100.00 900.00 23181.41

4. John has borrowed 10000, on which he is paying interest at 10% effective per
year. He is required to pay the interest on the loan annually, and is permitted to
repay only the entire loan, and only on an anniversary. He decides to accumulate
a sinking fund to accumulate the funds to repay the loan. Suppose that John
has 2400 available at the end of each year, out of which to pay both the interest
on the loan and an annual contribution to his sinking fund. If the sinking fund
accumulates at 6%, complete a table under the following headings to determine
when John will be able to repay the loan.

Duration Contribution Interest Interest Earned Balance of


(Years) to Sinking Fund on Loan in Sinking Fund Sinking Fund
0 0 0 0 0
... ... ... ... ...

Solution: The instructions asked that the student complete a table...to determine
when John will be able to repay the loan. The information could have been
obtained without the table, however, by finding the smallest value of n for which
1400sn 10000; this can be seen to be n = 6, where

10000 1400sn = 234.55 . (121)

From (121) we see that the shortfall in the balance of the sinking fund after the
last payment of 1400 is 234.55. The value of the sinking fund is not yet sufficient
to repay the loan. Even without a 7th payment the sinking fund will exceed 10000
by the time when that payment is due. It will, however, be necessary to pay the
interest charge of 1000 on the loan. If a full 6th payment of 1400 was made into the
sinking fund, there would be a refund of (1.06)(1400)s6 1000 = 351.38. However,
a better solution would have been for John to make a smaller 6th deposit into the
sinking fund just sufficient to bring the fund up to the level of 10000 at the time
of the 7th interest payment. The balance just after such a 6th payment would need
to be 10000
1.06
, and the balance just prior to the 6th deposit would be (1.06)1400s5 ;
Information for Students in MATH 329 2005 01 3023

so the appropriate 6th deposit would be


10000
(1.06)1400s5 = 9433.963 8365.446 = 1068.52 .
1.06
The first table below shows what would happen if John made a full 6th contribution:

Duration Contribution Interest Interest Earned Balance of


(Years) to Sinking Fund on Loan in Sinking Fund Sinking Fund
0 0.00 0. 0.00 0.00
1 1400.00 1000. 0.00 1400.00
2 1400.00 1000. 84.00 2884.00
3 1400.00 1000. 173.04 4457.04
4 1400.00 1000. 267.42 6124.46
5 1400.00 1000. 367.47 7891.93
6 1400.00 1000. 473.52 9765.45
7 -351.38 1000. 585.93 10351.38

The following table shows the result of a reduced 6th contribution:

Duration Contribution Interest Interest Earned Balance of


(Years) to Sinking Fund on Loan in Sinking Fund Sinking Fund
0 0.00 0. 0.00 0.00
1 1400.00 1000. 0.00 1400.00
2 1400.00 1000. 84.00 2884.00
3 1400.00 1000. 173.04 4457.04
4 1400.00 1000. 267.42 6124.46
5 1400.00 1000. 367.47 7891.93
6 1068.51 1000. 473.52 9433.96
7 0.00 1000. 566.04 10000.00

NOTE TO THE GRADER: PLEASE ACCEPT EITHER OF THESE TABLES.


5. (This is a complicated variant of [5, Exercise 4-6, p. 86]. It requires considerable
persistence, but is a very thorough exercise. Dont panic! This is not a typical
examination question.) Garfield is repaying a debt with 25 annual payments of
1000 each, at an annual interest rate of i = 10%. The terms of his loan permit
him to make additional payments on the date of any regular payment. After any
such additional payment, the terms of the loan require the borrower to continue
with payments of 1000 until a last payment of 1000 or less which settles the debt
completely.
Information for Students in MATH 329 2005 01 3024

(a) At the end of the 7th year Garfield proposes to make, in addition to his
regular annual payment of 1000, an extra payment of 5000. At that time
he also proposes to reduce his remaining payment period by 4 years, and to
make level payments over that time (replacing the originally agreed payments
of 1000). Find the revised annual level payment, computed using the interest
rate i = 10%.
(b) The lender is obliged to accept Garfields extra payment. But he is not obliged
to accept Garfields proposed method to repay the loan in fewer payments.
If, at the time of the change in the payment scheme, the lender insists on
charging an interest rate of i = 12% when the remainder of the loan will be
repaid over 14 equal annual payments, what will be the revised annual level
payment that will have to be paid at the end of the each of the next 14 years?
(c) Determine the premium Garfield is being asked to pay as a result of the
increased interest rate in part 5b. Express the amount as of the date of the
proposed change in the payment scheme. Make two sets of calculations:
i. when the cost of money37 is 10% per annum;
ii. when the cost of money is 12% per annum.
(d) As the loan is repaid, the lender is able to put his money to work. Suppose
that money now costs 12%, instead of the 10% that prevailed when the loan
was written. One might have expected the lender to encourage the borrower to
repay the loan faster. Faced with the lenders intransigence, Garfield makes
the supplementary payment of 5000, but decides to abandon his plans to
change the payment size; his payments will be 1000 per year until possibly
the last payment. Determine whether the lender has suffered from his own
stubbornness: express his loss (or gain) as of the time of the supplementary
payment made with the 7th payment.
(e) Suppose that, learning that the cost of money is 12% when he is about to
make his supplementary payment, Garfield changes his plans. He makes no
change to his loan, but invests his 5000 elsewhere in an annuity which will
provide him with payments of 1000 to apply to as many of the final payments
under his loan as possible. As of the beginning of the 8th year of the loan
(immediately following the 7th payment and any supplementary payment)
compare the cost of this scheme with
i. his commitment under the original loan contract;
37
By the statement The cost of money is i we intend that Garfield is able as of this particular date
to either borrow or lend money in any amount and for any period of time commencing immediately
at the interest rate i.
Information for Students in MATH 329 2005 01 3025

ii. his proposed scheme, whereby he would pay 5000 immediately and pay
the rest of the loan over 14 years at 10%;
iii. the lenders proposal, where an immediate payment of 5000 would be
followed by equal payments for 14 years, computed at a rate of 12%.
(f) Determine the yield earned by the lender under each of the following repay-
ment schemes:
i. the loan as originally written 25 annual payments of 1000;
ii. the repayment scheme proposed by Garfield: 1000 per year for 7 years,
5000 additional at the end of the 7th year; level payments for 14 years
thereafter, amount as computed in part 5a above;
iii. the repayment scheme proposed by the lender, in part 5b, where the
level payments are recomputed at 12% charged from the time of the 7th
payment; (in this case it suffices to write down an equation that must be
satisfied by the yield);
iv. the repayment scheme finally followed by Garfield in part 5d, where he
invests in an annuity to provide him with payments of 1000 for the final
payments, and pays the rest annually from his savings.

Solution:

(a) Using the Prospective Method, we find that the unpaid balance immediately
after the 7th payment, but prior to the extra payment, is 1000a180.1 ; after the
extra payment the amount owed is

1000a180.1 5000 = 8201.41 5000 = 3, 201.41 .

The level payment to repay this principal in 184 = 14 years is (with i = 10%
1
and v = 1.1 )

1000a180.1 5000 1000(1 v 18 ) 5000i


= (122)
a140.1 1 v 14
= 434.58.

(b) We will have to evaluate the same ratio as in (122), but where numerator and
denominator involve different interest rates.
1000a180.1 5000 1000(1 (1.1)18 ) 5000(0.1) 0.12
=
a140.12 1 (1.12)14 0.1
= 483.
Information for Students in MATH 329 2005 01 3026

(c) Lets first determine the nature of Garfields commitment under the loan after
he makes his supplementary payment. We have determined that the loan
balance is 3,201.41. We note that 1, 000a4.10 = 3, 169.87, while 1, 000a5.10 =
3, 790.79. Garfields loan contract requires him to make 4 payments of 1000;
and, at the end of the 5th year, to pay the balance of principal that would be
owing at that time. That balance would be

(1.1)5 (3201.41) 1000 s5.1 1 = 5155.90 5105.10 = 50.80 .
These 5 payments are prescribed under his contract, and the calculation of
their values is not affected by the cost of money today. What is affected is the
way in which Garfield finances these payments; or, equivalently, the present
value of these payments, which may not be equal to the loan balance.
i. If the cost of money is 10%, the present value of the 14 payments Garfield
would have to make would be 483a14.1 = 3558.11; the present value of the
payments required under the loan contract is the outstanding principal,
3201.41; so the premium would be 356.70.
ii. If the cost of money is 12%, the present value of the 14 payments would
be the outstanding principal, 3,201.41. The value of the 4 payments of
1000 and one final payment (i.e. the cost of financing them at 12%) is
1000a4.12 + (1.12)5 50.80 = 3037.35 + 28.83 = 3066.18 .
In this case he would be paying a premium of
3201.41 3066.18 = 135.23 .

(d) After his supplementary payment, Garfield owes an unpaid balance of 3201.41.
Had he been permitted to repay this with 14 annual payments of 434.58, the
present value of those payments would be 2880.47. But Garfield has now been
driven to repay the loan by continuing the planned payments of 1000 until a
final payment. In part 5c we have determined that the number of payments
of 1000 is 4, and these are followed by a payment one year later of 50.80. The
value of these payments today, when money costs 12%, is
1000a40.12 + (1.12)5 50.80 = 3037.35 + 28.83
= 3066.18 .
While neither of these repayment schemes yields the full amount owed
because interest rates are higher than at the outset the lender was wise
to be unwilling to accept Garfields offer: he has reduced his losses under the
loan by 3066.18 2880.47 = 185.71.
Information for Students in MATH 329 2005 01 3027

(e) i. Garfields commitment under the original contract is for payments of 1000
for 18 more years. At a rate of 12%, Garfield could buy an annuity to cover
his payments at a present cost of 1000a18.12 = 7249.67. We are asked to
compare this cost with the use of the 5000 to purchase a deferred annuity
to cover the last payments due under the contract. We will answer this
question naively and then, when the answer looks interesting, observe
that there is a much simpler solution.
The present value of an annuity that will cover the payments due in years
##k + 1, k + 2, ..., 18 is

1000 a18.12 ak.12 .

Since

1000a18.12 = 7249.67 5000 = 2249.67


1000a3.12 = 2401.83
1000a2.12 = 1690.05 ,

Garfields 5000 will buy him a deferred annuity paying 1000 per year,
starting at the end of 4 years from now until 18 years from now, costing
him 1000s18.12 1000s3.12 = 7249.67 2401.83 = 4847.84 and he will have
152.16 left over. The cost of the payments not covered by his 4847.84 is
2401.83; the total of his commitments today is therefore 7249.67, precisely
the same as computed above. This should be no surprise, as both sets
of computations are being made with an interest rate of 12%. Thus the
excess of one over the other is zero.
ii. An annuity to cover the payments of 434.58 per year for 14 years would
cost Garfield today 434.58a14.12 = 2880.47; under this scheme he would
also be making a payment of 5000, for a total of 7880.47: the deferred
annuity method would cost 7880.47 7249.67 = 630.80 less.
iii. The payments of 483 per year for 14 years are worth today 483a14.12 =
1000a180.1 5000 = 3201.41; the sum of the value of these payments and
the supplementary payment is 1000a180.1 = 8201.41 : the deferred annuity
method would cost 8201.41 7249.67 = 951.74 less.
(f) i. The loan was written to provide a yield of 10%. The fact that the cost
of money may have changed does not affect the yield, which is influenced
only by the lenders payments and receipts under the loan.
ii. Since Garfields computation of the new level payment is based on an
interest rate of 10%, there has been no change in the yield to the lender:
it remains 10%.
Information for Students in MATH 329 2005 01 3028

iii. When the lender demands that the computation of the replacement level
payment be based on an interest rate of 12%, he effects a partial im-
provement of the yield; but it cannot affect those funds that were already
repaid. Setting up an equation of value at time 7, just after the supple-
mentary payment and the 7th payment of 1000, we find that the yield
rate, i, will satisfy the equation:

(1 + i)7 1000a2510% + 1000s7i + 5000 + 483a14i = 0

which is equivalent to

7 (1 + i)7 1 (1 + i)14 1
9077.04(1+i) +1000 +5000+483 = 0.
i i(1 + i)14

It can be shown that i = 0.10345 approximately. Thus even the increase


in the interest rate for the final payments does not effect a marked increase
in the yield rate.
iv. In this case the yield rate is 10%, as there are, from the lenders perspec-
tive, no changes.

B.1.5 Fifth 2002/2003 Problem Assignment, with Solutions


1. (a) (cf. [5, Exercise 5.1, p. 105]) A 15-year bond with face value 20000, redeemable
at par, earns interest at 7.5%, convertible semiannually. Find the price to yield
an investor 8% convertible semiannually.
(b) What is the premium or discount at which the bond will be purchased?
(c) (cf. [5, Exercise 5-13, p. 109]) For the bond in part 1a find the market price38
and flat price at each of the following dates and times:
i. Just after the 7th coupon has been paid.
ii. 3 months after the 7th coupon has been paid. (Use simple interest for
fractions of a period.)
iii. Just before the 8th coupon is paid.
iv. Just after the 8th coupon is paid.
(d) What would the market price and flat price have been just before and just
after payment of the 8th coupon if the bond had been purchased at par?
Solution:
38
Market price=amortized value [5, p. 98] is obtained by interpolating linearly between book values
on coupon dates. Thus the market price is a continuous function of time.
Information for Students in MATH 329 2005 01 3029

(a) Both of the interest rates are nominal annual rates convertible semiannually;
we must divide each by 2. Using the general formula, we find the price of
the bond to be
(20000)(1.04)30 + (0.0375 20000)a30.04 (123)

30 750
= (20000)(1.04) + 1 (1.04)30 (124)
.04
30

= (20000)(1.04) + 18750 1 (1.04)30 (125)
= 18750 + (20000 18750)(1.04)30 (126)
= 18750.00 385.40 = 19135.40. (127)
Alternatively, using the alternate formula, we find it to be
200 + (750 800)a304%
50
= 2000 1 (1.04)30 = 19135.40.
0.04
(b) The bond is selling at a discount of 20000.00 19135.40 = 864.60 less than
its redemption value.
(c) The book value at the time of an interest payment is the present value of
the unpaid portions of the bond; the coupon payments will enter into the
accounting in some other way, e.g., as income. These computations make use
of the yield rate associated with the owners acquisition of the bond. The
market price at these times will equal the book value.
i. The remaining 30 7 coupons are worth 750a234% = 11142.53; the prin-
cipal is worth 20000(1.04)30+7 = 8114.53. The market price is the book
value, i.e., the sum, 19257.16.
ii. The book value immediately after the payment of the 8th coupon is
the sum of the value of the unpaid coupons, 750a224% = 10838.34 and
the present value of the principal, 20000(1.04)30+8 = 8439.11; together,
19277.45. The average of this book value and that after the payment of
the 7th coupon is 19267.30, and this is what we define to be the market
price.
iii. By our definition, market price is a continuous function of time: the
market price immediately before a coupon payment will be equal to that
after the payment here, 19277.45.
iv. As seen above, the book value is 19277.45. This could also have been
computed by subtracting the value of the coupon from 1.04 times the
book value after the payment of the 7th coupon:
(1.04 19257.16) 750.00 = 20027.45 750.00 = 19277.45 .
Information for Students in MATH 329 2005 01 3030

We compute the flat prices:


i. The flat price associated with yield rate 4% per interest period, just after
payment of the 7th coupon, is the same as the market price, as there is
no accrued interest. Here the value is, as above, 19257.16.
ii. The flat price is the book value at the time of the preceding coupon
payment plus accrued simple interest. That is,

1
1 + 0.04 19257.16 = 19642.30 .
2
In practice this is often quoted as the market price of 19267.30 plus ac-
crued interest of 375.00, (half of the next coupon).
iii. The flat price just before the payment of the 8th coupon can be deter-
mined in several different ways:
A. Viewed as book value plus accrued interest,

(1 + 0.04) 19257.16 = 20027.45 .

B. Viewed as the book value just after the payment of the coupon, plus
the value of the coupon, it is

19277.45 + 750.00 = 20027.45 .

iv. The flat price just after payment of a coupon is the book value, here
19277.45. (The flat price is discontinuous at such points in time: the
limit as time approaches the point from the right is different from the
limit from the left: they differ by the value of the coupon.)
(d) The flat price just after payment of the 8th coupon would have been

20000(1.0375)22 + 750a223.75%
= 8897.99 + 11102.01 = 20000.00 ;

are you surprised by this result? The flat price just before payment of the
coupon would have been 20000.00 increased by the interest that had been
earned but not paid, i.e. 20000.00 + 750.00 = 20750.00.
The market price would remain constant at 20000 throughout.

2. (a) [5, Exercise 5-3, p. 106] Prove the Alternate Price Formula:

P = C + (F r Ci)an

algebraically.
Information for Students in MATH 329 2005 01 3031

(b) (cf. [5, Exercise 5.5, p. 106]) Two bonds with face value 10000 each, redeemable
at par at the end of the same period, are bought to yield 10%, convertible
semiannually. The first bond costs 8246.56, and pays coupons at 7% per year,
convertible semiannually. The second bond pays coupons at 6% per half-year.
Find
i. the price of the second bond;
ii. the number of coupons remaining on each of the bonds.
Solution:

(a) We can derive the Alternate Price Formula from the General Formula [5,
(5.1)] as follows:

P = (F r)an + Cv n
= (F r)an + C(1 ian ) [5, (3.6), p. 45]
= C + (F r Ci)an

(b) We apply the Alternate Price Formula proved above to the two bonds. Denote
the price of the second bond by P2 , and the number of coupons remaining by
n. Then

8246.56 = 10000 + (10000(0.035) 10000(0.05))an (128)


P2 = 10000 + (10000(0.06) 10000(0.05))an (129)

From (128) we find that


8246.56 10000
an5% = = 11.6896 . (130)
10000(0.035 0.05)
i. Substituting in (129) yields P2 = 11168.96 as the price of the second
bond.
ii. We solve (130) for n:
1 vn
= 11.6896
0.05
1 v n = 0.58448
v n = 0.41552
n ln(1.05) = ln(0.41552)
n = 18

There are 18 coupons remaining: the bonds mature in 9 years.

UPDATED TO April 12, 2005


Information for Students in MATH 329 2005 01 3032

3. (cf. [5, Exercise 5-16, p. 108])

(a) Construct a bond amortization schedule for a 3 year bond of face amount
5000, redeemable at 5250 with semiannual coupons, if the coupon rate is 5%
and the yield rate is 6% both converted semiannually. Use the format
Time Coupon Interest Principal Book
Value Adjustment Value
0
..
.
(b) Construct a bond amortization schedule for a 3 year bond of face amount
5000, redeemable at 5250 with semiannual coupons, if the coupon rate is 6%
and the yield rate is 5% both converted semiannually.

Solution:

(a) The purchase price of the bond will be 5250(1.03)6 + 125a63% = 4396.79 +
677.15 = 5073.94.
Time Coupon Interest Principal Book
Value Adjustment Value
0 5073.94
1 125.00 152.22 -27.22 5101.16
2 125.00 153.03 -28.03 5129.19
3 125.00 153.88 -28.88 5158.07
4 125.00 154.74 -29.74 5187.81
5 125.00 155.63 -30.63 5218.44
6 125.00 156.55 -31.55 5249.99
(b) The purchase price of the bond will be 5250(1.025)6 + 150a62.5% = 4527.06 +
826.22 = 5353.28.
Time Coupon Interest Principal Book
Value Adjustment Value
0 5353.28
1 150.00 133.83 16.17 5337.11
2 150.00 133.43 16.57 5320.54
3 150.00 133.01 16.99 5303.55
4 150.00 132.59 17.41 5286.14
5 150.00 132.15 17.85 5268.29
6 150.00 131.71 18.29 5250.00
Information for Students in MATH 329 2005 01 3033

4. (cf. [5, Exercise 5-22, p. 109]) A 10-year bond of face value 12000 with semiannual
coupons, redeemable at par, is purchased at a premium to yield 10% convertible
semiannually.

(a) If the book value (just after the payment of the coupon) six months before the
redemption date is 11828.57, find the total amount of premium or discount in
the original purchase price.
(b) Determine the nominal annual coupon rate of the bond, compounded semi-
annually.
(c) Give the amortization table for the last one and one-half years.

Solution:

(a) The book value just after the pnultimate39 coupon is

11828.57 = 12000v + F r a10.05


12000 + F r
= 12000v + F r v =
1.05
so
F r = 1.05 11828.57 12000 = 420 .
Knowing the amount of each coupon we can now evaluate the purchase price
of the bond to have been

12000(1.05)20 + 420a200.05 = 4522.67 + 5234.13


= 9756.80 .

The bond was purchased at a discount of 12000 9756.80 = 2243.20.


420
(b) The rate per period was 12000 = 3.5%; hence the nominal rate compounded
semi-annually, is 2 3.5% = 7%.
(c) For convenience we will compile this table backwards, beginning with Time=20.
We were given that B19 = 11828.57. Hence the Principal Adjustment con-
tained in the 20th coupon is

11, 828.57 12, 000 = 171.43 .

book value at Time=18 will be (12000)(1.05)2 + 420(1.051 + (1.05)2 ) =


11665.31; the book value at Time=17 will be (12000)(1.05)3 + 420(1.051 +
(1.05)2 + (1.05)3 ) = 11509.81.
39
2nd last
Information for Students in MATH 329 2005 01 3034

Time Coupon Interest Principal Book


Value Adjustment Value
20 420.00 591.43 -171.43 12000.00
19 420.00 583.26 -163.26 11828.57
18 420.00 575.50 -155.50 11665.31
17 420.00 ... ... 11509.81

5. A 4.5% bond40 with par value of 100 and semiannual coupons is issued on July 1,
2003. It is callable at 110 on any coupon date from July 1, 2008 through January
1, 2011; at 105 on any coupon date from July 1, 2011 through January 1, 2013;
and at 102.50 on any coupon date from July 1, 2013 through January 1, 2015;
thereafter it is callable without premium on any coupon date up to January 1,
2018 inclusive; its maturity date is July 1, 2018. Determine the highest price that
an investor can pay and still be certain of a yield of

(a) 5% convertible semiannually;


(b) 4% convertible semiannually.
(c) 3% convertible semiannually.

[Hint: For each interest rate, and each range of payments for a given premium,
express the price of the bond as a function of the payment number.]
Solution: As a first step towards organizing data, the student should determine the
payment numbers being referred to. If we label the payment dates with natural
numbers, and define the issue date to be (non)-payment #0, the July dates will
have even numbers, and the January dates odd numbers. The premium of 10 is
payable when the calling date is ##10-15; the premium of 5 when the calling date
is ##16-19; the premium of 2.5 when the calling date is ##20-23; and no premium
is payable when the calling date is ##24-29 nor on the maturity date, which is
payment #30.
40
The convention in bonds is that, lacking any indication to the contrary, the term an r% bond refers
to a bond whose coupon rate is a nominal rate of r%; the rate is compounded (or converted ) as often as
indicated in the description of the bond, with the default being half-yearly if there is no indication to
the contrary. Under this convention the coupon rate for this bond is 2.25%. This convention is stated
in the textbook [5, p. 93, 1st paragraph]; however, the author usually supplies additional, redundant,
information in his problems.
Information for Students in MATH 329 2005 01 3035

(a) We tabulate the applicable price formul, based on the call or maturity date:
First Date Last Date Price
n
10 15 110.00 (1.025) + 2.25an2.5% = 90 + 20.00(1.025)n
16 19 105.00 (1.025)n + 2.25an2.5% = 90 + 15.00(1.025)n
20 23 102.50 (1.025)n + 2.25an2.5% = 90 + 12.50(1.025)n
24 30 100.00 (1.025)n + 2.25an2.5% = 90 + 10.00(1.025)n
One way to solve the problem would be to laboriously compute the price for
every possible call date, and then take the minimum. However, as the above
formul express the value in terms of a decreasing function v n , it suffices to
consider the smallest value in each interval, i.e. the largest value of n. So we
have to compare the following four prices:
Call Date Price
15 103.81
19 99.38
23 97.08
30 94.77
Thus the highest price that the investor may safely pay is 94.77. Because
the prices were expressible in the form 90 + A(1.025)n , where A is a non-
increasing function of n and (1.025)n also a non-increasing function of n, we
could have stated immediately that the lowest price would be that for the bond
held to maturity: it was not necessary to carry out all these computations.
The situation is not so clear when the yield rate is less than the coupon rate.
(b) Again we tabulate the applicable price formul, based on the call or maturity
date:
First Date Last Date Price
10 15 110.00 (1.02)n + 2.25an2% = 112.50 2.50(1.02)n
16 19 105.00 (1.02)n + 2.25an2% = 112.50 7.50(1.02)n
20 23 102.50 (1.02)n + 2.25an2% = 112.50 10.00(1.02)n
24 30 100.00 (1.02)n + 2.25an2% = 112.50 12.50(1.02)n
These formul express the value in terms of an increasing function v n , it
suffices to consider the largest value in each interval, i.e. the smallest value of
n. So we have to compare the following four prices:
Call Date Price
10 110.45
16 107.04
20 105.77
24 104.73
Information for Students in MATH 329 2005 01 3036

Thus the highest price that the investor may safely pay is 104.73.
(c) As before, we tabulate the applicable price formul, based on the call or
maturity date:

First Date Last Date Price


10 15 110.00 (1.015) + 2.25an1.5% = 150.00 40.00(1.015)n
n

16 19 105.00 (1.015)n + 2.25an1.5% = 150.00 45.00(1.015)n


20 23 102.50 (1.015)n + 2.25an1.5% = 150.00 47.50(1.015)n
24 30 100.00 (1.015)n + 2.25an1.5% = 150.00 50.00(1.015)n

As in the case of 4% we have to compare four prices:

Call Date Price


10 115.53
16 114.54
20 114.73
24 115.02

This time the highest price that the investor may safely pay is 114.54.

B.1.6 2002/2003 Class Tests, with Solutions


Versions 2 and 4 appear to have been slightly more difficult than Versions 1
and 3, and the grades were adjusted to compensate for this.

Versions 1 (white) and 3 (yellow)

1. Showing your work, solve each of the following problems:

(a) [2 MARKS] Determine the nominal annual interest rate, i1 , compounded every
3 months, which is equivalent to a nominal annual interest rate of 12% com-
pounded every 4 months.
(b) [4 MARKS] Determine the nominal annual interest rate compounded semi-
annually, i2 , which is equivalent to an effective annual discount rate of 6%.
(c) [4 MARKS] Determine the nominal annual interest rate, i3 , compounded in-
stantaneously (=convertible continuously), which is equivalent to an effective
monthly discount rate of 1%.

Solution:
Information for Students in MATH 329 2005 01 3037

4
(a) In one year a sum of 1 will grow, under the first rate, to 1 + i41 , and under
3 3
the second to 1 + 0.12
3
. Equating these two yields 1 + i41 = (1.04) 4 , so
3

i1 = 4 (1.04) 4 1 = 0.1194 = 11.94% .

(b) We know several relationships between i and the corresponding d. For exam-
i 1
ple, d = iv = 1+i = 1 1+i . Solving these equations for i when d = 0.06, we
obtain, corresponding to an effective annual discount rate of 6%, an effective
1 6
annual interest rate of 0.94 1 = 94 = 0.06383. The effective semi-annual
6 12
interest rate corresponding to this effective annual rate will be (1 + 94 ) 1 =
q
100
94
1 = 0.03142. Corresponding to this semi-annual rate, the nominal
annual interest rate compounded semi-annually will be twice this rate, i.e.
6.284 %.
99
(c) Since d + v = 1, the value of v corresponding to d = 1% is 100 , so 1 + i =
100 1 1
99
= 1 + and i = ; hence the corresponding effective annual rate of
99 1 12 99
interest is 1 + 99 1 = 12.81781%. The nominal annual interest rate, i3 ,
compounded instantaneously (=convertible continuously) will be the force
of interest
12 !
1
i3 = = ln 1+
99

1 100
= 12 ln 1 + = 12 ln = 0.1206 = 12.06%.
99 99

2. (a) [2 MARKS] Define the sequence of payments whose value is represented by


the symbol sni , using a time diagram showing the payments, and indicating
the point in time where the value of the various payments is being calculated.
(b) [4 MARKS] Derive a formula for sni by using formul known to you for the
summation of arithmetic or geometric progressions. Your final formula should
P
be expressed in closed form, i.e., without using summation symbols ( ) or
dots (. . .),
(c) [4 MARKS] Define what is meant by sni . Give, without proof, a formula
which expresses the value of sni in terms of i and n.
Solution:
Information for Students in MATH 329 2005 01 3038

1 1 1 1 1

0 1 2 3 n

(a)
(b)
sni = 1 + (1 + i) + (1 + i)2 + . . . + (1 + i)n1
(1 + i)n 1
= 1
(1 + i) 1
(1 + i)n 1
=
(1 + i) 1
(1 + i)n 1
=
i
(c) sni is the value of the sum of n payments of 1 at the beginning of each year,
n+1
evaluated one year after the last payment. Its value is sn+1i 1 = (1+i) i (1+i) ;
other formul would also have been acceptable.
3. A loan of 10,000 at i = 10% is to be repaid by ten equal annual payments.
(a) [5 MARKS] Determine the annual payment.
(b) [10 MARKS] Determine an amortization schedule for the first 5 payments,
showing, for each payment, the interest portion and the portion for reduction
of principal. Use the following format for your table.
Duration Payment Interest Principal Outstanding
(Years) Repaid Principal
...
(c) [5 MARKS] If the loan is sold to an investor immediately after the 5th payment
at a price to yield 12% effective annual interest, determine the price paid by
the investor.
Solution: (Source = Deferred/Supplemental Examination in Math 329, August,
2000, Problem 3.)
Information for Students in MATH 329 2005 01 3039

(a) If the annual payment is denoted by X, it must satisfy the equation X


1000 1000
a1010% = 10, 000. Solving this equation yields X = 1(1.1) 10 = 0.61445671 =

1627.45 as the level annual payment.


(b) If we were interested only in the interest portion of the 5th payment, we might
recall having proved that [5, p. 79] to be

X(1 (1.1)105+1 ) = 1627.45 (1 0.56447393) = 708.80.

The portion for reduction of principal would then be

X 708.80 = 1627.45 708.80 = 918.65 .

However, the problem required the construction of an amortization table, so


these data can be used only to verify our computations in the table:
Duration Payment Interest Principal Outstanding
(Years) Repaid Principal
0 10000.00
1 1627.45 1000.00 627.45 9372.55
2 1627.45 937.26 690.19 8682.36
3 1627.45 868.24 759.21 7923.15
4 1627.45 792.31 835.14 7088.01
5 1627.45 708.80 918.65 6169.36
(c) While the outstanding principal is shown as 6169.35, that will equal the
present value of the remaining 5 payments of 1627.45 each only if the interest
rate remains at 10%. If the interest rate changes to 12%, the present value
5
of the remaining 5 payments falls to 1627.45 a512% = 1627.45 1(1.12)
0.12
=
5866.59. This will be the price paid by an investor who expects the 5 remain-
ing payments to yield 12% effective interest.

Versions 2 (blue) and 4 (green)

1. [20 MARKS] A borrower takes out a loan of 2000 to be paid by one payment with
full interest at the end of two years. Construct a sinking fund schedule using the
headings
Duration Contribution to Interest Interest Earned Balance of Balance of
(Years) Sinking Fund on Loan in Sinking Fund Sinking Fund Principal
...
assuming that the lender receives 10% convertible semi-annually on the loan, and
the borrower replaces the amount of the loan with equal semi-annual deposits in a
Information for Students in MATH 329 2005 01 3040

sinking fund to mature when the loan becomes due, where the sinking fund earns
8% convertible semi-annually.
Solution: (Source = Final Examination in Math 329, April, 2000, Problem 3; the
present problem is simplified from that on the examination.)

(a) [7 MARKS] The sinking fund must attain the value of 2000(1.05)4 ; if we
denote the value of the semi-annual payments into this fund, than X s40.04 =
2000(1.05)4 , so

2000 (1.05)4 0.04 97.2405


X= 4
= = 572.48.
(1.04) 1 0.16986

(b) [13 MARKS] The schedule is as follows:


Duration Contribution to Interest Interest Earned Balance of Balance of
(Years) Sinking Fund on Loan in Sinking Fund Sinking Fund Principal
0.0 0.00 0.00 0.00 0.00 2000.00
0.5 572.48 100.00 0.00 572.48 2100.00
1.0 572.48 105.00 22.90 1167.86 2205.00
1.5 572.48 110.25 46.71 1787.05 2315.25
2.0 572.48 115.76 71.48 2431.01 2431.01

2. Showing your work, solve each of the following problems:

(a) [4 MARKS] Determine the nominal annual interest rate compounded semi-
annually, i1 , which is equivalent to an effective annual discount rate of 4%.
(b) [2 MARKS] Determine the nominal annual interest rate, i2 , compounded every
6 months, which is equivalent to a nominal annual interest rate of 24% com-
pounded every 3 months.
(c) [4 MARKS] Determine the nominal annual interest rate, i3 , compounded in-
stantaneously (=convertible continuously), which is equivalent to an effective
quarterly discount rate of 2%.

Solution:

(a) We know several relationships between i and the corresponding d. For exam-
i 1
ple, d = iv = 1+i = 1 1+i . Solving these equations for i when d = 0.04, we
obtain, corresponding to an effective annual discount rate of 4%, an effective
1 4
annual interest rate of 0.96 1 = 96 = 0.04167. The effective semi-annual
4 12
interest rate corresponding to this effective annual rate will be (1 + 96 ) 1 =
q
100
96
1 = 0.02062. Corresponding to this semi-annual rate, the nominal
Information for Students in MATH 329 2005 01 3041

annual interest rate compounded semi-annually will be twice this rate, i.e.
i1 = 4.124%.
2
(b) In one year a sum of 1 will grow, under the first rate, to 1 + i22 , and under
4
the second to 1 + 0.24
4
. Equating these two yields 1 + i21 = (1.06)2 , so

i2 = 2 (1.06)2 1 = 0.0472 = 24.72% .

98
(c) Since d + v = 1, the value of v corresponding to d = 2% is 100 , so 1 + i = 100
98
=
2 2
1 + 98 and i = 98 ; hence the corresponding effective annual rate of interest is

2 4
1 + 98 1 = 8.4166%. The nominal annual interest rate, i3 , compounded
instantaneously (=convertible continuously) will be the force of interest
4 !
2
i3 = = ln 1+
98

2 100
= 4 ln 1 + = 4 ln = 0.08081 = 8.081%.
98 98

3. (a) [2 MARKS] Define the sequence of payments whose value is represented by


the symbol ani , using a time diagram showing the payments, and indicating
the point in time where the value of the various payments is being calculated.
(b) [4 MARKS] Derive a formula for ani by using formul known to you for the
summation of arithmetic or geometric progressions. Your final formula should
P
be expressed in closed form, i.e., without using summation symbols ( ) or
dots (. . .).
(c) [4 MARKS] By allowing n to approach infinity, determine a (closed form)
formula for the value of ai .
Solution:

1 1 1 1 1

0 1 2 3 n

(a)
(b)

an = v + v 2 + . . . + v n
Information for Students in MATH 329 2005 01 3042

1 vn
= v
1v
1 vn
= (1 + i)v
(1 + i) (1 + i)v
n
1v 1 vn
= =
(1 + i) 1 i
lim 1 lim v n
1 1v n 10
(c) Since 1 + i > 1, 0 < 1+i
< 1, so ai = lim = n n
= = 1i .
n i i i

B.1.7 Final Examination, 2002/2003


1. (a) [3 MARKS] The total amount of a loan to which interest has been added is
5,000. The term of the loan was 4 years. If the nominal annual rate of interest
was 6% and interest was compounded semi-annually, determine the original
amount of the loan, showing all your work.
(b) [3 MARKS] Showing all your work, determine the simple interest rate under
which a sum of money will double in 5 years.
(c) [4 MARKS] Showing all your work, determine the effective annual compound
discount rate under which a sum of money will double in 8 years.
(d) [5 MARKS] Showing all your work, determine the rate of interest, convertible
continuously, that is equivalent to an effective interest rate of 1% per month.

2. (a) [8 MARKS] To repay a loan, X is obliged to pay Y 1,000 at the end of Decem-
ber, 2004, and 1,200 at the end of December, 2006. He proposes to replace
these two payments by a single payment of 2,196 at the end of December,
2005. If Y accepts this proposal, what yield rate will he be earning on his
loan? Show all your work.
(b) [7 MARKS] Showing all your work, determine the value at time t = 0 of
a continuous annuity that pays 10,000 per year for 2 years, at an effective
annual interest rate of 5%.

3. Express each of the following only in terms of `x , and v.

(a) [2 MARKS] d27


(b) [2 MARKS] 4 q24
(c) [2 MARKS] a20:25
(d) [2 MARKS] A120:25
(e) [2 MARKS] A20:25
Information for Students in MATH 329 2005 01 3043

(f) [2 MARKS] The probability that a 25-year old will survive 40 years, but will
die before reaching age 75.
(g) [3 MARKS] 12 |a20:25

4. The Wallace Widget Company is planning to borrow 150,000 from the Bank of
Antigonish, and to undertake to pay interest annually at a rate of 12%; they plan
to contribute equal annual payments to a sinking fund that earns interest at the
rate of 9%. The sinking fund will repay the principal at the end of 10 years.
Showing all your work, determine

(a) [2 MARKS] the annual interest payment,


(b) [3 MARKS] the annual payment into the sinking fund

At the end of 4 years, when Wallace has made its annual interest payment and its
4th payment to the sinking fund, it proposes that this should be the last payment
to the sinking fund. It will apply the balance X accumulated to date in the sinking
fund to repay principal, and it will amortize the remainder of the principal by equal
annual payments over the next 5 years, at a rate of 10%.

(c) [4 MARKS] Determine the annual level payment Y under this proposal.
(d) [6 MARKS] Construct an amortization table for this proposal, under the
following headings, beginning immediately after the 4th and last payment to
the sinking fund; assume also that all outstanding interest on the loan has
been made annually to date:
Duration Payment Interest Principal Outstanding
Repaid Principal
4 0.00 0.00 0.00 150000.00X
5 Y =
.. .. .. .. ..
. . . . .
9 0.00
5. Consider a 100 par-value 15-year bond, with semi-annual coupons at the nominal
annual interest rate of 4%, convertible every six months. Let t represent time in
half-years; assume that the bond is callable at 109.00 on any coupon date from
t = 10 to t = 20 inclusive, at 104.50 from t = 21 to t = 29 inclusive, but matures
at 100.00 at t = 30. In each of the following cases, determine what price an investor
should pay to guarantee himself

(a) [7 MARKS] a nominal annual yield rate of 5%, convertible semi-annually;


(b) [8 MARKS] an effective annual yield rate of 3%.
Information for Students in MATH 329 2005 01 3044

6. In addition to her down payment, Marys purchase of her new home is financed
by a mortgage of 60,000 payable to the vendor; the mortgage is amortized over 20
years, with a level payment at the end of each month, at a nominal annual rate of
6% compounded monthly.

(a) [3 MARKS] Determine the monthly payments under this mortgage.


(b) [2 MARKS] Divide the first payment into principal and interest.
(c) [3 MARKS] Determine the outstanding principal immediately after the 60th
payment.
(d) [4 MARKS] Divide the 60th payment into principal and interest.
(e) [3 MARKS] Determine the payment that Mary could make at the end of each
year which would be equivalent to the years 12 monthly payments.

7. (a) [5 MARKS] Define what is meant by (Da)n and (Ia)n , and explain verbally
why
(Da)30 + (Ia)30 = 31a30 .
(b) [10 MARKS] Showing all your work, find the present value (using effective
annual interest rate i = 6%) of a perpetuity which pays 100 after 1 year, 200
after 2 years, increasing until a payment of 2000 is made, after which payments
are level at 2000 per year forever. [For this problem you may assume that
an nv n
(Ia)n = (131)
i
a
(Ia) = (132)
i
sn n
(Is)n = .] (133)
i

8. In order to complete the sale of his home in Vancouver, John accepted, in partial
payment, a 200,000 mortgage amortized over 15 years with level semi-annual pay-
ments at a nominal annual rate of 5% compounded semi-annually. Fred has cash
available, and is prepared to buy the mortgage from John and to invest a fixed
portion of the semi-annual payments he receives in a sinking fund that will replace
his purchase capital in 15 years. The sinking fund will earn interest at only 4%,
compounded semi-annually. Showing all your work, determine the following:

(a) [3 MARKS] the amount of the semi-annual mortgage payments


(b) [4 MARKS] as a fraction of the purchase price Fred pays for the mortgage,
the semi-annual payment into the sinking fund
Information for Students in MATH 329 2005 01 3045

(c) [8 MARKS] the amount that Fred should pay for the mortgage in order to ob-
tain an overall yield rate of 6%, compounded semi-annually on his investment.
(Note that the sinking fund earns 4% compounded semi-annually.)

B.2 2003/2004
B.2.1 First 2003/2004 Problem Assignment, with Solutions
Distribution Date: Solutions mounted on the Web on Wednesday, 4 February, 2004
Assignment was mounted on the Web on Thursday, January 15th, 2005
Hard copy was distributed on Monday, January 19th, 2004
Solutions were to be submitted by Monday, January 26th, 2004
(This is a short assignment. Subsequent assignments can be expected to be
longer.)
1. It is known that the accumulation function a(t) is of the form b (1.1)t + ct2 , where
b and c are constants to be determined.
(a) If $100 invested at time t = 0 accumulates to $170 at time t = 3, find the
accumulated value at time t = 12 of $100 invested at time t = 1.
(b) Show that this function satisfies the requirement [1, p. 2, #2] that it be non-
decreasing.
(c) Determine a general formula for in , and show that lim in = 10%. (Use
n
LHopitals Rule.)
Solution: [1, Exercise 4, p. 30] Denote the corresponding amount function by A(t).
(a) An accumulation function must have the property that a(0) = 1; this implies
that 1 = a(0) = b + 0, so b = 1.
The given data imply that
170 = 100(a(3)) = 100(1(1.331) + c 32 ) (134)
which implies that c = 0.041. We conclude that
A(t)
= a(t) = (1.1)t + 0.041t2 , (135)
A(0)
implying that a(1) = 1.141, a(12) = 9.042428377. Then
A(12) a(12)
A(12) = A(1) = A(1)
A(1) a(1)
9.042428377
= A(1) = A(1)(7.925002959) = 792.5002959
1.141
Information for Students in MATH 329 2005 01 3046

so $100 at time t = 1 grows to $792.50 at time t = 12.


(b) It follows from (135) that a0 (t) = (1.1)t ln 1.1 + 0.082t, which is positive for
positive t; thus a(t) is an increasing function of t for positive t.
(This property may also be proved from first principles. Let t1 t2 . Then

a(t2 ) a(t1 ) = (1.1)t2 + 0.041t22 (1.1)t1 + 0.041t21



= (1.1)t1 (1.1)t2 t1 1 + (0.041) (t2 t1 ) (t2 + t1 )

where both of the summands are non-negative for 0 t1 t1 .


(c)

a(n) a(n 1) (0.1)(1.1)n1 + (0.041)(2n 1)


in = =
a(n 1) (1.1)n1 (0.041)(n 1)2

2n1
0.1 + 0.041 (1.1)n1
=
(n1)2
1 0.041 (1.1) n1

By LHopitals Rule
2x 1 2
lim = lim =0
x (1.1)x1 x (1.1)x1
ln 1.1
(n 1)2 2(x 1)
lim = lim
x (1.1)n1 x (1.1)x1 ln 1.1
2
= lim x1
=0
x (1.1) (ln 1.1)2

Hence
0.1 + 0.041 (0)
lim in = = 0.1 = 10%.
n 1 0.041 (0)
2. It is known that 1000 invested for 4 years will earn 250.61 in interest, i.e., that the
value of the fund after 4 years will be 1250.61. Determine the accumulated value
of 3500 invested at the same rate of compound interest for 13 years.
Solution: [1, Exercise 14, p. 30] Let i be the rate of compound interest. Then
1000(1 + i)4 = 1250.61. The accumulated value of 3500 after 13 years will be
134
13 1250.61
3500(1 + i) = 3500 = 7239.57 .
1000
Information for Students in MATH 329 2005 01 3047

3. It is known that an investment of 750 will increase to 2097.75 at the end of 25


years. Find the sum of the present values of payments of 5000 each which will
occur at the ends of 10, 15, and 25 years.
Solution: [1, Exercise 21, p. 31] Let i be the interest rate. The known fact is that
750(1 + i)25 = 2097.75. Hence (1 + i)25 = 2.797 , so v 25 = 0.357535924. The present
value of three payments of 5000 after 10, 15, and 25 years will, therefore, be

5000(v 10 + v 15 + v 25 )
10 15 25

= 5000 (0.357535924) 25 + (0.357535924) 25 + (0.357535924) 25
= 5000(0.662709221 + 0.539500449 + 0.357535924)
= 7798.73.

4. Find the accumulated value of 1000 at the end of 10 years:

(a) if the nominal annual rate of interest is 6% convertible monthly;


(b) if the nominal annual rate of discount is 5% convertible every 2 years.

Solution: [1, Exercise 32, p. 31]

(a) The accumulation factor for each month is 1 + 6%


12
= 1.005. After 10 years
1000 grows to
1000(1.005)1012 = 1819.40.
(b) The discount factor for each 2 years is 1 2 5% = 0.09 (moving backwards),
1
corresponding to an accumulation factor of 0.9 . After 10 years 1000 grows to
10
1000(0.09) 2 = 1693.51 .

5. Given that i(m) = 5 6 6 2 and d(m) = 2 8 0.06, find m, the equivalent annual
compound interest rate, and the equivalent annual compound discount rate.
Solution: [1, Exercise 30, p. 32] For an mth of a year the relationship between i(m)
and d(m) is given by
i(m) d(m)
1+ 1 =1
m m
which is equivalent to
(m + i(m) )(m d(m) ) = m2
or
i(m) d(m)
m= .
i(m) d(m)
Information for Students in MATH 329 2005 01 3048

Substituting the given values

i(m) = 0.041241452
d(m) = 0.040408205

gives m = 2. It follows that


2
i(2)
i = 1+ 1
2
1 1
= (1.020620726)2 1 = 4.1666667% = 4 % =
6 24

(2) 2
d
d = 1 1
2
1
= 1 0.96 = 4% =
25

B.2.2 Second 2003/2004 Problem Assignment, with Solutions


Distribution Date: Mounted on the Web on Friday, February 20th, 2004
Assignment was mounted on the Web on January 19th, 2004.
Hard copy was distributed on Wednesday, January 28th, 2004
Solutions were due by Monday, February 9th, 2004
(Solutions presented subject to correction of errors and omissions.)

1. Find the present value of 1000, to be paid at the end of 37 months under each of
the following scenarios:

(a) Assume compound interest throughout, and a (nominal) rate of discount of


6% payable quarterly.
(b) Assume compound interest for whole years only at a (nominal) rate of discount
of 6% payable quarterly, and simple discount at the rate of 1.5% per 3 months
during the final fractional period.
(c) Assume compound interest throughout, and a nominal rate of interest of 8%
payable semi-annually.

Solution: (cf. [1, Exercise 2, p. 53])


373
0.06
(a) Present value = 1000 1 = 829.94.
4
Information for Students in MATH 329 2005 01 3049

363
0.06 0.015
(b) Present value = 1000 1 1 = 10000.8728230.9995 =
4 3
829.96.
37
(c) Present value = (1.04) 6 = 1000 0.785165257 = 785.17.

2. The sum of 5,000 is invested for the months of April, May, and June at 7% simple
interest. Find the amount of interest earned

(a) assuming exact simple interest in a non-leap year


(b) assuming exact simple interest in a leap year (with 366 days);
(c) assuming ordinary simple interest;
(d) assuming the Bankers Rule.

Solution: (cf. [1, Exercise 6, p. 54])


91
(a) The number of days is 30 + 31 + 30 = 91; exact simple interest is 365
5000
(0.07) = 87.26.
91
(b) Exact simple interest is 366
5000 (0.07) = 87.02
30+30+30
(c) Ordinary simple interest is 360
5000 (0.07) = 87.50
91
(d) Interest under the Bankers Rule is 360
5000 (0.07) = 88.47.

3. Find how long 4,000 should be left to accumulate at 5% effective in order that it
will amount to 1.25 times the accumulated value of another 4,000 deposited at the
same time at a nominal interest rate of 4% compounded quarterly.
Solution: (cf. [1, Exercise 13, p. 55]) The equation of value at n years is

4000(1.05)n = (1.25)(4000)(1.01)4n

so
ln 1.25
n= = 24.82450822 .
ln 105 4 ln 101
4. The present value of two payments of 100 each, to be made at the end of n years
and 2n years is 63.57. If i = 6.25%, find n.
Solution: (cf. [1, Exercise 14, p. 55]) Solving the equation of value, 100v 2n +100v n =
63.57, we obtain
1 3.5428
vn = ,
2

UPDATED TO April 12, 2005


Information for Students in MATH 329 2005 01 3050

in which only the + sign is acceptable, since v n > 0. Taking logarithms gives
0.818446587
n= = 13.50023411.
ln 1.0625
We conclude, to the precision of the problem, that n = 13.5 years.

5. (a) Find the nominal rate of interest convertible quarterly at which the accumu-
lated value of 1000 at the end of 12 years is 3000.
(b) Find the nominal rate of discount convertible semi-annually at which a pay-
ment of 3000 12 years from now is presently worth 1000.
(c) Find the effective annual rate of interest at which the accumulated value of
1000 at the end of 12 years is 3000.
Solution:

(a) (cf. [1, Exercise 19, p. 55]) The equation of value at time t = 12 is
412
i(4)
1000 1 + = 3000 ,
4

implying that 1
i(4) = 4 3 48 1 = 9.260676%.

(b) The equation of value at time t = 12 is


212
d(2)
3000 1 = 1000 ,
2

implying that
1
d(2) = 2 1 3 24 = 9.3678763%.

(c) The equation of value at time t = 12 is

1000 (1 + i)12 = 3000 ,

implying that
1
i = 3 12 1 = 9.5872691%.

6. An investor deposits 20,000 in a bank. During the first 4 years the bank credits an
annual effective rate of interest of i. During the next 4 years the bank credits an
annual effective rate of interest of i 0.02. At the end of 8 years the balance in the
Information for Students in MATH 329 2005 01 3051

account is 22,081.10. What would the account balance have been at the end of 10
years if the annual effective rate of interest were i + 0.01 for each of the 10 years?
Solution: (cf. [1, Exercise 32, p. 57]) The equation of value is

20000(1 + i)4 (1 + (i 0.02))4 = 22081.10 ,

which we interpret as a polynomial equation. The equation is of degree 8, and we


dont have a simple algebraic method for solving such equations in general. But
this equation has the left side a pure 4th power, so we can extract the 4th roots of
both sides, obtaining
1
(1 + i)(1 + (i 0.02)) = (1.104055) 4 = 1.025056201 ,

which may be expressed as a quadratic equation in 1 + i:

(1 + i)2 0.02(1 + i) 1.025056 = 0

whose only positive solution is


p
0.02 + (0.02)2 + 4(1.025056)
1+i= = 1.0225
2
from which we conclude that i = 2.25%, and that the account balance after 10
years would be 20000(1.0225 + 0.01)10 = 20000(1.0325)10 = 27, 737.89.

7. A bill for 1000 is purchased for 950 4 months before it is due. Find

(a) the nominal rate of discount convertible monthly earned by the purchaser;
(b) the annual effective rate of interest earned by the purchaser.

Solution: (cf. [1, Exercise 25, p. 56])

(a) If d(12) be the nominal discount rate, then


4
d(12)
1000 1 = 950
12

implying that
d(12)
1 = 0.9872585
12
so d(12) = 15.29%.
Information for Students in MATH 329 2005 01 3052

(b) Let i be the effective annual interest rate. Then


1
950(1 + i) 3 = 1000

implies that 3
1000
i= 1 = 16.635% .
950
8. A signs a 2-year note for 4000, and receives 3168.40 from the bank. At the end
of 6 months, a year, and 18 months A makes a payment of 1000. If interest is
compounded semi-annually, what is the amount outstanding on the note at the
time if falls due?
Solution: If i0 be the rate of interest charged semi-annually, then

3168.40(1 + i0 )4 = 4000

so i0 = 6.00%; that is i(2) = 12.00%. The value of the 3 payments at the time the
note matures is

1000 (1.06)3 + (1.06)2 + (1.06)1 = 3374.62

so the amount outstanding before the final payment is 625.38.


9. The Intermediate Value Theorem for continuous functions tells us that such a
function f (x) whose value at x = a has the opposite sign from its value at x = b
will assume the value 0 somewhere between a and b. By computing the value of
f at the point 21 (a + b), we can infer that there is a 0 of f in an interval half as
long as [a, b], and this procedure may be repeated indefinitely to determine a zero
of f to any desired accuracy. Assuming that polynomials are continuous, use this
idea to determine the nominal quarterly compound interest rate under which the
following payments will accumulate to 1000 at the end of 4 years:
300 today

200 at the end of 1 year

300 at the end of 2 years


Your answer should be accurate to 3 decimal places, i.e., expressed as a percentage
to 1 decimal place.
Solution: (We will carry the computations to an accuracy greater than requested
in the problem.)
Information for Students in MATH 329 2005 01 3053

(a) Let the effective annual interest rate be i. The equation of value at the end
of 4 years is
300(1 + i)4 + 200(1 + i)3 + 300(1 + i)2 = 1000 . (136)

(b) We need a continuous function to which to apply the Intermediate Value


Theorem. Some choices may be better than others. We will choose
f (x) = 3x4 + 2x3 + 3x2 10 .
We observe that f (0) = 10, that f (2) = 48+16+12 10 = 66 > 0, and that
f (2) = 48 16 + 12 10 = 34 > 0. This tells us that there is a solution to
equation (136) for 2 x 0, equivalently for 3 i 1: such a solution
is of no interest to us, as it does not fit the constraints of this problem. But
the function is a cubic polynomial, and has 2 other zeros. We see that it also
has a solution in the interval 0 x 2, and we proceed to progressively halve
intervals.
(c) The midpoint of interval [0, 2] is 1;
f (1) = 3 + 2 + 3 10 = 2 < 0 < 66 = f (2) ,
so there must be a root in the interval [1, 2].
(d) The midpoint of [1, 2] is 1.5;
f (1.5) = 3(1.5)4 + 2(1.5)3 + 3(1.5)2 10
= 15.1875 + 6.75 + 6.75 10 = 18.6875
> 0
so there must be a zero in the interval [1, 1.5], whose midpoint is 1.25.
(e)
f (1.25) = 3(1.25)4 + 2(1.25)3 + 3(1.25)2 10
= 7.3242 + 3.9063 + 4.6875 10 = 5.918
> 0
so there must be a zero in the interval [1, 1.25], whose midpoint is 1.125.
(f)
f (1.125) = 3(1.125)4 + 2(1.125)3 + 3(1.125)2 10
= 4.8054 + 2.8477 + 3.7969 10 = 1.45
> 0
so there must be a zero in the interval [1, 1.125], whose midpoint is 1.0625.
Information for Students in MATH 329 2005 01 3054

(g)

f (1.0625) = 3(1.0625)4 + 2(1.0625)3 + 3(1.0625)2 10


= 3.8233 + 2.3989 + 3.3867 10 = 0.3911
< 0

so there must be a zero in the interval [1.0625, 1.125], whose midpoint is


1.09375.
(h)

f (1.09375) = 3(1.09375)4 + 2(1.09375)3 + 3(1.09375)2 10


= 4.2933 + 2.6169 + 3.5889 10 = 0.4991
> 0

so there must be a zero in the interval [1.0625, 1.09375], whose midpoint is


1.078125.
(i)

f (1.078125) = 3(1.078125)4 + 2(1.078125)3 + 3(1.078125)2 10


= 4.053197 + 2.506325 + 3.487061 10 = 0.046582
> 0

so there must be a zero in the interval [1.0625, 1.078125], whose midpoint is


1.0703125.
(j)

f (1.0703125) = 3(1.0703125)4 + 2(1.0703125)3 + 3(1.0703125)2 10


= 3.936984 + 2.452233 + 3.436707 10 = 0.174076
< 0

so there must be a zero in the interval [1.0703125, 1.078125], whose midpoint


is 1.07421875.
(k) f (1.07421875) = .064207958 < 0 so there must be a zero in the interval
[1.07421875, 1.078125], whose midpoint is 1.076172.
(l) f (1.076172) = .008925 < 0 so there must be a zero in the interval
[1.076172, 1.078125], whose midpoint is 1.077149.
(m) f (1.077149) = .018814 > 0 so there must be a zero in the interval
[1.076172, 1.077149], whose midpoint is 1.076661.
Information for Students in MATH 329 2005 01 3055

(n) f (1.076661) = .004952 > 0 so there must be a zero in the interval


[1.076172, 1.076661], whose midpoint is 1.076417.
(o) f (1.076417) = .001974 < 0 so there must be a zero in the interval
[1.076417, 1.076661], whose midpoint is 1.076539.
(p) f (1.076539) = .001488 > 0 so there must be a zero in the interval
[1.076417, 1.076539], whose midpoint is 1.076478.
(q) f (1.076478) = .000243 < 0 so there must be a zero in the interval
[1.076478, 1.076539], whose midpoint is 1.076509.
(r) f (1.076509) = .000637 > 0 so there must be a zero in the interval
[1.076478, 1.076509], whose midpoint is 1.076494.
(s) f (1.076494) = .000211 > 0 so there must be a zero in the interval
[1.076478, 1.076494], whose midpoint is 1.076486.
(t) f (1.076486) = .000016 > 0 so there must be a zero in the interval
[1.076486, 1.076494], whose midpoint is 1.07649.
(u) f (1.07649) = .000097 > 0 so there must be a zero in the interval
[1.076486, 1.07649], whose midpoint is 1.076488.
(v) f (1.076488) = .000041 > 0 so there must be a zero in the interval
[1.076486, 1.076488], whose midpoint is 1.076487.
(w) f (1.076487) = .000012. One zero will be approximately x = 1.07649. Thus
the effective annual interest rate is approximately 7.649%. This, however, is
not what the problem asked for. The accumulation function for 3-months will
1
then be (1.0749) 4 = 1.01822, so the effective interest rate for a 3-month period
will be 1.822%, and the nominal annual interest rate, compounded quarterly,
will be 7.288, or 7.3% to the accuracy requested.

THE FOLLOWING PROBLEM WAS CONSIDERED FOR INCLUSION IN THE AS-


SIGNMENT, BUT WAS (FORTUNATELY) NOT INCLUDED.

10. [1, Exercise 6, p. 88]

(a) Show that


amn = am v m sn = (1 + i)n am sn
where 0 < n < m.
(b) Show that
smn = sm (1 + i)m an = v n sm an
where 0 < n < m.
Information for Students in MATH 329 2005 01 3056

(c) Interpret the results in (a) and (b) verbally.

Solution:

(a) We prove the first of these identities by technical substitutions in sums, anal-
ogous to changes of variables in a definite integral. For the second identity we
give a less formal proof.
mn
X
amn = vr
r=1
m
X m
X
r
= v vr
r=1 r=mn+1
m
X m
X
= vr v m
v rm
r=1 r=mn+1
m
X 1
X
r m
= v v v 1s
r=1 s=n
under the change of variable s = m r + 1
Xm Xn
= vr vm v 1s
r=1 s=1
reversing the order of the 2nd summation
Xm Xn
r m
= v v (1 + i)s1
r=1 s=1
m
= am v sn
amn = v + v 2 + . . . + v mn
= v n+1 + v n+2 + . . . + v 0 + v 1 + v 2 + . . . + v mn

v n+1 + v n+2 + . . . + v 0

= v n v 1 + v 2 + . . . + v n + v n+1 + v n+2 + . . . + v m

(1 + i)n1 + (1 + i)n2 + . . . + (1 + i)v 0

= (1 + i)n v 1 + v 2 + . . . + v n + v n+1 + v n+2 + . . . + v m

(1 + i)0 + (1 + i)1 + . . . + (1 + i)v n1
= (1 + i)n am sn

(b) These identities could be proved in similar ways to those used above. Instead,
we shall show that these identities can be obtained from the preceding simply
Information for Students in MATH 329 2005 01 3057

by multiplying the equations by (1 + i)mn :

smn = (1 + i)mn amn [1, (3.5), p. 60]


mn n
= (1 + i) ((1 + i) am sn )
= (1 + i) am (1 + i)m (1 + i)n sn
m

= sm (1 + i)m an
smn = (1 + i)mn amn
= (1 + i)mn (am v m sn )
= (1 + i)n (1 + i)m am (1 + i)n sn
= (1 + i)n sm an

(c) i. An (m n)-payment annuity-immediate of 1 has the same present value


as an annuity for a total term of m = (m n) + n years minus a correc-
tion paid today equal to the value of the deferred n payments. Those n
payments are worth sm at time t = m, which amount can be discounted
to the present by multiplying by v m .
The preceding explanation was based on values at the commencement of
the first year of an m-year annuity-immediate. Let us now interpret the
m n payments as being the last payments of an m-year annuity whose
mth payment has just been made. That m-payment annuity was worth
am , n years ago a year before its first payment; today it is worth
(1 + i)n an , including the payments we attached at the beginning. Those
payments are worth sn today, for a net value as claimed.
ii. Consider an value of the first m n payments of an m-payment annuity-
immediate of 1, just after the (m n)th payment. Since these could be
considered simply the accumulated value of an (m n)-payment annuity,
they are worth smn . But the payments of the m-payment annuity not
yet made are worth an , and the entire annuity is worth sm at termination,
hence v n sm today; hence v m sm an is also the value today. This gives
the equality between the extreme members of the alleged inequality.
Now lets evaluate the same m n payments, but this time consider them
to be the last m n payments of an m-payment annuity-certain; again,
the m nth payment has just been made. From first principles, the
accumulated value of the payments actually received is smn , and we are
viewing them from the context of an annuity-certain of m payments that
would be worth sm today: lets determine the amount that would have
to be paid out today to correct for that expanded annuity. The value of
the n payments we have tacked on in the past was an one year before
Information for Students in MATH 329 2005 01 3058

the payments began, and (1 + i)m sn today; so we can also view the value
today as sm (1 + i)m an .

B.2.3 Third 2003/2004 Problem Assignment, with Solutions


Distribution Date: Mounted on the Web on Wednesday, March 3rd, 2004.
Assignment was mounted on the Web on February 8th, 2004,
hard copy of assignment was distributed on Wednesday, February 11th, 2004.
Solutions were to be submitted by 9 a.m., Monday, March 3rd, 2004
SUBJECT TO CORRECTION OF TYPOS AND OTHER ERRORS

Sketch a time diagram to accompany your solution of all problems except the last.

1. A skier wishes to accumulate 30,000 in a chalet purchase fund by the end of 8


years. If she deposits 200 into the fund at the end of each month for the first 4
years, and 200 + X at the end of each month for the next 4 years, find X if the
fund earns a nominal (annual) rate of 6% compounded monthly.
Solution: The equation of value at the end of 8 12 = 96 months is

200s96 + X s48 = 30000 ,

which we may solve to yield


30000 s
X = 200 96 0.005
s48 0.005 s48 0.005
30000 (1.005)96 1
= 200
s48 0.005 (1.005)48 1
= 30000s1
48 0.005
200((1.005)48 + 1)
= 30000(0.018485) 200(2.27049) from the tables
= 100.452.

2. A fund of 2500 is to be accumulated by n annual payments of 50, followed by n + 1


annual payments of 75, plus a smaller final payment of not more than 75 made 1
year after the last regular payment. If the effective annual rate of interest is 5%,
find n and the amount of the final irregular payment.
Solution: We shall interpret the payments to be made under two annuities-due: the
first, for 2n + 1 years, consists of an annual deposit of 50 in advance; the second,
for n + 1 years, deferred n years after the first, consists of an annual deposit of 25
in advance. It is at the end of year 2n + 1 that the final drop payment is to
Information for Students in MATH 329 2005 01 3059

be made, and it is to be under 75. (Note that this is the type of problem where the
drop payment could turn out to be negative.) We seek the smallest n for which

50 s2n+1 + 25 sn+1 > 2500 75 = 2425



n 2 n 0.05(2425)
2(1.05) ((1.05) ) + (1.05) 3 +
25(1.05)

n 2 1 n 1 0.05(2425)
((1.05) ) + (1.05) 3+
2.1 2.1 25(1.05)
2
n 1 1 1 0.05(2425)
(1.05) + > + 3+
4.2 (4.2)2 2.1 25(1.05)
21
n 1 1 1 0.05(2425)
(1.05) + > + 3+ = 1.919585178
4.2 (4.2)2 2.1 25(1.05)
ln 1.919585178
n> = 10.65133267
ln 1.05
Thus the drop payment will be when t = 11 + 12, i.e., 23 years after the first
payment under the annuity with payments of 50. Just before the drop payment
the accumulated value of all previous payments is
1.05
50s23 + 25s12 = 50 (1.05)23 + 25 (1.05)12 75 = 2592.924516
0.05
so the drop payment at time t = 23 is 2500 2592.924516 = 92.92.
Thus we have an example here of a negative drop payment. Could this mean
that we should have taken n = 10? No. In that case we would find that the
final payment would be larger than the permitted 75. (If tables like those in the
textbook were available, one could determine the value of n by inspecting the value
of 2s2n+1 + sn+1 . We observe from the 5% tables the following values:

n 2s2n+1 + sn
10 84.0165
11 97.0678
12 111.3713

We seek the smallest n such that

50s2n+1 + 25sn > 2500 75

i.e., such that


2s2n+1 + sn > 97 ,
Information for Students in MATH 329 2005 01 3060

equivalently,
97
2s2n+1 + sn > = 92.38 ,
1.05
and so can conclude that n = 11.)

3. On his 30th birthday, a teacher begins to accumulate a fund for early retirement
by depositing 5,000 on that day and at the beginnings of the next 24 years as
well. Since he expects that his official pension will begin at age 65, he plans that,
starting at age 55 he will make an annual level withdrawal at the beginning of each
of 10 years. Assuming that all payments are certain to be made, find the amount
of these annual withdrawals, if the effective rate of interest is 6% during the first
25 years, and 7% thereafter.
Solution: Let the constant amount of the withdrawals beginning at age 55 be X.
The equation of value at age 55, just before the first withdrawal, is

5000 s25 6% = X a10 7%


5000 s25 6%
Annual Withdrawal X =
a10 7%
1.06 0.07 (1.06)25 1
= 5000
1.07 0.06 1 (1.07)10
= 33477.74

4. At an effective annual interest rate of i it is known that

(a) The present value of 5 at the end of each year for 2n years, plus an additional
3 at the end of each of the first n years, is 64.6720.
(b) The present value of an n-year deferred annuity-immediate paying 10 per year
for n years is 34.2642.

Find i.
Solution: It is convenient to distinguish two cases.

Case i 6= 0: From (4a) we have an equation of value

64.6720 = 5 a2n + 3 an ; (137)

from (4b) we have the equation of value



34.2642 = v n 10 an = 10 a2n an . (138)

UPDATED TO April 12, 2005


Information for Students in MATH 329 2005 01 3061

Solving these equations, we obtain

a2n = 9.3689 (139)


an = 5.9425 , (140)

implying that

1 v 2n 9.3689
n
=
1v 5.9425
1 + v n = 1.5766
v n = 0.5766.

We can substitute in equation (140) to obtain

i = 0.07125 = 7.125%

which implies that


ln 0.5766
n= = 8.000 years.
ln 1.07125

Case i = 0: Here Equations (137) and (138) become

64.6720 = 5(2n) + 3n = 13n


34.2642 = 10n = 10(2n n)

which are inconsistent. Thus this case is impossible.

5. (a) Find a12 if the effective rate of discount is 5%.


(b) Charles has inherited an annuity-due on which there remain 12 payments of
10,000 per year at an effective discount rate of 5%; the first payment is due
immediately. He wishes to convert this to a 25-year annuity-immediate at the
same effective rates of interest or discount, with first payment due one year
from now. What will be the size of the payments under the new annuity?
Solution:

(a) Since (1 d)(1 + i) = 1, v = 1 d = 0.95 when d = 0.05.

1 (0.95)12
a12 = = 9.19279825.
0.05
Information for Students in MATH 329 2005 01 3062

(b) i = d(1 + i) = vd = 0.05


0.95
1
= 19 . Let X be the size of the new payments. We
must solve the equation of value,

91927.9825 = X a25i

where v = 0.95. Hence


91927.9825
X =
a25i
91927.9825i
=
1 v 25
91927.9825
=
(1 (0.95)25 ) 19
= 6695.606220

So the new annuity-immediate will pay 25 annual payments of 6695.61, be-


ginning one year from now.

6. Give an algebraic proof and a verbal explanation for the formula

m |an = a am v m+n a .

Solution:

(a)
1 1 vm 1
a am v m+n a = v m+n
i i i
1 (1 v m ) v m+n
=
i
v m (1 v n ) 1 vn
= = vm = v m an = m |an
i i
(b) a i is the present value of a perpetuity at rate i of 1 per year, payments
starting a year from now. am is the present value of the first m payments of
that perpetuity; if we subtract this we have the present value of a perpetuity-
immediate that starts m years from now, i.e., where the first payment is m + 1
years from now. v m+n a is the value of a perpetuity-immediate of 1 starting
m + n years from now, i.e., where the first payment is m + n + 1 years from
now; if we subtract this term as well, we are left with the present value of
payments at the ends of years m + 1, m + 2, . . ., m + n, i.e., with the present
value of an n-payment annuity-certain of 1, deferred m years, i.e., of m |an
Information for Students in MATH 329 2005 01 3063

7. A level perpetuity-immediate is to be shared by A, B, C, and D. A receives the


first n payments, B the next 2n payments, C payments ##3n + 1, . . . , 5n, and D
the payments thereafter. It is known that the present values of Bs and Ds shares
are equal. Find the ratio of the present value of the shares of A, B, C, D.
Solution: The present values of the shares of A, B, C, D are, respectively, an ,
v n a2n = a3n an , v 3n a2n = a5n a3n , and v 5n a = a a5n . The fact that Bs
and Ds shares are equal implies that
vn v 5n
(1 v 2n ) =
i i
which is equivalent to (v 2n )2 + v 2n 1 = 0, implying that

1 5
v 2n = .
2
Since v is positive, only the + sign is admissible:

2n 1 + 5
v = = 0.618033988...
2
so
v n = 0.7861513777... .
Then the shares of A, B, C, D will be in the ratio

1 v n : v n v 3n : v 3n v 5n : 1 v 5n

i.e.
0.2138486221 : 0.3002831059 : 0.1855851657 : 0.6997168937

8. (a) Find the present value of an annuity which pays 4,000 at the beginning of
each 3-month period for 12 years, assuming an effective rate of 2% interest
per 4-month period.
(b) Suppose that the owner of the annuity wishes to pay now so that payments
under his annuity will continue for an additional 10 years. How much should
he pay?
(c) How much should he pay now to extend the annuity from the present 12 years
to a perpetuity?
(It is intended that you solve this problem from first principles, not by substitu-
tion into formul in [1, Chapter 4].)
Solution:
Information for Students in MATH 329 2005 01 3064

(a) If i be the effective interest rate per 4-month period, the effective rate per
3
3-month period will be j = (1 + i) 4 1. Accordingly the value of the desired
annuity is

1 (1 + j)48
4000a48 j = 4000(1 + j)
j
3 1 (1 + i)36
= 4000(1 + i) 4 3
(1 + i) 4 1
1 (1.02)36
= 4000 3
1 (1.02) 4
= 138, 317.4894.

(b) Repeating the calculations above for 48 + 40 = 88 payments, we obtain

1 (1 + j)88
4000a88 j = 4000(1 + j)
j
3 1 (1 + i)66
= 4000(1 + i) 4 3
(1 + i) 4 1
1 (1.02)66
= 4000 3
1 (1.02) 4
= 197, 897.4338,

so the additional payments will cost 197, 897.4338138, 317.4894 = 59, 579.9444
today.
(c) The cost of the perpetuity-due today would be
1
4000a88 j = 4000(1 + j)
j
3 1
= 4000(1 + i) 4 3
(1 + i) 4 1
4000
= 3
1 (1.02) 4
= 271, 329.4837,

so the additional payments will cost

271, 329.4837 138, 317.4894 = 133, 011.9943

today.
Information for Students in MATH 329 2005 01 3065

9. (No time diagram is needed for the solution to this problem.) In Problem 9 of
Assignment 2 you were asked to apply the Bisection Method to determine the
solution to an interest problem to 3 decimal places. The equation in question was
(136):
300(1 + i)4 + 200(1 + i)3 + 300(1 + i)2 = 1000 .
and the solution given began with the values of
f (x) = 3x4 + 2x3 + 3x2 10 .
at x = 0 (f (0) = 10), x = 2 (f (2) = 66 > 0), and x = 2 (f (2) = 34 >
0), and we were interested in the solution between 0 and 2 a solution that
is unique because f 0 is positive in this interval. Apply Linear Interpolation 4
times in an attempt to determine the solution we seek. (You are not expected
to know the general theory of error estimation.) The intention is that you apply
linear interpolation unintelligently, using it to determine a point where you find the
function value and thereby confine the zero to a smaller subinterval: the point that
you find will replace the midpoint in the bisection method. In some situations, as
in the present one, the procedure may not be better than the bisection method.
Indeed, in the present example, it could take many more applications than the
bisection method to obtain the accuracy you obtained with that method.
Solution: We take x1 = 0, x2 = 2. Then
10
x3 = 0 + (2 0)
10 66
= 0.2631578947
f (x3 ) = 9.741407754
9.741407754
x4 = 0.2631578947 + (2 0.2631578947)
9.741407754 66
= 0.4865401588
f (x4 ) = 8.891376200
8.891376200
x5 = 0.4865401588 + (2 0.4865401588)
8.891376200 66
= 0.6662236083
f (x5 ) = 7.486007579
7.486007579
x6 = 0.6662236083 + (2 0.6662236083)
7.486007579 66
= 0.8020951913
f (x6 ) = 5.796139773
5.796139773
x7 = 0.8020951913 + (2 0.8020951913)
5.796139773 66
Information for Students in MATH 329 2005 01 3066

= 0.8988026707
f (x7 ) = 4.166425912
4.166425912
x8 = 0.8988026707 + (2 0.8988026707)
4.166425912 66
= 0.9641908820
f (x8 ) = 2.825434839
2.825434839
x9 = 0.9641908820 + (2 0.9641908820)
2.825434839 66
= 1.006713115
f (x9 ) = 1.837664218
1.837664218
x10 = 1.006713115 + (2 1.006713115)
1.837664218 66
= 1.033620406
f (x10 ) = 1.162055435
1.162055435
x11 = 1.033620406 + (2 1.033620406)
1.162055435 66
= 1.050340958
f (x11 ) = 0.721587971
0.721587971
x12 = 1.050340958 + (2 1.050340958)
0.721587971 66
= 1.060611435
f (x12 ) = 0.442976533
0.442976533
x13 = 1.060611435 + (2 1.060611435)
0.442976533 66
= 1.066874356
f (x13 ) = 0.270019571
0.270019571
x14 = 1.066874356 + (2 1.066874356)
0.270019571 66
= 1.070676410
f (x14 ) = 0.163879567
0.163879567
x15 = 1.070676410 + (2 1.070676410)
0.163879567 66
= 1.072978227
f (x15 ) = 0.099198908
0.099198908
x16 = 1.072978227 + (2 1.072978227)
0.099198908 66
= 1.074369462
Information for Students in MATH 329 2005 01 3067

f (x16 ) = 0.059950550
0.059950550
x17 = 1.074369462 + (2 1.074369462)
0.059950550 66
= 1.075209488
f (x17 ) = 0.036195811
0.036195811
x18 = 1.075209488 + (2 1.075209488)
0.036195811 66
= 1.075716385
f (x18 ) = 0.021840825
0.021840825
x19 = 1.075716385 + (2 1.075716385)
0.021840825 66
= 1.076022149
f (x19 ) = 0.013174269
0.013174269
x20 = 1.076022149 + (2 1.076022149)
0.013174269 66
= 1.076206548
f (x20 ) = 0.007944937
0.007944937
x21 = 1.076206548 + (2 1.076206548)
0.007944937 66
= 1.076317739
f (x21 ) = 0.004790698
0.004790698
x22 = 1.076317739 + (2 1.076317739)
0.004790698 66
= 1.076384781
f (x22 ) = 0.002888504
0.002888504
x23 = 1.076384781 + (2 1.076384781)
0.002888504 66
= 1.076425201
f (x23 ) = 0.001741533
0.001741533
x24 = 1.076425201 + (2 1.076425201)
0.001741533 66
= 1.076449571
f (x24 ) = 0.001049952
0.001049952
x25 = 1.076449571 + (2 1.076449571)
0.001049952 66
= 1.076464263
f (x25 ) = 0.000632996
Information for Students in MATH 329 2005 01 3068

0.000632996
x26 = 1.076464263 + (2 1.076464263)
0.000632996 66
= 1.076473120
f (x26 ) = 0.000381633
0.000381633
x27 = 1.076473120 + (2 1.076473120)
0.000381633 66
= 1.076478460
f (x27 ) = 0.000230079
0.000230079
x28 = 1.076478460 + (2 1.076478460)
0.000230079 66
= 1.076481679
f (x28 ) = 0.000138723
0.0001387233
x29 = 1.076481679 + (2 1.076481679)
0.0001387233 66
= 1.076483620
f (x29 ) = 0.000083635
0.000083635
x30 = 1.076483620 + (2 1.076483620)
0.000083635 66
= 1.076484790
f (x30 ) = 0.000050430
0.000050430
x31 = 1.076484790 + (2 1.076484790)
0.000050430 66
= 1.076485496
f (x31 ) = 0.000030391

The reason that the method is not efficient here is that the graph of the function
is far from linear in the interval under consideration.41

B.2.4 Fourth 2003/2004 Problem Assignment, with Solutions


Distribution Date: Mounted on the Web on Wednesday, March 31st, 2004
Solutions were due by Wednesday, March 17th, 2004
Corrected as of April 29th, 2004.
(Solutions presented subject to further correction of errors and omissions.)

1. (a) Find, to the nearest unit, the accumulated value 19 years after the first pay-
ment is made of an annuity on which there are 7 payments of 3000 each made
41
Issues of this type are beyond MATH 329, and are not covered adequately in the current textbook;
if you are interested in Numerical Anaylsis, consider taking MATH 317.
Information for Students in MATH 329 2005 01 3069

at 1 12 -year intervals. The nominal rate of interest convertible semiannually is


6%.
(b) Find, to the nearest unit, the present value of a 20-year annuity-due which
pays 200 at the beginning of each half-year for the first 8 years, increasing to
250 per half-year thereafter. The effective annual rate of interest is 6%.
Solution:
(a) We will interpret the payments as being made under an annuity-immediate
with time-intervals of 1 12 years. The clock starts ticking (i.e. t = 0) 1 12 years
before the first payment; the last payment is made at time t = 7 1.5 = 10.5.
The evaluation is to be made at time t = 19 1.5 = 17.5, i.e., 7 years after the
last payment; this is 14
3
intervals of length 1 12 years; it is simpler to view this
as 14 intervals of length 12 year, for each of which the effective interest rate is
3%. The effective interest rate call it j per 1 21 years is j = (1.03)3 1 .
The accumulated value is, therefore,
3000
3000(1.03)14 s7 = (1.03)14 (1 + j)7 1
j
3000
= 3
(1.03)14 (1.03)21 1
(1.03) 1
= 42100.12386
which is 42,100 to the nearest unit.
1 1
(b) With 1 + i = (1.06) 2 , v = (1.06) 2 ,
Present Value = 250a40 50a16

1 v 39 1 v 15
= (250 50) + 250 50
i i
39 15
250 (1 v ) 50 (1 v )
= 200 +
i
200 250v 39 + 50v 15
= 200 +
i
= 5342.993032.
To the nearest unit the present value is 5343.
2. (a) The present value of a perpetuity-immediate paying 1 at the end of every 5
years is 1.637975. Find i and d.
(b) The present value of a perpetuity-due paying 1 at the beginning of every 5
years is 1.637975. Find d and i.
Solution:
Information for Students in MATH 329 2005 01 3070

(a) A payment of 1 at the end of every 5th year for 5 years is equivalent to a
payment of s51 at the end of every year. The equation of value is

1
1.637975 = s1
5 i
1
1.637975 =
(1 + i)5 1
(1 + i)5 = 1.610500
i = 10.0000%,
0.1 1
d= = .
1.1 11

(b) A payment of 1 at the beginning of every 5th year is equivalent to a payment


of a1
5
at the beginning of every year. The equation of value is

1
1.637975 = s1
5 d
1
1.637975 =
1 (1 + i)5
(1 + i)5 = 0.389490
i = 20.7538%,
d = 15.1703%.

3. Determine the present value, at a nominal interest rate of 6% compounded quar-


terly, of the following payments made under an annuity: 120 at the end of the 3rd
year, 110 at the end of the 4th year, decreasing by 10 each year until nothing is
paid.
Solution: The payments decrease until a payment of 10 at the end of the 14th
year. We can think of a decreasing annuity-immediate of value 10 (Da)14 starting
with a payment of 140 at the end of the 1st year, and then make corrections. The
effective annual interest rate i is given by
4
0.06
1+i= 1+ = (1.015)4 .
4

We need only subtract the present values of the payments due at the end of the
first and second years.

Present Value = 10(Da)14 140v 130v 2


Information for Students in MATH 329 2005 01 3071


10 14 a14
= 140(1.015)4 130(1.015)8
i
56
10 14 1(1.015)
(1.015)4 1
= 140(1.015)4 130(1.015)8
(1.015)4 1
= 532.1438213

4. Find the present value, at an effective annual interest rate of 5.75%, of a perpetuity-
immediate under which a payment of 100 is made at the end of the 1st year, 300 at
the end of the 2nd year, increasing until a payment of 2500 is made, which level is
maintained for exactly a total of 10 payments of 2500 (including the first of them
in the count of 10), after which the payments fall by 400 each year until they reach
a level of 100, which is maintained in perpetuity. (Note: You are expected to show
explicitly how you decompose the payments; it is not sufficient to simply show a
few numbers and a sum.)
Solution: The payments reach the level of 2500 at the end of year 13, and continue
at that level until the end of year 22, after which they fall by 400 annually until
they reach 100 at the end of year 28.
Since the steady state is a constant perpetuity, we can begin with a perpetuity-
100
immediate of 100 per year, whose present value is . The remaining non-zero
0.0575
portions of payments are finite in number: there are additional amounts of 200
at the end of year 2, 400 at the end of year 3, . . . , 2400 at the ends of years 13
through 22, 2000 at the end of year 23, . . . , and a final amount of 400 at the end
of year 27.
The value at time t = 22 of the remaining amounts for years 23 through 27 is
400 (Da)5 , so the value at time t = 0 is (1.0575)22 400 (Da)6 . The present value
of the remainders of the payments at the ends of years 2, 3, ..., 12 is (1.0575)1
200 (Ia)11 . Thus the sum
100
+ (1.0575)22 400 (Da)5 + (1.0575)1 200 (Ia)11
0.0575
covers all the payments except for a level annuity of 2400 payable at the ends of
years ##13. . . 22, whose present value is

2400 a22 a12 .

Thus the present value of the entire scheme of payments is


100
+ (1.0575)22 400 (Da)5 + (1.0575)1 200 (Ia)11
0.0575
UPDATED TO April 12, 2005
Information for Students in MATH 329 2005 01 3072


+2400 a22 a12
100 22 5 a5 1 a11 11v 11
= + (1.0575) 400 + (1.0575) 200
0.0575 0.0575 0.0575
+2400 a22 a12
5
100 22 5 1(1.0575)
0.0575
= + (1.0575) 400
0.0575 0.0575
1.0575(1.0575)10
11(1.0575)11
+(1.0575)1 200 0.0575
0.0575
(1.0575)12 (1.0575)22
+2400
0.0575
= 1739.130435 + 1543.013920 + 8225.826240 + 9138.807264 = 20646.77786 .

5. Find, to the nearest unit, the present value of a 25-year annuity-due which pays
100 immediately, 104 at the end of the 1st year, 108.16 at the end of the 2nd year,
where each subsequent payment is obtained from its predecessor by multiplying by
a factor of 1.04. The annual effective rate of interest is 8.%.
Solution:
24
X
Present Value = 100v n (1.04)n
n=0
25
1 1.04
1.08
= 100 1.04
1 1.08
= 1648.998444

or 1649 to the nearest unit.

6. (a) A loan of 15,000 is being repaid with payments of 1,500 at the end of each
year for 20 years. If each payment is immediately reinvested at 6% effective,
find the effective annual rate of interest earned over the 20-year period.
(b) A loan of 15,000 is being repaid with payments of 1,500 at the end of each
year for 10 years. Determine the yield rate to the investor.
Solution:

(a) Let the effective yield rate be i. The payments do not become available until
the maturity date, after 20 years. Until that time they are locked into a
payment-scheme that accumulates to value 1500s20 6% . We are asked for the
interest rate that was earned. There are thus just two transactions: the loan

UPDATED TO April 12, 2005


Information for Students in MATH 329 2005 01 3073

at time 0, in the amount of 15,000, and the repayment at time 20, in the
amount given above. The equation of value at time t = 0 is

1500s20 6% (1 + i)20 = 15000


10(0.06)
(1 + i)20 =
(1.06)20 1
i = 6.7293555%.

(b) We have to determine i such that 1500 a10 i = 15000. This may appear to be
a difficult problem. But remember that

a10 = v + v 2 + v 3 + . . . + v 10

and that v 1. That means that the sum cannot be more than 10; in order
for it to equal 10, each of the summands must equal 1, so v = 1. 1 + i = 1,
and i = 0.

B.2.5 Fifth 2003/2004 Problem Assignment, with Solutions


Distribution Date: Mounted on the Web on Monday, April 7th, 2004.
Assignment was mounted on the Web on Wednesday, March 17th, 2004,
hard copy of the assignment was distributed on Friday, March 19th, 2004.
Solutions were to be submitted by Friday, April 2nd, 2004
(SUBJECT TO CORRECTION OF TYPOS AND OTHER ERRORS)

1. A loan is being repaid with instalments of 1000 at the end of each year for 15 years,
followed by payments of 2000 at the end of each year for 10 years. Interest is at
an effective rate of 4% for the first 10 years, and an effective rate 6% for the next
15 years.

(a) Showing all your work, find the numeric value of the amount of interest paid
in the 4th instalment without making use of a schedule.
(b) Showing all your work, find the amount of principal repaid in the 20th instal-
ment, without making use of a schedule.
(c) Then use the information you have computed to compile the lines of a schedule
corresponding to the payments at the ends of years 20, 21, . . . , 25.
(d) Now solve (a), (b), (c) again, this time assuming that all payments are at the
beginnings of the years: the interest rates remain precisely the same.

Solution:
Information for Students in MATH 329 2005 01 3074

(a) By the prospective method, the unpaid balance of the loan at time t = 0 is

1000 a10 4% + (1.04)10 2000 a15 6% 1000 a5 6%
1 (1.04)10
= 1000
0.04
1 (1.06)15 1 (1.06)5
+2000 (1.04)10 1000 (1.04)10
0.06 0.06
= 18387.66857 .

By the retrospective method, the unpaid balance just after the 3rd payment
of 1000 is, therefore,

(1.04)3 1
18387.66857(1.04)3 1000 s3 4% = 18387.66857(1.04)3 1000
0.04
= 17562.02642 .

The interest component of the 4th instalment is, therefore,

0.04(17562.02642) = 702.48 .

(b) By the prospective method, the unpaid balance just after the 19th instalment
is
1 (1.06)6
2000 a6 6% = 2000 = 9834.648653 .
0.06
The interest component of the 20th instalment is, therefore,

0.06(9834.648653) = 590.0789192 ,

so the component for reduction of principal is

2000 590.0789192 = 1409.921081 .

Payment Payment Interest Principal Outstanding


Number amount paid repaid loan balance
19 2000 ... ... 9834.65
20 2000 590.08 1409.92 8424.73
(c) 21 2000 505.48 1494.52 6930.21
22 2000 415.81 1584.19 5346.02
23 2000 320.76 1679.23 3666.78
24 2000 220.01 1779.99 1886.79
25 2000 113.21 1886.79 0
Information for Students in MATH 329 2005 01 3075

(d) Since the only interest rate affecting the last payments has not changed, there
will be no changes at all in (b) or (c).
By the prospective method, the unpaid balance of the loan at time t = 0, just
before the first payment, is

1000 a10 4% + (1.04)10 2000 a15 6% 1000 a5 6%
1 (1.04)10 1 (1.06)15
= 1000(1.04) + 2000(1.06) (1.04)10
0.04 0.06
5
1 (1.06)
1000(1.06) (1.04)10
0.06
= 19328.71077 .

By the retrospective method, the unpaid balance just after the 3rd payment
of 1000 is, therefore,

19328.71077(1.04)2 1000 s3 4%
(1.04)3 1
= 19328.71077(1.04)2 1000
0.04
= 17784.33357 .

The interest component of the 4th instalment is, therefore,

0.04(17784.33357) = 711.37 .

2. (This problem is modelled on [1, Exercise 23, p. 198].)


On a loan of 30,000, the borrower has agreed to pay interest at 7% effective at the
end of each year until the loan is repaid. The borrower has decided to deposit a
fixed amount at the beginning of each year into a sinking fund earning 4% effective.
At the end of 11 years the sinking fund is exactly sufficient to pay off exactly two-
thirds of the loan. He plans to continue accumulating the sinking fund until a year
when a deposit of not more than this fixed amount will bring the fund balance up
to 30,000 and the loan can be immediately repaid.

(a) Calculate the total amount the borrower has to pay out each year (at the
beginning, and at the end), except possibly in the year when the loan is
repaid.
(b) Complete the following table to show how the sinking fund attains the target
value of 30,000, and the net amount of the loan after payments ##10, 11, . . .
until the loan is paid off.
Information for Students in MATH 329 2005 01 3076

Payment Interest Sinking Interest earned Amount in Net amount


Number paid fund deposit on sinking fund sinking fund of loan
10
11
12
...

Solution:

(a) The borrower is paying a constant amount each year of 7% of 30,000, or 2,100
to cover the interest costs of servicing the loan; denote by X the constant
amount the borrower spends each year; thus the amount she contributes to
the sinking fund is X 2100. An equation of value at time t = 11 is (X
2100) s11 4% = 20000, so

20000 0.04
X = 2100 + = 3525.943064 .
((1.04)11 1) 1.04

Thus the level contribution to the sinking fund at the beginning of each year
except the last is 3525.94 2100.00 = 1425.94.
(b) The intention is that the column labelled Amount in sinking fund shows
the amount just after the payment with the given number.
Payment Interest Sinking Interest earned Amount in Net amount
Number paid fund deposit on sinking fund sinking fund of loan
10 2,100.00 1,425.94 603.62 17,120.03 12,879.97
11 2,100.00 1,425.94 684.80 19,230.77 10,769.23
12 2,100.00 1,425.94 769.23 21,425.64 8,574.36
13 2,100.00 1,425.94 857.03 23,708.61 6,291.39
14 2,100.00 1,425.94 948.34 26,082.89 3,917.11
15 2,100.00 1,425.94 1,043.32 28,552.15 1,447.85
16 2,100.00 305.76 1,142.09 30,000.00 0.00

3. A borrows 12,000 for 10 years, and agrees to make semiannual payments of 1,000,
plus a final payment. The lender receives 12% convertible semiannually on the
investment each year for the first 5 years and 10% convertible semiannually for the
second 5 years. The balance of each payment is invested in a sinking fund earning
8% convertible semiannually.

(a) Find the amount by which the sinking fund is short of repaying the loan at
the end of the 10 years.
Information for Students in MATH 329 2005 01 3077

(b) Complete the following table to show how the sinking fund attains its maxi-
mum value, and the net amount of the loan after payments.
Payment Interest Sinking Interest earned Amount in Net amount
Number paid fund deposit on sinking fund sinking fund of loan
09
10
11
12
Solution:
(a) The interest payments for the first 10 half-years are 6% of 12,000, i.e. 720
per half-year; and, for the second 10 half-years, 600 per half-year. This leaves
280 at the end of each of the first 10 half-years, and 400 at the end of each
of the second 10 half-years to accumulate in the sinking fund, which earns
4% effective every half year. The accumulated balance in the sinking fund at
maturity will be
1
120s10 4% + 280s20 4% = 120 (1.04)10 1 + 280 (1.04)20 1
0.04

= 25 120(1.04)10 + 280(1.04)20 400
= 9778.594855
implying that the shortfall to repay the loan will be 12, 000 9778.59 =
2221.41.
(b) Just after the 8th payment the balance in the fund is
280 ((1.04)8 1)
280 s8 4% = = 2579.98 .
0.04
Interest earned by the 9th payments is 103.20. We can now begin to fill in
the schedule:
Payment Interest Sinking Interest earned Amount in Net amount
Number paid fund deposit on sinking fund sinking fund of loan
09 720.00 280.00 103.20 2963.18 9036.82
10 720.00 280.00 118.53 3361.71 8638.29
11 600.00 400.00 134.47 3896.18 8103.82
12 600.00 400.00 155.85 4452.03 7547.97
4. (a) A borrower takes out a loan of 3000 for 10 years at 8% convertible semiannu-
ally. The borrower replaces one-third of the principal in a sinking fund earning
5% convertible semiannually, and the other two-thirds in a sinking fund earn-
ing 7% convertible semiannually. Find the total semiannual payment.
Information for Students in MATH 329 2005 01 3078

(b) Rework (a) if the borrower each year puts one-third of the total sinking fund
deposit into the 5% sinking fund and the other two-thirds into the 7% sinking
fund.
Solution:

(a) The semiannual contribution to the sinking funds is


1000 2000
+
s20 2.5% s20 3.5%

and the semiannual interest payment is 4% of 3, 000, or 120. Hence the total
semiannual payment is
1000 2000 25 70
+ + 120 = + + 120
s20 2.5% s20 3.5% (1.025) 1 (1.035)20 1
20

= 229.8692824

(b) Let the total sinking fund deposit be D. Then the equation of value at ma-
turity is
D 2D
s20 2.5% + s20 3.5% = 3000 ,
3 3
implying that
9000
D =
s20 2.5% + 2s20 3.5%
9000
= (1.025)20 1 (1.035)20 1
+2
0.025 0.035
= 109.6170427 ,

so the total semi-annual payment is 109.6170427+120=229.6170427.

5. A payment of 800 is made at the end of each month for 10 years to repay a loan
of 30,000. The borrower replaces the capital by means of a sinking fund earning a
nominal annual rate of 6% compounded monthly.

(a) Find the effective annual rate i paid to the lender on the loan.
(b) Suppose that the lender had elected to amortize the loan by equal monthly
payments, at the same rate as he is now paying to the lender. What would
be the amount of those equal payments?

Solution:
Information for Students in MATH 329 2005 01 3079

(a) The monthly contribution to the sinking fund is


30000 600
= = 183.0615058 .
s120 0.5% (1.005)120 1
Hence the monthly interest payment is 800 183.0615058 = 616.9384942,
i.e., 2.056461647% of the principal of 30,000. The effective annual rate is,
therefore,
(1.02056461647)12 1 = 27.6691838%.
(b) The level monthly payment necessary to amortize a loan of 30000 over 120
payments at an effective rate of 2.056461647% is
30000 (30000)(0.02056461647)
= = 675.6703969 .
a120 2.056461647% 1 (1.02056461647)120
(The amount is lower than the borrower is now paying because he is holding
funds in his sinking fund and earning less there than he his paying the lender
for the use of the capital.)

B.2.6 2003/2004 Class Test, Version 1


Instructions

The time available for writing this test is about 45 minutes.


This test booklet consists of this cover, Pages 3080 through 3080 containing ques-
tions together worth 60 marks; and Page 3080, which is blank.
Show all your work. All solutions are to be written in the space provided on the
page where the question is printed. When that space is exhausted, you may write
on the facing page, on the blank page, or on the back cover of the booklet, but you
must indicate any continuation clearly on the page where the question is printed!
(Please inform the instructor if you find that your booklet is defective.)
All your writing even rough work must be handed in.
Calculators. While you are permitted to use a calculator to perform arithmetic
and/or exponential calculations, you must not use the calculator to calculate such
actuarial functions as ani , sni , (Ia)ni , (Is)ni , (Da)ni , (Ds)ni , etc. without first
stating a formula for the value of the function in terms of exponentials and/or
polynomials involving n and the interest rate. You must not use your calculator in
any programmed calculations. If your calculator has memories, you are expected
to have cleared them before the test.
Information for Students in MATH 329 2005 01 3080

In your solutions to problems on this test you are expected to show all your work .
You are expected to simplify algebraic and numerical answers as much as you can.

Your neighbours may be writing a version of this test which is different from yours.

1. (a) [5 MARKS] Suppose that the nominal annual rate of interest, compounded
8 times per year, is 5%. Showing all your work, determine the equivalent
effective annual rate of discount.
(b) [5 MARKS] Suppose that the nominal annual rate of discount, compounded 3
times per year, is 7%. Showing all your work, determine the equivalent annual
rate of interest.
(c) [5 MARKS] State the nominal annual interest rate, compounded instanta-
neously, which is equivalent to an effective annual interest rate of 4%.
1
(d) [5 MARKS] Suppose that the effective interest rate for year is 3%. Deter-
4
mine the equivalent nominal interest rate, compounded every 2 years.

2. [15 MARKS] The accumulated value just after the last payment under a 12-year
annuity of 1000 per year, paying interest at the rate of 5% per annum effective,
is to be used to purchase a perpetuity at an interest rate of 6%, first payment to
be made 1 year after the last payment under the annuity. Showing all your work,
determine the size of the payments under the perpetuity.

3. [25 MARKS] A loan of 5000 is to be repaid by annual payments of 250 to commence


at the end of the 6th year, and to continue thereafter for as long as necessary. Find
the time and amount of the final payment if the final payment is to be larger than
the regular payments. Assume i = 4%.

continuation page for problem number

You must refer to this continuation page on the page where the problem is printed!

B.2.7 2003/2004 Class Test, Version 2


Instructions

The time available for writing this test is about 45 minutes.


Information for Students in MATH 329 2005 01 3081

This test booklet consists of this cover, Pages 3081 through 3082 containing ques-
tions together worth 60 marks; and Page 3082, which is blank.

Show all your work. All solutions are to be written in the space provided on the
page where the question is printed. When that space is exhausted, you may write
on the facing page, on the blank page, or on the back cover of the booklet, but you
must indicate any continuation clearly on the page where the question is printed!
(Please inform the instructor if you find that your booklet is defective.)

All your writing even rough work must be handed in.

Calculators. While you are permitted to use a calculator to perform arithmetic


and/or exponential calculations, you must not use the calculator to calculate such
actuarial functions as ani , sni , (Ia)ni , (Is)ni , (Da)ni , (Ds)ni , etc. without first
stating a formula for the value of the function in terms of exponentials and/or
polynomials involving n and the interest rate. You must not use your calculator in
any programmed calculations. If your calculator has memories, you are expected
to have cleared them before the test.

In your solutions to problems on this test you are expected to show all your work .
You are expected to simplify algebraic and numerical answers as much as you can.

Your neighbours may be writing a version of this test which is different from yours.

1. [25 MARKS] A loan of 1000 is to be repaid by annual payments of 100 to commence


at the end of the 5th year, and to continue thereafter for as long as necessary. Find
the time and amount of the final payment if the final payment is to be NO larger
than the regular payments. Assume i = 4.5%.
1
2. (a) [5 MARKS] Suppose that the effective interest rate for year is 0.02. Deter-
5
mine the equivalent nominal interest rate, compounded every 3 years.
(b) [5 MARKS] Suppose that the nominal annual rate of interest, compounded
9 times per year, is 6%. Showing all your work, determine the equivalent
effective annual rate of discount.
(c) [5 MARKS] Suppose that the nominal annual rate of discount, compounded 6
times per year, is 5%. Showing all your work, determine the equivalent annual
rate of interest.
(d) [5 MARKS] State the nominal annual interest rate, compounded instanta-
neously, which is equivalent to an effective annual interest rate of 8%.
Information for Students in MATH 329 2005 01 3082

3. [15 MARKS] The accumulated value just after the last payment under a 12-year
annuity of 1000 per year, paying interest at the rate of 5% per annum effective,
is to be used to purchase a perpetuity of 500 per annum forever, first payment to
be made 1 year after the last payment under the annuity. Showing all your work,
determine the effective interest rate of the perpetuity, assuming it comes into effect
just after the last payment under the annuity.

B.2.8 2003/2004 Class Test, Version 3


Instructions
The time available for writing this test is about 45 minutes.
This test booklet consists of this cover, Pages 3082 through 3083 containing ques-
tions together worth 60 marks; and Page 3083, which is blank.
Show all your work. All solutions are to be written in the space provided on the
page where the question is printed. When that space is exhausted, you may write
on the facing page, on the blank page, or on the back cover of the booklet, but you
must indicate any continuation clearly on the page where the question is printed!
(Please inform the instructor if you find that your booklet is defective.)
All your writing even rough work must be handed in.
Calculators. While you are permitted to use a calculator to perform arithmetic
and/or exponential calculations, you must not use the calculator to calculate such
actuarial functions as ani , sni , (Ia)ni , (Is)ni , (Da)ni , (Ds)ni , etc. without first
stating a formula for the value of the function in terms of exponentials and/or
polynomials involving n and the interest rate. You must not use your calculator in
any programmed calculations. If your calculator has memories, you are expected
to have cleared them before the test.
In your solutions to problems on this test you are expected to show all your work .
You are expected to simplify algebraic and numerical answers as much as you can.
Your neighbours may be writing a version of this test which is different from yours.

1. [15 MARKS] The accumulated value just after the last payment under a 9-year
annuity of 2000 per year, paying interest at the rate of 8% per annum effective,
is to be used to purchase a perpetuity at an interest rate of 4%, first payment to
be made 1 year after the last payment under the annuity. Showing all your work,
determine the size of the payments under the perpetuity.
Information for Students in MATH 329 2005 01 3083

2. [25 MARKS] A loan of 1000 is to be repaid by annual payments of 200 to commence


at the end of the 4th year, and to continue thereafter for as long as necessary. Find
the time and amount of the final payment if the final payment is to be larger than
the regular payments. Assume i = 5%.

3. (a) [5 MARKS] State the nominal annual interest rate, compounded instanta-
neously, which is equivalent to an effective annual interest rate of 6%.
1
(b) [5 MARKS] Suppose that the effective interest rate for year is 0.015. De-
6
termine the equivalent nominal interest rate, compounded every 4 years.
(c) [5 MARKS] Suppose that the nominal annual rate of interest, compounded
3 times per year, is 8%. Showing all your work, determine the equivalent
effective annual rate of discount.
(d) [5 MARKS] Suppose that the nominal annual rate of discount, compounded
12 times per year, is 6%. Showing all your work, determine the equivalent
annual rate of interest.

continuation page for problem number

You must refer to this continuation page on the page where the problem is printed!

B.2.9 2003/2004 Class Test, Version 4


Instructions

The time available for writing this test is about 45 minutes.

This test booklet consists of this cover, Pages 3084 through 3084 containing ques-
tions together worth 60 marks; and Page 3084, which is blank.

Show all your work. All solutions are to be written in the space provided on the
page where the question is printed. When that space is exhausted, you may write
on the facing page, on the blank page, or on the back cover of the booklet, but you
must indicate any continuation clearly on the page where the question is printed!
(Please inform the instructor if you find that your booklet is defective.)

All your writing even rough work must be handed in.


Information for Students in MATH 329 2005 01 3084

Calculators. While you are permitted to use a calculator to perform arithmetic


and/or exponential calculations, you must not use the calculator to calculate such
actuarial functions as ani , sni , (Ia)ni , (Is)ni , (Da)ni , (Ds)ni , etc. without first
stating a formula for the value of the function in terms of exponentials and/or
polynomials involving n and the interest rate. You must not use your calculator in
any programmed calculations. If your calculator has memories, you are expected
to have cleared them before the test.
In your solutions to problems on this test you are expected to show all your work .
You are expected to simplify algebraic and numerical answers as much as you can.
Your neighbours may be writing a version of this test which is different from yours.

1. (a) [5 MARKS] Suppose that the nominal annual rate of discount, compounded
6 times per year, is 0.5%. Showing all your work, determine the equivalent
annual rate of interest.
(b) [5 MARKS] State the nominal annual interest rate, compounded instanta-
neously, which is equivalent to an effective annual interest rate of 10%.
1
(c) [5 MARKS] Suppose that the effective interest rate for year is 2%. Deter-
5
mine the equivalent nominal interest rate, compounded every 6 years.
(d) [5 MARKS] Suppose that the nominal annual rate of interest, compounded
7 times per year, is 2%. Showing all your work, determine the equivalent
effective annual rate of discount.
2. [25 MARKS] A loan of 1000 is to be repaid by annual payments of 200 to commence
at the end of the 4th year, and to continue thereafter for as long as necessary. Find
the time and amount of the final payment if the final payment is to be NO larger
than the regular payments. Assume i = 5%.
3. [15 MARKS] The accumulated value just after the last payment under a 10-year
annuity of 1000 per year, paying interest at the rate of 6% per annum effective,
is to be used to purchase a perpetuity of 800 per annum forever, first payment to
be made 1 year after the last payment under the annuity. Showing all your work,
determine the effective interest rate of the perpetuity, assuming it comes into effect
just after the last payment under the annuity.

continuation page for problem number

You must refer to this continuation page on the page where the problem is printed!
Information for Students in MATH 329 2005 01 3085

B.2.10 Solutions to Problems on the 2003/2004 Class Tests


The first four problems listed are concerned with equivalent rates of interest and discount
(each in 4 parts); the next four concern annuities and perpetuities; and the last four are
concerned with unknown type and final balloon or drop payments.

1. (a) [5 MARKS] [VERSION 1 #1(a)] Suppose that the nominal annual rate of in-
terest, compounded 8 times per year, is 5%. Showing all your work, determine
the equivalent effective annual rate of discount.
Solution: We are given that i(8) = 0.05, and asked to determine d.
8
i(8)
1+ = 1+i
8
and (1 d)(1 + i) = 1
8
i(8)
d = 1 1+ = 0.0486225508 = 4.86%
8

(b) [5 MARKS] [VERSION 1 #1(b)] Suppose that the nominal annual rate of dis-
count, compounded 3 times per year, is 7%. Showing all your work, determine
the equivalent annual rate of interest.
Solution: We are given that d(3) = 0.07, and asked to determine i.
3
d(3)
1 = 1d
3
and (1 d)(1 + i) = 1
3
d(3)
i = 1 1 = 0.073398300 = 7.34%
3

(c) [5 MARKS] [VERSION 1 #1(c)] State the nominal annual interest rate, com-
pounded instantaneously, which is equivalent to an effective annual interest
rate of 4%.
Solution: The rate we seek is the solution i to the equation
n
i
lim 1 + = 1.04
n n

which is equivalent to ei = 1.04. Solving by taking logarithms, we obtain


i = ln(1.04) = 0.039220713151 = 3.92%.
Information for Students in MATH 329 2005 01 3086

(d) [5 MARKS] [VERSION 1 #1(d)] Suppose that the effective interest rate for
1
year is 3%. Determine the equivalent nominal interest rate, compounded
4
every 2 years.
i(4) 1
Solution: We are given the value of = 3%, and asked to determine i( 2 ) .
4
The equation we have to solve is
! 21
i (4) 4
i( 1
2 )
1+ =1+i= 1+ 1 .
4 2

This equation implies that


!
(4) 8
1 i
i( 2 ) =
1
1+ 1 = 0.1333850405 = 13.3%.
2 4

2. (a) [5 MARKS] [VERSION 2 #2(b)] Suppose that the nominal annual rate of in-
terest, compounded 9 times per year, is 6%. Showing all your work, determine
the equivalent effective annual rate of discount.
Solution: We are given that i(9) = 0.06, and asked to determine d.
9
i(9)
1+ = 1+i
9
and (1 d)(1 + i) = 1
9
i(9)
d = 1 1+ = 0.0580479306 = 5.80%
9

(b) [5 MARKS] [VERSION 2 #2(c)] Suppose that the nominal annual rate of dis-
count, compounded 6 times per year, is 5%. Showing all your work, determine
the equivalent annual rate of interest.
Solution: We are given that d(6) = 0.05, and asked to determine i.
6
d(6)
1 = 1d
6
and (1 d)(1 + i) = 1
6
d(6)
i = 1 1 = 0.051491358 = 5.15%
6
Information for Students in MATH 329 2005 01 3087

(c) [5 MARKS] [VERSION 2 #2(d)] State the nominal annual interest rate, com-
pounded instantaneously, which is equivalent to an effective annual interest
rate of 8%.
Solution: The rate we seek is the solution to the equation
n
i
lim 1 + = 1.08
n n

which is equivalent to ei = 1.08. Solving by taking logarithms, we obtain


i = ln(1.08) = 0.07696104114 = 7.70%.
(d) [5 MARKS] [VERSION 2 #2(a)] Suppose that the effective interest rate for
1
year is 0.02. Determine the equivalent nominal interest rate, compounded
5
every 3 years.
i(5) 1
Solution: We are given the value of = 2%, and asked to determine i( 3 ) .
5
The equation we have to solve is
5 ! 31
i( 3 )
1
i(5)
1+ =1+i= 1+ 1 .
5 3

This equation implies that


!
(5) 15
1 i
i( 3 ) =
1
1+ 1 = 0.1152894460 = 11.5%
3 5

3. (a) [5 MARKS] [VERSION 3 #3(c)] Suppose that the nominal annual rate of in-
terest, compounded 3 times per year, is 8%. Showing all your work, determine
the equivalent effective annual rate of discount.
Solution: We are given that i(3) = 0.08, and asked to determine d.
3
i(3)
1+ = 1+i
3
and (1 d)(1 + i) = 1
3
i(3)
d = 1 1+ = 0.0759156521 = 7.59%
3

(b) [5 MARKS] [VERSION 3 #3(d)] Suppose that the nominal annual rate of
discount, compounded 12 times per year, is 6%. Showing all your work,
determine the equivalent annual rate of interest.
Information for Students in MATH 329 2005 01 3088

Solution: We are given that d(12) = 0.06, and asked to determine i.


12
d(12)
1 = 1d
12
and (1 d)(1 + i) = 1
12
d(12)
i = 1 1 = 0.061996367 = 6.20%
12

(c) [5 MARKS] [VERSION 3 #3(a)] State the nominal annual interest rate, com-
pounded instantaneously, which is equivalent to an effective annual interest
rate of 6%.
Solution: The rate we seek is the solution to the equation
n
i
lim 1 + = 1.06
n n
which is equivalent to ei = 1.06. Solving by taking logarithms, we obtain
i = ln(1.06) = 0.05826890812 = 5.83%.
(d) [5 MARKS] [VERSION 3 #3(b)] Suppose that the effective interest rate for
1
year is 0.015. Determine the equivalent nominal interest rate, compounded
6
every 4 years.
i(6) 1
Solution: We are given the value of = 1.5%, and asked to determine i( 4 ) .
6
The equation we have to solve is
6 ! 41
i( 4 )
1
i(6)
1+ =1+i= 1+ 1 .
6 4

This equation implies that


!
(6) 24
1 i
i( 4 ) =
1
1+ 1 = .1073757030 = 10.7%
4 6

4. (a) [5 MARKS] [VERSION 4 #1(d)] Suppose that the nominal annual rate of in-
terest, compounded 7 times per year, is 2%. Showing all your work, determine
the equivalent effective annual rate of discount.
Solution: We are given that i(7) = 0.02, and asked to determine d.
7
i(7)
1+ = 1+i
7
Information for Students in MATH 329 2005 01 3089

and (1 d)(1 + i) = 1
7
i(7)
d = 1 1+ = 0.197733746 = 1.98%
7

(b) [5 MARKS] [VERSION 4 #1(a)] Suppose that the nominal annual rate of
discount, compounded 6 times per year, is 0.5%. Showing all your work,
determine the equivalent annual rate of interest.
Solution: We are given that d(6) = 0.005, and asked to determine i.
6
d(6)
1 = 1d
6
and (1 d)(1 + i) = 1
6
d(6)
i = 1 1 = 0.005014616 = 0.5014616%
6

(c) [5 MARKS] [VERSION 4 #1(b)] State the nominal annual interest rate, com-
pounded instantaneously, which is equivalent to an effective annual interest
rate of 10%.
Solution: The rate we seek is the solution to the equation
n
i
lim 1 + = 1.10
n n
which is equivalent to ei = 1.10. Solving by taking logarithms, we obtain
i = ln(1.10) = 0.09531017980 = 9.53%.
(d) [5 MARKS] [VERSION 4 #1(c)] Suppose that the effective interest rate for
1
year is 2%. Determine the equivalent nominal interest rate, compounded
5
every 6 years.
i(5) 1
Solution: We are given the value of = 2%, and asked to determine i( 6 ) .
5
The equation we have to solve is
5 ! 61
i( 6 )
1
i(5)
1+ =1+i= 1+ 1 .
5 6

This equation implies that


!
(5) 30
1 i
i( 6 ) =
1
1+ 1 = 0.1352269307 = 13.5%.
6 5
Information for Students in MATH 329 2005 01 3090

5. [15 MARKS] [VERSION 1 #2] The accumulated value just after the last payment
under a 12-year annuity of 1000 per year, paying interest at the rate of 5% per
annum effective, is to be used to purchase a perpetuity at an interest rate of 6%,
first payment to be made 1 year after the last payment under the annuity. Showing
all your work, determine the size of the payments under the perpetuity.
Solution: Let X be the level payment under the perpetuity. The equation of value
just after the last annuity payment is

1000 s12 5% = X a 6%

implying that
s12 5%
X = 1000
a 6%
6
= 1000 (1.05)12 1 = 955.03.
5

6. [15 MARKS] [VERSION 2 #3] The accumulated value just after the last payment
under a 12-year annuity of 1000 per year, paying interest at the rate of 5% per
annum effective, is to be used to purchase a perpetuity of 500 per annum forever,
first payment to be made 1 year after the last payment under the annuity. Showing
all your work, determine the effective interest rate of the perpetuity, assuming it
comes into effect just after the last payment under the annuity.
Solution: Let i be the interest rate of the perpetuity. The equation of value just
after the last annuity payment is
500
1000 s12 5% = 500 a i =
i
implying that
500 1
i =
1000 s12 5%
500 0.05
=
1000 ((1.05)12 1)
= 3.14%.

7. [15 MARKS] [VERSION 3 #1] The accumulated value just after the last payment
under a 9-year annuity of 2000 per year, paying interest at the rate of 8% per
annum effective, is to be used to purchase a perpetuity at an interest rate of 4%,
first payment to be made 1 year after the last payment under the annuity. Showing
all your work, determine the size of the payments under the perpetuity.
Information for Students in MATH 329 2005 01 3091

Solution: Let X be the level payment under the perpetuity. The equation of value
just after the last annuity payment is

2000 s9 8% = X a 4%

implying that
s9 8%
X = 2000
a 4%
4
= 2000 (1.08)9 1 = 999.00
8

8. [15 MARKS] [VERSION 4 #3] The accumulated value just after the last payment
under a 10-year annuity of 1000 per year, paying interest at the rate of 6% per
annum effective, is to be used to purchase a perpetuity of 800 per annum forever,
first payment to be made 1 year after the last payment under the annuity. Showing
all your work, determine the effective interest rate of the perpetuity, assuming it
comes into effect just after the last payment under the annuity.
Solution: Let i be the interest rate of the perpetuity. The equation of value just
after the last annuity payment is
800
1000 s10 6% = 800 a i =
i
implying that
800 1
i =
1000 s10 6%
800 0.06
=
1000 ((1.06)10 1)
= 6.07%.

9. [25 MARKS] [VERSION 1 #3] A loan of 5000 is to be repaid by annual payments


of 250 to commence at the end of the 6th year, and to continue thereafter for
as long as necessary. Find the time and amount of the final payment if the final
payment is to be larger than the regular payments. Assume i = 4%.
Solution: (cf. [1, Exercise 32, p. 91]) Let the time of the last balloon payment
be n, and let the amount of the last payment be X. Then n is the largest integer
solution to the inequality

1 (1.04)(n5)
5000 250(1.04)5 an5 = 250(1.04)5
0.04
Information for Students in MATH 329 2005 01 3092

5000 0.04 (1.04)5


(1.04)(n5) 1
250
(n 5) ln 1.04 ln 1 20 0.04 (1.04)5
ln (1 0.8(1.04)5 )
(n 5)
ln 1.04
ln (1 0.8(1.04)5 )
n5 = 97.39832188.
ln 1.04
Thus we conclude that the balloon payment is made at time t = 97. The equation
of value at time t = 22 is

5000(1.04)97 = 250s92 + (X 250)

implying that
250
X = 250 + 5000(1.04)97 (1.04)92 1 = 346.8818 .
0.04

10. [25 MARKS] [VERSION 2 #1] A loan of 1000 is to be repaid by annual payments
of 100 to commence at the end of the 5th year, and to continue thereafter for
as long as necessary. Find the time and amount of the final payment if the final
payment is to be NO larger than the regular payments. Assume i = 4.5%.
Solution: Let the time of the last drop payment be n, and let the amount of
the last payment be X. Then n is the smallest integer solution to the inequality
1 (1.045)(n4)
1000 100(1.045)4 an4 = 100(1.045)4
0.045
4
1000 0.045 (1.045)
(1.045)(n4) 1
100
(n 4) ln 1.045 ln 1 10 0.045 (1.045)4
ln (1 0.45(1.045)4 )
(n 4)
ln 1.045
ln (1 0.45(1.045)4 )
n4 = 21.47594530.
ln 1.045
Thus we conclude that the drop payment is made at time t = 22. The equation of
value at time t = 22 is
1000(1.045)22 = 100s17 + X
implying that
100 1.045
X = 1000(1.045)22 (1.045)17 1 = 48.143638 .
0.045
Information for Students in MATH 329 2005 01 3093

11. [25 MARKS] [VERSION 3 #2] A loan of 1000 is to be repaid by annual payments
of 200 to commence at the end of the 4th year, and to continue thereafter for
as long as necessary. Find the time and amount of the final payment if the final
payment is to be larger than the regular payments. Assume i = 5%.
Solution: (cf. [1, Exercise 32, p. 91]) Let the time of the last balloon payment
be n, and let the amount of the last payment be X. Then n is the largest integer
solution to the inequality

1 (1.05)(n3)
1000 200(1.05)3 an3 = 200(1.05)3
0.05
1000 0.05 (1.05)3
(1.05)(n3) 1
200
1000 0.05 (1.05)3
(n 3) ln 1.05 ln 1
200
3
ln (1 0.25(1.05) )
(n 3)
ln 1.05
ln (1 0.25(1.05)3 )
n3 = 10.00252595
ln 1.05
Thus we conclude that the balloon payment is made at time t = 10. The equation
of value at time t = 10 is

1000(1.05)10 = 200s7 + (X 200)

implying that
200
X = 200 + 1000(1.05)10 (1.05)7 1 = 200.492935 .
0.05

12. [25 MARKS] [VERSION 4 #2] A loan of 1000 is to be repaid by annual payments
of 200 to commence at the end of the 4th year, and to continue thereafter for
as long as necessary. Find the time and amount of the final payment if the final
payment is to be NO larger than the regular payments. Assume i = 5%.
Solution: Let the time of the last drop payment be n, and let the amount of
the last payment be X. Then n is the smallest integer solution to the inequality

1 (1.05)(n3)
1000 200(1.05)3 an3 = 200(1.05)3
0.05
1000 0.05 (1.05)3
(1.05)(n3) 1
200
(n 3) ln 1.05 ln 1 5 0.05 (1.05)3
Information for Students in MATH 329 2005 01 3094

ln (1 0.25(1.05)3 )
(n 3)
ln 1.05
ln (1 0.25(1.05)3 )
n3 = 10.00252595.
ln 1.05
Thus we conclude that the drop payment is made at time t = 11. The equation of
value at time t = 11 is
1000(1.05)11 = 200s7 + X
implying that
200 1.05
X = 1000(1.05)11 (1.05)7 1 = 0.517581.
0.05

B.2.11 Final Examination, 2003/2004


1. In each of the following problems you are expected to show all your work.

(a) [3 MARKS] If v = 0.97, determine the value of d(4) .


1
(b) [3 MARKS] If i(12) = 6%, determine the value of i( 2 ) .
(c) [3 MARKS] Showing all your work, determine the nominal interest rate, com-
pounded semi-annually, under which a sum of money will triple in 12 years.
(d) [3 MARKS] Showing all your work, determine the rate of interest, convertible
continuously, that is equivalent to a nominal interest rate of 8% per annum,
convertible monthly.

2. In each of the following problems, give a formula in terms of i alone; then evaluate
the formula and determine the numerical value.

(a) [5 MARKS] Determine the present value of a perpetuity-due of 100 payable


every three months, at an effective annual interest rate of 6%.
(b) [5 MARKS] The present value of a perpetuity-immediate paying 1000 at the
125, 000
end of every 3 years is .Determine the effective annual interest rate.
91
3. In each of the following problems, give a formula in terms of i alone; then evaluate
the formula and determine its numerical value.

(a) [7 MARKS] At a nominal annual interest rate of 8% compounded quarterly,


determine the value, 2 years after the last payment, of a decreasing annuity
paying 5,000 at the end of the first half-year, 4,500 at the end of the 2nd half-
year, and continuing to decrease at 500 per half-year until the final payment
of 500.
Information for Students in MATH 329 2005 01 3095

Table 3: Several Useful Formulas that you were not expected to memorize

an i nv n
(Ia)n i = i
nan i
sn i n (Da)n i = i
(Is)n i = i
n(1+i)n sn i
s
n+1 i
(n+1) (Ds)n i = i
(Is)n i = i

(b) [8 MARKS] Three years before the first payment, determine the value of an
annuity that pays 4,000 the first year, 3,900 the second year, with payments
continuing to decrease by 100 until it pays 2,000 per year, after which it pays
2,000 forever. The interest rate is 5% effective per year until the payment of
3,000, after which the interest rate becomes 4% effective forever.

4. One of the following equations is always true, and one is true only when i = 0.
1 1
I. = +i
an i sn i
1 1 1
II. = +
sn i an i i
1 1 1
III. = +
an i sn i i
1 1
IV. = +i
sn i an i

(a) Explain which is always true, and prove it


i. [4 MARKS] algebraically; and
ii. [4 MARKS] by a verbal argument, referring to a sinking fund.
(b) [4 MARKS] Prove algebraically that one of the other equations is true for
i = 0.

5. The purchase of a new condominium is partially financed by a mortgage of 120,000


payable to the vendor; the mortgage is amortized over 25 years, with a level pay-
Information for Students in MATH 329 2005 01 3096

ment at the end of each half-month, at a nominal annual rate of 6.6% compounded
every half-month.

(a) [3 MARKS] Determine the half-monthly payments under this mortgage.


(b) [2 MARKS] Divide the 1st payment into principal and interest.
(c) [3 MARKS] Determine the outstanding principal immediately after the 50th
payment.
(d) [4 MARKS] Divide the 52nd payment into principal and interest.
(e) [3 MARKS] The amortization by half-monthly payments was designed to ac-
commodate the purchase, whose salary was being deposited automatically to
his bank account every half-month. The purchase changes his profession, after
2 years, and now would prefer to make a single payment once every half-year.
Determine the amount of that payment if the interest rates are unchanged, but
if the mortgage is now amortized to be paid off 5 years earlier than previously.

6. (a) [8 MARKS] Find the price of the following bond, which is purchased to yield
6% convertible semi-annually: the bond has face value of 10,000, matures in
15 years at a maturity value of 11,500, and has a nominal coupon rate of 9%
per annum, compounded semi-annually; the investor is replacing the principal
by means of a sinking fund earning 7% convertible semi-annually.
(b) [7 MARKS] Suppose that the bond is callable when t = 13 at a premium of
1, 000 above the maturity value. Explain what price the investor should pay
if he is no longer plans to deposit any of the interest in a sinking fund.

7. A loan is being repaid with 15 annual payments of 1,000 each. At the time of the
5th payment the borrower is permitted to pay an extra 2000, and then to repay
the balance over 5 years with a revised annual payment.

(a) [5 MARKS] If the effective annual rate of interest is 6%, find the amount of
the revised annual payment.
(b) [8 MARKS] Complete an amortization table for the full 10 years of the loan,
with the following columns:
Payment Payment Interest Principal Outstanding
number amount paid repaid loan balance
0
1
... ... ... ... ...
10
Information for Students in MATH 329 2005 01 3097

8. [8 MARKS] It was n years ago when James deposited 10,000 in a bank paying
2.4% interest compounded monthly. If he had, instead, placed his deposit in a
syndicate paying interest by cheque annually at the rate of 5% per annum, and
he had invested only this interest with the bank, how much more interest would
he have earned altogether? Show all your reasoning, and express your answer in
terms of n.

B.2.12 Supplemental/Deferred Examination, 2003/2004


1. In each of the following problems you are expected to show all your work.

(a) [3 MARKS] If v = 0.95, determine the value of d(3) .


(b) [3 MARKS] Showing all your work, determine the nominal interest rate, com-
pounded quarterly, under which a sum of money will double in 10 years.
(c) [3 MARKS] Showing all your work, determine the rate of discount, convertible
continuously, that is equivalent to a nominal discount rate of 8% per annum,
convertible semi-annually.
(d) [3 MARKS] If i( 2 ) = 1 , determine the value of i(12) .
1

30

2. You must show all your work in solving the following problems:

(a) [5 MARKS] Determine the present value of a perpetuity-immediate of 1000


payable every three months, at an effective annual interest rate of 8%.
(b) [5 MARKS] The present value of a perpetuity-immediate paying 100 at the
129, 600
end of every 4 years is . Determine the effective annual interest rate.
1, 105

3. Show detailed work in your solutions to each of these problems.

(a) [7 MARKS] At a nominal annual interest rate of 6% compounded quarterly,


determine the value 5 years after the last payment of a decreasing
annuity paying 6,000 at the end of the first half-year, 5,500 at the end of the
2nd half-year, and continuing to decrease at 500 per half-year until the final
payment of 500.
(b) [8 MARKS] Three years before the first payment, determine the present value
of an annuity that pays 6,000 the first year, 5,900 the second year, with
payments continuing to decrease by 100 until it pays 4,000 per year, after
which it pays 4,000 forever. The interest rate is 8% effective per year until
the first payment of 4,000, after which the interest rate becomes 5% effective
forever.
Information for Students in MATH 329 2005 01 3098

Table 4: Several Useful Formulas that you were not expected to memorize

an i nv n
(Ia)n i = i
nan i
sn i n (Da)n i = i
(Is)n i = i
n(1+i)n sn i
s
n+1 i
(n+1) (Ds)n i = i
(Is)n i = i

4. The purchase of a new condominium is partially financed by a mortgage of 120,000


payable to the vendor; the mortgage is amortized over 35 years, with a level pay-
ment at the end of each half-month, at a nominal annual rate of 9.6% compounded
every half-month.

(a) [3 MARKS] Determine the half-monthly payments under this mortgage.


(b) [2 MARKS] Divide the 1st payment into principal and interest.
(c) [3 MARKS] Determine the outstanding principal immediately after the 60th
payment.
(d) [4 MARKS] Divide the 62nd payment into principal and interest.
(e) [3 MARKS] The amortization by half-monthly payments was designed to ac-
commodate the purchaser, whose salary was being deposited automatically to
his bank account every half-month. The purchaser changes his profession 4
years after the mortgage is executed, and now wishes to make a single payment
once every half-year. Determine the amount of that payment if the interest
rates are unchanged, but if the mortgage is now amortized to be paid off after
25 years.

5. One of the following equations is always true, and one is true only when i = 0.
1 1
I. = +i
sn i an i
1 1 1
II. = +
sn i an i i
Information for Students in MATH 329 2005 01 3099

1 1
III. = +i
an i sn i
1 1 1
IV. = +
an i sn i i

(a) Explain which is always true, and prove it


i. [4 MARKS] algebraically; and
ii. [4 MARKS] by a verbal argument, referring to a sinking fund. (A detailed
explanation is expected.)
(b) [4 MARKS] Prove algebraically that one of the other equations is true for
i = 0.

6. (a) [8 MARKS] Find the price of the following bond, which is purchased at a
premium to yield 5% convertible semi-annually: the bond has face value of
10,000, matures in 12 years at a maturity value of 11,500, and has a nominal
coupon rate of 8% per annum, compounded semi-annually; the investor is
replacing the premium by means of a sinking fund earning 4% convertible
semi-annually.
(b) [7 MARKS] Suppose that the bond is callable at the end of 9, 10, or 11 years
at a premium of 1, 000 above the maturity value. Explain what price the
investor should pay if she is no longer plans to deposit any of the interest in
a sinking fund.

7. [8 MARKS] It was n years ago when James deposited 10,000 in a bank paying
1.8% interest compounded monthly. If he had, instead, placed his deposit in a
syndicate paying interest by cheque annually at the rate of 6% per annum, and
he had invested only this interest with the bank, how much more interest would
he have earned altogether? Show all your reasoning, and express your answer in
terms of n.

8. A loan is being repaid with 16 annual payments of 1,000 each. At the time of the
4th payment the borrower requests permission, and is permitted to pay an extra
3000, and then to repay the balance over 8 years with a revised annual payment.

(a) [5 MARKS] If the effective annual rate of interest is 6%, find the amount of
the revised annual payment.
(b) [8 MARKS] Complete an amortization table for the last 8 payments, with the
following columns:
Information for Students in MATH 329 2005 01 3100

Payment Payment Interest Principal Outstanding


number amount paid repaid loan balance
5
... ... ... ... ...
12

You might also like